Sie sind auf Seite 1von 224

m

o
.c
s
s
re
p
rd
o
.w
b
o
o
b
h
a
m
rs
u
o
y
y
o
u
rs
m
a
h
b
o
o
b
.w
o
rd
p
re
• Head Office : B-32, Shivalik Main Road, Malviya Nagar, New Delhi-110017

s
s
.c
o
• Sales Office : B-48, Shivalik Main Road, Malviya Nagar, New Delhi-110017

m
Tel. : 011-26691021 / 26691713

Price : ` 150

Typeset by Disha DTP Team

DISHA PUBLICATION
ALL RIGHTS RESERVED

© Copyright Publisher
No part of this publication may be reproduced in any form without prior permission of the publisher. The author
and the publisher do not take any legal responsibility for any errors or misrepresentations that might have crept
in. We have tried and made our best efforts to provide accurate up-to-date information in this book.

For further information about the books from DISHA,


Log on to www.dishapublication.com or email to info@dishapublication.co.in
y
o
u
rs
m
a
h
b
o
o
b
.w
Preface

o
rd
p
re
s
s
.c
o
m
Recent developments in competitive examinations, where the examiner has tried to assess the
intellect of the examinee by gauging his ability to analyze and interpret the data, we find DATA
INTERPRETATION to be of paramount importance to get through with the examinations
like CAT, XAT, CSAT, Banking & other competitive examinations. The weightage of DATA
INTERPRETATION questions in competitive examinations are increasing regularly and more
number of accurate attempts in DI can reduce the burden of solving Quantitative questions in
a substantial way.
However, students find this topic abstract and abstain themselves from attempting these sets of
questions during the stipulated time of examinations. During the course of this book, we have
tried to make this topic conducive for the students and one can get a good grasp on the subject
matter by following the levels of the book.
Questions are divided in 5 different difficulty level :
Concept Applicator (CA) : Everyone should attempt this section (Basic questions for banking
aspirants)
Concept Builder (CB) : For CAT/MAT/BANK PO/Govt Job etc
Concept Cracker (CC) : For CAT / XAT/ IIFT/SNAP
Concept Deviator(CD) : For CAT / XAT/ IIFT/SNAP
Concept Eleminator (CE) : To check your understanding.
[Questions after 2008 onwards are memory based for banking and CAT]
We would like to thank our students who not only helped us in compiling our book but also helped
us while checking its answers, these include but not limited to DIVYA REYYA, GAURAV JAIN.
We would like to extend our sincere gratitude to Mr. Deepak Agarwal, Founder-Director of Disha
Publication for consistently guiding us to bring this book as immaculate.
We would be really happy to receive critical observation and suggestion from the students and
esteemed teachers for further improvement of the book. You can reach out to Us on below given
email ids.
GajendraKumar and Abhishek Banerjee
(gajendra591@gmail.com, abhishekb004@gmail.com)

Dedicated To
My Parents PRODYUT BANERJEE and RITA BANERJEE
— Abhishek Banerjee
y
o
u
rs
m
a
h
b
o
o
b
.w
o
rd
p
re
s
Contents

s
.c
o
m
Page No.

1. Line Graph 1-24

2. Data Table 25-80

3. Bar Graph 81-122

4. Pie Chart 123-146

5. Mixed Graph 147-170

6. Data Sufficiency 171-210

7. Miscellaneous 211-220
y
o
u
rs
m
a
h
b
o
o
b
.w
o
rd
1
% EXPORTED

p
45

re
42
40 40

Line

s
35 35 35

s
32

.c
30 30
28

o
25 25 25

m
20 20 20
15
10

0
5
12
10
15
10
Graph
1996 1997 1998 1999 2000 2001
Car Scooter Motorbike

Topic No. of Questions Level


Concept Applicator 25 Basic - Bank PO, MAT
Concept Builder 25 Advance - Bank PO, MAT
Concept Cracker 40 Basic - CAT IIFT XAT
Concept Deviator 27 Advance - CAT, IIFT, XAT
Concept Eliminator 15
Total 132

INTRODUCTION Line graph questions comes in the examinations


from data interpretation are generally based on various
In line graph type questions information of a variable is types –
given with respect to the other variable. This information 1. Single line graph : graphs representing single line
is given on Cartesian co-ordinate plane that has two axes which shows changes in a single variable over a
namely X axis and Y axis. certain period of time.
Consider an example where marks of 3 students A, B, 2. More than one line graph: in this type of graphs
C is given for their 4 semester. two or more dependent variable are represented.
Sem 1 Sem 2 Sem 3 Sem 4 3. Mixed variables : in the same graph, when two
continuous variables having different units of
A 60 65 80 75
measurement.
B 70 75 80 85 The above mentioned types are most frequently asked
C 75 65 70 80 graphs in different types of examinations e.g banking,
any govt. jobs , CAT, MAT, IBPS etc.
The same information can be given by line graph where
Line graphs are very useful for visual representa-
it is easier to compare the rate of increase or decrease.
tions from time series, categorical data, frequency
distributions etc.
90
80
70 Example 1:
60 Salary Increment
50 A 3000
40 B
30 C 2000
20 Salary
1000 Increment
10
0 0
Sem 1 Sem 2 Sem 3 Sem 4 2012 2013 2014 2015
y
o
u
rs
m
a
h
b
o
o
b
.w
o
rd
2 Line Graph

p
re
Above graph is showing Annual salary increment of Sol. (c) X takes 2 days to make a table, Y takes 6 days

s
s
.c
Radhekishan. Answer the following questions carefully: to make a table and Z takes 3 days to make

o
1. What is the percentage increment in 2013( approx)? 1 1 1 6

m
a chair. Together they take + + = = 1
(a) 53 % (b) 54% day to make a table 2 6 3 6
(c) 56% (d) 58% 3. What is the total number of days that Carpenter
Sol. (a) Salary of Radhekishan in 2012 : `1500 in Z will take to make one piece each of all the four
2013 salary is : `2300. items together?
2300 − 1500 (a) 32 days (b) 23 days
Percentage = ×100 = 53.33% (c) 11/59 days (d) 11/32 days
1500
So, increment in the salary is 53.33%. Sol. (b) Z will take 2 + 3 + 8 + 10 = 23 days to make
each piece.
Example 2: Study the given graph carefully to answer
the questions that follow: RATE OF INCREASE OR DECREASE
Number of days taken by three carpenters to finish Rate of increase or decrease can be determine just by
making one piece each of four different items of furniture observation of line graph. Rate of increase is more for
16 line graph whose slope (Or inclination) with respect to
14 horizon or X-axis is more.
12 Consider two graphs A and B and observe the pattern
10 Carpenter X and determine rate of increase of which variable is more.
Carpenter Y
8
Carpenter Z
Consider business done by two corporate houses from
6
January to April.
4
18
2
16
0
Chair Table Bed Cupboard 14

1. If carpenter X and carpenter Y were to make a 12

chair together how many days would they take? 10 A


B
8
(a) 1 day (b) 4 days
6
(c) 12/7 days (d) 2 days 4
Sol. (c) Carpenter X takes 3 days to make a chair 2
and carpenter Y takes 4 days to make a chair. 0
1 1 7 12 January February March April
+ = = i.e., days to make a chair
3 4 12 7 From the graph we can observe that the slope of A is
together. more than that of B or in other words we can say that
2. If carpenter X, Y and Z were to make a table increment of business done by corporate House A is
together how many days would they take? more than increment done by B.
(a) 4 days (b) 3 days Similarly we can observe that average business done by
(c) 1 day (d) 2 days corporate house A is more than that done by B.
y
o
u
rs
m
a
h
b
o
o
b
.w
o
rd
Line Graph 3

p
re
s
s
.c
o
m
Directions (Qs. No. 1-5): Study the following graph (a) 50 (b) 45
and answer the questions that follow: (c) 40 (d) 35
Number of students ( males and females ) passed out Directions (Qs. No. 6-8):  Study the following graph
from various colleges ( A, B, C, D and E) in a year. carefully to answer these questions.
( Number in thousands) [SBI Associates PO 2011]
40
35
The line graph below shows per cent profit earned by two
30 companies producing electronic goods over the years
25 % Profit = Profit earned / Total Investment × 100
20 males
Profit Earned = Total Income – Total Investment in the year
15 females
100
10 90
5 80
0 70
A B C D E
60
1. What is the average number of students (males and 50 Company A
females ) passed out from all the colleges together? 40 Company B

(a) 38000 (b) 48000


30
20
(c) 42000 (d) None of these 10
2. The number of females passed out from college 0
C is approximately what percent of the total 2004 2005 2006 2007 2008 2009 2010

number of females passed out from all the colleges 6. If the profit earned in 2006 by Compnay B was
together? `8,12,500, what was the total income of the
(a) 28 (b) 30 company in that year?
(c) 36 (d) 25 (a) `12,50,000 (b) `20,62,500
3. What is the difference between the total number of (c) `16,50,000 (d) None of these
students passing out from college A and the total 7. If the amount invested by the two companies in
number of students passing out from college E? 2005 was equal, what was the ratio of the total
(a) 20500 (b) 21000 income of the company A to that of B in 2005?
(c) 10,000 (d) None of these (a) 31 : 33 (b) 33 : 31
4. What is the ratio of the total number of males to (c) 34 : 31 (d) 14 : 11
the total number of females passed out from all the 8. If the total amount invested by the two companies
colleges together? in 2009 was ` 27 lakh while the amount invested
(a) 19:23 (b) 18:25 by company B was 50% of the amount invested by
(c) 23:19 (d) 25:18 Company A, what was the total profit earned by the
5. The number of males passing out from colleges A two companies together?
and B together is what percent of the number of (a) ` 21.15 lakh (b) ` 20.70 lakh
females passing out from colleges C and D together? (c) ` 18.70 lakh (d) ` 20.15 lakh
y
o
u
rs
m
a
h
b
o
o
b
.w
o
rd
4 Line Graph

p
re
Directions (Qs. No. 9-13): Study the following graph % profit Earned by a company over the years Per Cent

s
s
.c
carefully and answer the questions that follow. Number Profit = (Income – Expenditure) / expenditure × 100.

o
of Girl students in ‘000s [Syndicate bank PO – 2010]

m
Per Cent of Profit
35 80
30 70
25 60

20 50
School A
15 School B 40
School C
10 30

5 20
10
0
2004 2005 2006 2007 2008
0
9. What was the ratio between the number girls 2005 2006 2007 2008 2009
enrolled in the school-C in the year 2007 and the Per Cent of Profit
total number of girls enrolled in school-A and 14. What is the average per cent profit earned by the
school-B together in the same year? company over the years?
(a) 11 : 3 (b) 3 : 11 (a) 55 (b) 51
(c) 4 : 11 (d) 4 : 7 (c) 62 (d) 59
10. In which school was the difference between the 15. If the expenditure of the company in the year 2006
number of girls enrolled in the year 2008 and 2004 was ` 75,000, what was the ratio of income to
minimum? expenditure of the company in that year?
(a) Only school-A (a) 3 : 2
(b) Only School-B
(b) 5 : 4
(c) Both school-A and school-B
(c) 4 : 3
(d) Both school-A and school-C
(d) Cann’t be determined
11. What was the approximate average number of girls
16. The per cent profit earned by the company in the
enrolled in the year 2006 in all the three schools
year 2009 was what percent of the per cent profit
together?
earned by the company in the year 2005?
(a) 20,800 (b) 23,000
(c) 20,000 (d) 21,600 (a) 200 (b) 240
12. Total number of girls enrolled in all the three (c) 260 (d) 255
schools in the year 2004 was what percentage of 17. If the income of the company in the year 2005 and
the number of girls enrolled in school-C in the year the year 2007 was `6,80,000 each, then what was
2007? the difference in expenditures of the company in
(a) 208 (b) 230 the year 2005 and 2007?
(c) 200 (d) 225 (a) ` 1,24,000 (b) ` 1,40,000
13. In which year was the total number of girls enrolled (c) ` 1,50,000 (d) None of these
in all the three schools together second highest? 18. If the profit earned by the company in the year
(a) 2005 (b) 2006 2008 was `90,000, what was the income of the
(c) 2007 (d) 2008 company in that year?
(a) ` 2,90,000
Directions (Qs. No. 14-18): Study the following graph
(b) ` 2,00,000
carefully and answer the questions given below it.
(c) ` 1,50,000
[Syndicate bank PO 2010]
(d) Cannot be determined
y
o
u
rs
m
a
h
b
o
o
b
.w
o
rd
Line Graph 5

p
re
Directions (Qs. No. 19-25): Study the data presented in 21. What is the average monthly expenditure during

s
s
.c
the following graph to answer these questions. the year 1999 covering the period shown in the

o
Monthly expenditure of a firm from January to July graph?

m
during the years 1997, 1998, 1999 (a) ` 2,75,000 (b) ` 2,70,000
Chart Title (c) ` 3,14,000 (d) ` 2,47,000
320
22. Which month has been the least expensive during
Expenditure In '000 Rupees

315 1999?
310 (a) June (b) April
305 1997 (c) May (d) July
1998 23. The expenditure in April 1999 was . . . . . . . . higher
300 1999
than that of corresponding period in 1998.
295 (a) 1.5% (b) 2%
290 (c) 2.5% (d) 0.94%
0 1 2 3 4 5 6 7 24. The expenditure in 1997 was ….. less than that of
19. What is the total expenditure during the period the corresponding period in 1999.
under review (7 months) in 1997? (a) 3.27% (b) 2.5%
(a) ` 21,07,000 (b) ` 21,96,000 (c) 1.5% (d) 2%
(c) ` 21,54,000 (d) ` 21,24,000 25. In how many months out of the given 7 months
20. What total expenditure has been made during the year expenditure in 1998 is more than that in 1997 but
1997 and 1998 in the period covered in the graph? less than that in 1999.
(a) ` 42,87,000 (b) ` 2,70,000 (a) 2 (b) 3
(c) ` 48,27,000 (d) ` 42,78,000 (c) 4 (d) 5

Directions (Qs. No. 1-6): Study the following graph to Profit = Sales – Cost – Tax
answer these questions. Gross Profit = Sales – Cost
Index of sales, cost, and profit 1. What is the profit in the year 1998?
200
Chart Title
(a) ` 110
190 188
180 188 (b) ` 143
162 153
160 154
173
163
173 166
(c) ` 95
136 152 143
140 125 131 125 124 134 (d) cannot be determined
130
120 120 121 110 105
110 115
2. What is the gross profit in 1993?
100 100 98 95 110
80
95 (a) ` 27 (b) ` 30
60 (c) ` 35 (d) None of these
40 3. If the profit was `600 in 1993, then what was the
20 profit in 1990?
0
1990 1991 1992 1993 1994 1995 1996 1997 1998 1999 2000 (a) ` 441 (b) ` 395
Sales Cost Profit (c) ` 480 (d) ` 545
y
o
u
rs
m
a
h
b
o
o
b
.w
o
rd
6 Line Graph

p
re
4. If the base of the profit index is changed to 1993 10. In which year, the maximum profit was generated

s
s
.c
= 100, then what would be the index of profit for vis-à-vis in?

o
the year 2000? (a) 1991 (b) 1992

m
(a) 104 (b) 100 (c) 1993 (d) 1994
(c) 109 (d) None of these 11. Which year registered the maximum sales turnover?
5. If sales were `1200 crore in 1990, then what were (a) 1994 (b) 1995
the total sales in the period 1990-1995? (c) 1992 (d) 1993
(a) ` 8628 crore (b) ` 9828 crore 12. In which of the following years, there was the
(c) ` 9156 crore (d) ` 8136 crore maximum net growth in car sales as compared to
6. What was the difference between the average sales its earlier years?
index and the average cost index? (a) 1994 (b) 1992
(a) 7.3 (b) 7.7 (c) 1993 (d) 1995
(c) 7.5 (d) 7.9 Directions (Qs. No. 13-17): Refer the graph below to
Directions (Qs. No. 7-12): Study the graph given below answer these questions.
to answer these questions. Quarterly Profits Departmental Store
300 240
210
250
180
150
200 `
lakhs 120
150 90
60
100 30
0
1997 1998 1999 2000
50
13. Approximately, what was the actual profit made by
0
the department store in the second quarter of 1999?
1991 1992 1993 1994 1995 1996 (a) ` 160 lakh (b) ` 170 lakh
Selling price per car (in thousands) (c) ` 180 lakh (d) ` 210 lakh
No. of cars sold (in thousands) 14. In which of the following quarters, did the
Manufacturing cost per car (in thousands) departmental store make the least amount of profits?
7. In which year, were the gross sales proceeds the (a) Third quarter of 2000
highest? (b) Second quarter of 1999
(a) 1992 (b) 1993 (c) First quarter of 1999
(c) 1994 (d) 1995 (d) Third quarter of 1998
8. Which year was the least profitable from the fiscal 15. During the period 1998-2000, how many quarters
point of view? exceeded the profit of ` 150 lakh?
(a) 1993 (b) 1994 (a) 6 (b) 5
(c) 1995 (d) 1996 (c) 4 (d) 3
9. In which year the number of cars sold was the 16. In the year 2000, total profit made by the
lowest but financial turnover, the maximum? departmental store was approximately :
(a) 1993 (b) 1994 (a) ` 540 lakh (b) ` 630 lakh
(c) 1995 (d) 1996 (c) ` 720 lakh (d) ` 770 lakh
y
o
u
rs
m
a
h
b
o
o
b
.w
o
rd
Line Graph 7

p
re
17. The total amount of profit made by the departmental

s
s
.c
store increased by approximately what percent 70 70

o
High
from 1997 to 2000 ? 60 60

m
(a) 40% (b) 50% 50 50

Avg.
(c) 90% (d) 120% { 40 40
30 30
Directions (Qs. No. 18-20): Study the following graph,
{

Low
20 20
showing the sales forecast for the next ten weeks, to 10 10
answer these questions.
- Sales Forecast Theoretical Economic Aesthetic Social Political Religious

Series 1 Average male profile Average female profile


500 Personal profile
450
400 21. Compare the three and state which of the given
350
300
values is the highest in the personal profile of
250
Series 1 the student?
200
150 (a) Theoretical (b) Religious
100
50
(c) Social (d) Economic
0
1 2 3 4 5 6 7 8 9 10
22. In the given personal profile, which is the value
with the lowest score?
18. If the forecasted demand is met by having uniform (a) Theoretical (b) Religious
production during the weeks at an average level,
then the number of weeks during which demand (c) Social (d) Aesthetic
will not be met is : 23. In which values score, there exists maximum
(a) 2 (b) 3 difference between average female profiles and
(c) 4 (d) None of these personal profile?
19. If the production is uniform, then what should be (a) Theoretical (b) Religious
the minimum capacity of the storage space to store (c) Economic (d) Political
the units in excess of demand?
24. In which value score, there exists convergence
(a) 25 (b) 50
between personal profile and average female
(c) 100 (d) 200
profile?
20. If the maximum production capacity is 300 units,
then the unmet demand will be (a) Theoretical (b) Social
(a) 225 (c) Aesthetic (d) None of the above
(b) 275 25. In which value score, there exists a no difference
(c) 175 state between the personal profile and average
(d) All the demand will be met male profile?
(a) Economic
Directions (Qs. No. 21-25): Use the graph given below
to answer these questions : Given is graph is the profile (b) Social
of values of a college student marked as personal profile. (c) Aesthetic
The normative profiles are given as average male profile
(d) None of the above.
and average female profile.
y
o
u
rs
m
a
h
b
o
o
b
.w
o
rd
8 Line Graph

p
re
s
s
.c
o
m
Directions (Qs. No. 1-3): Refer to the graph given 5. The performance for which of the following houses
below which gives the circulation growth of GRAMSEWA is the best?
magazine from July to December 2003. (a) Pearl (b) Ruby
Series 1 (c) Topaz (d) Sapphire
250000 6. For which of the following houses is the percentage
200000 173182
211885
change in the results maximum for any years over
200189 204933
150000 175395 189277 the previous years?
100000 Series 1 (a) Topaz (b) Pearl
(c) Sapphire (d) Ruby
50000
7. If the trend observed between 1999 and 2000
0
JUL AUG SEPT OCT NOV DEC
continues in the next year, what will be number of
students passing in the examination in 2001?
1. During November and December, there was an (a) 245 (b) 237
even growth rate, the average of which is (c) 263 (d) 300
(a) 2.36% (b) 2% 8. The number of students keeps on increasing by 50
(c) 2.88% (d) 3.36% every years. In 1998, there were 250 students. For
2. The circulation in October is . . . times than that of which of the following years is the performance
July. best in the school?
(a) 1.5 (b) 2 (a) 1998
(c) 1 (d) 1.15 (b) 2000
3. The growth rate is very marginal during the month (c) 1999
of : (d) Cannot be determined
(a) August (b) October Directions (Qs. No. 9-13): Study the following line
(c) November (d) December graphs which show the production (in, 0000 units) and
Directions (Qs. No. 4-8): The scatter diagram shows percentage expected of scooters, motor bikes and car
the number of students passing in the high school respectively over the years
examination in the given years from the four houses of Production (in ‘0000 units)
a public school. Chart Title
100 100
80 90
84 86
80
60 75
70
40
60 60 60 60 60
20 56
50 48 48
0 44
1997 1998 1999 2000 2001 40 40 42
38 40
Reby Topaz Pearl Sapphire 34 34
30
26
4. The average number of students for each house 20
who have passed in the given years is 10
(a) 61 (b) 52 0
1996 1997 1998 1999 2000 2001
(c) 63 (d) 56 Car Scooter Motorbike
y
o
u
rs
m
a
h
b
o
o
b
.w
o
rd
Line Graph 9

p
re
s
% EXPORTED

s
800 H

.c
45 a d
750

o
42

Cumulstive distance (in kms)


40 40 G

m
35 35 35 a d
574 a d F
32
30 30 468
28 a d E
25 25 25 380 D
20 20 20
a d
15 15 231 C
12 a d
10 10 10 140 B
5
0 A

5:00
6.40
6.50
8.10
8.12
9.55
10.00
11.20
11.30
12.45
13.00
15.00
15.10
16.25
1996 1997 1998 1999 2000 2001
Car Scooter Motorbike Clock Time

9. In which of the following years was the production 14. The average speed the train maintained between
of cars more than 50% of the production? two successive stations was maximum between
(a) 2000 (b) 2001 (a) E–F (b) F–G
(c) 1998 (d) 1996 (c) G–H (d) Both G–H and F–G
10. Find the total number of automobiles exported in 15. Between how many pairs of consecutive stations
the year 1999 : does the speed run below the overall average speed
(a) 2,27,600 (b) 2,07,600 of the entire trip?
(c) 2,17,600 (d) 2,20,000 (a) 4 (b) 1
11. Find the ratio of cars, scooters and motorbikes
(c) 3 (d) 2
exported in 1996 :
16. If the train stops at each city for 30% more time
(a) 25 : 16 : 19 (b) 16 : 25 : 19
that what it is at the moment, then at what time will
(c) 19 : 16 : 25 (d) 6 : 5 : 1
it reach the city H after departing from city A as per
12. If the ratio of export prices of a cars, scooter and
schedule?
motorbike was 2 : 1 : 1.5 in 1998, what was the
(a) Data insufficient (b) 17 : 03
proportion of their export earnings?
(a) 4 : 2 : 3 (c) 16 : 41 (d) 16 : 58
(b) 6 : 1 : 21 17. The train begins its onward journey from City A
(c) 45 : 24 : 31.5 and it is extended to beyond City H to a City M
(d) Cannot be determined due to some unavoidable reason. The train starts
13. In which of the following years was the production its return journey immediately after it reached City
of motorbikes exactly 40% of the total production M. the train returns with a speed of 90km/h without
of automobiles in that years? any stoppages in between and reaches City A at
(a) 1997 (b) 2000 2 : 25 am. Find the distance between city H and
(c) 1999 (d) 1996 city M.
(a) 40 km (b) 90 km
Directions (Qs. No. 14-17): Study the following line (c) 70 km (d) None
graph to answer these questions.
Railway Time Schedule of an Express Train X Running Directions (Qs. No. 18-21) :  Consider the following
Between City A and City H graph where the prices of timber are given for the period
1997 – 2003. The prices for plywood and sawn timber
a → Arrival of train b → Departure of train A, B, C,
are given in Rs/m^3 while the price of logs is given in
D, E, F, G and H → Cities throughout which the train
Rs/tonne. Assume 1 tonne in equal to 1,000 kg and one
runs. a – d → indicates stoppage/halting of the train at
cu. M. of log weights 800kg.
the city station.
y
o
u
rs
m
a
h
b
o
o
b
.w
o
rd
10 Line Graph

p
re
Timber Product Prices

s
140

s
.c
120

o
m
20000 100
80

15000 60
40
20
10000
0
Jan Feb Mar Apr May June
A B C D
5000
22. Which share showed the greatest percentage
increase in market value in any month during the
0
entire period?
1997 1998 1999 2000 2001 2002 2003 (a) A (b) B
Plywood Sawn Timber Logs (c) C (d) D
23. In which month was the greatest absolute change
18. Which product had the largest percentage increase in market value for any share recorded?
in price per cubic metre over the 7 year period? (a) March (b) April
(a) Sawn timber (c) May (d) June
24. In which month was the greatest percentage
(b) Logs
increase in market value for any share recorded?
(c) Plywood (a) February (b) March
(d) Cannot be determined (c) April (d) May
25. An individual wishes to sell 1 share of C and
19. The maximum increase in price per cubic metre for
1 share of D to buy 1 share of A at the end of a
any product between two years was month. At which month-end would the individual’s
(a) ` 2,500 (b) ` 3,000 loss from this decision, due to share value changes,
(c) ` 2,000 (d) ` 4,125 be the most?
(a) February (b) March
20. In 2003, the total sales of the company measured in (c) April (d) June
cubic metres was made up of 40% plywood, 30% 26. An individual decides to sell 1 share of C and 1
sawn timber and 30% logs. The average realization share of D to buy 1 share of A at the end of the
per cubic metre in 2003 was closest to month. What can be the individual’s greatest gain
(a) ` 16,500 (b) ` 13,500 from this decision, due to share value changes?
(a) 5 (b) 10
(c) ` 15,425 (d) ` 18,000 (c) 15 (d) None
21. In 2004, the prices of plywood, sawn timber and Directions (Qs. No. 27-30): Study the graph below
logs went up by 5%, 1% and 10% respectively and and answer the questions that follow. [CAT – 1993]
the total sales were made up of 40% plywood, 30% 140

sawn timber and 30% logs. The average realization 120


per cubic metre in 2004 was closest to 100
(a) ` 15,500 (b) ` 16,135.5 80 Sales
(c) ` 14,500 (d) ` 18,500 60 Expenditure

Directions (Qs. No. 22-26): The graph below shows 40 Equity


the end of the month market values of 4 shares for the 20
period from January to June. Answer the following 0
questions based on this graph. [CAT – 1991] 1990 1991 1992 1993
Profit is defined as Sales – Expenditure
y
o
u
rs
m
a
h
b
o
o
b
.w
o
rd
Line Graph 11

p
re
27. In which year is the profit per rupee of equity the (a) 0.04 (b) 0.4

s
s
.c
highest? (c) 4 (d) 0.35

o
(a) 1991 (b) 1992 33. By what % is the solubility of Potassium Chlorate

m
(c) 1993 (d) 1990 – 1991 in water increased as the water is heated from 30°C
28. The simple annual growth rate in sales was the to 80°C?.
highest between the years? (a) 100 (b) 200
(a) 1990 – 91 (b) 1991 – 92 (c) 250 (d) 300
(c) 1992 – 93 (d) 1990 – 92
34. If 1 mole of Potassium Chloride weighs 0.07456
29. In which year is the sales per rupee of expenditure
kg, approximately. How many moles of Potassium
the lowest?
Chloride can be dissolved in 100 litres of water at
(a) 1990 (b) 1991
36°C?
(c) 1992 (d) 1993
30. In which year is sales per rupee of equity the (a) 700 (b) 650
highest? (c) 480 (d) 540
(a) 1990 (b) 1991 35. Which of the salts has greater change in solubility
(c) 1992 (d) 1994 in kg / litre of water between 15°C and 25°C?
(a) Potassium Chlorate
Directions (Qs. No. 31-35): Are based on the graph
(b) Potassium Nitrate
given below:
(c) Sodium Chlorate
Solubility-Temperature relationships for various salts.
(The Y-axis denotes Solubility (kg/litre of water)) (d) Sodium Nitrate
[CAT – 1994] Direction(Qs. No. 36-40): Answer the questions based
2 on the following graph. [CAT – 1996]
1.8 45
40
1.6
35
1.4 30
Sales
1.2 25
Cost
20
1 Employees
15
0.8 10
0.6 5
0.4 0
Jan Mar May Jul Sept Nov
0.2 Employees in thousands Sales – Cost = Profit
0 36. Which month records the highest profit?
0 10 20 30 40 50 60 70 80 90 100
Potassium Chlorate Potassium Chloride (a) September (b) July
Potassium Nitrate Sodium Chloride (c) March (d) May
Sodium Chlorate Sodium Nitrate 37. In which month is the total increase in the cost
31. Which of the following salts has greatest solubility? highest as compared to two months ago?
(a) Potassium Chlorate at 80°C. (a) March (b) September
(b) Potassium Chloride at 35°C. (c) July (d) May
(c) Potassium Nitrate at 39°C. 38. In which month is the percentage increase in sales
(d) Sodium Chloride at 85°C. two months before, the highest?
32. Approximately, how many kg of Potassium Nitrate (a) March (b) September
can be dissolved in 10 litres of water at 30°C? (c) July (d) May
y
o
u
rs
m
a
h
b
o
o
b
.w
o
rd
12 Line Graph

p
re
39. Which month has the highest profit per employee? 40. Assuming that no employees left the job, how

s
s
.c
(a) September (b) July many more people did the company take on in the

o
(c) January (d) March given period?

m
(a) 4,600 (b) 5,000
(c) 5,800 (d) 6,400

Directions (Qs. No. 1-3): Answer the questions on the Assets ( ` crore)
180
basis of the graph given below. [XAT – 2008] 160 160
140 150
2400 Sales and Costs of XYZ Co. 120 125 135
2300 100 110
2200 80 100
2100 60
2000 40
1900 20
1800 0
1700 2004 2005 2006 2007 2008 2009 2010
1600
1500 CSR Spending (% of sales)
1400 3.00%
1300 2.50%
1200 2.50%
1 2 3 4 5 6 7 8 9 10 2.00% 2.09% 2.22%
Month 2.08%
Figure in 1.50% 1.67%
Rupees Sales Cost of Goods Sold 1.50%
1.00%
1. In which month did the company earn maximum 0.50%
profits? 0.00%
2004 2005 2006 2007 2008 2009
(a) 5 (b) 4
Sales (% of Assets)
(c) 3 (d) 2 1.4
2. In which month did the company witness maximum 1.2 1.25

sales growth? 1
0.8
(a) 9 (b) 6 0.6 0.67
0.8
0.64
(c) 7 (d) 1 0.4
0.6 0.5
3. What were average sales and costs of figures for 0.2
XYZ Co. over the period of ten months? 0
2004 2005 2006 2007 2008 2009
(a) 1819, 1651 (b) 1919, 1751
(c) 1969, 1762 (d) 1719, 1601 4. In which year was the increase in spending on
Directions (Qs. No. 4-6): Answer the questions based on CSR, vis-a-vis the previous year, the maximum?
the following information. [XAT – 2011] (a) 2006 (b) 2007
The following graphs give annual data of Assets. Sales (c) 2008 (d) 2009
(as percentage of Assets) and Spending on Corporate 5. Of the years indicated below, in which year was
Social Responsibility (CSR) (as percentage of Sales), the ratio of CSR Assets the maximum?
of a company for the period 2004 - 2009. (a) 2004 (b) 2005
(c) 2006 (d) 2008
y
o
u
rs
m
a
h
b
o
o
b
.w
o
rd
Line Graph 13

p
re
6. What was the maximum value of spending on CSR Directions (Qs. No. 11-14): These questions are based

s
s
on the price fluctuations of four commodities - arhar,

.c
activities in the period 2004 – 2009?

o
(a) ` 0.5 Crore (b) ` 1.0 Crore pepper, sugar and gold during February - July 1999 as

m
(c) ` 2.0 Crore (d) ` 3.0 Crore described in the figures below: [CAT – 1999]
Arhar (Friday quotations)
Directions (Qs. No. 7-10): These questions are based 2500

on the situation given below: [CAT – 1999] 2300

The figure below presents sales and net profit, in ` Crores, 2100

Price (`/qrt)
of IVP Ltd for the five years from 1994-95 to 1998-99. 1900
During this period, the sales increased from `100 Crores 1700
to `680 Crores. Correspondingly, the net profit increased 1500
from `2 Crores to `12 Crores. Net profit is defined as the 1300
excess of sales over total costs. F1 F4 M3 A2 A5 M3 JN2 JL1
700 14 Weeks (1999)
600 12   
500 10 Pepper (Friday quotations)
400 8 20000
300 6 19500
200 4
19000
Price (`/qrt)

100 2
0 0 18500
94-95 95-96 96-97 97-98 98-99
Sales Net Profit 18000

17500
7. The highest percentage of growth in sales, relative
to the previous year, occurred in 17000
F1 F4 M3 A2 A5 M3 JN2 JL1
(a) 1995-96 (b) 1996-97 Weeks (1999)
(c) 1997-98 (d) 1998-99 Sugar (Friday quotations)
8. The highest percentage growth in net profit, 1520

relative to the previous year, was achieved in 1500

(a) 1998-99 (b) 1997-98 1480


Price (`/qrt)

(c) 1996-97 (d) 1995-96 1460

9. Defining profitability as the ratio of net profit to 1440


sales, IVP Ltd. recorded the highest profitability in 1420
(a) 1998-99 (b) 1997-98 1400
(c) 1994-95 (d) 1996-97 F1 F4 M3 A2 A5 M3 JN2 JL1
10. With profitability as defined in question 137, it can Weeks (1999)
  
be concluded that Gold (Friday quotations)
4400
(a) Profitability is non-decreasing during the five 4300
years from 1994-95 to 1998-99.
Price (`/10 gms)

4200
(b) Profitability is non-increasing during the five 4100
years from 1994-95 to 1998-99. 4000
(c) Profitability remained constant during the 3900
five years from 1994-95 to 1998-99. 3800
(d) None of the above 3700
F1 F4 M3 A2 A5 M3 JN2 JL1
Weeks (1999)
y
o
u
rs
m
a
h
b
o
o
b
.w
o
rd
14 Line Graph

p
re
11. Price change of commodity is defined as the Note: Average unit cost, AC = Total monthly cost /

s
s
absolute difference in ending and beginning prices

.c
monthly production, and Marginal cost, MC is the rate of

o
expressed as a percentage of the beginning. What change in total cost for unit change in quantity produced.

m
is the commodity with the highest price change? 15. Total production in July is 40 units. What is the
(a) Arhar (b) Pepper
approximate average unit cost for July?
(c) Sugar (d) Gold
12. Price volatility (PV) of a commodity is defined as (a) 3600 (b) 90
follows: (c) 140 (d) 115
PV = (highest price during the period lowest price 16. ABC Ltd. is considering increasing the production
during the period) / average price during the period. level. What is the approximate marginal cost of
What is the commodity with the lowest price increasing production from its July level of 40 units?
volatility?
(a) Arhar (b) Pepper (a) 110 (b) 130
(c) Sugar (d) Gold (c) 150 (d) 160
13. Mr.X, a fund’s manager with an investment 17. From the data provided it can be inferred that, for
company invested 25% of his funds in each of the production levels in the range of 0 to 60 units,
four commodities at the beginning of the period.
(a) MC in an increasing function of producing
He sold the commodities at the end of the period.
quantity.
His investments in the commodities resulted in:
(a) 17% profit (b) 5.5% loss (b) MC is a decreasing function of production
(c) no profit, no loss (d) 3% profit quantity.
14. The price volatility of the commodity with the (c) Initially MC is a decreasing function of
highest PV during the February-July period is production quantity, attains a minimum and
approximately equal to: then it is an increasing function of production
(a) 3% (b) 40% quantity.
(c) 20% (d) 12%
(d) None of the above
Directions (Qs. No. 15-20): Answer the questions based
18. Suppose that each widget sells for `150. What is
on the following information: [CAT – 2000]
the profit earned by ABC Ltd. in July? (Profit is
ABC Ltd. Produces widgets for which the demand
is unlimited and they can sell all of their production. defined as the excess of sales revenue over total
The graph below describes the monthly variable costs cost.)
incurred by the company as a function of the quantity (a) 2400 (b) 1600
produced. In addition, operating the plant for one shift (c) 400 (d) 0
results in a fixed monthly cost of `800. Fixed monthly 19. Assume that the unit price is `150 and profit is
costs for second shift operation are estimated at `1200. defined as the excess of sales revenue over total
Each shift operation provides capacity for producing 30 costs. What is the monthly production level of
widgets per month. ABC Ltd. at which the profit is highest?
Variable Cost
7000 (a) 30 (b) 50
6000 (c) 60 (d) 40
5000 20. For monthly production level in the range of 0 to
30 units,
4000
(a) AC is always higher than MC.
3000
(b) AC is always lower than MC.
2000
(c) AC is lower than MC up to a certain level and
1000 then is higher than MC.
0 (d) None of the above is true.
1 6 11 16 21 26 31 36 41 46 51 56
y
o
u
rs
m
a
h
b
o
o
b
.w
o
rd
Line Graph 15

p
re
Directions (Qs. No. 21-24): Answer the questions on 21. After which month did Seeta’s rate of growth start

s
s
.c
the basis of the information given below. [ CAT – 2003] to decline?

o
The length of an infant is one of the measures of his / (a) Second month (b) Third month

m
her development in the early stages of his / her life. The (c) Fourth month (d) Never
figure below shows the growth chart of four infants in 22. Who grew at the fastest rate in the first two months
the first five months of life. of life?
65 (a) Geeta (b) Seeta
(c) Ram (d) Shyam
60 23. The rate of growth during the third month was the
Length of (cm)

lowest for
55
(a) Geeta (b) Seeta
(c) Ram (d) Shyam
50
24. Among the four infants, who grew the least in the
45 first five months of life?
0 1 2 3 4 5 (a) Geeta (b) Seeta
Month
Seeta Geeta Ram Shyam (c) Ram (d) Shyam

Directions (Qs. No. 1-5): Answer the questions on the Ratio of Revenue generated by these five restaurants in
basis of following information. Indian and Chinese food is as given below
The line graph given below shows the revenue (in Take away Dine In
million $) generated by five restaurants in its Take away (Indian: Chinese) (Indian: Chinese)
and Dine In facility. The table below gives the break-up Foodie 7:3 4:1
of the revenue generated through Take away and Dine In
Lazeez 3:2 1:1
facility of five restaurants in Indian and Chinese foods.
5
4.5 Dhaba 9:1 7:3
4.5
4.3 4.4
4
3.8 Pizza hut 4:1 3:2
3.5
3.5
3
2.8 Sarson 1:1 9:1
2.5 Dine In
2.4 2.5
2 Take Away
1.5
1.8 1.7 Performance of a restaurant A is better than restaurant
1 B either in Take away or in Dine In only if sales of
0.5 restaurant A is better than that of restaurant B in both
0
Foodie Lazeez Dhaba Pizza hut Sarson Indian as well as Chinese food.
y
o
u
rs
m
a
h
b
o
o
b
.w
o
rd
16 Line Graph

p
re
1. How many of the restaurant definitely performed

s
40000

s
.c
better than Dhaba in both Take away and Dine In? 35000

o
m
34192
(a) 0 (b) 1 30000 29619

(c) 2 (d) 3 25000


22572

2. How many restaurants performed better than 20000


15702 16416
17169 14525
sarson but not better than Foodie either in Take 15000
10702
away or Dine in section? 10000
12564 9115 12324
6123
7652
(a) 0 (b) 1 5000 7811 8216
5119 5289 5782
(c) 2 (d) 3 0
2009 2010 2011 2012 2013 2014
3. For which restaurant the revenue earned from
Total Members Number of female members
Indian food is maximum Number of highly educated members
(a) Foodie (b) Dhaba 6. Which of the following years witnessed the highest
(c) Pizza hut (d) Sarson % increase in new members over the previous years?
(a) 2009 (b) 2010
4. How many restaurants performed better than Sarson
(c) 2013 (d) None of these
in Take away and better than Dhaba in Dine In?
7. In which year women and men ratio is greatest?
(a) 0 (b) 1
(a) 2011 (b) 2012
(c) 2 (d) 3 (c) 2013 (d) None of these
5. For which restaurant difference is maximum in 8. The number of men who joined the club in 2010
revenue earned through Chinese food in Take-away was what % of the total number of people who
and Dine-in section? joined the club in the same year?
(a) 0.63 (b) 0.75
(a) Foodie (b) Dhaba
(c) 0.27 (d) 0.11
(c) Lazeez (d) Sarson
9. If there were 30 % old men (approx.) in the club
Directions (Qs. No. 6-8): Answer the questions on the in 2013 then what is the percentage of new young
men to the young men in the group in 2014,
basis of given information:
considering there were no old man join the club?
The information are for the members of a society club (a) 74.52 (b) 57.36
“Bidhanpark Unayan Club” run by the residents of
(c) 39.24 (d) 18.07
Bidhanpark. The club started its operation for social
welfare and welfare for society on 1st January 2009. 10. The number of educated and illiterate members
Every year after 2009, some new volunteers joined the (not highly educated) who joined the club in 2013
as a percentage of total number of men who joined
club and still no one left the group from 2009 to 2014. the club in the same year could not be more than
All the values are recorded on the very second day of
(a) 11.43 (b) 15.65
each year.
(c) 9.63 (d) 17.34
y
o
u
rs
m
a
h
b
o
o
b
.w
o
rd
Line Graph 17

p
re
Directions (Qs. No. 11-15): At Praxis Business School 11. Maximum score of which team is the highest?

s
s
.c
Kolkata, four teams namely A, B, C and D participated (a) B (b) A

o
in an in house Quiz which is conducted on 4 stages. (c) D (d) C

m
Scores of these 4 stages are as given below. 12. If A has achieved the highest score then what is the
Further it is known that at any stage score of highest and score achieved by C
lowest scorer is entered correctly and remaining two (a) 330 (b) 335
groups may be correct or may be interchanged. viz. at (c) 325 (d) Cant determine
stage 1 score 75 is of D (Highest) and score 60 is of B 13. Score of how many of the groups can be determined
(Lowest) and remaining two scores 65 and 70 may be uniquely?
entered correctly or it is shuffled means A may have got
(a) 1 (b) 2
65 or 70 and similarly C may have got 70 or 65. No two
(c) 3 (d) 4
groups got same score at any stage.
14. If C has achieved the lowest possible score then-
100
90
95 90 95 (a) A has 4 possible total score
85 85 90
80
75 75 80 80 (b) D has 4 possible total score
70
70
65 70 65 65 (c) B has 2 possible total score
60
50
60 A (d) D has 3 possible total score
B
40 C 15. Which group could get maximum percentage
30 D increase in the score in any stage when compared
20 to previous stage?
10
(a) B (b) A
0
Stage 1 Stage 2 Stage 3 Stage 4 (c) D (d) C
y
o
u
rs
m
a
h
b
o
o
b
.w
o
rd
p
re
s
s
.c
o
m
CONCEPT APPLICATOR ⇒ 37500 =I – 75000 ⇒ I = 112500. Ratio of income
to expenditure in 2006 = 112500 : 75000 = 3 : 2
sum of all males + sum of all females
1. (c) Average = Alternate : Since profit is 50% so required ratio
total number of colleges
150 3
(115+95) is =

= = 42 or 42 thousands 100 2
5 35 16. (c) Percent profit earned in the year 2009 was 65
2. (b) required % = × 100 = 30% (approx)
115 65
and 2005 was 25. Therefore, × 100 = 260.
3. (d) say 38000 passed from A and 18000 passed 25
17. (d)
from E, difference is 20,000
95 19 Year 2005 Year 2006
4. (a) ratio = = 25 = 6,80,000 – E / E × 70 = 6,80,000 – E / E
115 23
32.5 100 ; × 100
5. (a) ×100 = 50%
65 25 E = 6,80,00000 – 70 E = 6,80,00000 –
6. (b) Let x be the investment, 55% of x = profit 100E ; 100E
earned (8,12,500). E = 6,80,00000 / 125 E = 6,80,00000 / 170
X = 12,50,000 then income = profit earned + = 5, 44, 000 = 4,00,000
investment = 20,62,500 (approx.).
Difference = 5,44,000 – 4,00,000 = 1,44,000
70 90,000
7. (c) profit earned Company A = x 18. (a) As per the formula 45 = ×100
100 E
55 ⇒ E = 2,00,000. Therefore, income =
profit earned Company A = x (where x is
investments) 100 expenditure + profit i.e., 2,00,000 + 90,000
170 = 2,90,000
required ratio = = 34 : 31 19. (d) add all the values of expenditure of 1997 –
155
8. (b) Let x be the sum invested amount by A (306 +300 +300 + 306 + 300 + 306 +306) ×
x 1000 = 2124000.
So, x + = 27, x = 18lakh. 20. (d) Total expenditure of 1998 = 2154 and
2
Profit : Company A = 75 % of 18lakh = 13.5lakh expenditure of 1997 = 2124
Company B = 80 % of 9lakh = 7.2lakh Hence , total expenditure = 4278000
21. (c) average monthly expenditure of 1999 = 2196/7
9. (c) 20 : (30 + 25) = 20 : 55 i.e. 4 : 11
= 313.714 or 314000( approx)
10. (c) If we go through the line graph it is clearly
22. (c ) in 1999 , least expenses month is may.
evident that the difference is minimum in both
23. (b) difference in expenditure ( 1999 – 1998 )
School A and School B
= 318 – 312
(15000 + 20000 + 28000) Required % = ( 6/ 312 ) x 100 = 1.92 % or 2%
11. (a) = 21000 ≈ 20800
3 ( approx)
45
12. (d) × 100 = 225% 24. (a) Total expense in 1997 is 2124000 and that in
13. (c) 20 1999 is 313714 required percentage is
14. (b) Average percent profit earned by the company 2196000 − 2124000
× 100 = 3.27%
25 + 50 + 70 + 45 + 65 2196000
of all years = = 51.
5 25. (c) From the line graph in 2nd, 3rd, 4th and 7th
I − 75000 month expenditure in 1998 is more than that
15. (a) As per the formula 50 =
75000 in 1997 but less than that in 1999.
y
o
u
rs
m
a
h
b
o
o
b
.w
o
rd
Line Graph 19

p
re
15. (a) profit of 6 quarter exceeds `150 lakh

s
CONCEPT BUILDER

s
.c
16. (d) addition of all quarter profits of 2000 = 770
1. (a) if we follow the graph we can clearly

o
lakh

m
understand the profit is 110 in 1998 17. (c) profit of 1997 = 400 lakh
2. (d) Lets follow the formula, gross profit = sales – 770 − 400
cost = 152 – 110 = 42 Increased % = × 100
400
3. (d) we can solve it by ratio, if the profit of 1993 = 92.5% or 90 % approx
was 600 and cost = 110. Let the profit be x for 18. (d) we can find out, average sales of forecast is
600 x
1990 and the cost was 100, then = 322. From the graph we came to know that
110 100 there are four weeks during which demand
[assuming the ratio of profit and cost be same] will not be met.
then x = 545. 19. (d)
4. (a) profit index = 104 20. (c) demand which doesn’t met = 475 – 300 = 175
5. (b) Total sales from 1990 – 1995 = 819. If the sales 21. (d) as we follow the graph we can clearly observe
of `100 becomes `1200 crore then sales of that personal profile is the highest.
1200
` 819 becomes, then × 819 = 9828 crore 22. (b)   23. (b) 24. (c)   25. (d)
100
6. (c) average sales index – average cost index
CONCEPT CRACKER
= 147.27 – 139.73 = 7.54
7. (d) Hint : if we product selling price per car 1. (c) we have to find the growth of the month Nov
with the no. of cars sold each year we get and Dec first than their average.
respective gross sales. compare the each year 204933 − 200189
Growth of the month nov =
in the options. answer is 1995 200189
4744
8. (c) manufacturing cost = × 100 = 2.36%
200189
1993 = 160 × 130 = `20800 million 211885 − 204933
1994 = 150 × 170 = `25500 million Growth of the month dec =
6952 204933
1995 = 195 × 190 = `37050 million = × 100 = 3.39 %
1996 = 160 × 210 = `33600 million 204933
2.36 + 3.39
Now, we apply Profit = sales – cost, we get Average = = 2.87%
least profit earned in the year 1995 2
2. (d) circulation of October is 200189 and July has
9. (d)
200189
years 1993 1994 1995 1996 173182, then = 1.15 times
173182
No. of cars 160 150 195 160 3. (a) we can easily answer it by seen the graph that
turnover 33120 31500 40950 38800 august has very marginal growth.
Solution from 4 - 8
10. (a) 11. (b)
12. (d) from the graph it is clear 1998 1999 2000 Total
13. (a) it is clear from the graph that profit made is Reby 40 35 35 110
160 lakh Topaz 70 55 70 195
14. (d) comparing the options we come to know that Pearl 45 75 75 195
3rd quarter 1998 make least profit. i.e
Sapphire 60 80 90 230
3rd quarter 2nd quarter 1st quarter 3rd quarter Total 215 245 270 730
2000 1999 1999 1998
730
210 lakh 160 lakh 120 lakh 75 lakh 4. (a) average = = 61
12
y
o
u
rs
m
a
h
b
o
o
b
.w
o
rd
20 Line Graph

p
re
5. (d) From the line graph we can observe that 16. (c)

s
s
.c
performance of Sapphire is the best.
17. (d) Let distance between H and M be d km. time

o
6. (b) in 1999, percentage increase in pearl

m
860
75 − 45 taken by the train = = 9 and 5/9 hrs
= × 100 = 66.66% 90
45 follow the trend the total number of
7. (d) as if we The train returns from city A in 26 and 25/60 hrs
students passed in 1999 was 245 and 2000
were 270, so the no. of students passed in Therefore the train reaches H ( 26 + 25/60 – 9
2001 would be around 300. + 5/9 ) = hrs
8. (a) In 1998 the performance was the best. Time taken the train from H to M and return
607 5 607
9. (a) in 2000, percentage of produced cars out of on the same path = – 16 + = –
197 4 36 12 36
86 = hr
the total = × 100 = 53.08%
86 + 42 + 34 12 3
10. (b) Total automobiles exported in 1999 = 10% of As the train runs from G to H 88 km/h.
40,0000+ 12% of 480000 + 20% of 60,0000 therefore the train runs 88km/h from city H to
= 40,000 + 57,600 +120000 = 217600. city M also
11. (b) cars = 28% of 480000, scooters = 35% of So, speed of the train between city H and city
600000 and motorbike = 42% of 380000.
M = 90 km/h
Ratios of these = 16:25:19
Therefore distance covered be x km, i.e
12. (c) required ratio = 30% of 75 × 2 : 40% of 60 ×
x x 4
1 : 25 % of 84 × 1.5 = 45 : 24 : 31.5 + = or x = 59 km
80 90 3
13. (a)
18. (b) percentage change or increase of the
14. (d) average speed maintained by the train between
106 followings
E and F ( 106 kms in 1and ¼ hrs) =
1 20000 − 14000
1+ For plywood = × 100 = 42.8%
4 14000
= 84.8 km/h. in the similar way we get, F and
G = 88 km /h and G and H = 88 km /h 16000 − 10000
For swan timber = × 100 = 60%
15. (d) average speed for the entire trip is – 10000
Total distance 860 km and 11h 25 min i.e 7000 − 4000
For logs = × 100 = 75%
860 4000
= 75.2 km /hr 19. (b)
5
11 +
12 20. (c) Total sales of the company in 2013 be
Lets calculate the average speed maintained 40% of 20000 + 30% of 16000 + 30% of 7000
by the train between stations i.e A and B
= 14900 which is nearer to 15425.
140 × 3
140 km in 1 + 2/3 hr = = 84 km/h. 40 101 30
5 21. (b) 20000 × (1 + 5%) × + 16000 × ×
similarly we can calculate rest one – 100 100 100
B and C = 68.25 km/h, C and D = 86.8km/h, D  110   30   1000 
+700 
and E = 66km/h, E and F = 84.8 km/h, F and  100   100   800 
G = 88 and Gand H = 88 km/h.
= 8400 + 4848 + 2887.5
= ` 16,135.5
Hence for two stations it runs below the over
all average speed.
y
o
u
rs
m
a
h
b
o
o
b
.w
o
rd
Line Graph 21

p
re
Solution from 22 - 26 : The values of the graph are mentioned below:

s
s
.c
A % Change B % Change C % Change D % Change

o
m
Jan 100 - 70 - 60 - 40 -
Feb 95 -5% 72 2.85% 55 -8.33% 50 25%
Mar 115 21% 74 2.77% 60 9.09% 50 -
Apr 105 -8.70% 76 2.70% 69 15% 41 -18%
May 100 -4.70% 78 2.63% 60 -13% 44 7.31%
Jun 110 10% 80 2.56% 55 -8.33% 45 2.27%

22. (d) The highest % increase is for D in Feb. viz.25%


23. (a) For share “A” for month of March, The absolute change in the market value for any share recorded is
= 115 – 95 = 20
24. (a) The percentage change in any share was recorded for share D for the month of February viz. 25%
25. (d) Follow the graph
C D Total Earning A Gain/Loss
Jan 60 40 100 100 -
Feb 55 50 105 95 +5
Mar 60 50 110 115 -5
Apr 69 41 110 105 +5
May 60 44 104 100 +4
Jun 55 45 100 110 -10

26. (a) refer the above graph


Solution from 27 - 30: The graph can be represented in the following table:
Company Sales (1) Expend (2) Profit (3) = Equity (4) Pro/Equ Sal/Equ Sal/Exp Growth rate
(1) –(2) (3)/(4) (1)/(4) (1)/(2) sales
1990 80 76 4 8 0.5 10 1.05 -
1991 92 88 4 8 0.5 11.5 1.04 5%
1992 106 100 6 22 0.27 4.81 1.06 15.21%
1993 128 114 14 22 0.51 5.81 1.12 20.75%

27. (c)   28. (c)   29. (b)   30. (b) 33. (d) percentage increase in the solubility of


31. (c) Follow the table below potassium chlorate = (0.4 – 0.1) 100/0.1 = 300%.
Option Description Solubility 34. (d) Solubility of potassium chloride at 36°C =
(a) Potassium chlorate at 80° 0.4 0.4 kg./lt. so, in 100lir solution the amount of
(b) Potassium Chloride at 35° C 0.4
Potassium chloride dissolved in = 40 kg.
Number of moles = 40 / 0.075 = 533 approx.
(c) Potassium Nitrate at 39° C 0.48
540 moles can be dissolved in 100 lt. of water
(d) Sodium Chloride at 85° C 0.4 at 36°C.
35. (c) From the graph. It can clearly seen that
32. (c) At 30°C, solubility of potassium nitrate is
solubility of sodium chlorate is maximum.
0.38 kg./lt. Hence in 10 lt. 3.8 kg., Approx. =
36. (b) Profit in July = 37 – 34 = 3
4 kg. of potassium nitrate can be dissolved.
y
o
u
rs
m
a
h
b
o
o
b
.w
o
rd
22 Line Graph

p
re
37. (d) Difference between the Cost in March and 8. (d) Again as we did in the previous question.

s
s
May is the highest. Thus, in May the Cost is 4.5 − 2.5

.c
1995 – 96 = × 100 = 80%

o
the highest as compared to two months ago. 2.5

m
38. (d) The difference between the Cost in March and In 1996 – 97 = 33.33%, in 1997– 98 = 41.66%,
May is more or highest. On the same way, the in 1998 – 99 = 41.17%
Sales in March is less. So, percentage increase Hence it is maximum in 1995 – 96
in sales two months before the highest is in may 9. (b) Gap between profit and sales is largest with
39. (d) If we observe increase in the number of profit being above sales substantially as
employees from January to March is less than compared to other years. Hence the highest
the increase in profit of this period. Thus, we profitability was recorded in 97-98.
can easily say that profit per employee is the 10. (d) Profitability decreased as well as increased
highest in March. over the course of 5 years (can be observed
40. (b) From January to November the company from the graph as profit curve has surged
takes the number of employees upwards wrt sales graph in some years and
= (16 – 11) × 1000 = 5000. slumped below in some other years). Hence
the first 3 options are not valid.
CONCEPT DEVIATOR 2120 − 1700
11. (a) price change for arhar = × 100
1. (a) easily we can answer from the graph. 1700
2. (a) = 24.7%
3. (a) Lets add all the values of sales and costs and 19100 − 18500
Price change for pepper = × 100
then calculate the average we will get average, = 3.2% 18500
1825 and 1820 (after taking approximation) Similarly we will calculate for sugar and gold
4. (b) Lets follow the table and we will find that it is maximum for arhar
Year Assets Value Sales Value CSR spend 12. (c) This question requires you to do a bit more
2004 100 60 1.00 calculation.
2005 110 55 1.15 Let consider for Arhar, avg price = [beginning
2006 125 80 2.0 price + end price + highest price +lowest price]/4
2007 135 90 2.0 = [1700 + 2120+2300+1500]/4 = 1905
2008 150 120 2.50 Difference between highest and lowest price
2009 160 200 3.0
= 2300-1500 = 800
800
Hence price volitality of Arhar = = 0.421.
So, the increase in spending on CSR was 1900
maximum in 2006. Similarly we will calculate for others and we
5. (d) 6. (d) will observe that it is lowest for sugar.
7. (a) We will eliminate options one by one. 13. (d) As we did in question no 145 again we need to
(a) growth in 1995 – 96 find the price change for each of them, we did in
250 − 100 question number 145, For Arhar 25%, pepper
= × 100 = 150%
100 = 4%, sugar = –0.35 and that for gold = –7.7%
(b) growth in 1996 – 97 = here we don’t need Hence total profit percentage is the average of
to calculate as graph is almost horizontal of all these = [25 + 4 – 0.35 – 7.7]/4 = 5.1%
so percentage increase must be less than the approximate value is 3%
150%. 14. (b) As we did in question number 146 we calculated
(c) growth in 1997 – 98, again we don’t that lowest for Arhar and it is equal to 40%
need to calculate as in th is case there is a 15. (c) 40 widgets means 2 shifts in operation since in
decrease in the sales. one shift more than 30 is not possible, Fixed
(d) growth in 1998 – 99 cost (FC) = 800 + 1200 = ` 2000
680 − 280 400 Variable cost (VC) = 3500
= × 100 = × 100 < 150%
280 280 Total cost (TC) = 2000 + 3500 = 5500
Hence it is maximum in the year of 1995 – 96. Avg. Cost AC = 5500 / 40 = 137.5  140
y
o
u
rs
m
a
h
b
o
o
b
.w
o
rd
Line Graph 23

p
re
16. (b) At 40 units the Total cost (TC) = 2000 + 3500 21. (b) If we carefully notice at growth chart of

s
s
= 5500 seeta,1st to 2nd month growth rate is slightly

.c
o
Marginal cost (MC) = Change in TC/Change high but 2nd to 3rd month is maximum. After

m
in no of units that growth rate comes down in respect of 2nd
To find MC we will check at 2 places at to 3rd. So, decline rate start from 3rd month.
production units of 46. The total cost = 2000 22. (a) By observation we can get the required result.
+ 4300 = 6300. Among all infants Geeta is the only child ,
At production units of 51, TC = 2000 + 5050 whose length was minimum at initial but after
= 7050 2 months her length is maximum. Hence her
MC from 40 to 46 = (6300 – 5500)/6 =133 growth is maximum.
MC from 40 to 51 = (7050 – 5500)/12 = 129 23. (a) Same way by observation, change in height of
So we cant take the approximate value of MC Geeta between 2-3rd month is minimum. So,
= 130 growth must be least for Geeta.
17. (d) To find the trend we will find the MC in 24. (d) Growth after 5 months-(Final length – initial
different interval of production length)
MC between 1 to 6 units = (1100 – 850)/5 = 50 For, seeta = 60 – 50 = 10
MC between 16 to 21 units = (2300 – 1800)/5 = 100 Geeta = 64 – 49 = 15
MC between 26 to 30 units = ( 3150 – 2700)/4 Ram = 63 – 51 = 12
=112.5 Shyam = 62 – 53 = 9
MC between 30 to 31 units = (4500 – 3150)/1
Least growth = shyam = 9
= 1350
MC between 31 to 36 units = (500 – 4500) /5 = 100 CONCEPT ELIMINATOR
As can be seen the MC is an increasing
function for a period and then it is a decreasing Take away Dine In
function hence none of the above given
Total Indian Chinese Indian Chinese Total
statements is true.
18. (c) Profit = Profit per quantity × Quantity = (Price 4.3 3.01 1.29 Foodie 1.92 0.48 2.4
– Avg cost ) × quantity = (150 – 140)40 = 400 2.5 1.5 1 Lazeez 2.2 2.2 4.4
19. (a) We know that Profit = Price × quantity (Q) – 3.8 3.42 0.38 Dhaba 1.26 0.54 1.8
T.C = 150Q – T.C
Now we will check each of the options and find 4.5 3.6 0.9 Pizza 1.68 1.12 2.8
out which one is giving us maximum value. Hut
From Option (A) Q = 30, T.C = 800 + 2500 = 1.7 0.85 0.85 Sarson 3.15 0.35 3.5
3300. So Profit = 1200
1. (a) From the table no restaurant performed better
From Option (B) Q = 50, T.C = 2000 + 5000
than Dhaba in Take Away as in Take away
= 7000. Profit = 7500 – 7000 = 500
From Option (C) Q = 60, Sales = 150 × 60 = revenue of Indian food market Dhaba is
9000, TC = 2000 + 6500 = 8500. Profit = 500 the highest.
From Option (D) Q = 40 Sales = 150 × 40 = 2. (b) In indian Take away Lazeez satisfy the
6000 TC = 2000 + 3500 = 5500 Profit = 500. condition In Chinese Take away Lazeez,
We can make out that Option (a) gives us the and Pizza hut satisfy the condition
maximum profit So in Take away section performance of Lazeez
20. (d) Average cost for 1 unit = 850 is better than sarson but not better than Foodie
Average cost for 6 units = 1100/6 = 183 In indian Dine In, none of the restaurants
Average cost for 11 units = 1500/11 = 137 satisfy the condition
Average cost for 16 units = 1800/16 = 112 So in Dine In section none of the restaurants
Average cost for 21 units = 2200/21 < 100 satisfy the condition
We can observe AC is higher than MC till a So in total only one restaurants satisfy the
certain level and then decreases condition.
y
o
u
rs
m
a
h
b
o
o
b
.w
o
rd
24 Line Graph

p
re
3. (c) Revenue earned is as follows- Total number of educated and illiterate( not

s
s
(a) Foodie :- 3.01 + 1.92 = 4.93

.c
highly qualified) = total number of members

o
(b) Dhaba:- 3.42 + 1.26 = 4.68 educated and illiterate in 2014 - total number

m
(c) Pizza hut :- 3.6 + 1.68 = 5.28 of members educated and illiterate in 2013 =
(d) Sarson:- 0.85 + 3.15 = 4.0 21868 – 21403 = 465
So for Pizza hut it is the maximum Required % = 465/2682 × 100 = 17.33%
4. (c) In Take away section Foodie, Lazeez, and Solution from 11 - 15:-
Pizza hut performed better than Sarson.
As per the given line graph score of four groups are
In Dine In section Lazeez and Pizza hut
performed better than Dhaba as follows with highest and lowest score is marked is
So only Lazeez and Pizza hut satisfy both the marked with shade means that is fixed and can not be
conditions. changed.
5. (c) Consider the options one by one- So at stage 1 A can get either 70 or 65 and so on
(a) Foodie :- 1.29 – 0.48 = 0.81 A B C D
(b) Dhaba:- 0.54 – 0.38 = 0.16
(c) Lazeez:- 2.2 – 1 = 1.2 Stage 1 70/65 60 65/70 75
(d) Sarson:- 0.85 – 0.35 = 0.50 Stage 2 75/85 70 95 85/75
Solution from 6 - 10:- Stage 3 85/80 90 80/85 65
6. (d) Since, 2009 is the year when the club is Stage 4 80/90 65 95 90/80
inaugurated. So 2009 is rejected
Now, Check the % change Highest score achieved by A = 70 + 85 + 85 + 90 = 330
15702 − 12564 Highest score achieved by B = 60 + 70 + 90 + 65 = 285
% change in 2010 = × 100 Highest score achieved by C = 70 + 95 + 85 + 95 = 345
= 24.97% 12564
Highest score achieved by D = 75 + 85 + 65 + 90 = 315
On similar way we observe that 2011 witnessed
9.34%, 2012 witnessed 31.46%, 2013 So C could have achieved the highest score
witnessed 31.22% and 2014 witnessed 15.43%. 11. (d)
7. (a) Lets find the ratio first 12. (b) If A has achieved the highest score then score
of C is 65 + 95 + 80 + 95 = 335
2009 6123 : 6441
13. (b) From the table above score of only B is fixed
2010 7652 : 8050 14. (d) If C has achieved the lowest possible score
2011 9115 : 8054 then-
2012 10702:11870 A B C D
2013 14525 : 15094 Stage 1 70 60 65 75
2014 16416 : 17776 Stage 2 75/85 70 95 85/75
Stage 3 85 90 80 65
4
8. (c) Required % = × 100 = 0.27 Stage 4 80/90 65 95 90/80
1467
310/320/330 295/305/315
9. (d) 30% of 29619 = 8885.7 or 8886 persons are
old men 15. (d) For A maximum change could be from 65 to
As number of old men are same then in 2014, 85 from stage 1 to stage 2
(34192 – 8886) = 25306 are young men are For B maximum change is be from 60 to 70
there. from stage 1 to stage 2
25306 − 20732 For C maximum change could be from 65 to
Hence required % = × 100 95 from stage 1 to stage 2
25306
= 18.07% For D maximum change could be from 65 to
10. (d) Total number of men joined in 2014 are = 90 from stage 3 to stage 4
17776 – 15094 = 2682 So C could get maximum % Change.
y
o
u
rs
m
a
h
b
o
o
b
.w
o
rd
2

p
re
Data

s
s
.c
o
m
Table
Topic No. of Questions Level
Concept Applicator 54 Basic PO + MAT (basic level)
Concept Builder 75 Basic PO + MAT (advance level)
Concept Cracker 49 CAT + XAT + IIFT (basic level)
Concept Deviator 50 CAT + XAT + IIFT (advance level)
Concept Eliminator 12 Tough questions
Total 240

INTRODUCTION 1. What is the Minimum fare of the BUS running on


the route no 06?
DATA TABLE is one of the frequently ask questions.
(a) 15 (b) 10
Tables are one of the most versatile methods of systematic
presentation of quantitative data. It is very easier than (c) 7 (d) 5
other graphs to extract information from the tables. Sol. (d) As it is very clear from the table that the
Usually questions are asked on Single data type minimum bus fare is 5. Hence option (d) is
and multiple data type questions. the answer.

Example 1: The following table shows the bus fare from 2. If a person board himself in garia, for him how
different places of one way bus fare from one place to much percent fare increase from rashbihari
another. Answer the following questions from the table. to hazra?
BUS FARE chart of Route no 06 Sol. It is clearly understand that bus fare increases
Places Garia Tollygunj
Anwar Rash
Hazra
` 5 so it is 50 % change in the fare.
Shah Bihari
Garia - 7 10 10 15 5
i.e., × 100 = 50 %
Tollygunj 7 - 5 7 10 10
Anwar Shah 10 5 - 7 10

Rash Bihari 10 7 7 - 5
Hazra 15 10 10 5 -
y
o
u
rs
m
a
h
b
o
o
b
.w
o
rd
26 Data Table

p
re
Example 2: Study the following table carefully and answer the questions

s
s
.c
Marks obtained out of 100

o
m
Student
SOCIAL SCs SANSKRIT MATHS SCIENCE ENGLISH HINDI
P 45 53 100 96 50 45
Q 56 58 96 50 46 50
R 48 60 92 45 52 42
S 62 67 88 67 48 34
T 66 55 78 88 60 32
U 54 64 64 96 53 40

3. What is the average percentage marks obtained by 4. Which student has scored in maths closest to the
all the students in Science? average marks in maths?
(a) 99 (b) 73.66 (a) P (b) T
(c) 64.66 (d) None of these
Sol. (b) As total marks is 100, so total marks obtained (c) R (d) None of these
= ( 96 + 50 + 45 + 67 + 88 + 96) = 442 Sol. (d) Average marks obtained in maths
Average marks = 442/6 = 73.66 %
= ( 100 + 96 + 92 + 88 + 78 + 64)/6 = 86.33
73.66 Following the table clearly we can understand
Now average percentage = × 100
100 that S (88) is nearer to the Average marks.
= 73.66%
y
o
u
rs
m
a
h
b
o
o
b
.w
o
rd
p
re
s
s
.c
o
m
Direction (Qs. No. 1 – 5): Study the following table 2. In which year at Churchgate, the monthly rent
carefully to answer the questions that follow. Monthly rent increased more than 100 per cent from the previous
(in ` thousand) at five different places in six different years year ?
(a) 2006 (b) 2007
Place (c) 2008 (d) 2009
Years
Churchgate Dadar Kandivali Borivali Virar 3. What is the difference between the monthly rent
2005 5.3 3.8 1.5 2.7 1.1 at Dadar in the year 2009 and Borivali in the year
2007 ?
2006 12.5 8.3 3.4 4.8 2.1
(a) ` 7,600 (b) ` 7,900
2007 16.7 11.7 5.5 6.6 1.8 (c) ` 8,100 (d) ` 8,600
2008 20.9 13.6 9.8 12.7 3.6 4. Monthly rent at Kandivali in the year 2008 was
2009 25.8 14.5 11.5 14.1 5.5 approximately what per cent of the total monthly
rent at Virar over all the years together ?
2010 30.3 20.9 15.6 15.9 7.8
(a) 30 (b) 38
1. In which place did the monthly rent not increase (c) 42 (d) 45
consistently from year 2005 to 2010? 5. Which city was most expensive in terms of rent ?
(a) Churchgate (b) Dadar (a) Churchgate (b) Dadar
(c) Kandivali (d) Virar (c) Kandivali (d) Borivali
Direction (Qs. No. 6-10): Study the table carefully to answer the questions that follow : Number of girls and boys
(in hundreds) in six different years in five different schools.
School A B C D E
Years Boys Girls Boys Girls Boys Girls Boys Girls Boys Girls
2005 3.3 3.6 5.2 3.1 5.5 4.5 2.4 1.4 6.5 6.6
2006 6.6 4.2 4.9 2.2 6.9 3.3 4.4 2.3 5.5 3.6
2007 9.3 6.9 4.7 4.2 5.8 4.9 6.4 3.3 2.7 2.4
2008 5.4 9.6 6.3 5.4 6.6 5.2 5.3 5.4 5.4 5.7
2009 8.4 12.9 7.5 5.9 8.7 6.6 12.1 5.2 6.8 6.5
2010 12.3 14.4 9.8 4.4 11.7 4.2 12.2 9.4 10.8 12.7

6. What is the approximate percentage decrease in 7. The number of girls in School B in the year 2009
the number of boys in School D in the year 2008 as is approximately what per cent of the total number
compared to that in the previous year ? of students (both boys and girls) in School E in the
(a) 17 (b) 12 year 2006?
(a) 46 (b) 52
(c) 9 (d) 23
(c) 70 (d) 65
y
o
u
rs
m
a
h
b
o
o
b
.w
o
rd
28 Data Table

p
re
8. What is the average number of girls in School A in the semester fee charged for M Sc course in the

s
s
.c
all the years taken together ? year 2009 ?

o
(a) 760 (b) 800 (a) 67 (b) 84

m
(c) 860 (d) None of these (c) 80 (d) 72
9. What is the ratio of the number of boys in School 15. What was the total semester fee charged for all the
C in the year 2009 to the number of girls in School courses together in the year 2006 ?
A in the year 2009? (a) ` 42,500 (b) ` 41,500
(a) 29 : 41 (b) 36 : 11 (c) ` 41,600 (d) ` 42,200
(c) 29 : 43 (d) None of these Directions (Qs. No. 16-21): Following six questions
10. In which year is the total number of students are on the basis of the following table which gives data
(both girls and boys together) the third highest in yearwise and disciplinewise for candidates selected in
School E ? an industry (in thousands). [Mat 1998]
(a) 2006 (b) 2007
(c) 2008 (d) 2005 Year Discipline 1991 1992 1993 1994 1995 1996
Direction (Qs. No. 11-15): Study the following table
carefully to answer the questions that follow. Science 16 20 25 15 14 16
Semester fees (in ` thousand) for five different courses Arts 4 8 9 10 11 13
in six different years. Commerce 8 12 12 11 15 13
Law 7 9 5 3 6 8
Courses
Years Computer Science 10 14 18 20 25 30
B .Tech. M. Sc. B. Ed. M. Phil. Diploma
Others 1 1 2 1 1 1
2005 11.5 5.8 7.5 4.7 1.8
Total 46 64 71 60 72 81
2006 14.5 6.4 11.6 5.8 3.2
16. In which year for the first time, Computer Science
2007 20.0 10.2 13.9 8.6 4.8
discipline constituted more than 25% of the
2008 22.2 14.6 15.8 12.7 5.6 selected candidates?
2009 35.8 17.7 18.5 25.1 12.5 (a) 1991 (b) 1992
2010 50.7 20.9 22.6 18.9 14.9
(c) 1993 (d) 1994
17. Which discipline showed the greatest difference
11. What was the approximate per cent increase in the between 1992 and 1994 selections?
semester fees of B.Ed. course in the year 2007 as (a) Science & law
compared to the previous year? (b) Computer Science& law
(a) 26 (b) 30 (c) Arts & commerce
(c) 20 (d) 16 (d) Law & others
12. What was the average semester fee charged for 18. In which year the percentage of Science discipline
M. Sc. course over all the years together ? candidates was the highest?
(a) ` 12,700 (b) ` 12,600 (a) 1991 (b) 1992
(c) ` 12,060 (d) ` 12,070 (c) 1993 (d) 1994
13. What was the difference between the total semester 19. Which discipline has shown the greatest rate of
fee charged for Diploma course over all the years increase in its selection from 1994 to 1996?
together and the fee charged for B. Tech. course in (a) Arts (b) Computer Science
the year 2009 ? (c) Law (d) Commerce
(a) ` 8,500 (b) ` 8,000 20. For which discipline the total recruitment for all the
(c) ` 6,500 (d) None of these 6 years is closest to the total recruitment for any year?
14. The semester fee charged for M. Phil. course in the (a) Arts (b) Computer Science
year 2008 was approximately what per centage of (c) law (d) Commerce
y
o
u
rs
m
a
h
b
o
o
b
.w
o
rd
Data Table 29

p
re
21. For which discipline there is no change between 23. For which toy category there has been a continuous

s
s
.c
the candidates selected in 1991 and in 1996? increase in the production over the years?

o
(a) Commerce (b) Science (a) Ludo (b) Chess

m
(c) Law (d) Arts (c) Monopoly (d) Carrom
Directions (Qs. No. 22-27) : Following six questions 24. What is the percentage drop in the production of
are on the basis of the data given in the following table, Ludo from1 992 to 1994?
which gives the annual production (in thousands) of 5 (a) 30 (b) 50
products of a famous toy company. [Mat 1998] (c) 20 (d) 10
Year Ludo Scrabble Chess Monopoly Carrom
25. For which of the following toys, the total
production of all the 5 years is the maximum?
1992 200 150 78 90 65 (a) Ludo (b) Scrabble
1993 150 180 100 105 70 (c) Chess (d) Carrom
1994 180 175 92 110 85 26. The production of chess in 1993 was what percent
of the production of Carrom in 1993?
1995 195 160 120 125 75
(a) 142 (b) 70
1996 220 185 130 135 80 (c) 35 (d) 20
22. What is the approximate percentage increase in the 27. For which of the following toys the ratio of the
production of Monopoly from 1993 to 1995? production in 1996 to that in 1992 is the highest?
(a) 10 (b) 20 (a) Chess (b) Ludo
(c) 5 (d) 25 (c) Monopoly (d) Carrom
Directions (Qs. No.28-32): Answer these question based on the table given below. [Mat 2002]
The Hotel Company of India (HCI) owns seven Hotels with the same capacity. The occupancy rates across the
seven hotels are given in the following table.
Hotel Pleasant Dessert Black Lake Classic Radiant Plaza
Name Stay Palace Lagoon View
2001 65% 55% 70% 49% 71% 47% 59%
2000 43% 72% 76% 46% 64% 64% 63%
1999 63% 71% 65% 61% 58% 66% 65%
1998 72% 68% 60% 64% 61% 72% 49%
1997 81% 67% 64% 63% 59% 69% 45%

28. In which year did HCI witness the highest 30. Which of the following statement(s) is/are false ?
occupancy rate? (a) The average occupancy rate of Plaza was
(a) 2000 (b) 1999 greater than that of Lake View.
(c) 1997 (d) 1998 (b) The greatest average occupancy rate was
29. Which one of the following statements is true ? witnessed in Pleasant Stay.
(a) The lowest average occupancy rate was in (c) The average occupancy rate for Dessert
the year 2000. Palace is greater than that of Black Lagoon.
(b) The average occupancy rate in 1997 was
(d) All of the above statements are false.
greater than that in the year 1998.
(c) There is a gradual decrease in the average 31. In which year was the rate of growth in occupancy
occupancy rate over the years. rate the highest ?
(d) The highest average occupancy rate was (a) 1997 (b) 1998
witnessed in 1997. (c) 1999 (d) 2000
y
o
u
rs
m
a
h
b
o
o
b
.w
o
rd
30 Data Table

p
re
32. Every year HCI gives special awards to the 33. Which shop has the lowest sales of both type B and

s
s
.c
managers of those hotels that had achieved the best type E as compared to other shops ?

o
and the second best occupancy rates. Which of the (a) P (b) Q

m
hotels has won this award at least twice ? (c) R (d) T
(a) Pleasant Stay and Lake View 34. Which shop has a share of 15% sales of the total
(b) Dessert Palace and Classic type D sold by all the shops ?
(c) Black Lagoon and Radiant (a) P (b) Q
(d) Lake View and Plaza (c) R (d) T
Directions (Qs. No. 33-37): Study the following table, 35. Which shop has the highest sale of cars of all the
showing monthly sales of cars of five types by five types ?
automobile shops to answer these questions. [MAT 2002] (a) P (b) Q
(c) R (d) None of these
Automobile shops 36. Which shop sales cars of type B seven times to that
Type
P Q R S T of type E sold by it ?
(a) Only P (b) Only Q
A 1250 3500 1360 2240 210 (c) Both P and Q (d) Only T
B 2100 3080 3700 4200 920 37. Among all the shops the lowest sale of type A is the
highest sale of which of the following types ?
C 3460 4400 4860 4860 4760 (a) B
D 900 680 700 1120 600 (b) C
(c) D
E 300 440 1200 1250 280
(d) No such type exists
Directions (Qs. No. 38-41): Study the table given below to answer these question.
NHPC has undertaken massive afforestation which is an effective tool in arresting soil erosion and enrichment of
environment. The details are as under. [MAT 2002]

Forest area involved Area afforested No. of trees No. of trees


S. No. Name of the project
(in ha) (in ha) affected planted
1. Chamera I (HP) 982.50 2000 40,000 3981,186
2. Dulhasti (J & K) 1.1 18 700 7,85,673
3. Rangit (Sikkim) 34.60 38 5000 3,31,000
4. Tanakpur 293.35 350.00 17,368 6,66,165
5. Uri (J & K) 54.71 62.70 4,000 3,21,000
6. Dhauliganga-I (Uttarakhand) 138.60 140.73 1,571 2,87,887
7. Chamera-II (HP) 78.78 172.58 1,380 2,30,000
Total 69,965 6,60,3911

38. The maximum number of trees has been affected (a) Dulhasti (J & K) (b) Uri (J & K)
by the project : (c) Rangit (Sikkim) (d) Chamera I HP
(a) Chamera-II (HP) (b) Tanakpur 40. Assuring that the trees have been planted on more
or less even distribution the density of the trees
(c) Uri (J & K) (d) Chamera I (HP)
planted has been maximum in the case of :
39. Out of the given projects, the minimum forest area (a) Chamera-I (HP) (b) Dulhasti (J & K)
involved has been in respect of : (c) Rangit (Sikkim) (d) None of these
y
o
u
rs
m
a
h
b
o
o
b
.w
o
rd
Data Table 31

p
re
41. Out of the given projects maximum forest area 42. In 1979, the maximum reduction in per capita

s
s
.c
involved is in the State of : consumption of cement took place in :

o
(a) Himachal Pradesh

m
(a) USSR (b) Italy
(b) Jammu and Kashmir
(c) Sikkim (c) Japan (d) India
(d) Uttarakhand 43. In 1979, USSR produced more cement than the
Directions (Qs. No. 42-45): Refer the table given combined total of four other countries excluding :
below to answer these questions. [MAT 2002]
(a) India (b) Japan
Production and Consumption of Cement
(c) Italy (d) France
Production Per Capita 44. The adverse effect of decline in the consumption
Country
(in million tonnes) Consumption (in kg) of cement in 1979 in comparison to 1978 is likely
1978 1979 1978 1979 to be more in:
Japan 84.89 87.80 689 631 (a) USSR (b) Italy
Italy 38.32 39.72 656 582 (c) France (d) India
USSR 129.28 123.01 483 388 45. In 1978 … had 15 times more per capita cement
West Germany 33.50 35.47 520 482 consumption than that in India
France 29.06 28.89 506 447 (a) France (b) West Germany
India 19.56 18.26 32 25 (c) USSR (d) Japan

Directions (Qs. No. 46-50) : Refer to the table given below to answer these questions.
Participation in Elections (Persons in Millions)
1984 1988 1992
Characteristics Persons of Percent voted Persons of Percent voted Persons of Percent voted
voting age voting age voting age
Total 330 69 348 68 408 63
Male 156 72 162 70 192 64
Female 174 67 186 66 216 62
Urban 96 57 102 60 126 55
Rural 234 75 246 71 282 66
Age 18 -24 30 51 36 50 75 50
25 - 44 135 69 138 67 147 63
45 - 64 114 76 120 75 126 71
65 years above 51 66 54 66 60 63

46. Which of the following groups had the highest 47. In 1992 approximately what per cent of persons of
percentage of voters in 1988? voting age were females ?
(a) Male (b) 25 – 44 years (a) 42 (b) 53
(c) Rural (d) 65 years and above (c) 60 (d) 64
y
o
u
rs
m
a
h
b
o
o
b
.w
o
rd
32 Data Table

p
re
48. In 1988 how many males of voting age voted ? 51. Compared to the food demand in the year 2000 the

s
s
.c
(a) 113,400,000 (b) 114,400,000 demand in the year 2020 is expected to be over 4

o
(c) 123,400,000 (d) 134,000,000 times in respect of :

m
49. How many persons of 65 years and above did not (a) Fruits (b) Meat,Fish and Eggs
vote in 1992 ? (c) Milk (d) Edible oils
(a) 37,000,000 (b) 37,800,000 52. The table shows projected food demand at 7
(c) 23,000,000 (d) 22,200,000 per cent income Growth. If for some reason, the
50. If X be the number of persons (in million) of Income Growth is less than 7 per cent, then which
voting age 25 – 44 living in rural areas in 1984, of the following can result in the year 2020 ?
then which of the following includes all possible
(a) The projected demand will more or less,
values and only possible values of X ?
remain the same.
(a) 0 ≤ X ≤ 145 (b) 39 ≤ X ≤ 135
(c) 39 ≤ X ≤ 234 (d) 135 ≤ X ≤ 234 (b) The demand will decrease.
Directions (Qs. No. 51-55): Refer the following table (c) The demand may increase due to increased
to answer these questions. [MAT 2003] population
Projected Household Demand for Food in India at 7% (d) Cannot say
Income Growth 53. In the three decades from 1990 to 2020 the demand
increase in three times in the case of :
Annual household demand
(million metric tones) (a) Edible oils (b) Vegetables
Commodity
1990 2000 2010 2020 (c) Milk (d) Sugar
Food grains 168.3 208.6 266.4 343.0 54. Which of the following groups of commodities
Milk 48.8 83.8 153.1 271.0 show a closer similar increased demand for the
Edible oils 4.3 6.3 9.4 13.0 decade 2000 – 2010 ?
Vegetables 56.0 80.0 117.2 168.0 (a) Food grains and Sugar
Fruits 12.5 22.2 42.9 81.0 (b) Milk and Edible Oil
Meat, Fish and Eggs 3.4 6.2 12.7 27.0 (c) Edible Oil and Vegetables
Sugar 9.6 12.8 17.3 22.0 (d) Vegetables and Fruits

Directions (Qs. No. 1-3): Use the data in the table 1. What will be the cost of purchasing x grams of
given below to answer these questions. [MAT 2003] food A, y grams of food B and z grams of food C ?
% of % of % of Cost (a) ` (0.32x + 1.82y + 2.57z )
protein carbohydrate fat per 100g
(b) ` (1.8x + 0.3z + 2.75y)
Food A 10 20 30 ` 1.80
0.9 0.3 0.11 
(c) `  x+ y+ z
Food B 20 15 10 ` 3.00  50 10 4 
Food C 20 10 40 ` 2.75 (d) ` (x + y + z)
y
o
u
rs
m
a
h
b
o
o
b
.w
o
rd
Data Table 33

p
re
2. Which of the following diets would supply the

s
40-49 50-59 60-69 70-79 80-89 90-99

s
.c
most grams of protein ?
50 - 59 3 3 6 1

o
(a) 500 g of A

m
(b) 250 g of B 40 - 49 2 7 7
(c) 350 g of C 4. How many students received grades 80 and above
(d) 200 g of B and 200 g of C in Mathematics ?
3. All the following diets would supply at least 75 g (a) 20 (b) 30
of fat. Which of the diets costs the least ? (c) 23 (d) 25
(a) 300 g of A 5. How many students would qualify for admission to
(b) 200 g of C a prime Engineering College that stipulates above
(c) 150 g of A and 100 g of C 80% in Mathematics and Physics ?
(d) 500 g of B and 100 g of A (a) 9 (b) 12
Directions (Qs. No. 4-7): The following table gives (c) 16 (d) 18
the frequency distribution of the final grades of 100 6. The School Trust provides scholarships for higher
students in Mathematics and Physics. Analyse the data studies to students to students who secure 90% and
presented to answer these questions [MAT 2003] above in Mathematics and Physics. How many
Mathematic Grade (Row wise) and Physics Grade students are eligible for scholarships for higher
(Colum wise ) studies?
40-49 50-59 60-69 70-79 80-89 90-99 (a) 5 (b) 3
(c) 7 (d) 4
90 - 99 4 2 5
7. What percentage of students got less than 70% in
80 - 89 2 3 7 2
both Mathematics and Physics ?
70 - 79 1 7 6 3 (a) 34 (b) 43
60 - 69 2 5 9 8 5 (c) 39 (d) 44
Directions (Qs. No. 8-12): The following table gives the quarterly output of three production units of machine
manufacturing company. [MAT 2003]
Year 1st Quarter 2nd Quarter 3rd Quarter 4th Quarter
Unit 1 2 3 1 2 3 1 2 3 1 2 3
1998 30 34 38 40 44 42 36 37 39 34 35 37
1999 34 45 42 52 54 56 50 49 48 44 45 48
2000 35 45 43 58 53 58 54 56 56 48 49 43
2001 54 58 57 76 77 79 68 69 70 62 64 66
2002 80 82 87 92 97 93 86 88 87 82 84 83
8. Which of the following statement is correct ? 9. Incentives based on the production are given on
(i) The performance of Unit 1 has been the quarterly basis. In which quarter would the workers
lowest during the 5 years period get maximum incentive ?
(ii) The performance of Unit 2 has been the
highest during the 5 years period (a) 1st Quarter (b) 2nd Quarter
rd
(c) 3 Quarter (d) 4th Quarter
(iii) The performance of Unit 3 has been the
highest during the 5 years period 10. Which unit has shown the least growth in
(iv) The performance of Unit 2 has been the production during the period 1998 – 2002 ?
lowest during the 5 years period
(a) (i) (b) (iii) and (iv) (a) Unit 1 (b) Unit 2
(c) (i) and (iii) (d) None of these (c) Unit 3 (d) Cannot say
y
o
u
rs
m
a
h
b
o
o
b
.w
o
rd
34 Data Table

p
re
11. During 1998-2002 the highest quarterly production Toshiba TVs, then what percentage of the total

s
s
.c
has been achieved by Unit … in the …… quarter sales in 2000 is for Toshiba TVs ?

o
of 2002 ? (a) 17 % (b) 7.8 %

m
(a) 1, 1st (b) 2, 2nd (c) 9.3 % (d) 6.3 %
rd
(c) 3, 3 (d) 2, 4th 17. The brand which showed a decrease of 50 % during
12. The highest increase in the yearly average has been the given five-year period registered maximum
over the period : percentage decrease during the period ?
(a) 1998-1999 (b) 2001-2002 (a) 1995 - 96 (b) 1996 - 97
(c) 1999-2000 (d) 2000-2001 (c) 1997 - 98 (d) 1998 - 99
Directions (Qs. No. 13-17): Study the data given below Directions (Qs. No. 18-23): Loan disbursed by five banks
to answer the following questions: [MAT 2003] (in ` crore) are given in the table below: [MAT 2003]
No. of Units of TVs Sold Year
Banks
1990 1991 1992 1993 1994
1995 1996 1997 1998 1999
A 18 23 45 30 70
LG 30,000 38,000 36,000 42,000 40,000 B 27 33 18 41 37
C 29 29 22 17 11
Samsung 17,000 28,000 33,000 32,000 27,000
D 31 16 28 32 43
Sony 12,500 20,000 35,000 40,000 50,000 E 13 19 27 34 42
National 18. In which year was the distribution of loans of
30,000 25,000 22,000 20,000 15,000
Panasonic
all the banks put together least compared to the
Toshiba 15,725 18,625 13,275 14,375 16,000 average disbursement of loans over the years ?
(a) 1990 (b) 1991
13. The average annual sale of which brand is the highest ? (c) 1992 (d) 1994
(a) LG (b) Sony 19. What was the percentage increase of disbursement
(c) National Panasonic (d) Toshiba of loans of all the banks together from 1992 to 1993 ?
14. Which of the following statements is/are true ? (a) 110% (b) 14%
I. LG is showing an increase in sales every year. (c) 10% (d) 11%
II. Samsung has recorded a fall in sales thrice 20. In which year was the disbursement of loans
during the given five years period. of banks A and B exactly equal to the total
III. The percentage increase in the number of disbursement of loans of banks D and E?
units sold from 1995 to 1999 is the highest (a) 1991 (b) 1992
for Sony. (c) 1994 (d) None of these
IV. The average annual sale of Samsung is more 21. If the minimum target in the succeeding year was
than that of Sony. 20% of the total disbursement of loans, then how
(a) III alone (b) I and II many banks reached the target in 1991 ?
(c) III and IV (d) III and II (a) 1 (b) 2
15. The installed capacity of each company is 75,000 (c) 3 (d) 4
units and all the units produced by each company 22. In which banks was the loan disbursement more
are sold. Then the least and the highest values for than 25% of the disbursement of all the banks in
1994?
annual capacity utilization for any company are
(a) A (b) B
respectively:
(c) C (d) D
(a) 16.67% and 66.67% (b) 17.5% and 72.5%
23. By what per cent did the loan disbursement
(c) 14.28% and 75.5% (d) 5.24% and 95.34% increase from 1990 to 1994 ?
16. If for 2000 there is a 25% increase in the total sale (a) 72 (b) 82
of TVs and there is a 10% decrease in the sale of (c) 94 (d) 91
y
o
u
rs
m
a
h
b
o
o
b
.w
o
rd
Data Table 35

p
re
Directions (Qs. No. 24-27) : Answer these 28. How much more money would be paid on a loan of

s
s
.c
questions based on the data given in the table ` 20,00,000 taken out over 20 years compared to

o
below. The table shows the trends in the relative the same loan taken over a period of 15 years ?

m
value in the market of select groups of commodities (a) `3,00,000 (b) ` 4,25,000
(1999-2003). [MAT 2003] (c) `5,50,000 (d) ` 6,12,000
Years 29. What is the total amount repaid over 25 years on a
Commodities 1999 2000 2001 2002 2003 loan of `15,00,000 ?
Milk 95 92 86 72 76
(a) ` 22,50,000 (b) ` 37,95,000
(c) ` 45,30,000 (d) ` 55,70,000
Cereals 75 68 62 66 60
30. The monthly repayment on a loan of `15,00,000
Fats and Oils 76 70 68 62 58
over 20 years is reduced to `12500. By how much
Gas 82 76 100 98 96 would this reduce the total amount on the loan over
Vegetables 80 62 64 84 88 the full period ?
Fruits 79 74 72 66 73 (a) `1,30,000 (b) `2,40,000
24. What is the average difference in the relative value (c) `2,24,000 (d) `1,26,000
of the six commodities in 2003 compared to 1999 ? 31. Instead of taking a loan of ` 10,00,000 with a
(a) +8 (b) –5 repayment period of 15 years, the society proposes
(c) –6 (d) +4 to take a loan of `15,00,000 to be paid back in 10
25. Which value showed the greatest amount of change years to provide for a generator set. What is the
in 1999 compared to 2003 ? cumulative financial impact ?
(a) Milk (b) Gas (a) ` 4,54,000
(c) Fats and Oils (d) Vegetables (b) ` 4,74,000
26. Which commodity showed the least variation in (c) ` 5,67,000
value over the period 1999-2003 ? (d) Cannot be assessed
(a) Cereals (b) Gas Directions (Qs. No. 32-35) : These questions are based
(c) Vegetables (d) Fruits on the following table. The table shows the number of
27. For which commodities is there a clearly discernible emergencies attended by 6 fire brigade substations
trend of decreasing relative value between 1999- during May October 2002: [MAT 2003]
2003? Sub-Station May June July Aug. Sep. Oct.
(a) Cereals and Gas A 12 15 17 21 13 17
(b) Milks, Cereals, Fats and Oil B 18 21 15 18 18 19
(c) Milk, Gas Fats and Oils C 10 11 19 21 23 18
(d) None of the above D 17 17 19 12 18 10
Directions (Qs. No. 28 - 31): Answer these questions E 12 15 18 10 21 11
based on the data given in the following table. The table F 14 15 12 13 18 19
shows Instalment amounts for monthly repayments ( in
Rupees) on housing society loans for different period. 32. Number of emergencies attended by the 6 substa-
[MAT 2003] tions was the same in the months of :
(a) May and June
Years
(b) June and July
Loan 10 15 20 25 (c) August and September
1,00,000 1250 1050 900 850 (d) June and October
2,00,000 2500 2050 1800 1600 33. Which of the following substations showed a
10,00,000 12950 10300 9000 8450 greater increase in the number of emergencies
attended in August as Compared to July ?
15,00,000 19400 15450 13500 12650
(a) A (b) E
20,00,000 25900 20600 18000 16800 (c) D (d) C
y
o
u
rs
m
a
h
b
o
o
b
.w
o
rd
36 Data Table

p
re
34. Which substation attended to the maximum (b) The above statement is not supported by the

s
s
.c
number of complaints in the given period data given in the table

o
(a) A (b) B (c) Additional information is required to arrive

m
(c) C (d) F at the above conclusion
35. Which two months aggregated over 36% of the total (d) None of the above
number of emergencies in the six-month period ? Directions (Qs. No. 39-41) : Answer these questions
(a) May and June based on the table given below. The table shows
(b) July and October number of new female and male employees engaged by
(c) August and September 5 employers from 1999 to 2003. [MAT 2003]
(d) July and September Gender of
Employer 1999 2000 2001 2002 2003 Total
Directions (Qs. No. 36-38): The table below gives the employees
details of foreign tourist arrivals and foreign exchange Female 4 4 5 10 12 35
A
earnings during the period 1995-1996 to 2001-2002. male 5 6 8 12 12 43
Answer these question based on the data given in the B
Female 10 11 9 13 15 58
following. male 12 12 13 23 14 74
Female 67 66 74 57 89 353
Foreign Tourist Arrivals and Foreign Exchange Earnings C
male 13 11 10 6 9 49
Estimated Female 4 6 8 2 9 29
Foreign Per Per D
foreign male 3 5 8 6 4 26
Tourists cent cent
Year exchange Female 4 5 4 3 2 18
Arrivals change change E
(in million male 4 5 2 6 3 20
(in lakh)
US$) Total 126 131 141 138 169 705
1995 – 96 2190 – 2713 – 39. What was the total number of employees (female
1996 – 97 2334 6.6 2878 6.1 and male) in all the companies in 1999 and 2000 ?
1997 – 98 2371 1.6 2914 1.3 (a) 234 (b) 257
(c) 235 (d) 256
1998 – 99 2397 1.1 2993 2.7
40. What is the average number of new female
1999 – 00 2505 4.5 3036 1.4 employees per company in 2001 ?
2000 – 01 2699 7.7 3168 4.3 (a) 25 (b) 30
2001 – 02 2423 –10.2 2910 –8.1 (c) 20 (d) 18
41. What was the ratio of the new female employees to
36. The maximum percentage increase in foreign new male employees in Company C in 2000 ?
tourist arrivals during the given period has been in: (a) 1 : 6 (b) 6 : 1
(a) 2001-2002 (b) 2000-2001 (c) 2 : 3 (d) 3 : 2
(c) 1999-2000 (d) 1996-1997 Directions (Qs. No. 42-45): Answer these questions
37. The estimated foreign exchange earnings has been based on the following table which gives the circulation
steadily increasing from the period : in thousands of five English dailies in the four States
(a) 1995-96 to 2001-02 during 2002-2003. [MAT 2004]
(b) 1995-96 to 2000-01
News Papers Kerala Punjab UP HP
(c) 1999-00 to 2001-02
(d) None of the above A 123 227 96 78
38. “ As a result of September 11, 2001 incidents in the B 105 220 117.2 97
United States the tourist arrivals dropped by about C 12.2 14.6 9.7 17.2
10 percent when compared, with the previous year” ? D 82.4 44 145 9.3
(a) The data given in the table supports the above
E 24.4 23 10 100
statement
y
o
u
rs
m
a
h
b
o
o
b
.w
o
rd
Data Table 37

p
re
42. Of the five dailies, which has the highest number 46. Following the above table in which category will

s
s
.c
of circulation? you classify the given data for IQ scores?

o
(a) A (b) B Data: 20, 47, 24, 51, 55, 42, 38, 61, 65

m
(c) D (d) E (a) Superior (b) Average
43. What is the difference in the circulation among the (c) Mentally defective (d) No given category
top two newspapers? 47. In which category will you classify the given data
(a) 14,200 (b) 15,200 for IQ scores?
(c) 13,200 (d) 12,200 Data: 140, 127, 135, 120, 125, 131, 139, 144, 138
44. The newspaper A’s circulation in Punjab is x times (a) Superior and very superior
that of the newspaper B’s circulation in HP. What (b) Bright and superior
is x? (c) Average and dull
(a) 2.35 (b) 2 (d) No given category
(c) 2.75 (d) 2.25 48. According to the given table in which category
will you classify the given data for I.Q scores?
45. The ratio of the circulation of newspaper D in
Data : 99, 100, 101, 105, 95, 92, 90, 108, 98, 103, 93
Punjab and HP is:
(a) Very superior (b) Superior
(a) 5.5 :2 (b) 5 :2
(c) Average (d) Border line
(c) 6 : 3.41 (d) 5.5 : 1.16
49. According to the given table which of the following
Directions (Qs. No. 46-50): Study the given table to is the category with the highest percentage of cases
answer these questions: [MAT 2004] included in it?
(a) Very superior (b) Bright normal
Table of Intelligence Classification
(c) Average (d) Border line
I.Q. Classification % Included 50. What is the similarity between Very Superior
130 and above Very Superior 2.2 and Mentally defective categories in terms of
120 -129 Superior 6.7 percentage of cases included in each?
(a) Both are different categories for intelligence
110 - 119 Bright normal 16.1
classifications
90 - 109 Average 50.0 (b) Both are extreme categories for differentiating
80 - 89 Dull normal 16.1 maximally between the people
70 - 79 Border line 6.7 (c) No similarity at all
(d) Smallest percentage of cases belong to both
69 and below Mentally Defective 2.2
of these categories
Directions (Qs. No. 51-53): Study the given table to answer these questions. [MAT 2004]
1989 1991
Industry Bank Overdues Bank Overdues
No. of Sick Units No. of Sick Units
( ` In crore) ( ` In crore)
19474 416 24941 476.92
Engineering
(10.4) (108.6) (11.3) (17.1)
4123 97.42 6057 134.16
Cotton Textiles
(2.2) (4.3) (2.7) (4.8)
15308 151.12 16149 171.78
Electricity
(8.2) (6.7) (7.3) (6.2)
203 12.75 287 6.18
Sugarcane
(0.1) (0.6) (0.1) (0.2)
y
o
u
rs
m
a
h
b
o
o
b
.w
o
rd
38 Data Table

p
re
s
1989 1991

s
.c
Industry Bank Overdues Bank Overdues

o
No. of Sick Units No. of Sick Units

m
( ` In crore) ( ` In crore)
1881 46.02 2634 36.51
Paper
(1.0) (2.1) (1.2) (1.3)
953 36.50 2063 39.63
Copper
(0.5) (1.6) (0.9) (1.4)
405 13.61 737 20.35
CR Steel
(0.2) (0.6) (0.3) (0.7)
2356 110.49 2952 129.68
HR Steel
(1.3) (4.9) (1.3) (4.6)
371 14.27 384 14.06
Cement
(0.2) (0.6) (0.2) (0.5)
6751 206.45 8208 262.15
Petroleum
(3.6) (9.2) (3.7) (10.1)
134616 1138.00 157075 1080.61
Miscellaneous
(72.3) (50.8) (71.0) (53.1)
186441 2242.63 221487 2772.03
Total
(100.0) (100.0) (100.0) (100.0)

51. Which of the following statements is/are true ? News papers Kerela Punjab UP HP
(a) There has been a net increase in the number A 123 227 96 78
of sick units between 1989 to 1991?
B 105 220 117.2 97
(b) While the number of sick units under the
C 12.2 14.6 9.7 17.2
miscellaneous category has gone up, the
bank overdues such sick units a as percentage D 82.4 44 145 9.3
of total bank overdues have gone down. E 24.4 23 10 100
(c) Cement companies have been doing worse 54. Of the five dailies, which has the highest number
during the period. of circulation?
(d) Both (a) and (b) (a) A (b) B
52. Which of the following industries have shown an (c) C (d) E
increase in the number of sick units from 1989 to 1991? 55. What is the difference in the circulation among the
I. Cotton Textiles II. Petroleum top two newspapers ?
III. Paper (a) 14,200 (b) 15,200
(a) I only (b) II only (c) 13,200 (d) 12,200
(c) I and II only (d) I, II and III only 56. The newspaper A’s circulation in Punjab is x times
53. Which sector has shown the maximum percentage that of the newspaper B’s circulation in HP. What
increase in the number of sick units? is x ?
(a) Electricity (b) Engineering (a) 2.35 (b) 2
(c) Cotton Textiles (d) None of these (c) 2.75 (d) 2.25
Directions (Qs. No. 54-57): Answer these questions 57. The ratio of the circulation of newspaper D in
based on the following table which gives the circulation Punjab and HP is :
in thousands of five English dailies in the four States (a) 5.5 : 2 (b) 5 : 2
during 2002-2003. [MAT 2004] (c) 6 : 3.41 (d) 5.5 : 1.16
y
o
u
rs
m
a
h
b
o
o
b
.w
o
rd
Data Table 39

p
re
Directions (Qs. No. 58-59): Study and analyse the table 60. Which is the fuel whose proportion in the total

s
s
.c
given below to answer these questions. [MAT 2005] energy demand will increase continuously over the

o
Productions, Export and Consumption period 2005-2020 in Asia?

m
of Coffee (Qty. in lakh tonnes) (a) Natural gas
Produc- (b) Both Natural gas and Hydropower
Exports Domestic
tion (c) Hydropower
Year US $ Consumption
Qty. Qty. ` Crore Qty (d) None of the above
Million
61. Which is the fuel whose proportion in the total
1990-91 1.70 1.00 279 108 0.54 energy demand will remain unaltered from 2005 to
1996-97 2.05 1.81 1467 452 0.55 2010 in Asia?
(a) Petroleum (b) Solid fuels
1997-98 2.28 1.79 1708 477 0.50
(c) Natural Gas (d) Nuclear
1998-99 2.65 2.12 1752 431 0.50 62. For which source of energy is the demand in 2020
as a ratio of demand in 2005 in the Asian region the
1999-2000 2.92 2.45 1901 447 0.55
greatest?
2000-2001 3.01 2.47 1374 334 0.58 (a) Natural Gas (b) Nuclear
2001-2002 3.06 2.13 1050 246 0.60 (c) Solid fuels (d) Hydropower
63. Which is the fuel for which demand in the rest
58. The highest percentage of domestic consumption of the World (excluding Asia) as a proportion of
with reference to the production of coffee has been total energy demand of the world (including Asia)
in the year…. During the given period: shows continuous decrease over the period?
(a) 2000-01 (b) 2001-02 (a) Solid fuels and Natural Gas
(c) 1996-97 (d) 1990-91
(b) Hydropower and Petroleum
59. During the given period though the largest quantity
(c) Solid fuels and hydropower
of coffee exported has been …….. lakh tones, the
foreign exchange earned in dollars has been the (d) None of the above
highest in the year 64. Over 2005-2020, which two fuels meet more than
(a) 2.47, 1997-98 (b) 3.01, 1999-2000 60 percent of the total energy demand of the World
(c) 3.06, 1997-98 (d) None of these and Asia both?
Directions (Qs. No. 60-64): The forecasts of the World (a) Nuclear and Hydropower
and Asian energy demand for the years 2005, 2010 and (b) Nuclear and Solid fuels
2020 are given in the table. The demand is given in (c) Hydropower and Solid fuels
million barrels per day crude oil equivalent. (d) None of the above
[MAT 2006] Directions (Qs. No. 65-67): Study the following table
2005 2010 2020 to answer these questions. [MAT 2006]
Availability and demand for various categories of
World Asia World Asia World Asia
steel in Indian railways (in ‘000 tonnes)
Petroleum 56.0 8.0 86.0 11.5 108.0 18.0
1999-2000 2003-2004
Natural Gas 38.0 1.5 67.0 2.5 96.0 4.5 S.No Category
Demand Availability Demand Availability
Solid fuels 46.0 6.0 54.0 13.0 67.0 15.4
1. Shapes 6960 5725 9745 9368
Nuclear 8.0 2.0 9.0 4.8 12.0 5.5
2. Flats 4360 5020 6300 6600
Hydropower 12.0 1.5 14.0 2.2 17.0 4.6
Railway
Total 160.0 19.0 230.0 34.0 300.0 48.0 3. 400 550 450 560
material
y
o
u
rs
m
a
h
b
o
o
b
.w
o
rd
40 Data Table

p
re
65. If the demand for each category of steel is to be 70. For China, assuming a linear growth in LMVs

s
s
.c
met in 2003-04,then the additional quantity of steel population, extrapolate nearly, when will the growth

o
that is to be produced is in population be 108% beyond the year 2030?

m
I. 110 thousand tones of railway material (a) 2048 (b) 2050
II. 300 thousand tones of flats (c) 2032 (d) 2038
III. 385 thousand tones of shapes 71. The percentage growth of the projected LMVs
(a) I only (b) III only population between 1975 and 2030 among the last
(c) II only (d) Both II and III five countries is maximum in
66. The expected percentage growth in the demand (a) Italy (b) Switzerland
for railway material over the five-year period from (c) Canada (d) UK
1999-2000 to 2003-2004 is Directions (Qs. No. 72-75): Study the following table
(a) 11 (b) 1/8 to answer these question. [MAT 2006]
(c) 37.5 (d) 12.5 Allotment of shares By a Multinational Company
67. The percentage change in the shortfall of shapes
No. of Ratio of
over the five-year period from 1999-2000 to 2003- No. of shares
Shares Allottees to
No. of
04 expected to be Applied for Allottees
Allotted Applicants
(a) + 40 (b) + 221
(c) – 68 (d) – 221 100 100 1:50 8001
Directions (Qs. No. 68-71): Study the following table 200-500 100 2:41 7624
to answer these questions. [MAT 2006]
600-900 200 1:15 6202
Projected Population of Light Motor Vehicles
(In Millions) 1000-3000 200 3:28 1515
S.No Country 1975 2030 3100-10000 200 1:6 1633
1 United States 141 382 10200-21000 300 2:5 404
2 Japan 120 238 25000 350 1:1 11
3 France 67 164
72. Find the total number of applicants who had
4 China 63 117
applied for 3100-25000 shares.
5 Italy 18 61 (a) 2048 (b) 10819
6 Germany 21 58 (c) 445 (d) 7562
7 UK 15 47 73. Find the average number of shares allotted to an
8 Canada 5 17 allottee?
9 Switzerland 15 3 (a) 100 (b) 150
(c) 140 (d) 200
68. The average population of LMVs of the middle 74. Find the ratio between the number of applicants
three countries in 1975 bears to the average who applied for 1000-3000 shares and those for
population of LMVs of the last three countries a 10200-21000 shares.
ratio of nearly
(a) 56:15 (b) 15:56
(a) 19:4 (b) 11:3
(c) 70:3 (d) 14:1
(c) 7:2 (d) 5:1
69. The percentage growth of the average population 75. If the face value of a share is ` 100 and the company
of LMVs for the last three countries between the wanted a subscription of 1 lakh rupees, then how
years 1975 and 2030 is approximately much was it oversubscribed?
(a) 71 (b) 212 (a) ` 45,000 (b) ` 4,500
(c) 172 (d) 221 (c) ` 15,000 (d) ` 10,000
y
o
u
rs
m
a
h
b
o
o
b
.w
o
rd
Data Table 41

p
re
s
s
.c
o
m
Directions (Qs. No. 1-5): Answer the following questions on the state of affairs of Indian economy using the data given in the
following table: [XAT 2006]
Description Units Scale 2002 2003 2004 2005 2006
Current account balance US dollars Billions 7.061 6.853 – 0.837 – 13.524 – 16.408
Current account balance in percent of GDP Ratio 1.4 1.2 – 0.1 – 1.8 –2
Gross domestic product based on
purchasing-power-parity (PPP) per capita US dollars Units 2682.165 2877.698 3079.727 3315.702 3547.148
GDP
Gross domestic product based on
purchasing-power-parity (PPP) share of Percent 5.65 5.833 5.913 6.049 6.179
world total
Gross domestic product based on
purchasing-power-parity (PPP) valuation of US dollars Billions 2769.335 3023.022 3290.8 3602.894 3918.637
country GDP
National currency
Implied PPP conversion rate 8.691 8.87 9.155 9.276 9.447
per US dollar
Price Index Index, 2000 - 100 108.239 112.358 116.591 121.105 127.272
1. Which year India witnessed highest annual growth 5. In which year Indian GDP (at current prices) in
in GDP based on FPP valuation? national currency v/s approximately equal to `
(a) 2003 (b) 2004 30,128 billion?
(c) 2005 (d) 2006 (a) 2003 (b) 2004
2. From the year 2003 to 2006, highest annual rate of (c) 2005 (d) None of the above
inflation is equal to Questions (Qs. No. 6-9): Read the data below and
(a) 6 (b) 5 choose the correct option for the questions that follow.
(c) 4 (d) 3 [XAT 2007]
3. From year 2003 to 2006, growth rate of population Queen Airlines offers the following Privilege
in India was highest in programme: There are 5 membership tiers, each with
(a) 2005
its own set of enhanced tier-specific benefits. New
(b) 2004
(c) 2003 members join at the Blue tier level, then upgrade to
(d) Cannot be determined the Blue Plus, with the added benefits of Tele check-in.
4. Which year witnessed highest annual changed in The programme also has three elite tiers –Silver, Gold
world’s total GDP (based on PPP)? and Platinum. The 6-month fast-track upgrade and the
(a) 2006 12-month standard upgrade system operate in parallel
(b) 2005 – implying that whenever a particular passenger
(c) 2004 satisfies conditions, either in term of number of flights
(d) Cannot be determined in the stipulated period or in terms of number of flights
y
o
u
rs
m
a
h
b
o
o
b
.w
o
rd
42 Data Table

p
re
in the stipulated period or in terms of accumulated BENEFITS AND PRIVILEGES

s
s
.c
Queen – Miles in either of the two systems, they are
Blue

o
automatically upgraded to the next tier. The 6-month Blue Silver Gold Platinum

m
Plus
upgrade system considers the data in rolling 6 months Tele Check-in √ √ √ √
period. The same holds for the 12-month period. If
Additional
conditions for more that one upgrade are satisfied, the baggage 10 kgs 20 kgs 35 kgs
passenger is given the higher of the two. allowance
Cumulative Queen Airways Confirmed
Flight/cumulative Status Queen-Miles upgrade 1 3 5
vouchers
6 month Fast 12-month Standard
Guaranteed
Tier upgrade Track Upgrade Upgrade
reservations
System System
up to 24 √ √
Blue to Blue plus 3 / 3000 Not Applicable hours prior To
Blue Plus to Silver Not Applicable 5 / 12500 departure
Cancellation
Silver to Gold 5 / 12500 10 / 20000 fees waived on √
Gold to Platinum 10 / 20000 20 / 30000 published fares

Distance between cities in Queen–miles


Kolkata 1461
Mumbai 1407 1987
Hyderabad 1499 1516 711
Bangalore 2061 1881 998 562
Coimbatore 2401 2167 1265 902 340
Guwahati 1959 1081 2746 2370 2932 3209
Chennai 2095 1676 1329 688 331 491 2718
Delhi Kolkata Mumbai Hyderabad Bangalore Coimbatore Guwahati

Mr. Kakkar, a newly recruited MBA from a business Date From To


school, started his career with the start of the year-
04.07.2007 Chennai Guwahati
2007. his travel plans for the year of 2007 is given
below in the table 20.07.2007 Guwahati Kolkata
02.09.2007 Kolkata Hyderabad
Date From To
11.09.2007 Hyderabad Guwahati
02.01.2007 Mumbai Hyderabad
22.09.2007 Guwahati Delhi
08.01.2007 Hyderabad Mumbai
01.10.2007 Chennai Bangalore
03.02.2007 Mumbai Delhi
11.10.2007 Guwahati Chennai
08.03.2007 Delhi Guwahati
04.11.2007 Chennai Delhi
20.03.2007 Guwahati Kolkata
29.11.2007 Delhi Hyderabad
11.04.2007 Kolkata Guwahati 01.12.2007 Hyderabad Guwahati
30.04.2007 Guwahati Chennai 31.12.2007 Guwahati Mumbai
y
o
u
rs
m
a
h
b
o
o
b
.w
o
rd
Data Table 43

p
re
6. In which month will Mr. Kakkar become eligible (a) 15.0 percent (b) 5.7 percent

s
s
.c
for guaranteed reservations up to 24 hours prior to (c) 88.6 percent (d) 37.8 percent

o
departure? (e) None of the above

m
(a) January (b) February 11. In which year annual growth rate in the aggregate
(c) March (d) April Salaries and Wages expense was maximum?
(e) May (a) 2005 (b) 2004
7. The number of complete calendar months, (c) 2003 (d) 2002
discounting any partial months, for which Mr. (e) 2001
Kakkar avails the gold tier membership is: Direction (Qs. No. 12-16): Study the tables of the
(a) 7 (b) 8 Indian foreign trade given below to answer the
(c) 9 (d) 10 question. [XAT 2007]
(e) 11 Principal
8. The difference in complete calendar months commodities’ Weight (%)
discounting any partial months, between the first tier Export
upgrade and the last tier upgrade for Mr. Kakkar is - COMMODITES 2003-04 2004-05 2005-06
(a) 7 (b) 8 plantations 0.92 0.78 0.71
(c) 9 (d) 10 agri & allied products 8.39 7.61 7.21
marine products 2.08 1.60 1.40
(e) 11 ores & minerals 3.69 5.29 6.02
9. If 6-month upgrade had not been in operation then leather & mfrs. 3.19 2.89 2.56
Mr. Kakkar would have reached Gold tier in the gems & jewellery 16.56 17.29 15.13
sports goods 0.15 0.12 0.13
month of chemicals & related
(a) June (b) July 15.43 16.00 15.10
products
(c) August (d) September engineering goods 16.41 18.41 18.66
(e) October electronic good 2.74 2.28 2.18
project goods 0.09 0.06 0.13
Direction (Qs. No. 10-11): Study the aggregate textiles 18.86 15.16 14.80
financial ratios of all registered Indian manufacturing handicrafts 0.70 0.43 0.40
companies in the table below to answer the questions carpets 0.90 0.75 0.81
cotton raw incl.
that follow. [XAT 2007] 0.28 0.10 0.61
waste
All figures are as % of net sales unless otherwise petroleum products 5.54 8.57 11.21
mentioned unclassified exports 4.07 2.66 2.94
Grand total 100.00 100.00 100.00
2000 2001 2002 2003 2004 2005
Total exports in
PBDIT 13.1 11.7 12.3 13.3 14.4 14.7 293,366.75 375,339.53 454,799.97
Rupees Crore
PBDT 8.1 7.1 8 9.9 11.8 12.7 US Dollar Exchange
PBIT 9.4 8.4 8.7 9.9 11 11.6 45.9513 44.9315 44.2735
Rate
PAT 3.2 2.8 2.7 4.4 6 6.9 Principal
Raw Material exp. 41 40.6 43.1 45.5 45.7 47.1 commodities’ Weight (%)
Salaries and wages 5.9 5.7 5.6 5.3 4.9 4.4 import
Interest payments 4.6 4.3 4 3.1 2.3 1.7 COMMODITES 2003-04 2004-05 2005-06
Operating system 5.2 4.2 4.9 6.7 8 8.7 bulk imports 37.87 39.09 42.56
Net sales(% Growth pearls, precious &
18.4 19.3 2.6 15.7 15.2 19.9 9.25 8.80 6.42
over previous year) semi precious stones
machinery 10.63 10.00 10.94
10. What is the annual growth rate in aggregate PAT project goods 0.49 0.54 0.57
of the Indian manufacturing companies in the Others 41.76 41.57 39.51
TOTAL IMPORTS 100.00 100.00 100.00
financial year 2005 as compared to that in the Total imports (in
financial year 2004? 359,107.66 501,064.54 630526.8
Crores of Rupees)
y
o
u
rs
m
a
h
b
o
o
b
.w
o
rd
44 Data Table

p
re
12. The three commodities which had highest export Directions (Qs. No. 17-21): On the basis of the data

s
s
.c
growth rate in the year 2004-05 as compared to given below. [XAT 2008]

o
the previous year, arranged in descending order of

m
Area/ Jamshedpur
growth rates are: January February March
Month Electronic
(a) petroleum products, ores and minerals,
Sales in Bistupur
engineering goods
(b) ores and minerals, gems and jewellery, Television 900 1050 1200
chemicals & related products Ipods 15750 16800 17850
(c) gems and jewellery, chemicals & related Sales in Sakchi
products, agriculture & allied products. Television 1800 2100 2400
(d) ores and minerals, chemicals & related
Ipods 9450 10080 10710
products, agriculture and allied products
(e) ores and minerals, engineering goods, Sales in Kadma
chemicals and related products Television 6300 7350 8400
13. In the year 2005-06 the commodity which Ipods 6300 6720 7140
witnessed maximum growth in exports (in Indian Units ordered = Units Sold + Ending Inventory –Beginning
Rupees) as compared to the year 2004-05 is Inventory
(a) petroleum products (b) project goods All sales figure are in Rupees thousand.
(c) ores & minerals (d) sports goods All other things are constant.
(e) None of the above All Rupees figures are in thousands.
14. In the two year period from 2004-05 to 2005-06,
the average growth in import (in Indian Rupees) of 17. In a period from January to March, Jamshedpur
which commodity to India was maximum? Electronics sold 3150 units of Television, having
(a) bulk imports started with a beginning inventory of 2520 units
(b) pearls, precious & semi-precious stones and ending with an inventory of 2880. What was
(c) machinery the value of order placed (Rupees in thousands) by
Jamshedpur Electronics during the three months
(d) project goods
period? [Profits are 25% of cost price, uniformly.]
(e) others
(a) 2808 (b) 26325
15. Growth of trade imbalance (exports less imports) (c) 22320 (d) 25200
in dollar terms in the year 2005-06 as compared to (e) 28080
the previous year was. 18. What was the total value of surcharge paid - at
(a) 39.77 (b) 41.85 the rate of 14% of sales value - by Jamshedpur
(c) 91.24 (d) 95.98 Electronics, over the period of 3 months?
(e) None of the above (a) 18522 (b) 18548
16. Given that the weight (%) of petroleum crude & (c) 18425 (d) 18485
products in the total imports of India 26.70, 27.87, (e) Cannot be determine
and 30.87 in the year 2003-04, 2004-05, and 19. 10% of sales price of IPods and 20% of sales
2005-06 respectively. What is the ratio of yearly price of Television contribute to the profits of
difference in the export of Petroleum Products and Jamshedpur Electronics. How much profit did
import of Petroleum crude & products, in dollar the company earn in the month of January from
terms, in the year 2005-06 versus 2004-05? Bistupur and Kadma from the two products?
(a) 1.36 (b) 1.38 (a) 513 (b) 4410
(c) 1.46 (d) 1.48 (c) 3645 (d) 5230
(e) None of the above. (e) 5350
y
o
u
rs
m
a
h
b
o
o
b
.w
o
rd
Data Table 45

p
re
20. In the period from January to March, consider 21. For Jamshedpur Electronics Beginning inventory

s
s
.c
that Jamshedpur Electronics ordered 7560 units was 720 for Televisions and 1800 for IPods and

o
Ending inventory was 840 for Televisions and

m
of IPods for all three areas put together. What was
1920 for IPods in the month of January. How many
unit sales price of IPod during the period? The units of Televisions and IPods did Jamshedpur
ending inventory was 6120 units and the beginning Electronics order for the month of January?
inventory stood at 5760. Additional Data: In the month of February, 1050
units of Television and 2400 units IPods were sold
(a) 14.00 (b) 14.65 in all three areas put together.
(c) 14.80 (d) 13.00 (a) 1020, 2270 (b) 1020, 2370
(e) 13.60 (c) 2270, 1030 (d) 1030, 2370
(e) 1020, 2280
Direction (Qs. No. 22-25): On the basis of the data given below. [XAT 2008]
Gender bias is defined as disproportion in percentage of drop-out rate of the two genders.

Drop Out Rates, in Percentage, at Primary, Elementary and Secondary Classes in India

Year Primary (I-V) Classes Elementary (I-VIII) Classes Secondary (I-X) Classes

Boys Girls Total Boys Girls Total Boys Girls Total

1996-97 39.7 40.9 40.2 54.3 59.5 56.5 67.3 73.7 70.0

1997-98 37.5 41.5 39.2 53.8 59.3 56.1 66.6 73 69.3

1998-99 40.9 41.3 41.5 54.2 59.2 56.3 64.5 69.8 66.7

1999-00 38.7 42.3 40.3 52.0 58.0 54.5 66.6 70.6 68.3

2000-01 39.7 41.9 40.7 50.3 57.7 53.7 66.4 71.5 68.6

2001-02 38.4 39.9 39.0 52.9 56.9 54.6 64.2 68.6 66

2002-03 35.8 33.7 34.8 52.3 53.5 52.8 60.7 65.0 62.6

2003-04 33.7 28.6 31.5 51.9 52.9 52.3 61.0 64.9 62.7

2004-05 31.8 25.4 29.0 50.4 51.2 50.8 60.4 63.8 61.9

22. Based on the data above, choose the true statement (d) Gender bias was consistently highest for
from the following alternatives: secondary classes.
(a) Gender bias in primary education has (e) None of the above.
consistently decreased over the years. 23. Assume that girls constituted 55% of the students
(b) Gender bias decreases as students move from entering school. In which year, as compared to
primary to secondary classes. the previous year, number of boys in secondary
(c) Total drop-out rate decreased consistently education would be more than the number of girls?
(a) 1996-97 (b) 1997-98
for primary classes children from 1996-97 to
(c) 2000-01 (d) 1998-99
2004-05. (e) 2001-02
y
o
u
rs
m
a
h
b
o
o
b
.w
o
rd
46 Data Table

p
re
24. Suppose, every year 7,000 students entered Class I, 1996-97 to 2004-05, were (approximately)?

s
s
out of which 45% were boys. What was the average

.c
(a) 18500 (b) 24500

o
number (integer value) of girls, who remained in (c) 19500 (d) 16000

m
educational system after elementary classes, from (e) 11500
1996-97 to 2004-05? Direction (Qs. No. 26-28): On the basis of the data
(a) 1475 (b) 1573 given in the table. [XAT 2009]
(c) 1743 (d) 1673
(e) 3853 A cake chain manufactures two types of products –
25. Suppose the total number of students in 1996-97 ‹cakes/pastries/gateaux› and savouries. The chain was
were 1000 and the number of students increased concerned about high wastage (in terms of leftover)
every year by 1000, up to 2004-05. The total and wanted to reduce it. Table 1 provides information
number of drop outs from primary classes, from about sales, costs and wastage for both products.
Table 1: Revenue Statement
Cakes/ Pastries/ Gateaux Savouries
Leftover as Leftover as
Year Sales in ` lac Costs in ` lac Sales in ` lac Costs in ` lac
%age of sales %age of sales
1993 81.47 80.06 1.52 41.79 41.07 9.38
1994 171.42 168.03 1.58 80.69 79.09 10.61
1995 326.95 323.7 1.43 146.88 145.42 10.08
1996 591.77 576.52 1.23 220.96 215.26 10.45
1997 667.1 657.89 2.06 516.23 509.1 8.43
1998 936.52 928.95 1.74 468.39 464.6 11.04
1999 978.69 966.97 1.19 528.31 521.98 6.98
2000 752.09 743.2 1.5 637.63 630.09 5.61
2001 713.46 687.83 1.83 694.42 669.47 5.96
2002 885.29 845.83 1.76 869.15 830.4 5.66
2003 1,071.81 1,014.87 1.81 913.68 865.14 6.72
2004 1,225.00 1163.75 2.78 1,075.00 1,021.25 6.14

26. Which of the following statement(s) is (are) right? 27. Maximum decline in amount of leftover of cakes/
1. The worth of leftover for cakes/pastries/ pastries/gateaux occurred in the year:
gateaux increased from 1993 to 2004. (a) From 1997 to 1998
2. The worth of leftover for cakes/pastries/ (b) From 1995 to 1996
gateaux, kept on fluctuating, many a times (c) From 1998 to 1999
between 1993 and 2004. (d) There was always an increase in worth of
3. The worth of leftover for savouries and leftover.
(e) Cannot be calculated from the data.
cakes/pastries/gateaux was highest in 2004.
28. If profit = sales – cost – leftover, in which year did
4. The worth of leftover for savouries kept on the cake chain was in losses?
fluctuating, many a times, between 1993 and 1. 1993 2. 1997
2004. 3. 1998 3. 2000
Choose the right combination from the following: Choose the right option:
(a) 1 and 4 (b) 3 and 4 (a) 1, 2, 3, 4 (b) 3, 4
(c) 1 and 2 (d) 3 only (c) 2, 3 (d) 1, 2, 3
(e) 2 and 3 (e) It was always in profit.
y
o
u
rs
m
a
h
b
o
o
b
.w
o
rd
Data Table 47

p
re
Directions (Qs. No. 29-31): Based on the information 29. Compared to January, the total expenditure on

s
s
.c
given below. [XAT 2010] carnations in March

o
The retail prices of flowers, consumer expenditure on (a) increased by 6.27%

m
flowers and sales of flowers for the calendar year 2009, (b) decreased by 6.64%
in Phoolgaon, a small town with a population of 70000,
(c) increased by 6.69%
is summarized in table 3.
(d) decreased by 7.11%
Average
Price Total (e) did not change
consumer Sales of
(retail) of consumer
Month expenditure carnations 30. Compared to January, the sales of roses in July
roses expenditure
on roses and (dozens)
( ` /dozen) on roses ( ` ) (a) decreased by 39.15%
carnations ( ` )
Jan 99 47.4 1136916 13848 (b) decreased by 28.13%
Feb 112.5 51.9 1051650 20486 (c) increased by 4.53%
Mar 135 49.5 1137915 12928 (d) increased by 4.33%
Apr 130.5 51.6 1315310 14021 (e) did not change
May 126 59.4 1116612 18774 31. Compared to January, the price of carnations in
Jun 157.5 55.8 979020 17579 December
Jul 144 56.4 1188432 17521
(a) increased by 26.57%
Aug 117 54.0 940446 20355
(b) increased by 28.12%
Sep 162 55.5 1287900 16031
Oct 126 55.2 772884 22897 (c) increased by 36.19%
Nov 189 52.8 597240 19128 (d) increased by 38.16%
Dec 166.5 56.4 977688 18859 (e) did not change

Direction (Qs. No. 32-37: Study the table below and answer the questions that follow it. [IIFT 2007]
World merchandise exports by Select Countries (Million Dollars)

Countries 1999 2000 2001 2002 2003 2004 2005

Cambodia 1129 1389 1500 1923 2118 2798 3100

China 194931 249203 266098 325596 438228 593326 761954

India 35667 42379 43361 49250 57085 75562 95096

Japan 417610 479249 403496 416726 471817 565675 594905

South Korea 143686 172267 150439 162471 193817 253845 284419

Myanmar 1136 1646 2381 3046 2483 2380 2925

Singapore 114680 137804 121751 125177 159902 198637 229649

Thailand 58440 69057 64968 68108 80324 96248 110110

Viet Nam 11540 14449 15029 16530 20176 25625 31625


y
o
u
rs
m
a
h
b
o
o
b
.w
o
rd
48 Data Table

p
re
32. The third highest average annual export growth 35. Which of the following statement is false?

s
s
.c
over the entire period (1999-2005) has been (a) Average annual levels of SO2 emission for

o
experienced by: Pondichery over 1998-2003 is approximately

m
(a) Cambodia (b) India. 51.40 mg/m3
(c) Myanmar (d) Vietnam
(b) Average annual levels of SO2 emission for
33. Which of the Following statement is not true?
(a) During 1999-2000, Myanmar registered the Anpara over 1997-2003 is approximately
highest annual export growth rate. 48.24 mg/m3
(b) India witnessed second highest annual export (c) Average annual levels of SO2 emission for
growth rate during 2003-04. Madras over 1999-2003 is approximately
(c) Cambodia registered third highest annual 25.02 mg/m3
export growth rate during 2001-02. (d) Average annual levels of SO2 emission
(d) The change in Thailand’s export growth rate for Yamuna Nagar during 1997-2003 is
from 2000-01 to 2001-02 was more than 10 approximately 23.74 mg/m3
percent.
36. If the highest average annual level of SO2 emission
34. Which of the following statement is not false?
among the given cities is noted year-wise, then their
(a) South Korea registered the third lowest
export growth rate during the year 2000-01. difference would be maximum for the following
(b) The sum of the export growth rate of India pair of years:
and Vietnam during 2001-02 is lower than (a) 1998 and 2003. (b) 1997 and 2001.
the export growth rate of China during that (c) 1997 and 2003. (d) 1998 and 2002.
particular year. 37. Which of the following statement is true?
(c) Myanmar witnessed maximum number of
(a) Bangalore in 2003 registered the lowest level
years of positive export growth rate during
of SO2 emission in relation to the given dataset.
the entire period.
(d) The difference between the export growth (b) The difference between the average annual
rate of China and Japan during 2004-05 was level of SO2 emission during 1997-2003 for
lower than the export growth rate of Vietnam Pondichery and Gajroula is lower than the
during that particular year. corresponding average annual level of Mysore.
Direction (Qs. No. 35-39): Study the table below and (c) The average annual level of SO2 emission
answer the questions that follow it. [IIFT 2007] during 1997-2000 for Faridabad is higher
Industrial Emission Level of SO2 is Select City Points than the average annual level for the city for
(1997-2003). Annual Mean Concentration Range the entire period (1997-2003).
(mg/m3)
(d) The sum of the average annual level of SO2
City 1997 1998 1999 2000 2001 2002 2003 emission during 1997-2003 for Bombay and
Anpara 54.1 57.9 59.3 64.6 53 30.1 18.7
Bangalore 28.1 37.2 37.9 19.1 19.3 12.1 7.6 Calcutta is lower than the corresponding
Bombay 36 21.7 22.3 11.8 12 9.7 7.4 average annual level for Anpara.
Calcutta 33.2 40.8 66.9 25.3 21.9 13.3 18.1
Cochin 7.4 4.9 10.5 41.6 24.6 31.5 23.4 38. If the SO2 emission level change for the given
Faridabad 37.8 35.7 31.3 37.3 23.1 13.1 9.5 cities is noted year-wise, the difference would be
Gajroula 25.8 19.7 25.7 26.6 35.5 41.1 39.3
Madras 26.3 14.2 11.7 20.1 26.1 40.9 26.3 maximum for which of the following option?
Mysore 32.4 32.1 31 30.7 24.1 20.6 11.2 (a) Cochin and Pondichery during 1999-2000.
Nagda 81.8 55.2 26.9 52.6 46.5 36.5 36.5
Pondichery 112.3 114.9 93.3 37.6 17.5 19.8 25.3 (b) Calcutta and Nagda during 1998-1999.
Solapur 19.4 17.2 18 18.9 19.4 20.1 19.9 (c) Madras and Anpara during 2001-2002.
Yamuna
27.8 32.2 9.8 18.9 22.1 28.6 28.2 (d) Nagda and Pondichery during 1997-98.
Nagar
y
o
u
rs
m
a
h
b
o
o
b
.w
o
rd
Data Table 49

p
re
39. Which of the following statement is true? (c) The absolute decline in annual SO2 emission

s
s
.c
(a) While for Anpara, the SO2 annual emission level in for Bombay during 1997-1998 was

o
level declined consistently during 2000- lower than the corresponding figure for

m
2001 and 2002-2003, the same increased Faridabad during 2000-2001.
consistently for Yamuna Nagar between (d) The number of cities which experienced
1998-99 and 2001-02. a decline in their annual SO2 emission
(b) During 2002-2003, the SO2 annual emission level during 1999-2000 was more than the
level declined for maximum number of cities. corresponding figure during 1998-1999.

Direction (Qs. No. 40-43): Study the table below and answer the questions that follow it. [IIFT 2007]
Major Regional Trade Flows in World Exports (Annual Percentage Change) (in Percentage)

Region Chemicals Iron and Steel Automotive Parts

2003 2004 2005 2003 2004 2005 2003 2004 2005

Intra-Europe 22 21 11 27 45 10 24 20 1

Intra-Asia 23 33 19 31 42 22 39 23 12

Europe to North America 19 11 6 –9 81 21 14 8 6

Intra-North America 11 18 14 4 41 23 0 9 6

Europe to Asia 18 24 8 0 16 39 0 9 –1

40. If the region-wise increase in the export growth 42. The difference between the highest and the lowest
rate of the three industries during 2003-2004 is average export growth rate during 2005 among all
noted, _________ would be ranked second. three industries and regions is:
(a) 40 percent.
(a) Intra-North America iron and steel export (b) 33 percent.
(b) Intra-Europe iron and steel export. (c) 29 percent.
(c) Intra-Asia chemicals export. (d) None of the above.
43. If the region-wise average export growth rates of
(d) Europe to asia the three industries are analyzed, ________ would
41. If the region-wise decline in the export growth rate be ranked second.
of the three industries during 2004-2005 is noted, (a) Intra-Asia average export of chemicals
________ would be ranked second. during 2003-05.
(b) Intra-Europe average export of iron and steel
(a) Intra-Europe automotive parts export. during 2004-05.
(b) Intra-Europe iron and steel export. (c) Intra-Asia average export of automotive part
during 2003-05.
(c) Intra-Asia chemicals export.
(d) Intra-North America average export of iron
(d) Europe to Asia chemicals export. and steel during 2003-05.
y
o
u
rs
m
a
h
b
o
o
b
.w
o
rd
50 Data Table

p
re
Directions (Qs. No. 44-49): Answer the questions based on the following table. [IIFT 2008]

s
s
.c
Growth Trend in Rail Wagons

o
m
Open Open
Depart- Special Total Average
Total Covered High Low
Year mental type wagon wagon
Wagons wagons sided sided
wagons wagons capacity capacity
wagons wagons
Number Number Number Number Number Number (Million Tones) (Tonnes)
1993 337562 157581 105469 12221 12009 50282 11.79 34.9
1994 312405 138642 101160 11922 11473 49208 11.32 36.2
1995 291360 121946 98795 11507 11185 47927 10.76 36.9
1996 280791 114065 98297 11196 11008 46225 10.62 37.8
1997 272127 106634 98906 10601 10645 45341 10.64 39.1
1998 263981 102217 97616 9726 10569 43853 10.69 40.5
1999 252944 96371 95613 9106 9612 42242 10.7 42.3
2000 234397 86024 91415 7735 8907 40316 10.26 43.8
2001 222193 75768 91099 7999 8443 38884 10.19 45.9
2002 216717 71950 90371 7585 9536 37275 10.09 46.6
2003 214760 68467 90765 7160 10718 37650 9.98 46.5
2004 227752 67870 100211 8882 11388 39401 10.66 46.8
2005 222379 64417 101757 8787 10964 36454 10.6 47.7

44. Find the True Statement: 47. Find out the Lowest annual growth rate among the
(a) The number of covered wagons expressed following:
as a percentage of total wagons declined (a) Annual growth rate of total wagons in 1999
consistently from 1993 to 2002, but increased (b) Annual growth rate of covered wagons in 1998
marginally in 2003 as compared to the (c) Annual growth rate of special type wagons in
previous year level. 2002
(b) The special type wagons expressed as a (d) Annual growth rate of total wagon capacity
percentage of total wagons is maximum in 2000
during 2003. 48. Find out the False statement:
(c) The open high sided wagons expressed as a (a) The annual growth rate of covered wagons in
percentage of total wagons increased during 1996 was higher than the same in 2000.
1994 to 2001, but declined from the 2001 (b) The annual growth rate of open high sided
level in 2002. wagons in 1997 was higher than the same in 2003.
(c) The annual percentage growth rate of average
(d) None of the above.
wagon capacity has been maximum in 1999.
45. The special type wagons expressed as a percentage (d) None of the above.
of total wagons were at almost same level during 49. Find out the Highest annual growth rate among the
the following pair of years: following:
(a) 1995 and 2001 (b) 1998 and 2004 (a) Annual growth rate of total wagons in 1995.
(c) 2000 and 2002 (d) 1993 and 1994 (b) Annual growth rate of covered wagons in 2002.
46. The Departmental wagons expressed as a (c) Annual growth rate of open Low sided
percentage of total wagons was maximum during: wagons in 1998.
(a) 2002 (b) 2005 (d) Annual growth rate of departmental wagons
(c) 2004 (d) 2003 in 2000.
y
o
u
rs
m
a
h
b
o
o
b
.w
o
rd
Data Table 51

p
re
s
s
.c
o
m
Direction (Qs. No. 1-3): Based on the information (c) Between Jan-Mar 2009 and Apr-Jun 2009,
given below. [XAT 2010] both profit and profit rate of Paharpur Cement
Cost and price data for Portland cement manufactured decreased, whereas profit rate of Bahsin
by Paharpur Cement and Bahsin Cement, for four Cement decreased but its profit increased,
consecutive quarters, are given in table. (d) Between Jan-Mar 2009 and Apr-Jun 2009,
Paharpur Cement Bahsin Cement profit rate of Paharpur Cement decreased but
its profit increased, whereas both profit and
Cost Cost ( as %
Price Price profit rate of Bahsin Cement increased.
(as % of sales of sales
( `/bag) ( `/ bag) 3. If between Apr-Jun 2009 and Jul-Sept 2009 sales
revenue) revenue)
Oct-Dec of Paharpur Cement increased by 2.25%, it profit
92.11 352 94.21 438 increased by
2008
Jan-Mar (a) 2.08% (b) 2.25%
87.56 304 91.34 440 (c) 2.96% (d) 3.42%
2009
Apr-Jun Direction (Qs. No. 4-6): Based on the information
91.03 340 89.96 430
2009 given below. [XAT 2010]
Jul-Sep The retail prices of flowers, consumer expenditure on
90.42 322 90.38 434
2009
flowers and sales of flowers for the calendar year 2009,
Sales Revenue = Price × Sales Quantity in Phoolgaon, a small town with a population of 70000,
Profit = Sales Revenue – Cost is summarized in table.
Profit Rate = Profit / Sales Quantity Month Price Average Total Sales of
1. Profit rate of Paharpur Cement is more than the (retail) of consumer consumer carnations
profit rate of Bahsin Cement in: roses expenditure on expenditure (dozens)
(a) Oct-Dec 2008 and Jan-Mar 2009 (` / roses and on roses (`)
(b) Jan-Mar 2009 and Apr-Jun 2009 dozen) carnations (`)
(c) Oct-Dec 2008 only
(d) Jan-Mar 2009 only
Jan 99 47.4 1136916 13848
2. If between Jan-Mar 2009 and Apr-Jun 2009 sales of Feb 112.5 51.9 1051650 20486
Paharpur Cement increased from 543278 to 698236 Mar 135 49.5 1137915 12928
and that of Bahsin Cement decreased from 526532 Apr 130.5 51.6 1315310 14021
to 499874, then which of the following is true? May 126 59.4 1116612 18774
(a) Between Jan-Mar 2009 and Apr-Jun 2009, Jun 157.5 55.8 979020 17579
profit and profit rate of Paharpur Cement Jul 144 56.4 1188432 17521
increased, whereas profit and profit rate of Aug 117 54.0 940446 20355
Bahsin Cement decreased. Sep 162 55.5 1287900 16031
(b) Between Jan-Mar 2009 and Apr-Jun 2009, Oct 126 55.2 772884 22897
profit rate of Paharpur Cement increased but Nov 189 52.8 597240 19128
its profit decreased, whereas both profit and
Dec 166.5 56.4 977688 18859
profit rate of Bahsin Cement increased.
y
o
u
rs
m
a
h
b
o
o
b
.w
o
rd
52 Data Table

p
re
4. Compared to January, the total expenditure on (c) increased by 4.53%

s
s
carnations in March

.c
(d) increased by 4.33%

o
(a) increased by 6.27% 6. Compared to January, the price of carnations in

m
(b) decreased by 6.64% December
(c) increased by 6.69% (a) increased by 26.57%
(d) decreased by 7.11% (b) increased by 28.12%
5. Compared to January, the sales of roses in July (c) increased by 36.19%
(a) decreased by 39.15%
(d) did not change
(b) decreased by 28.13%
Directions (Qs. No. 7-11): Answer the questions based on the following Table. [IIFT – 2009]
Crops Area under production Quantity of Production
(‘000 Hectare) (‘000 Tonne)
2006 2007 2008 2006 2007 2008
Cereals
Rice 107 108 110 153 170 190
Jowar 598 673 720 173 368 380
Bajra 4992 4890 4800 2172 3421 3350
Maize 1004 1020 1101 1102 1116 1182
Millets 16 16 15 5 4 4
Pulses
Moong 799 751 725 130 270 260
Urd 124 101 102 31 30 35
Moth 1228 1151 1199 149 191 250
Arhar(Tur) 20 19 19 13 9 9
Chaula 126 110 101 26 45 30
Other Pulses 5 5 5 2 3 2
Oil Seeds
Sesamum 422 273 280 63 89 70
Groundnut 317 302 298 491 396 374
Soyabean 744 641 650 856 771 799
Sunflower 472 350 325 880 751 699
Castor Seed 106 79 80 135 104 106
Note: Yield (or Productivity)is defined as quantity produced per hectare

7. What is the ratio between Jowar yield (2007) and (c) Castor Seed, Sunflower, Rice
Soyabean yield (2008)? (d) Bajra, Maize, Castor Seed
(a) 1.00 : 2.10 (b) 1.21 : 1.89 9. Bottom 3 crops by yield in the year 2008 are:
(c) 0.89 : 2.00 (d) 0.78 : 1. 61 (a) Moth, Sesamum, Millets
8. Top 3 crops by yield in the year 2006 are: (b) Moong, Moth, Millets
(a) Castor Seed, Groundnut, Maize (c) Arhar, Urd, Moong
(b) Sunflower, Groundnut, Rice (d) Moong, Sesamum, Chaula
y
o
u
rs
m
a
h
b
o
o
b
.w
o
rd
Data Table 53

p
re
10. Examine the following statements: 11. Examine the following statements:

s
s
.c
I. Total productivity of pulses has gone down I. Over the period total cereal productivity has

o
over the years gone up

m
II. Maize is the most stable cereal in terms of II. Area, Production and yield of the total oil
productivity over the years seeds is on decline
III. Percentage growth in area and quantity of III. Though there is a decline in the area under
production is highest in the case of Jowar Urd production but the quantity of production
during the entire period. Select the best and yield has gone up over the years.
option: Select the best option:
(a) Statement I and II are correct (a) Statement I and III are correct
(b) Statement I and III are correct (b) Statement I and II are correct
(c) Statement II and III are correct (c) Statement I alone is correct
(d) Statement III alone is correct (d) Statement III alone is correct
Directions (Qs. No. 12-16): Study the following carefully and answer the questions. [IIFT 2009]
Oil Sales by Type of Use and Production Loss (MT)
Oil Used Oil
Production
Metro City Suburban
Loss
Year House Transport Industrial

Hold
1996 10 700 1794 104 650
1997 16 920 1864 114 732
1998 22 1012 1732 104 834
1999 22 1054 1668 90 1102
2000 20 1092 1700 108 702
2001 22 1320 1752 168 492
2002 14 1854 1618 172 386
2003 6 2046 1270 178 444
2004 16 2118 1480 208 506
2005 4 2252 1132 230 1032
2006 4 2384 1228 210 1142
2007 8 2096 1672 254 1454
2008 0.2 2240 1646 266 1428
Note: Whatever is produced in a year is either used or lost in the production in the same year.

12. During which year the Oil used for House Hold as (a) 2002 (b) 2003
a percentage of Total Oil Used is highest? (c) 2004 (d) 2006
(a) 1998 (b) 1999 14. During which year use of oil by Suburban as a
(c) 2000 (d) 2001 proportion of Total Oil Used was the highest?
13. During which year the Oil Production Loss as a (a) 2005 (b) 2006
proportion of Total Oil Produced is the lowest? (c) 2007 (d) 2008
y
o
u
rs
m
a
h
b
o
o
b
.w
o
rd
54 Data Table

p
re
15. For how many number of years the growth rate in 17. What are the countries on the coverage frontier?

s
s
.c
Production of Oil is more than the growth rate in (a) India and China

o
Total Oil Used? (b) Sri Lanka and Indonesia

m
(a) 3 years (b) 4 years (c) Philippines and Bangladesh
(c) 5 years (d) 6 years (d) Nepal and Pakistan
16. Which of the below statements are true, based on
18. Which of the following statements are true?
the data in the above table?
A. India > Pakistan and India > Indonesia
(a) Oil is used for Transport purpose by Metro
B. India > China and India > Nepal
City is increasing every since 1996.
(b) Oil is used for Industrial purpose by Metro C. Sri Lanka > China
City is increasing every since 1996. D. China > Nepal
(c) Oil used by Suburban is increasing every (a) A and C (b) B and D
year since 2000. (c) A, B and C (d) B, C and D
(d) Total Oil Produced is increasing every year 19. Using only the data presented under Sanitation
since 2003. facilities columns, it can be concluded that rural
Directions (Qs. No. 17-21): These questions are based population in India, as a percentage of its total
on the table below presenting data on percentage of the population is approximately
population covered by drinking water and sanitation (a) 76
facilities in selected Asian countries.  [CAT 1999] (b) 70
Population Covered by Drinking (c) 73
Water and Sanitation Facilities (d) Cannot be determined
Percentage Coverage 20. Again, using only the data presented under
Drinking Water Sanitation Facilities Sanitation facilities columns, sequence China,
Urban Rural Total Urban Rural Total Indonesia and Philippines in ascending order of
India 85 79 81 70 14 29 rural population as a percentage of their respective
Bangladesh 99 96 97 79 44 48
total populations. The correct order is:
China 97 56 67 74 7 24
(a) Philippines, Indonesia, China
Pakistan 82 69 74 77 22 47
Philippines 92 80 86 88 66 77 (b) Indonesia, China, Philippines
Indonesia 79 54 62 73 40 51 (c) Indonesia, Philippines, China
Sri Lanka 88 52 57 68 62 63 (d) China, Indonesia, Philippines
Nepal 88 60 63 58 12 18 21. India is not on the coverage frontier because
Source: World Resources 1998-99, p. 251, UNDP, UNEP
A. it is lower than Bangladesh in terms of
and World Bank
coverage of drinking water facilities.
Country A is said to dominate B or A > B if A has B. it is lower than Sri Lanka in terms of coverage
higher percentage in total coverage for both drinking of sanitation facilities.
water and sanitation facilities, and, B is said to be
C. it is lower than Pakistan in terms of coverage
dominated by A, or B < A.
of sanitation facilities.
A country is said to be on the coverage frontier if no
D. it is dominated by Indonesia.
other country dominates it. Similarly, a country is not
(a) A and B (b) A and C
on thecoverage frontier if it is dominated by at least one
other country. (c) D (d) None of these
y
o
u
rs
m
a
h
b
o
o
b
.w
o
rd
Data Table 55

p
re
Directions (Qs. No. 22-24): Answer these questions Directions (Qs. No. 25-27): Answer the following

s
s
.c
with reference to the table given below: Questions based on the information given below.

o
Information Technology Industry in India There are 100 employees in an organization across five

m
(Figures are in million US dollars) departments. The following table gives the department-
wise distribution of average age, average basic pay and
1994-95 1995-96 1996-97 1997-98 1998-99
allowances. The gross pay of an employee is the sum of
Software: his/her basic pay and allowances.
Domestic 350 490 670 950 1250
Average
Average Allowances
Exports 485 734 1083 1750 2650 Number of Basic
Department Age (% of
Hardware: Employees Pay
(Years) Basic Pay)
(Rupees)
Domestic 590 1037 1050 1205 1026
HR 5 45 5000 70
Exports 177 35 286 201 4
Marketing 30 35 6000 80
Peripherals:
Finance 20 30 6500 60
Domestic 148 196 181 229 329
Business
Exports 6 6 14 19 18 35 42 7500 75
Development
Training 107 143 185 263 302
Maintenance 10 35 5500 50
Maintenance 142 172 182 221 236
Networking There are limited numbers of employees considered
36 73 156 193 237 for transfer/promotion across departments. Whenever
and others
Total 2041 2886 3807 5031 6052 a person is transferred/promoted from a department of
lower average age to a department of higher average
22. The total annual exports lay between 35 and
age, he/she will get an additional allowance of 10% of
40 percent of the total annual business of the IT
basic pay over and above his/her current allowance.
industry, in the years:
There will not be any change in pay structure if a person
(a) 1997-98 & 1994-95
is transferred/promoted from a department with higher
(b) 1996-97 & 1997-98
average age to a department with lower average age.
(c) 1996-97 & 1998-99
Questions below are independent of each other.
(d) 1996-97 & 1994-95
25. There was a mutual transfer of an employee
23. The highest percentage growth in the total IT
business, relative to the previous year was achieved between Marketing and Finance departments and
in: transfer of one employee from Marketing to HR.
(a) 1995-96 (b) 1996-97 As a result, the average age of Finance department
(c) 1997-98 (d) 1998-99 increased by one year and that of marketing
24. Which one of the following statement is correct? department remained the same. What is the new
(a) The annual software exports steadily average age of HR department?
increased but annual hardware exports (a) 30 (b) 35
steadily declined during 1994-1999. (c) 40 (d) 45
(b) The annual peripheral exports steadily 26. What is the approximate percentage change in the
increased during 1994-1999. average gross pay of the HR department due to
(c) The IT business in training during 1994- transfer of a 40-yr old person with basic pay of Rs.
1999 was higher than the total IT business in 8000 from the Marketing department?
maintenance during the same period. (a) 9% (b) 11%
(d) None of the above statements is true. (c) 13% (d) 15%
y
o
u
rs
m
a
h
b
o
o
b
.w
o
rd
56 Data Table

p
re
27. If two employees (each with a basic pay of ` 6000)

s
s
Sectional Cut-off Marks Aggregate

.c
are transferred from Maintenance department Cut-off

o
Section Section Section Section
to HR department and one person (with a basic

m
Marks
A B C D
pay of ` 8000) was transferred from Marketing College 1 42 42 42 176
department to HR department, what will be the College 2 45 45 175
percentage change in average basic pay of HR College 3 46 171
department? College 4 43 45 178
(a) 10.5% (b) 12.5%
College 5 45 43 180
(c) 15% (d) 30%
College 6 41 44 176
Directions (Qs. No. 28-30) Answer the following 28. Aditya did not get a call from even a single college.
questions based on the information given below: What could be the maximum aggregate marks
For admission to various affiliated colleges, a university obtained by him?
conducts a written test with four different sections, (a) 181 (b) 176
each with a maximum of 50 marks. The following (c) 184 (d) 196
table gives the aggregate as well as the sectional cut-off 29. Bhama got calls from all colleges. What could be
marks fixed by six different colleges affiliated to the the minimum aggregate marks obtained by her?
(a) 180 (b) 181
university. A student will get admission only if he/she
(c) 196 (d) 176
gets marks greater than or equal to the cut-off marks in
30. Charlie got calls from two colleges. What could be
each of the sections and his/her aggregate marks are at the minimum marks obtained by him in a section?
least equal to the aggregate cut-off marks as specified (a) 0 (b) 21
by the college. (c) 25 (d) 35

Directions (Qs. No. 31-34): Answer the following questions based on the information given below:   [CAT 2007]
The Table below shows the comparative costs, in US Dollars, of major surgeries in USA and a select few Asian
countries.
Procedure Comparative costs in USA and some Asian countries (in US Dollars)
USA INDIA THAILAND SINGAPORE MALAYSIA
Heart Bypass 130000 10000 11000 18500 9000
Heart Valve Replacement 160000 9000 10000 12500 9000
Angioplasty 57000 11000 13000 13000 11000
Hip Replacement 43000 9000 12000 12000 10000
Hysterectomy 20000 3000 4500 6000 3000
Knee replacement 40000 8500 10000 13000 8000
Spinal Fusion 62000 5500 7000 9000 6000

The equivalent of one US Dollar in the local currencies is given below


1 US Dollar equivalent
India 40.928 Rupees
Malaysia 3.51 Ringits
Thailand 32.89 Bahts
Singapore 1.53 S Dollars
y
o
u
rs
m
a
h
b
o
o
b
.w
o
rd
Data Table 57

p
re
A consulting firm found that the quality of the health services were not the same in all the countries above. A poor

s
s
.c
quality of a surgery may have significant repercussions in future, resulting in more cost in correcting mistakes. The

o
cost of poor quality of surgery is given in the table below:

m
Comparative costs of poor quality in USA and some Asian countries
Procedure
(in US Dollars x 1000)
USA INDIA THAILAND SINGAPORE MALAYSIA
Heart Bypass 0 3 3 2 4
Heart Valve Replacement 0 5 4 5 5
Angioplasty 0 5 5 4 6
Hip Replacement 0 7 5 5 8
Hysterectomy 0 5 6 5 4
Knee replacement 0 9 6 4 4
Spinal Fusion 0 5 6 5 6

31. A US citizen is hurt in an accident and requires (a) 23500 (b) 40500
in angioplasty, hip replacement and a knee (c) 57500 (d) 67500
replacement. Cost of foreign travel and stay is not 33. Taking the cost of poor quality into account, which
a consideration since the government will take country/countries will be the most expensive for
care of it. Which country will result in the cheapest knee replacement?
package, taking cost of poor quality into account? (a) India (b) Thailand
(a) India (b) Thailand (c) Malaysia (d) Singapore
(c) Malaysia (d) Singapore 34. The rupee value increases to ` 35 for a US Dollar,
32. Approximately, what difference in amount in and all other things including quality, remain the
Bahts will it make to a Thai citizen if she were to same. What is the approximate difference in cost,
get a hysterectomy done in India instead of in her in US Dollars, between Singapore and India for a
native country, taking into account the cost of poor Spinal Fusion, taking change into account?
quality? It costs 7500 Bahts for one-way travel (a) 700 (b) 2500
Thailand and India. (c) 4500 (d) 8000
Directions (Qs. No. 35-39): Answer the following questions based on the information given below : [CAT 2007]
A low-cost airline connects ten Indian cities, A to J. The table below gives the distance between a pair of airports
and the corresponding price charged by the company. Travel is permitted only from a departure airport to an arrival
airport. The customers do not travel by a route where they have to stop at more than two intermediate airports.
Sector No Airport Departure Airport Arrival Distance between the Price ( `)
Airport (KM)
1 A B 560 670
2 A C 790 1350
3 A D 850 1250
4 A E 1245 1600
5 A F 1345 1700
6 A G 1350 2450
7 A H 1950 1850
y
o
u
rs
m
a
h
b
o
o
b
.w
o
rd
58 Data Table

p
re
s
Sector No Airport Departure Airport Arrival Distance between the Price ( `)

s
.c
Airport (KM)

o
m
8 B C 1650 2000
9 B H 1750 1900
10 B I 2100 2450
11 B J 2300 2275
12 C D 460 450
13 C F 410 430
14 C G 910 1100
15 D E 540 590
16 D F 625 700
17 D G 640 750
18 D H 950 1250
19 D J 1650 2450
20 E F 1250 1700
21 E G 970 1150
22 E H 850 875
23 F G 900 1050
24 F I 875 950
25 F J 970 1150
26 G I 510 550
27 G J 830 890
28 H I 790 970
29 H J 400 425
30 I J 460 540
35. What is the lowest price, in rupees, a passenger has (a) 2275 (b) 2615
to pay for travelling by the shortest route from A to J? (c) 2850 (d) 2945
(a) 2275 (b) 2850 38. If the prices include a margin of 10% over the total
(c) 2890 (d) 2930 cost that the company incurs, what is the minimum
36. The company plans to introduce a direct fight
cost per kilometer that the company incurs in
between A and J. The market research results
indicate the all its existing passengers travelling flying from A to J?
between A and J will use this direct flight if it is (a) 0.77 (b) 0.88
priced 5% below the minimum price that they (c) 0.99 (d) 1.06
pay at present. What should the company charge 39. If the price include a margin of 15% over the total
approximately, in rupees, for this direct flight? cost that the company incurs, which among the
(a) 1991 (b) 2161 following is the distance to be covered in flying
(c) 2707 (d) 2745 from A to J that minimizes the total cost per
37. If the airports C, D and H are closed down owing
kilometer for the company?
to security reasons, what would be the minimum
price, in rupees, to be paid by a passenger travelling (a) 2170 (b) 2180
from A to J? (c) 2315 (d) 2350
y
o
u
rs
m
a
h
b
o
o
b
.w
o
rd
Data Table 59

p
re
Directions (Qs. No. 40-43): Answer the following Directions (Qs. No. 44-47: Answer the question on the

s
s
basis of the information given below. [CAT 2004]

.c
questions based on the information given below:

o
A health-drink company’s R & D department is trying to Prof. Singh has been tracking the number of visitors to

m
make various diet formulations, which can be used for his homepage. His services provider has provided him
certain specific purposes. It is considering a choice of 5 with the following data on the country of origin of the
alternative ingredients (O, P, Q, R and S), which can be visitors and the university they belong to
used in different proportions in the formulations. The Number of visitors
table below gives the composition of these ingredients.
The cost per unit of each of these ingredients is O:150, Country DAY
P: 50, Q: 200, R: 500, S:100. Day 1 Day 2 Day 3
Composition Canada 2 0 0
Ingredient Carbohydrate Protein Fat Minerals
(%) (%) (%) (%) Netherlands 1 1 0
O 50 30 10 10 India 1 2 0
P 80 20 0 0
Q 10 30 50 10 UK 2 0 2
R 5 50 40 5 USA 1 0 1
S 45 50 0 5
40. Which among the following is the formulation
Number of visitors
having the lowest cost per unit for a diet having
10% fat and at least 30% protein? The diet has to DAY
UNIVERSITY
be formed by mixing two ingredients. Day 1 Day 2 Day 3
(a) P and Q (b) P and S University 1 1 0 0
(c) P and R (d) Q and S
University 2 2 0 0
41. In what proportion P, Q and S should be mixed to
make a diet having at least 60% carbohydrate at the University 3 0 1 0
lowest per unit cost? University 4 0 0 2
(a) 2:1:3 (b) 4:1:2 University 5 1 0 0
(c) 2:1:4 (d) 4:1:1
University 6 1 0 1
42. The company is planning to launch a balanced
diet required for growth needs of adolescent University 7 2 0 0
children. This diet must contain at least 30% University 8 0 2 0
each of carbohydrate and protein, no more than
25% fat and at least 5% minerals. Which one 44. To which country does University 5 belong?
of the following combination of equally mixed (a) India or Netherlands but not USA
(b) India or USA but not Netherlands
ingredients is feasible?
(c) Netherlands or USA but not India
(a) O and P (b) R and S
(d) India or USA but not UK
(c) P and S (d) O and S
45. University 1 can belong to
43. For a recuperating patient, the doctor recommended
(a) UK (b) Canada
a diet containing 10% minerals and at least 30% (c) Netherlands (d) USA
protein. In how may different ways can we prepare 46. Visitors from how many universities from UK
this diet by mixing at least two ingredients? visited Prof.Singh’s homepage in the three days?
(a) One (b) Two (a) 1 (b) 2
(c) Three (d) Four (c) 3 (d) 4
y
o
u
rs
m
a
h
b
o
o
b
.w
o
rd
60 Data Table

p
re
47. Which among the listed countries can possibly 48. Which of the following institute has the lowest ratio

s
s
.c
host three of the eight listed universities? of fee per student to Exp on faculty per student?

o
(a) None (b) Only UK (a) AAA (b) BBB

m
(c) Only India (d) Both India and UK (c) CCC (d) DDD
49. Which of the following institutes has lowest ratio
Directions (Qs. No. 48-50): There are four coaching of profit per student?
institutes AAA, BBB, CCC & DDD are running (a) AAA (b) BBB
their centers at Delhi. The following table gives (c) CCC (d) DDD
the information about their revenue generation and 50. Which of the following statements is true?
expenditure. (i) For AAA ratio of total profit to total exp =
1.07
AAA BBB CCC DDD
(ii) For BBB ratio of total profit to total exp =
No of Students 1710 2710 1920 3920 0.922
Revenue x 1000 3505.5 5284.5 4128 7252 (iii) For CCC ratio of total profit to total exp =
1.40
Exp. on Faculty
x 1000
850 1220 1640 2840 (iv) For DDD ratio of total profit to total exp =
0.58
Exp on Infra. x
650 1150 1560 1280 (a) (i), (ii) & (iii) are true
1000
(b) (i), (ii) & (iv) are true
Cost of material (c) (ii), (iii) & (iv) are true
110 140 130 120
per student (`) (d) None of the above.

Direction (Qs. No. 1-3): Answer the questions given 1st 1st 1st 1st
below. August September October November
Krishna, Shyam and Murli are three friends and all are 2014 2014 2014 2014
very fond of comics. All of them had collected 100 Krishna to
3 6 1 8
Shyam
different comics as on 31st july 2014. They decided
Shyam to
to exchange some of their comics among themselves 1 5 4 6
Krishna
on the first day of every upcoming four months. Table Shyam to
3 4 3 7
depicts the number of comics transferred from one Murli
person to another. Murli to
7 3 2 3
Shyam
Note: i. On a single day, No comic was transferred
Krishna to
more than once. 9 2 6 4
Murli
ii. Any transferred of a comic necessarily Murli to
involved the original owner of the comic 2 3 5 1
krishna
y
o
u
rs
m
a
h
b
o
o
b
.w
o
rd
Data Table 61

p
re
1. On 2nd nov 2014 , the number of comics originally (a) 91 (b) 94

s
s
.c
owned by Murly Among those in possession of (c) 98 (d) None of these

o
Shyam cannot be less than Direction of (Qs. No. 4-7): Answer the questions given

m
(a) 0 (b) 3 below.
(c) 2 (d) 4 The tables given below shows the score card of a
2. Which among the following can be the minimum twenty twenty tournament between four teams of
number of comics originally owned by Krishna four different clubs namely Lucky Club (LC), Youth
among those in possession of krishna on 2nd nov Sporting Club(YSC), Star Dust Club (SDC) and Citizen
2014? Club(CC) played against each other in the initial stage
(a) 64 (b) 63 and weighted criteria. Table 1 shows the number of
(c) 62 (d) 61 runs scored by the batting side in each of the matches.
3. The number of comics owned by Murli among The two teams with the highest number of wins and
possession of Murli cannot be more than, as on 2nd weightage qualifies for final.
oct 2014?
TABLE 1:
Fielding side
LC YSC SDC CC
Batting side

LC 162 ( 120 balls) 179 (114 balls) 132 (d)

YSC 159 ( 118 balls) 180 (c) 210(108 balls)


SDC 164 (96 balls) 140 (b) 144 (110 balls)
CC 133 (a) 165 (120 balls) 154 (115 balls)

TABLE 2: 4. If a = 90 and d = 120, which team qualifies for the


Criterian Slab Weightage final ( not considering the weightage)?
200 and above 8 (a) LC and YSC
180 -199 7 (b) LC and CC
150 -179 6 (c) YSC and CC
120 -149 5 (d) cannot be determind
90 - 119 4 5. Which team has highest weighted average?
89 0r less 3 (a) LC (b) YSC
(c) SDC (d) CC
NOTE:
1. (a, b, c and d) are the figures of balls which is 6. How many team(s) definitely had a positive value
neither less than 90 nor more than 120. of NRR at the end of the first Six matches?
2. Where NRR (Net Run Rate) = RR1 – RR2. (a) more than 2 (b) 2
RR1 for team is defined as (c) 1 (d) 0
total run score in all the matches 7. If a = b = c = d = 90, then which team was at the
=
total number of ball scored 2nd position at the end of the initial stage?
RR2 for a team is defined as
total run score against a team (a) LC (b) YSC
= (c) SDC (d) CC
total number of ball thrown by that team
y
o
u
rs
m
a
h
b
o
o
b
.w
o
rd
62 Data Table

p
re
Directions (Qs. No. 8-12): On the basis of the information given below.

s
s
.c
In a Class X Board examination, ten papers are distributed over five Groups of PCB, Mathematics, Social Science,

o
m
Vernacular and English. Each of the ten papers is evaluated out of 100. The final score of a student is calculated
in the following manner. First the Group Scores are obtained by averaging marks in the papers within the Group.
The final score is the simple average of the Group Scores. The data for the top ten students are presented below.
(Dipan’s score in English Paper II has been intentionally removed in the table).

Social
Vernacular English
PCB Group Science
Mathe Group Group
Group Final
matics
Name of Score
Group Paper Paper Paper Paper
the Phy. Chem. Bio. Hist. Geo.
I II I II
student
Ayesha (G) 98 96 97 98 95 93 94 96 96 98 96.2
Ram (B) 97 99 95 97 95 96 94 94 96 98 96.1
Dipan (B) 98 98 98 95 96 95 96 94 96 ?? 96.0
Sagnik (B) 97 98 99 96 96 98 94 97 92 94 95.9
Sanjiv (B) 95 96 97 98 97 96 92 93 95 96 95.7
Shreya (G) 96 89 85 100 97 98 94 95 96 95 95.5
Joseph (B) 90 94 98 100 94 97 90 92 94 95 95.0
Agni (B) 96 99 96 99 95 96 82 93 92 93 94.3
Pritam (B) 98 98 95 98 83 95 90 93 94 94 93.9
Tirna (G) 96 98 79 99 85 94 92 91 87 96 93.7

Note: B or G against the name of a student respectively 11. Each of the ten students was allowed to improve
indicates whether the student is a boy or a girl. his/her score in exactly one paper of choice with
8. How much did Dipan get in English Paper II? the objective of maximizing his/her final score.
(a) 94 (b) 96.5 Everyone scored 100 in the paper in which he or
(c) 97 (d) 98 she chose to improve. After that, the topper among
the ten students was:
9. Students who obtained Group Scores of at least
95 in every group are eligible to apply for a prize. (a) Ram (b) Agni
Among those who are eligible, the student obtaining (c) Pritam (d) Dipan
the highest Group Score in Social Science Group is
12. Had Joseph, Agni, Pritam and Tirna each obtained
awarded this prize. The prize was awarded to:
Group Score of 100 in the Social Science Group,
(a) Shreya (b) Ram then their standing in decreasing order of final
(c) Ayesha (d) Dipan score would be:
10. Among the top ten students, how many boys (a) Pritam, Joseph, Tirna, Agni
scored at least 95 in at least one paper from each of
the groups? (b) Joseph, Tirna, Agni, Pritam
(a) 1 (b) 2 (c) Pritam, Agni, Tirna, Joseph
(c) 3 (d) 4 (d) Joseph, Tirna, Pritam, Agni
y
o
u
rs
m
a
h
b
o
o
b
.w
o
rd
Data Table 63

p
re
s
s
.c
o
m
Concept Applicator 12. (b) Average of all the semester fees of M.Sc

Solutions from 1-5: Answer can be easily obtained 5800 + 6400 + 10200 + 14600 + 17700 + 20900
=
from the table. 6
1. (d) As we follow the table, we can observe that 75600
= = 12, 600
the rent of all the cities gradually increases 6
except virar, whose rent fluctuates from 2005 13. (d) Total fee charged for diploma course over
to 2010. all the years is 42.8 and btech is 35.8.
2. (a) If we observe that the Churchgate, monthly 42.8 – 35.8 = 7 i.e. 7000
rent increases more than 100% in 2006. 14. (d) Fees charged for M Phil course in 2008 is 12.7
3. (b) and M Sc in 2009 is 17.7
4. (d) In 2008 kandiwali was 9.8. virar = 21.9, 12.7
× 100 = 71.75% i.e. 72 (approx.)
required % = [9.8/21.9] × 100 = 44.7% or 45% 17.7
5. (a) from the table we can observe that cruchgate 15. (b) Total fee charged = 14.5 + 6.4 + 11.6 + 5.8 + 3.2
was most expensive. = 41.5 i.e. 41.500
Solutions from 6-10: Solutions from 16-21:
6. (a) In 2008, Number of boys were 5.5 (in 16. (d) In 1994 computer science constituted more
hundreds). In 2007 number of girls were 6.4 than 25% i.e. 25% of 60 = 15 and it is 20.
(in hundreds) 17. (b) It is clearly visible from the table that computer
1.1 science & law showed the greatest difference.
Difference × 100 = 17 (approx.)
6.4 18. (c) in the year 1993 the percentage is highest as
OR 25
compared to the rest three ×100 = 35.21%
We know that 20% of 64 is 12.8 which is 71
much higher than 11. So, it must be less than 19. (c) It is clearly visible that the greatest rate of
20% and more than 15% (by observation). So, 8−3
increase is in computer science × 100 =
only one answer satisfies i.e. 17. 166.67% 3
7. (d) Number of girls in B in 2009 = 5.9 20. (d) It can be calculated form the table that
Total student in E in 2006 = 9.1 commerce discipline total is equal to the total
5.9 of the year 1993.
So required % = × 100 = 64.835
9.1 21. (b) It is visible from the table that there is no
i.e. approximately 65. change in the science discipline.
5160 Solutions from 22 – 27:
8. (c) Add all the girls = 860
6 125 –105
8.7 29 22. (b) ×100 = 20 (approx.)
9. (c) Ratio = = 105
12.9 43 23. (c) It is visible that monopoly was continuously
10. (d) It can be clearly answered from the table. increasing over the years.
Solutions from 11–15: 200 − 180
24. (d) × 100 = 10% .
11. (c) Required % 200
13.9 − 11.6 2.3 25. (a) Ludo has the maximum total production of
= × 100 = × 100 = 20% 945 toys in all the years together.
11.6 11.6
y
o
u
rs
m
a
h
b
o
o
b
.w
o
rd
64 Data Table

p
re
26. (a) 100/70 = 1.42 × 100 = 142 % 52. (c) If the growth is less than the demand may

s
s
.c
27. (a) The ratio is highest for Chess i.e. 1.67 increase due to increased population.

o
28. (b) HCI encountered the highest occupancy rate 53. (b) The demand increase of three times exactly is

m
in 1999. in vegetables i.e. 3 × 56 i.e. 168.
29. (b) Is true. The average occupancy rate in 1997 54. (c)
was 64 % and 1998 was 63.71%
30. (d) All the statements were false. Concept Builder
31. (c) The rate of growth is highest in 1999. 1. (c) Let Cost of purchasing x gm of food A, y gm
32. (c) It is visible that Black Lagoon and Radiant of food B and z gm of food C (convert it into
won this award atleast twice. 1.80 3 2.75
one gram) = x + y +
33. (d) Shop T has the lowest sales of 920 and 280 in 100 100 100
both B and E. 0.9 0.3 0.11
= x + y + z
34. (d) 15% of 4000 = 600 50 10 4
35. (d) S has the highest sale of cars 13670. 2. (d) From the graph, Protein in 500 g of A
36. (c) Both P & Q sells 300 × 7 = 2100 & 440 × 7 = 3080. = 500 × 10% = 50
37. (b) 250 g of B = 250 × 20 % = 50
38. (d) It is clearly visible that Chamera I (HP) has 350 g of C = 350 × 20 % = 70
the maximum no. of tress effected. 200 g of B and 200 g of C = 200 × 20 % + =
39. (a) Dulhasti (J & K) has minimum forest area 200 × 20% = 80
involved i.e. 1.1 Hence (d) provides the maximum protein.
1.8
40. (b) Chamera I (HP) has the maximum density of 3. (a) The cost of 300 g of A = ×300 = 5.40
the trees planted. 100
41. (a) 2.75
The cost of 200 g of B = ×200 = 5.5
42. (a) The maximum reduction in per capita 100
consumption took place in USSR i.e. 95 The cost of 150 g of A and 100 g of C
43. (b) Japan is excluded as it will increase than the 1.8 3
= ×150 + ×100 = 5.7
USSR Production. 100 100
44. (d) USSR has the adverse decline of 95 in the 500 g of B and 100 g of A
consumption of cement. 3 1.8
= × 500 + ×100 = 16.8
45. (b) USSR had 15 times more per capita 100 100
consumption i.e. 32 × 15 = 480 and USSR has The cost of 300 gm of A is least.
483 in the year 1978. 4. (b) (2 + 7 + 6 + 5 + 5 + 2 + 3) = 30
46. (c) Rural has the highest percentage of voters i.e. 5. (c) (2 + 7) + (5 + 2) = 16
71% 6. (b)
216
47. (b) × 100 = 53% (approx..) 7. (d) Only 44 % of students score less than 70%.
408
8. (c) Following table provides the Production of all
48. (a) 70% of 162 million = 113,400,000
the units year wise.
49. (d) (100 – 63)% = 37% didn’t vote. Therefore,
1998 1999 2000 2001 2002 Total
37% of 60 million = 22,200,000
50. (b) Unit 1 140 180 195 260 340 1115
51. (b) Meat, Fish and Eggs demand in the year 2000 Unit 2 150 193 203 268 351 1165
is 6.2 4 times of 6.2 is 24.8 and in 2020 it is Unit 3 156 194 200 272 350 1172
more than 24.8 i.e. 27.0
y
o
u
rs
m
a
h
b
o
o
b
.w
o
rd
Data Table 65

p
re
it is clear that performance of unit 1 is the III. % increase in the number of units sold

s
s
.c
lowest and that of unit 3 is the highest during for sony (95 – 99) is highest

o
the 5 yrs period. 50000 − 12500 ×

m
9. (b) Table below give full idea = 100 = 300 %
12500
Total Production in 1st quarter 764 IV. The average annual sale of Samsung is
Total Production in 2nd quarter 971
less than that of Sony.
15. (a) By following the table, we can answer the
Total Production in 3rd quarter 893 question.
Total Production in IVth quarter 824 Highest
Least annual
Total 3452 Brand annual
capacity
name capacity
utilisation
10. (c) From the table in question 150 growth unit utilisation
wise during (1998 − 2002) is given below 12500 50000
340 − 140 × × 100 × 100
Unit 1 = 100 = 142 % Sony 75000 75000
140 = 16.67 % = 66.67 %
351 − 150 × LG 40 % 56 %
Unit 2 = 100 = 134 %
150 Samsung 22.67 % 44 %
350 − 156 × National 20 % 40 %
Unit 3 = 100 = 124 %
11. (b) 156
Toshiba 17.7 % 24.83 %
12. (d) below the table give us clear idea.
16. (b) Total Sale of TV in 1999 = 148000
Year Average Growth
Production Total Sale of TV in 2000
1998 111.50 − = 148000 × 1.25 = 185000
1999 141.50 27.13% Sale of Toshiba in 1999 = 16000
2000 149.50 5.46% Sale of Toshiba in 2000 = 16000 × 0.9 = 14400
2001 200.00 33.11% 14400
Required % = × 100 = 7.8 %
2002 260.25 30.12% 185000
17. (d) National Panasonic shows a decrease of sales
13. (a) Average annual sale of
30000 − 15000
186000 by 50 % = × 100 = 50 %
LG = =37200 30000
5 On the similar way decreasement process
157500 30 − 25
Sony = = 31500 follows % decrease in 1996 = × 100
5 30
112000 = 16.6 %
National Panasonic = = 22400 25 − 22 ×
5 % decrease in 1997 = 100 = 12 %
25
78000
Toshiba = = 15600 22 − 20
5 % decrease in 1998 = × 100 = 9.09 %
22
14. (a) I. LG shows increase and decrease in sales
20 − 15
during 5 years. % decrease in 1999 = × 100 = 25 %
20
II. Samsung has recorded a fall in sales only Maximum % decrease is shown during
twice. (1998–99)
y
o
u
rs
m
a
h
b
o
o
b
.w
o
rd
66 Data Table

p
re
Solutions from 18-23 29. (b) Total repaid amount over 25 years

s
s
.c
Year = 25 × 12 × 12650 = ` 3795000

o
Banks Total
30. (b) Reduction = (13500 – 12500) × 20 × 12 =

m
1990 1991 1992 1993 1994
` 240000
A 18 23 45 30 70 186
31. (b) Amount payable for 15 years on a loan of
B 27 33 18 41 37 156 ` 1000000 = 15 × 12 × 10300 = 1854000
C 29 29 22 17 11 108 Amount payable for 10 years on a loan of
D 31 16 28 32 43 150 ` 1500000 = 10 × 12 × 19400 = 2328000
Difference = (2328000 – 1854000)
E 13 19 27 34 42 135
= ` 474000
Total 118 120 140 154 203 735 32. (d) Number of emergencies attended by 6
substations in the month of June and October
18. (a) Average distribution of loan during the years
735 was same and equal to 94.
(1990 -1994) = = 147 crore 33. (a)
5
Clearly from the table, in 1990 it was least 34. (b) Substation B attended the maximum number
118 as compared to average 147 crore of complaints and is equal to 109.
154 − 140 35. (d) These two months are July and September
19. (c) % increase × = 100 = 10%
140 211
20. (d) × 100 = 36.56 %
577
21. (b) Total loan disbursed in the year 1991 = 120 crore
36. (b) Clearly from the table maximum percentage
Therefore target for each bank = 120 × 20 % increase in foreign tourist arrivals is in the
= 24 crore. It is clear that only banks B and C year 2000-2001.
reached the target.
37. (b) Increasing from 1995-96 to 2000-2001.
22. (a) Total loan disbursed in 1994 = 203 crore
Loan disbursed by bank A = 70 crore 38. (a)
39. (b) Total number of new employees = 126 + 131
∴   % = 70 × 100 = 34.45 % = 257.
203 5 + 9 + 74 + 8 + 4 100
85 40. (c) Required average = =
23. (a) Required % = × 100 = 72 % 5 5
118 = 20
487
24. (c) Average value of six commodities =
6 41. (b) Required ratio = 66 : 11 = 6 : 1
= 81.17 in 1999
451 Solutions from 42-45
Average value of six commodities =
6 News Kerela Punjab UP HP Total
= 75.17 in 2003
A 123 227 96 78 524
∴   Required average difference = – 6
25. (a) B 105 220 117.2 97 539.2
26. (d) C 12.2 14.6 9.7 17.2 53.7
27. (d) D 82.4 44 145 9.3 280.7
28. (d) Amount paid in 15 years = 180 × 20600 = E 24.4 23 10 100 157.4
` 3708000 42. (b) Clearly news paper B has the highest number
Amount paid in 20 years = 240 × 18000 = of circulation.
` 4320000 43. (b) Difference = (539.2 – 524) = 15.2 thousand
∴   ` 612000 more will be paid. = 15200
y
o
u
rs
m
a
h
b
o
o
b
.w
o
rd
Data Table 67

p
re
227 58. (d) Percentage of domestic consumption -

s
∴  x = = 2.34 ≈ 2.35

s
44. (a) x × 97 = 227

.c
97 0.58 ×
2000-01 = 100 = 19.26

o
45. (d) Required ratio = 44 : 9.3 = 440 : 93 = 5.5 : 1.16 3.01

m
46. (c) From 69 and below it is mentally defective. 0.60
2001-02 = × 100 = 19.60
47. (a) From 120 and above it is superior and very 3.06
superior . 0.55 ×
1996-97 = 100 = 26.82
48. (c) 2.05
49. (c) 50% is the highest % which is classified in 0.54
1990-91 = × 100 = 31.76
average category. 1.70
50. (d) Clearly smallest percentage of cases belong to 59. (a) The largest quantity (2.47 lakh tones) of
both of these categories coffee was exported in 2000-01 and the
51. (a) From the table it is clear that there has been a highest foreign exchange earned in dollars
increase in the number of sick units between (477 million) in the year 1197-98.
1989 to 1991. 60. (d) Proportion in the total energy demand in Asia
52. (d) All the three sectors cotton textiles, petroleum in 2005,2010 and 2020 :
and paper have shown an increase in the Natural Gas: 7.89 %, 735 %, 9.37 %
number of sick units 1989 – 1991 61. (c)
53. (d) % increase in number of sick unit in 62. (d) The demand in 2020 as a ratio of demand in
16149 − 15308 2005 in the Asian region for :
Electricity = × 100 = 5.49 % 4.5
15308 Natural gas = = 3
24941 − 19474 1.5
Engineering = × 100= 28.07 % 5.5
19474 Nuclear = = 2.75
6075 − 4123 2
Cotton textiles = × 100 = 46.90 % 15.4
4123 Solid fuels = = 2.56
6
Thus, cotton textiles shows the max. percentage 4.6
Hydropower = = 3.06
increase in number of sick units. 1.5
Solutions from 54-57 63. (c) Required proportion for :
Solid fuels = 25.69, 19.25
News Kerela Punjab UP HP Total
Hydropower = 6.71, 5.31, 4.88
A 123 227 96 78 524 64. (d)
B 105 220 117.2 97 539.2 65. (b) Only in shapes category the demond exceed
C 12.2 14.6 9.7 17.2 53.7 availability (9745–9360) = 385
D 82.4 44 145 9.3 280.7 66. (d) Required percentage growth rate
50
E 24.4 23 10 100 157.4 = × 100 = 1.25
400
54. (b) Clearly news paper B has the highest number 67. (c) Shortfall of shapes in 1999-2000 = 6960 –
of circulation. 5725 = 1235 thousand tonnes Shortfall of
55. (b) Difference = (539.2 – 524) = 15.2 thousand shapes in 2003-2004 = 9745 – 9360 = 385
= 15200 thousand tonnes changes in the shortfall of
227 shapes from 1999-2000 to 2003-04 = -850
56. (a) x × 97 = 227  ∴   x = = 2.34 ≈ 2.35
97 thousand tones. Percent change in shortfall of
57. (d) Required ratio = 44 : 9.3 = 440 : 93 = 5.5 : 1.16 850
shares – × 100 = –68
1235
y
o
u
rs
m
a
h
b
o
o
b
.w
o
rd
68 Data Table

p
re
68. (a) 63 + 18 + 21 : 15 + 5 + 1.5 = 102 : 21.5 72. (b) No. of application who applied for 3100 –

s
s
= 1020 : 215 = 204 : 43 ≈ 19 : 4 10000 shares = 1633 × 6 = 9798

.c

o
69. (b) 15 + 5 + 1.5 to 47 + 17 + 13 i.e 21.5 to 67 No. of applicants who applied for 10200 –

m

 Growth of the average population of 404 × 5
21000 shares = = 1010
LMVs = 45.5, 2
No. of applicants who applied for 25000
45.5
i.e × 100%, i.e 212% shares = 11
21.5 ∴   Total number of applicants who applied
70. (d)
71. (c) Italy : 239%, UK : 213%, Canada : 240%, for 3100 − 25000 shares = 9798 + 1010 + 11
Switzerland : 100% = 10819
73. (c) Average number of shares allotted to an allotte
100 × 8001 + 100 × 7624 + 200 × 6202 + 200 × 1515 + 200 × 1633 + 300 × 404 + 350 × 11
8001 + 7624 + 6202 + 1515 + 1633 + 404 + 11
800100 + 762400 + 1240400 + 303300 + 326600 + 121200 + 3850 355750
= = ≈ 140
25390 25390
74. (a) Total No. of applicants who applied for 1000 – 7. (b) Mr. Kakkar becomes a Gold member on
1515 × 28 20.3.2007 and he becomes a Platinum
3000 Shares = = 505 × 28 = 14140 member on 31.12.2007. Hence he enjoys the
3
Total No. of applicants who applied for 10200 Gold membership for 9 months,
404 × 5 8. (e) He completed his 1st tier in February(2)
– 21000 Shares = = 1010
2 Last tier in 20th trip in November(11).
14140 Difference Between this two = 8 months.
∴   Required ratio = = 14
1010 9. (d) Had there been no 6 – month system, then Mr.
75. (a) Total subscription = (100 + 100 + 200 + 200 + Kakkar membership, after completing 20,000
200 + 300 + 350) × 100 = 145000 Queen Miles which happens in the month of
∴   Oversubcription = ( 1,45,000 – 1,00,000) September.
= ` 45,000 10. (d) In 2004 PAT = (100 × 6)/100; In 2005 PAT
Concept Cracker = (119.9 × 6.9)/100 Chage in % = [{(119.9 ×
6.9)/100 – (100 × 6)/100}/(100 × 6)/100] × 100
1. (c) Growth in GDP based on PPP valuation in
= 37.8%
2003 = 9.16%
2004 = 8.85% , 2005 = 9.48% and 2006 = 8.76% 11. (a) Salaries and Wages change In respect of
2. (b) For inflation we use price induces. Rate of previous year is maximum in2001. As Change
inflation in 2003 = 3.8 in value in respect of 2000 is minimum than
2004 = 3.76, 2005= 3.87, 2006 = 5.09 others year. So Ratio of change value with
3. (d) Dose not tells about the growth of population. previous year is also minimum .Annual
4. (d) No data regarding the world GDP. growth rate must be maximum in 2001.
5. (d) Ambiguity in currency valuation of GDP 12. (a) If we carefully observe the chart among
all commodities some product changed
6. (a) Mr.Kakkar get guaranteed reservation ship uo
maximum with respect of previous year. We
to 24 hrs ,when he reach Gold tier.To reach
don’t need to find each ratio, from observation
there he need to cover 12500 km or 5 journey
petroleum and engineering good changed
with in restricted period.
maximum.So,3rd commodity must be Ore
He covered his 5th journey in March and
and Minerals.
Total distance covered is less than 12500.
y
o
u
rs
m
a
h
b
o
o
b
.w
o
rd
Data Table 69

p
re
13. (b) Grand total in both 2004-05 and 2005-06 18. (a) From the given table we can conclude that-

s
s
.c
are 100. Difference of two years value in Total sales of Television and IPods in

o
individual sector indicates contribution in (Thousand) = 31500 + 100800 = 132300

m
growth. Here petroleum Products shown It is given in this question that the surcharge
maximum change = (11.21 – 8.57 ) = 2.64 paid is 14% of the total sales
14. (d) Among 5 commodities bulk imports shows Hence Surcharge paid in (Thousand) =
maximum import in two years = (39.09 + 132300 × 0.14 = 18522
42.56) = 81.62 19. (c) From table we can find that-In the month of
15. (b) Difference in 2004-05 imports minus export = January, Sales of Television in (Thousand)
(from Bistupur + Kadma) = 900 + 6300 =
(501064.54 – 375339.53)
7200 and sales of IPods in (Thousand) (from
Difference in 2005-06 imports minus export Bistupur + Kadma) = 15750 + 6300 =22050
= (630526.8 – 454799.97) Hence, Now 20% of the sales of Television in
% of change = [{(630526.8 – 454799.97) – Thousand = 7200 × 0.20 = 1440
(501064.54 – 375339.53)}/ And 10% of the sales of IPods in Thousand =
(501064.54 – 375339.53)] × 100 = 39.77% 22050 × 0.10 = 2205
16. (a) In 2005 – 06, export of petroleum products = Hence Profit earned by the company in the
0.1121 × 454799.97 = 50983.08 cr & import month of January from Bistupur and Kadma
of petroleum crude and products = 0.3087 × = 2205 + 1440 = 3645
63.526.77 = 194643.61 cr rupees. 20. (a) It is given that-Units ordered = Units sold +
⇒ yearly difference = (50983.08 – 194643.61) Ending Inventory – Beginning Inventory
= 143660.53 cr In 2004 – 05, export of Units Sold = 7560 – 6120 + 5760 = 7200
petroleum products = 0.0857 × 375339.53 = Total sales from I-Pods (thousand) = 15750
32166.59 crore rupees & import of petroleum + 16800 + 17850 + 9450 + 10080 + 10710 +
crude and products = 0.2787 × 501064.54 = 6300 + 6720 + 7140 = 100800
139646.68 cr yearly difference = (32166.59 Sales price per unit = Sales/Units hence
– 139646.68) =107480.09 cr Ratio = (yearly Sales price of IPod during this 3 month period
difference in 2005 06)/(yearly difference in in Thousand =100800/7200 =14
2004-05) = 1.336 21. (b) In a month of February, 1050 units of
17. (e) It is given that-Units ordered = Units sold + Television and 2400 units of IPods were sold
in all three areas.
Ending Inventory – Beginning = 3150 + 2880
From the table sales from Television in all 3
– 2520 = 3510
areas in February = 10500
Total sales of Television in Thousand
And it is given that 1050 units were sold
(Revenue) = 900 + 1800 + 6300 + 1050 +
hence we can calculate the selling price per unit.
2100 + 7350 + 1200 + 2400 + 8400 = 31500
Sell price of Television per unit in (Thousand)
Sales price per unit of television in Thousand sales/unit = 10500/1050 = 10
= Sales/Units Sold = 31500/ 3150 = 10 Similarly price of IPod per unit in Thousand =
It is given that profits made is 25% , and we 33600/2400 = 14
have calculated S.P =10 In the month of January total sales (Thousand)
Hence C.P (in Thousand) = 10/1.25 = 8 = 9000
The value of order = unit ordered × C.P per Hence no of Units of Television sold in the
unit = 3510 × 8 = 28080 month of January = 9000/10 = 900
y
o
u
rs
m
a
h
b
o
o
b
.w
o
rd
70 Data Table

p
re

Similarly no of Units of IPods sold in the month of January = 31500/14 = 2250

s
s
.c

We know that Units ordered = Units sold + Ending Inventory – Beginning Inventory

o

Hence For Television: Units ordered = 900 + 840 – 720 = 1020

m

And For IPod: Units ordered = 2250 + 1920 – 1800 = 2370
22. (e) Given table is-
Drop out Rates, in Percentage, at Primary, Elementary and Secondary Classes in India.
Primary Elementary Secondary
Year
(I-V) Classes (I-VIII) Classes (I-X) Classes
Boys Girls Total Boys Girls Total Boys Girls Total
1996-97 39.7 40.9 40.2 54.3 59.5 56.5 67.3 73.7 70.0
1997-98 37.5 41.5 39.2 53.8 59.3 56.1 66.6 73 69.3
1998-99 40.9 41.3 41.5 54.2 59.2 56.3 64.5 69.8 66.7
1999-00 38.7 42.3 40.3 52.0 58.0 54.5 66.6 70.6 68.3
2000-01 39.7 41.9 40.7 50.3 57.7 53.7 66.4 71.5 68.6
2001-02 38.4 39.9 39.0 52.9 56.9 54.6 64.2 68.6 66
2002-03 35.8 33.7 34.8 52.3 53.5 52.8 60.7 65.0 62.6
2003-04 33.7 28.6 31.5 51.9 52.9 52.3 61.0 64.9 62.7
2004-05 31.8 25.4 29.0 50.4 51.2 50.8 60.4 63.8 61.9


As we can see from the given table, none of the first four option is correct hence option E is correct.

23. () Data given doesn’t match with the data given Statement 3: It states that the worth of
in table. leftover for savories and cakes/pastries/
24. (d) It is a direct question that requires just gateaux was highest in 2004. From the given
calculation. From the table we can calculate table we can conclude that leftover is highest
that D is the correct option. in 2004 hence this statement is true.
25. (d) From the table we can calculate that D is the Statement 4: It states that worth of left over
correct option. for savories kept on fluctuating many times
26. (d) We will evaluate the statements one by one- between 1993 to 2004. But, from the given
Statement 1: It states that leftover for cakes/ table we can conclude that it is fluctuating
pastries/gateaux increased from 1993 to 2004, only in 1999 and 2000 hence it is false.
but from the given table we can conclude that 27. (c) We can observe that the maximum decline is
it decreased in the year 1999 and 2000. Hence from 1998 to 1999 which is 4.65, hence
it is false. C is correct.
Statement 2: It states that the worth of 28. (a) In the given table we can observe in the last
leftover keeps on fluctuating many time, but column that there were losses in the years
from the given table we can observe that the 1993, 1997, 1998 and 2000.
leftover increases from 1993 to 1998 then 29. (c) In January, Total expenditure on roses and
decreases in 1999 and 2000 and again starts carnations = 47.4 × 70,000 = 3318000
increasing from 2001 to 2004 so statement 2 And expenditure on carnations = 3318000 –
is also false. 1136916 = 2181084
y
o
u
rs
m
a
h
b
o
o
b
.w
o
rd
Data Table 71

p
re
In March total expenditure on roses and

s
year avg. emission

s
.c
carnation = 49.5 × 70,000 = 3465000
2000 64.6

o
Expenditure on carnation = 3465000 –

m
1137915 = 2327085. Change in expenditure 2001 46.5
on carnations from January to march 2002 40.9
= 2327085 – 2181084 = 146001 2003 39.3
Hence Percentage change on carnations
= 146001/2181084 x 100 = 6.69% The difference is maximum for the year 1998
and 2003.
30. (b) For the month of January, Total consumer
expenditure on roses = ` 1136916 37. (c) Emission level in the years 2001, 2002 &
2003 is less than in any years from 1997 –
Price of roses per dozen = ` 99
2000. So, The average level of SO2 emission
Hence number of roses sold = (1136916 ÷ 99)
during 1997 – 2000 is higher than the average
= 11484 dozens
annual level for the city for the period from
Similarly, for the month of July, number of 1997 – 2003.
Roses sold = 8253 dozens
38. (a) If we go with options we have difference
So, percentage decrease in the sales between cochin and pondichery is more
= {(11484 − 8253)/11484}100 = 28.13%
39. (b) The annual emission level decline is maximum
31. (d) From the solution of previous question, we during in 2002-2003
have, for the month of January, the total
40. (a)
expenditure on carnations = ` 2181084 and
Carnations sold = 13848 dozens Region Iron and steel increase
Price of carnations per dozen = (2181084 ÷ 2003 2004
13848) = ` 157.5 per dozen Intra europe 27 45 18
Similarly, for the month of December, The Intra asia 31 42 11
total expenditure on carnations = ` 2970312
Europe to north
Carnations sold = 18859 dozens –9 81 90
america
Price of carnations per dozen = ` 157.5 per
Intra north
dozen. Thus, we can say that the price did not america
4 41 37
change.
Europe to asia 0 16 16
32. (d) From the data we observed that China is first,
than Cambodia than third one is Vietnam. 41. (b) Rank 1 = europe to north america (export) = 90
33. (c) C is not true because it exhibits 28.2% which rank 2 = intra europe (export) = 90
is highest in 2001-2002. 42. (a) The difference between the highest and lowest
34. (d)
D is correct because difference between average export growth rate during 2005 is 40.
growth of china and Japan is 23.3% 43. (b) Europe to north america export(iron and steel)
35. (d) During 1997 – 2003 yamuna nagar has 23.94 = 31
average annual level of SO2. 44. (b) After finding out the ratio of special type of
36. (a) The highest average annual level of SO2 wagon to total we will find out that special
emission on different year is as in the table type wagons expressed as a percentage of
year avg. emission total wagons is maximum during 2003
1997 112.3 45. (c) In 2000 special type wagons to total wagons
1998 114.9
in % = (40316/ 234397) × 100 = 17.19%
In 2002 special type wagons to total wagons
1999 93.3
in % = ( 37275/216717) × 100 = 17.19%
y
o
u
rs
m
a
h
b
o
o
b
.w
o
rd
72 Data Table

p
re
46. (c) In 2004 departmental wagons was maximum 1. (c) From the table we can conclude that Profit

s
s
.c
i.e (11388/227752) × 100 = 5% rate of Paharpur Cement is more than the

o
47. (a) Annual growth rate(AGR) = wagons no. Of profit rate of Bahsin Cement in Oct-Dec 2008

m
the year/ summation of the total years only
AGR for total wagon = –11037/263981 × 100 2. (d) We will calculate profit for both company.
= –4.18% For Jan- Mar 2009,
AGR for covered wagons in 1998 = –4.14% Paharpur Cement
AGR for special wagons in 2002 = –4.13% Sales Quantity = 543278
AGR of total wagon in 2000 –4.11% Cost (as a percentage of revenue) = 87.56 %
48. (c) From the table we can observe that the annual Hence Profit = 100 − 87.56 = 12.44%
percentage growth rate of average wagon 12.44
∴  Profit = × 543278 × 304
capacity was not maximum in 1999. 100
49. (b) From the table we will get the AGR of total = 20545470
wagons in 1995 = – 6.7% Profit Rate = 37.82
AGR of covered wagons in 2002 = – 5% For Bahsin Cement
AGR of open low sided wagon in 1998 100 − 91.34
= – 8.25% Similarly, Profit = × 526532 × 440
100
AGR of departmental wagon in 2000 = 7.33% = 20062975.3
Hence Profit Rate = 38.10
Concept Deviator
Now For April - June 2009
Solutions from 1 – 3 : We have been given the cost
Paharpur Cement
as a percentage of sales revenue of both the factories
Similarly Profit rate = 30.5
Let SR be the sales revenue, SQ be the sales quantity
In the same way Profit Rate of Bashin Cement
and C be the cost
= 43.17
Profit S.R. S.R. − C
Profit rate (P.R) = = = 3. (d) Let the sales quantity of Paharpur cements in
S.Q. S.Q. S.Q. Apr-June 2009 is 100 units.
S.Q. × Price − C C Then sales quantity in July-Sept 2009 is
= = Price – …(1)
S.Q. S.Q. 102.25. Now we can tabulate the information
 
as follows.
But C = S. R’ % = Price ‘S.Q’ % …(2)
April- June July - Sept
From equations (1) and (2).
Price × S.Q. × % Sales Revenue 34000 32924.5
P.R = Price –
S.Q. Cost 309502 29770.33
From this we can complete the table Profit 3049.8 3154016
Profit Rate Profit Rate
of Paharpur of Bahsin From the table we can observe Change in
Cement Cement profit = [(3154.16 – 3049.8)/3049.8] × 100
= 3.42%
October-December 2008 27.78 25.36
Thus profit is increased by 3.42%.
January-March 2009 37.82 38.1 4. (c) In January, Total expenditure on roses and
April-June 2009 30.5 43.17 carnations = 47.4 × 70,000 = 3318000
July-September 2009 30.85 41.75 And expenditure on carnations
= 3318000 – 1136916 = 2181084
y
o
u
rs
m
a
h
b
o
o
b
.w
o
rd
Data Table 73

p
re
In March total expenditure on roses and 12. (b) Complete the table from the given information

s
s
.c
carnation = 49.5 x 70,000 = 3465000 and we will get-

o
Expenditure on carnation = 3465000 – 1137915

m
House hold as
= 2327085 House Total oil
Year percentage of
hold used
Change in expenditure on carnations from total oil used
January to march = 2327085 – 2181084 1998 22 2870 0.766
= 146001
1999 22 2834 0.776
Hence Percentage change on carnations
2000 20 2120 0.684
= 146001/2181084 x 100 = 6.69%
2001 20 3262 0.674
5. (b) For the month of January, Total consumer
expenditure on roses = ` 1136916
From the table we can observe that it is
Price of roses per dozen = ` 99 highest in the year 1999
Hence number of roses sold = (1136916 ÷ 99) 13. (a) As we know that Oil production loss for
= 11484 dozens different year is given and also the total oil
Similarly, for the month of July, number of production hence we can calculate their ratio.
Roses sold = 8253 dozens
Oil Total oil
So, percentage decrease in the sales Year production produced L:T
= {(11484 − 8253)/11484}100 = 28.13% loss (L) (T)
6. (d) From the solution of previous question, we
2002 386 4044 0.095
have, for the month of January,
2003 444 3944 0.1125
The total expenditure on carnations =
` 2181084 and Carnations sold = 13848 dozens 2004 506 4328 0.1169
Price of carnations per dozen = (2181084 ÷ 2006 1142 4968 0.2298
13848) = ` 157.5 per dozen
From the table we can observe that the lowest
Similarly, for the month of December, ratio is for the year 2002
The total expenditure on carnations =
14. (d) To compare the ratio of oil used by suburban
` 2970312 Carnations sold = 18859 dozens
to that by total oil used we will form the table
Price of carnations per dozen = ` 157.5 per as follows-
dozen
Thus, we can say that the price did not change. Oil used by Total oil
Year S:T
sububan (S) used (T)
7. (c) Jower yield in 2007 = 368/673 and Soyabean
in 2008 = 799/650 2005 230 3618 0.06357
Their ratio be nearly 0.445. 2006 210 3826 0.05488
8. (b) After calculating the yield for crops, we get 2007 254 4030 0.06302
the top three crops are sunflower, ground nut 2008 266 4152 0.06406
and rice.
From the above table we can observe that the
9. (a) The bottom three crops by yield are moth,
ratio is highest for the year 2008.
seasamum and millets.
15. (d) The growth rates in production of oil are more
10. (c) Total productivity of pulses has increased and
than that of total oil used during the years
decreased.
1997 to 2007.
Statements II and III are correct.
16. (d) We can observe from the data that the total oil
11. (a) Statement: I and III are correct.
produced is increasing every year since 2003.
y
o
u
rs
m
a
h
b
o
o
b
.w
o
rd
74 Data Table

p
re
17. (c) From the table we can observe that Bangladesh 21. (d) We will evaluate each of the statement

s
s
.c
for Drinking Water and Philippines for Statement A – It is true as 81 (India) < 97

o
Sanitation facilities are on the coverage

m
(Bangladesh)
frontier.
Statement B – It is true as 29 (India) < 63
18. (b) From the table we can observe that India and
China both dominates Nepal while China is (Sri Lanka)
dominated by India. Statement C – It is true as 29 (India) < 47
19. (c) Considering urban population to be x and rural (Pakistan)
population to be y, we get the equation---- Statement D – It may or may not be true. True
(70x+14y=29(x+y)). From this equation we for Sanitation and false for drinking water.
get (y/x=41/15) and hence (y/(x+y)=41/56)).
Hence (y/(x+y)) in percentage comes as 73%.
So, some of the required reasons are A, B, C
20. (a) Solving by the same method as above. We will and D.
get the correct order of population is given in 1
Statements A, B and C are correct.

22. (b) Write down the table

1994-95 1995-96 1996-97 1997-98 1998-99

Exports (E) 485+177+6= 668 734+35+6=775 1083+286+14=1383 1750+201+19=1970 2650+4+18=2672

Total business (B) 2041 2886 3807 5031 6052

E/BX100 32.73 26.85 36.32 39.15 44.15

Hence there are two periods 1996-97 and 1997-98

23. (a) Consider the year 1995-96, % increase = 25. (c) Let the age of the employee being transferred
(2886-2041) x 100/2401 = 41.40% from the-
Year 1996-97, % increase = (3807-2886) x Marketing department to the Finance
100 /2886 = 31.9% department be y.
Year 1997-98 % increase = (5031-3807) x Finance department to the Marketing
100 / 3807 = 32.15% department be x.
Year 1998-99 % increase = (6052-5031) x Marketing department to the HR department
100 / 5031 = 20.29% be z.
24. (c) (a) is incorrect because Hardware export
The sum of the ages of all employees in
increases from 1995-96 to 1996-97
Finance originally was 30 × 20 = 600
(b) is incorrect because Peripheral export
decreases from 1997-98 to 1998-99 Hence for finance (600 – x + y)/20 = 31
or x – y = 20 ……. (I)
Now we will check (c),
Training (business volume) = 107 + 143 + Similarly for Marketing = (30x35 – y + x – z)/29
185 + 263 + 302 = 10000 = 35. Or x – y + z = 35 …….(II)
Maintenance (businessvolume) = 142 + 172 From the equation (I) & (II) we will get z =15
+ 182 + 22 + 236 = 953               So average age for HR = (45 × 5 +15)/6
Hence (c) is correct. = 240/6 = 40 Years
y
o
u
rs
m
a
h
b
o
o
b
.w
o
rd
Data Table 75

p
re
26. (c) It is given that the average age of the Marketing values to ensure he doesn’t get any call. By

s
s
.c
department is 35 years and that of the HR reducing the marks obtained in section C to

o
department is 45 years. So, the employee is 41, we ensure colleges 1, 2, 3 & 5 are ruled

m
being transferred from a department with out. Now for colleges 4 & 6, reducing the
a lower average age to one with a higher marks obtained in section D to 43, ensures
average age, which means that he gets an these colleges are also ruled out. Here we are
additional allowance of 10% of basic pay reducing the score to 1 less than the minimum
over his current allowance. So his current cut-off across all colleges for that particular
allowance = 80% of 8000 = 6400 Therefore, section. In the other two sections A and B,
his new allowance = 6400 + 10% of 6400 = Aditya may score 50 each. So, the maximum
6400 + 640 = 7040 After the transfer, his gross possible aggregate marks = 50 + 50 + 41 + 43
pay = 8000 + 7040 = 15040 = 184
Initially, the average gross pay of the HR 29. (b) It is given that Bhama got calls from all
department = 5000 + 70% of 5000 = 8500 colleges, that means she has to score marks
The new average gross pay of the HR in each section equal to at least the maximum
department (i.e. after the transfer of the 40-yr of the cut-offs across colleges which means
old) 45, 45, 46 & 45 in section A, B, C, & D
= [(8500 × 5) +15040]/6 = 9590 respectively. So that cut-off in all the sections
Hence the percentage change in the average in all the institute is covered. Hence total
gross pay of the HR department marks = 45 + 45 + 46 + 45 = 181
= [9590 – 8500]/8500 × 100 = 12.823% 30. (c) The minimum aggregate marks that Charlie
27. (b) Here we need, the percentage change in should score to get call from 2 colleges is
basic pay. According to the data given, only 175 then he can get calls from college 2 and
the allowances (and hence the gross pay) is college 3.
affected when a person is transferred. The Since cut off of section A and D of these
basic pay of a person remains unaltered. two college not given in table, hence we can
∴  The average basic pay after the transfers distribute the 175 in such a manner that in one
have taken place section he will get minimum score.
= [(5000 × 5) + (6000 × 2) + (8000)]/8 Section A-50 Section –B-50
= 45000/8 = 5625 Section C-50 Section D- 25.
∴ The percentage change in the average basic 31. (c) Total cost of angioplasty, hip replacement and
pay of the HR department is given by- a knee replacement (including the cost of poor
(5625 – 5000)/5000 × 100 = 12.5% quality) in the 5 given country,
28. (c) It is given that Aditya did not get a call from India – (11000 + 9000 + 85000) + (5000 +
any of the colleges, so for each college, he 7000 + 9000) = 28500 + 21000 = 49500$
either did not clear one of the sectional cut- Thailand – 35000 + 16000 = 51000$
offs or he did not clear the aggregate cut-off Malaysia – 29000 + 18000 = 47000$
or both. Now if he did not clear one of the Singapore – 38000 + 13000 = 51000$
sectional cut-offs, then for that section he
USA – 140000 + 0 = 140000$
scored less marks than the least cut-off among
the given cut-offs for all the colleges. Since Hence cheapest is in Malaysia
we have to maximize Aditya’s marks, let 32. (d) The total cost of hysterectomy including the
use take the base values of marks in each poor quality cost in India
section and try to reduce that by minimum = 3000 +5000 = 8000$
y
o
u
rs
m
a
h
b
o
o
b
.w
o
rd
76 Data Table

p
re
Total cost of the same in Thailand including

s
Route Distance (Km) Price (`)

s
.c
the cost of poor quality = 10500$
ACGJ 2530 3340

o
The difference = 10500 – 8000 = 2500$ =

m
2500 × 32.89 Baths = 82225 Baths ADJ 2500 3700
Total travelling cost = 7500 + 7500 = 15000 ADFJ 2445 3100
Baths ADGJ 2320 3290
Hence required difference = 82225–15000 = ADHJ 2200 2925
67225, closest is 67500 Baths in 4 AEFJ 3465 4450
33. (a) Cost of knee replacement including the cost of AEGJ 3045 3640
poor quality in the four given countries is
AEHJ 2495 2900
India – 17500 $
AFJ 2315 2850
Malaysia – 12000$
AFGJ 3075 3640
Thailand – 16000$
Singapore – 17000 $ AFIJ 2680 3190
Hence we can conclude that it is most AGJ 2180 3340
expensive in India AGIJ 2320 3540
34. (b) Total cost of spinal fusion in india AHJ 2350 2275
Cost of surgery = 5500 x 40.928/35 = 6432$ AHIJ 3200 3360
Cost of poor quality in India = 5000x40.928/35
= 5846$ Now from the table we can find out the
shortest route from A to J is ACFJ that has a
Total cost in India = 6432 + 5846 = 12278$
distance of 2170Km and price to travel by this
Cost of surgery in Singapore = 9000$
route is 2930.
Cost of poor quality in Singapore = 5000$ 36. (b) The lowest cost from A to J is 2275 with route
Total cost in Singapore = 9000 +5000 =14000$ AHJ,
The difference = 14000 – 12278 = 1722$ Hence company should charge 0.95 × 2275 =
None of the is even close to this one, but if we ` 2161 for the direct flight.
see the question carefully it is not mentioned 37. (c) It is given that airports C, D and H are closed,
in this question (unlike other questions) that then from the table the minimum price will be
the cost of poor quality is to be considered. along the route AFJ and the price is ` 2850.
Hence we will find the difference only in the 38. (b) We have seen that minimum price is ` 2275
cost of surgery for a distance of 2350 Km (with route AHJ)
The required difference = 9000- 6432 =2568$ The minimum price that company incurred =
approximately equal to 2500$ 2275/1.1 = ` 2068
35. (d) From the given data we will find out the So price per Km = 2068/2350 = 0.88/KM
different routes keeping it in mind that not
39. (d) 1- distance 217km, then price per km = 2390/
more than two intermediate stops in any route.
(1.15 x 2170) = 135/115
Route Distance (Km) Price (`) 2- distance 2180km, then price per km =
ABJ 2860 2945 3340/(1.15 × 2180) = 153/115
ABHJ 2710 2995 3- distance -2315 km, then price per km =
ABIJ 3120 3660 2850/(1.15 × 2315) =123/115
ACDJ 2900 4250
4- distance – 2350 km, then price per km
=2275/(1.15 × 2350)= 96/115
ACFJ 2170 2930
5 – distance -2390 km,
y
o
u
rs
m
a
h
b
o
o
b
.w
o
rd
Data Table 77

p
re
40. (d) Given condition is diet must contain Fat -10% 42. (d) The given condition is Carbohydrate ≥ 30%

s
s
.c
and Protein ≥ 30% Protein ≥30%

o
We will eliminate option one by one to get the Fat ≥ 25%

m
correct combination from the given options. Minerals ≥ 5%
1. P and Q is mixed, to get Fat 10% we have Now we will evaluate and eliminate the
to mix them in the ratio of 4:1 and then options one by one.
the protein content would be 110/500 < 1. protein content = (30+20)/200 × 100
30 hence this option is eliminated. = 25% < 30% hence eliminated.
2. P & S, None of them contain Fat hence 2. carbohydrate < 30% hence eliminated
this option also eliminated.
Similarly option 3 doesn’t satisfy the
3. P & R, these two must be mixed in the ratio Mineral content and option 4 doesn’t
of 3:1 to get Fat =10%, and then protein satisfy the Carbohydrate content but
content is 110/400 < 30 hence eliminated option 5 satisfy all the conditions
4. Q & S they must be mixed in the ratio of 43. (a) Here in this question Diet must contain
1:4 to get 10% Fat, in that case protein Minerals 20% and protein ≥30%
content = 230/500 > 30 hence this
The only one combination that gives 10%
combination is allowed and then cost per
minerals is O and Q in 1:1 ratio, no other
unit would be (200 + 4x100)/500 = 6/5
combination can give us minerals 10%
5. R & S, they must be mixed in the ratio of
Solutions of Questions 44 – 47:
1:3 to get 10% fat, then protein content
will be (150 + 50)/400 ½ >30% hence Number of visitors
this combination is also allowed Country DAY
The cost per unit in this case = 800/400 = 2 > 1 2 3
6/5, hence option 4 gives us the minimum cost
Canada 2 0 0
41. (d) Here we want higher % of carbohydrate and
lower cost both of this given by P, it has the Netherlands 1 1 0
highest Carbohydrate % and the lowest cost, India 1 2 0
hence in the mixture P should be maximum UK 2 0 2
1. 2:1:3, P is not the highest hence USA 1 0 1
eliminated
2. 4;1:2, then content of carbohydrate =
Number of visitors
(4×80 + 10 + 2×45)/700 = 420/700 = 0.6
= 60% and cost per unit = (200 + 200 + DAY
UNIVERSITY
200)/700 = 600/7 1 2 3
3. 2:1:4 again p is not the highest hence University 1 1 0 0
eliminated
University 2 2 0 0
4. 3;1;2 Carbohydrate content = (3×80
+ 10 + 2x 45)/600 < 0.6 = 60% hence University 3 0 1 0
eliminated. University 4 0 0 2
5. 4:1:1, carbohydrate content = (4 × 80 + University 5 1 0 0
10 + 45) /600 =375/600 > 60% University 6 1 0 1
Cost per unit = (200 + 200 + 100)/6 = 500/6 University 7 2 0 0
Bust cost in option 4 (i.e 700/6) is more than
University 8 0 2 0
that of option 5 (500/6)
y
o
u
rs
m
a
h
b
o
o
b
.w
o
rd
78 Data Table

p
re
Comparing Table 1 and 2, university 4 corresponds to UK 44. (a)

s
s
.c
and university 6 corresponds to USA (after as day 3 values 45. (c)

o
are concerned and university 8 corresponds to India and

m
46. (b)
university 3 to Netherlands now Indian or Netherlands
can take university 1 or university 5. Now university 47. (a)
2 and 7 belongs to either UK or Canada (only one). 48. (c) Fee per student x Number of students =
UNIVERSITY DAY COUNTRY Revenue
1 2 3 Exp on Faculty per student x Number of
student = Total exp on Faculty
University 1 1 0 0 India / Netherlands

Hence required ratio is the ratio of revenue to
University 2 2 0 0 UK / Canada
Exp on Faculty
University 3 0 1 0 Netherlands

For AAA – 3505.5/850 = 4.124
University 4 0 0 2 UK
Similarly For BBB – 5284.5/1220 = 4.33
University 5 1 0 0 India/Netherlands

For CCC – 4128/1640 = 2.51 and
University 6 1 0 1 USA For DDD – 7252/2840 = 2.55
University 7 2 0 0 UK/Canada
Hence least ratio is for college CCC.
University 8 0 2 0 India

49. (c) Complete the given table and find the profit of each institute.
AAA BBB CCC DDD
No Of Students 1710 2710 1920 3920

Revenue x 1000 3505.5 5284.5 4128 7252

Exp. On Faculty x 1000 850 1220 1640 2840

Exp On Infra. x 1000 650 1150 1060 1280

Cost of material x 1000 110 x 1710 = 188.1 K 140×2710=379.4K 130 x 1920= 249.6K 120x 3920= 470.4 K

Total Exp 1688.1 2749.4 2949.6 4590.4

Profit 1817.4 2535.1 1178.4 2661.6

Profit / Student 1062 935 613 679


Hence lowest value is for CCC.

50. (b) Evaluate the options one by one


(a) From the above table for AAA ratio of total profit to total exp = 1817/1688 = 1.07
(b) From the above table for BBB ratio of total profit to total exp = 2535/2749 = 0.922
(c) From the above table for CCC ratio of total profit to total exp = 1178/2949 = 0.40
(d) From the above table for DDD ratio of total profit to total exp = 2661/4590 = 0.58
y
o
u
rs
m
a
h
b
o
o
b
.w
o
rd
Data Table 79

p
re
Concept Eleminator

s
s
.c
Solutions from 1 – 3:

o
m
1. (a) To find the required quantity of comics in possession of Shyam. The follow the table carefully. Assuming
returning process starts from second month
On 1st AUGUST ON 1st SEP On 1st OCTOBER On 1st November
S given 3comics to M Say M returns 3 comics of S S given 3comics to M Say M returns 3 comics of S
(3 comics was with M) (0 comics in hand of M) (3 comics was with M) (0 comic of S in hand of M)
S returns 5 comics + 2
S taken 7 comics from M S taken 2 comics from M
S returns 4 comics of M comics new to M = 7
( +7comics) M (+2comics)M
comics.
7 comics in possession 3 comics in possession 5 comics in possession 0 possession

Or
Assuming that returning process starts from 3rd month
On 1st AUGUST ON 1st SEP On 1st OCTOBER On 1st November
S given 3comics to M S given 4comics to M (total S return 3comics to M S return 7 comics to M
(3 comics was with M) 7 comics was with M) (7 comics was with S) (0 comics was with S)
S taken 7 comics from S taken 3 comics from M M returns 3 comics to S
M returns 2 comics to S
M (+7comics) M (10comics was with S) = 7 comics.
7 comics in possession 10 comics in possession 7 comics in possession 0 possession

Note: S – shyam and M – murli. (a)- minimum zero

2. (d) lets say that the comics that Krishna had As murli will have = 100 + ( + 3 – 7) = 96
given to either Shyam or Murli has not been = (+ 4 – 3) = 97 = ( +3 – 2 ) = 98
returned. Than total no. of comics is 100 – So total comics murli has is 98
(3 + 6 + 1 + 8) – (9 + 2 + 6 + 4) = 61 So, c cannot have more than 98 comics
3. (c) Comics of Murli with Krishna Solutions from 4 -7:
1st August 1st September 1st October Lets find the NRR values
2014 2014 2014 For team LC RR1 can be calculated as
+9 (Krishna 3 comics 5 comics 162 + 179 + 132
gives i.e return to return to = = 473/( 234 + d)
120 + 144 + d
addition) krishna krishna
If d = 90, then RR1 = 1.46 (max) and if d = 120, then
6 comics RR1 = 1.34 (mini)
2 comics 159 + 164 + 133
-2 (out going) given by For Team LC RR2 = = 456/(214 + a)
return to murli 118 + 96 + a
krishna
On the similar basis we can find the RR1 and RR2 of

As murli will have = 100 + ( +9 –2) = 107 + the teams
(–3 +2) = 106 + ( + 6 – 5) = 107 LC YSC SDC CC

So total comics murli has is 100 and 7 comics
Wins 2 2 0 2
in possession

On the same way Murli with shyam Losses 1 1 3 1
y
o
u
rs
m
a
h
b
o
o
b
.w
o
rd
80 Data Table

p
re
s
s
Team Min Max Min Max

.c
o
LC 1.34 (d = 120) 1.46 (d = 90) 1.37 (a = 120) 1.50 (a = 90)

m
YSC 1.59 (c = 120) 1.74 (c = 90 ) 1.30 (b = 120) 1.42 (b = 90)
SDC 1.37 ( b = 120) 1.51 (b = 90) 1.47 (c = 120) 1.61(c = 90)
CC 1.27 (a = 120) 1.39 (a = 90) 1.44 (d = 120) 1.58 (d = 90)

4. (b) If a = 90 and d = 120, then 8. (c) From the table it is given that average
NRR for team LC and CC are – 0.16 and score of Dipan in PCB group is 98 and
– 0.05 respectively hence we can say team his average scores in Mathematics, Social
YSC and SDC are in final. Science, and Vernacular groups are 95, 95.5
5. (b) Weighted average of YSC is more. and 95 respectively. Let the average score
6. (c) Team YSC will definitely qualify owing to in English group be x then total average =
its positive NRR. Which is higher than the (98+95+95.5+95+x)/5 = 96 as 96 is the total
maximum possible NRR of the other teams. average, then x = 96.5 so total score in English
is 96.5 × 2= 193. Score of Dipan in English 1 =
If a = 90 and d = 120, then NRR for team LC
96 hence his score in English 2= 193 – 96 = 97.
= 1.34 – 1.5 = – 0.16, NRR for team CC =
1.39 – 1.44 = – 0.05 9. (d) From the table we can observe that only Dipan
is eligible to apply for the prize as he is the
7. (a) If a = b = c = d = 90, then
only one who satisfy the condition of at least
NRR for team LC = 1.46 – 1.5 = – 0.04, NRR 95 in every group. So Dipan gets the prize.
for team YSC = 1.74 – 1.42 = 0.32, NRR for
10. (a) From the table we can observe that only Dipan
team CC = – 0.19
scored at least 95 in at least one paper from
Hence team LC will placed second. each of the groups
11. (d) In order to maximize scores, each student would choose to improve score in the paper in which would
affect the group score the most or in the group that has minimum average.
STUDENT Change in Group Change in Group Avg Change in Avg Final Average
Ram 94 to 100 (Vern. Grp) 6/2 = 3 3/5 = 0.6 96.1 + 0.6 = 96.7
Agni 82 to 100 (Vern paper I) 18/2 = 9 9/5 = 1.8 94.3 + 1.8 = 96.1
Pritam 83 to 100 ( History) 17/2 = 8.5 8.5/5 = 1.7 93.9 + 1.7 = 95.6
Ayesha 93 to 100 (geo. ) 7/2 = 3.5 3.5/5 = 0.7 96.2 + 0.7 = 96.9
Dipan 95 to 100 (Math.) 5/1 = 5 5/5 = 1 96 + 1 = 97

Hence from the table Dipan has the highest score.


12. (a) Group scores of Joseph, Agni, Pritam and Tirna in Social Science Group are 95.5, 95.5, 89 and 89.5
respectively. Their final scores are 95, 94.3, 93.9, 93.7 respectively. If their group scores in social science
change to hundred their final scores will be affected by For Joseph (100-95.5)/5 = 4.5/5 = 0.9 and final
score 95.9
For Agni (100 – 95.5)/5 = 4.5/5 = 0.9, and final score = 94.3 + 0.9 = 95.2
For Pritam (100 – 89)/5 = 11/5 = 2.2 and final score = 93.9 + 2.2 = 96.1
For Tirna (100 – 89.5)/5 = 10.5/5 = 2.1and final score = 93.7 + 2.1 = 95.8
Their standing in decreasing order of final score would be Pritam, Joseph, Tirna, Agni.
y
o
u
rs
m
a
h
b
o
o
b
.w
o
rd
3
50

p
45

re
40

Bar

s
35

s
30

.c
25

o
m
20
15

Graph
10
5
0
Asia Pacific Europe North Latin South
America America Africa
Book($5) Video/DVD Games ($10)
Airline Tckt Reservation ($20) Clothing etc ($15)

Topic No. of Questions Level


Concept Applicator 25 Basic - Bank PO, MAT
Concept Builder 36 Advance - Bank PO, MAT
Concept Cracker 57 Basic - CAT IIFT XAT
Concept Deviator 68 Advance - CAT, IIFT, XAT
Concept Eliminator 11
Total 204

INTRODUCTION 1. What is the percentage increase in the number of


runs scored by Team B in match 4 as compared to
A bar graph is a chart that uses either horizontal or that in the previous match (Match 3)?
vertical bars to show comparisons among categories. (a) 40 (b) 30
One axis of the chart shows the specific categories (c) 20 (d) 25
being compared, and the other axis represents a discrete Sol. (c) Score of team B in Match 4 = 300 runs
value. Score in match 3 = 250 runs, required %
Example 1: increase = ( 50/250 ) × 100 = 20%.
Directions: Study the following graph carefully to 2. What is the ratio of the number of runs scored
answer the questions that follow: by team A in match 2 to the number of runs
Number of runs scored by three different teams in scored by Team C in Match 6?
six different cricket matches (a) 5 : 4 (b) 2 : 5
400 (c) 2 : 3 (d) None of these
350 Sol. (d) Score of team A : score of team C = 200 : 10 =
300 20 : 1.
250 3. What is the average number of runs scored by
200 Team A Team B in all the matches together?
150
Team B (a) 250 (b) 275
Team C
(c) 200 (d) 300
100
Sol. Average runs scored by team B
50
200 + 300 + 250 + 300 + 250 + 200
0 = = 250
match 1 match 2 match 3 match 4 match 5 match 6 6
y
o
u
rs
m
a
h
b
o
o
b
.w
o
rd
82 Bar Graph

p
re
Example 2: The following bar chart shows the 1. Total Consumption of Nitrogenous Fertilizers,

s
s
.c
consumption of fertilizers in nutrient terms. Examine Phosphatic fertilizers and Potassic Fertilizers

o
the following graph to answer these questions: during the period 1997-2002 has been in the ratio

m
(a) 10 : 28 : 74 (b) 37 : 14 : 5
14,000 (c) 5 : 2 : 1 (d) None of the above
12,000 Sol. (d) Required ratio = nitrogenous : phosphate :
10,000 potassic = 54500 : 21500 : 3500 = 109;43:13.
8,000 Nitrogenous
2. Which of the following fertilizers has shown a
Fertilizers
6,000 consumption pattern of increase and decrease in
4,000 Phosphatic alternate years?
2,000 Fertilizers (a) Nitrogenous Fertilizers
0 Potassic (b) Phosphatic Fertilizers
Fertilizers (c) Potassic Fertilizers
(d) No such trend is discernable
Sol. No such trend is encountered by any types of the
fertilizers.
y
o
u
rs
m
a
h
b
o
o
b
.w
o
rd
p
re
s
s
.c
o
m
Directions (Qs. No. 1-5): Study the following graph Directions (Qs. No. 6-10): Study the following graph and
carefully to answer these questions. answer the questions that follow. [Central Bank PO 2010]
[Andhra bank PO 2009] Number of Appeared candidates and Passed candidates
Populations of two states (in lakhs) over the years. (in hundreds) in a test from seven different Institutions
120 16
14
100
12
80 10
8 Appeared Candidates
60 State A
6 Passed Candidates
State B
40 4
2
20
0
A B C D E F G
0
2001 2002 2003 2004 2005 2006 2007 6. What is the difference between the number of
1. The population of State B in 2002 is what per cent candidates appeared from institutions B, C, D and
of the total population of State B in all the years F together and candidates passed from institutions
together? (Rounded off to two digits after decimal) A, E and G together?
(a) 8.26 (b) 7.26
(a) 100 (b) 900
(c) 8.32 (d) None of these
(c) 1000 (d) 540
2. What is the ratio of the total population of State A
7. What is the average number of candidates passed
for the years 2001, 2002, and 2003 together to the
total population of state B for the years 2005, 2006 from all the institutions together?
and 2007 together? (a) 700 (b) 490
(a) 27 : 53 (b) 54 : 29 (c) 350 (d) 675
(c) 29 : 54 (d) 53 : 27 8. The number of candidates passed from institutions
3. For which state and in which year was the per cent C and E together is approximately what percentage
rise in population from the previous year was the of the total number of candidates appeared from
highest? institutions A and G together?
(a) State B-2003 (b) State B-2002 (a) 72 (b) 62
(c) State A-2004 (d) State A-2005 (c) 54 (d) 67
4. What is the percent rise in the population of State 9. From which institution is the difference between
B from 2003 to 2004? the appeared candidates and passed candidates the
1 2 maximum?
(a) 16 (b) 16
3 3 (a) B (b) G
2 1 (c) D (d) F
(c) 18 (d) 18 10. What is the ratio of the candidates who have failed
3 3
5. Approximately what is the average population of from Institution B to the number of candidates who
State A for all the given years? have appeared from institution F?
(a) 56 lakhs (b) 50 lakhs (a) 2 : 5 (b) 2 : 3
(c) 48 lakhs (d) 58 lakhs (c) 4 : 3 (d) 1 : 3
y
o
u
rs
m
a
h
b
o
o
b
.w
o
rd
84 Bar Graph

p
re
Directions (Qs. No. 11-15): Study the following graph 15. What is the ratio of the number of students who

s
s
.c
and answer the questions that follow: No. of students (in opted for English in the years 2006 and 2008

o
thousand) who opted for three different specialization together to the number of students who opted for

m
during the given five years in a university [IBPS PO 2011] Hindi in the year 2005 and 2009 together?
40 (a) 11 : 5 (b) 12 : 7
35 (c) 11 : 7 (d) None of these
Direction (Qs. No. 16-21): The following 7 questions
30
are to be answered on the basis of the sales turnover
25 bar-chart for the years 1990 to 1995 given below
20 [Dec. 1997]
440
15 420
400
440
420
380 400 400

Sales in Thousands of Rupees


10 360
340
320 340
300 320
5 280
260
240
0 220
200
2005 2006 2007 2008 2009 180
160
Hindi English Mathematics 140
120
100
11. Out of the total number of students who opted for 80
60
the given three subjects, in the year 2009, 38% 40
20
0
were girls. How many boys opted for mathematics 1990 1991 1992 1993 1994 1995
in the same year?
16. By what amount are the sales in 1993 more than
(a) 1322 (b) 1332 those in 1991 ?
(c) Cannot be determined (d) None of these
(a) ` One hundred (b) ` Ten thousand
12. If the total number of students in the university
(c) ` One lakh (d) ` Ten lakh
in the year 2007 was 455030, the total number of
17. The sales in 1991 are how many times to those of
students who opted for the given three subjects was
1992 ?
approximately what percent of the total subjects?
(a) 8 times (b) 0.8 times
(a) 19 (b) 9
(c) 12 (d) 5 (c) 2.5 times (d) 3 times
13. What is the total number of students who opted for 18. In which year did the sales show the least per cent
Hindi and Mathematics in the years 2006, 2007 increase to that of the preceding year ?
and 2009 together? (a) 1990 (b) 1992
(a) 97000 (b) 93000 (c) 1993 (d) 1994
(c) 85000 (d) None of these 19. The sales in 1994 are what per cent of those in
14. The total number of students who opted for 1992?
Mathematics in the years 2005 and 2008 together (a) 40 (b) 4
is approximately what percent of the total number (c) 110 (d) 1.1
of students who opted for all three subjects in the 20. What is the appropriate average sales (in thousands)
same years? for the years 1992 to 1995 ?
(a) 38 (b) 28 (a) 420 (b) 415
(c) 42 (d) 32 (c) 430 (d) 425
y
o
u
rs
m
a
h
b
o
o
b
.w
o
rd
Bar Graph 85

p
re
21. For which of the earlier years, was the sales 22. Which section has the largest number of students ?

s
s
.c
turnover the same as for the year 1995 ? (a) E (b) D

o
(a) 1991 (b) 1992 (c) B (d) C

m
(c) 1993 (d) 1994 23. Which section has twice the number of students as
Direction (Qs. No. 22-25): These four questions are to compared to another ?
be answered on the basis of the following bar-graph, (a) A (b) C
showing the number of students in five sections A to E (c) D (d) E
of a class in a school [Jan. 1999] 24. If the students were to be uniformly divided in
40 each section, for which section would the strength
35 38
40 change most drastically?
36
30 (a) A (b) B
No. of Students

32
25
(c) D (d) E
20
20 25. What is the number of students that have to be moved
15
10
from one section to another so that there are three
5 sections with exactly the same number of students?
0 (a) 1 (b) 2
A B C D E
(c) 3 (d) 4
Class-Sections

Direction (Qs. No. 1-8): Study the following chart to 2. The difference in the slum populations of Bangalore
answer these questions [Dec. 2000] and Hyderabad was :
Slum Population in Metropolis: 1991 (a) 4.1 Lakh (b) 3.71 Lakh
Lakh (c) 2.43 lakh (d) 2 Lakh
3. The city with the highest slum population was :
Calcutta 35% 91.9

Mumbai 38% 82.4 (a) Mumbai (b) Calcutta


Delhi 30% 57.3 (c) Delhi (d) Chennai
Chennai 32% 42.9 Slum Population as
per cent of total
4. Two cities with nearly equal slum population were:
Ahmedabad 26% 25.5 population (%) (a) Ahmedabad and Hyderabad
Hyderabad 21% 25.5 (b) Delhi and Chennai
Bangalore 10% 29.2 (c) Hyderabad and Bangalore
0% 20% 40% 60% 80% 100% (d) Mumbai and Calcutta
1. The total slum population of Calcutta in 1991 was 5. The slum population of Delhi was more than 3
approximately : times the slum population of :
(a) 30 Lakh (b) 31 Lakh (a) Hyderabad (b) Ahmedabad
(c) 32 lakh (d) 33 Lakh (c) Bangalore (d) Chennai
y
o
u
rs
m
a
h
b
o
o
b
.w
o
rd
86 Bar Graph

p
re
6. The slum population of all the seven cities nearly Direction (Qs. No. 13-16): Refer the following bar

s
s
.c
equaled the total population of: graph to answer these questions. [MAT 2002]

o
(a) Calcutta and Bangalore

m
(b) Delhi and Chennai
(c) Delhi and Hyderabad
(d) Mumbai and Ahmedabad

17,118,339
16,985,503
15,809,735
7. The ratio of slum population to total population in

15,714,116
14,748,928
14,626,492
13,120,958
12,819,750
Income
Calcutta is ….. times the same ratio in Bangalore.

11,469,444
11,486,757
Expenses
(a) 3 (b) 3.5
(c) 4 (d) 5
8. In terms of slum population, the second city with
the least population was : 1997 1998 1999 2000 2001
(a) Delhi (b) Bangalore
(c) Ahmedabad (d) Hyderabad 13. In which year was the profit (Income minus
Direction (Qs. No. 9-12): Study the bar-chart of All Expenses) the highest?
India Monsoon Rainfall given below to answer these (a) 1998 (b) 1999
questions [MAT Dec. 2001] (c) 2000 (d) 2001
All India Monsoon Rainfall (1990 to 1991) June -September
14. In how many years was the profit larger to that in
140
119 the preceding years ?
Rainfall (% of Normal)

110
120 100 100 103 102 106 96
100 91 93 (a) 0 (b) 1
80 (c) 2 (d) 3
60 15. Which of the following statements can be inferred
40 from the bar graph?
20 (A) The company made a profit in all the years
0
1990 1991 1992 1993 1994 1995 1996 1997 1998 1999
shown on the graph.
YEAR (B) The company’s profit increased every year
All India Monsoon Rainfall (1990 to 1991) June -September between 1999 and 2001.
9. The normal rainfall during the period 1990 – 1999 (C) The company’s expenses increased every
was experienced in the year (s) : year shown on the graph.
(a) 1994 (b) 1994 and 1995 (a) A only (b) B only
(c) 1996 and 1997 (d) 1990 (c) C only (d) All of the above
10. The year …………. Witnessed the least rainfall,
16. How much reduction in the expenses in the year
(a) 1991 (b) 1999
2000 would have sustained the same profit as in
(c) 1992 (d) 1993
11. Out of the 10 years, how many years had above the year 1999?
normal rainfall ? (a) ` 25817 (b) ` 26817
(a) 3 (b) 7 (c) ` 28715 (d) ` 27621
(c) 5 (d) 6
Directions (Qs. No. 17-20): The following bar graph
12. Maximum flood damage has been during the year :
depicts the annual rates of Inflation in percentages for
(a) 1990 (b) 1994
1975 and 1987
(c) 1999 (d) Inadequate data
y
o
u
rs
m
a
h
b
o
o
b
.w
o
rd
Bar Graph 87

p
re
21. The percentage increase of production of item A

s
40

s
36.2

.c
35 from April to May is

o
(a) 33% (b) 66%

m
30
26.6
23.5
(c) 74% (d) 25%
25
21.7 22. Which item has maintained a rise over the three
20
1975 months?
16
15 14.4 14.2
12.7 (a) A (b) B
11.8 1987
10
10.3 (c) C (d) B and C
23. The overall production of items A, b and C during
5
April and May is in the ratio
0 (a) 1 : 1 (b) 3 : 4
World Developed Third USA UK
Nations World (c) 9 : 11 (d) 11 : 9
17. From 1975 to 1987, inflation rate increased in the Directions (Qs. No. 24-28): These questions are based
third world countries approximately by : on the graph which shows the demand and production
(a) 10% (b) 20% statistics of the five companies [MAY 2003, DEC 2002]
(c) 30% (d) 35% 3500
18. In the year 1975,the lowest rate of inflation was in the
(a) Developed Nations (b) UK 3000

(c) USA (d) Third World 2500


19. In the year 1987,the inflation rate in the third
world countries vis-à-vis the whole world jumped 2000
Demand
approximately by : 1500
(a) 50% (b) 100% Production
(c) 125% (d) 200% 1000
20. Comparing the figures for the USA vis-à-vis the 500
developed nations,it can be concluded that :
(a) USA had better control on inflation. 0
A B C D E
(b) Developed Nations had better control on
inflation, 24. What is the ratio of companies having more
(c) The control on inflation rate continues to be the demand than production to those having more
same for the USAand for the developed nations. production than demand?
(d) no conclusion can be drawn. (a) 2: 3 (b) 4 : 1
Directions (Qs. No. 21-23): Answer these questions (c) 2 : 2 (d) 3 : 2
on the basis of the following graph which shows the 25. What is the difference between the average demand
and the average production of five companies
production of items A, B and C during the months
taken together?
April, My and June.
(a) 1400 (b) 480
350
(c) 280 (d) 138
300 26. The production of the company D is how many
250 times of the production of the company A?
200 Product A (a) 1.8 (b) 1.5
150 (c) 5.4 (d) 1.11
Product B
100 27. The demand of company B is what per cent of the
Product C demand of company C?
50
(a) 4 (b) 24
0
APRIL MAY JUNE (c) 20 (d) 60
y
o
u
rs
m
a
h
b
o
o
b
.w
o
rd
88 Bar Graph

p
re
28. If company A desires to meet the demand by 29. What was the percentage increase in the number of

s
s
procuring TV sets from a single company, then which employees of the company from 1995-96 to 1997-98?

.c
o
one of the following can meet the need adequately? (a) 21.33% (b) 25%

m
(a) B (b) C (c) 33.33% (d) 16.67%
(c) D (d) None 30. What was the average number of employees who
Directions (Qs. No. 29 to 32): The graph below gives worked in the company over the given years?
the data of the number of employees working in a (a) 185 (b) 195
company the total expenditure of the company, and the (c) 235 (d) 175
total salary paid to the employees by the company over 31. What was the difference between the average of the
the years. Study the graph carefully to answer these total salaries paid by the company over the given
questions. [MAY 2008] years and the total salary paid by the company in
500 the year 1997-98?
No of employees of
400 the company (a) ` 2, 00,000 (b) ` 2, 50,000
(c) ` 4, 00,000 (d) ` 1, 50,000
300
Total exp. Of the 32. The total expenditure of the company in 1995-96
200 company in lakh was approximately what percent of the average
100 of the total expenditures of the company over the
0 Total salary paid to given years?
-00
-99
-98

(a) 82% (b) 79%


-97
-96

the employees by the


company in ` 0000
99
98
97
96
95

(c) 76% (d) 87%


19
19
19
19
19

Directions (Qs. No. 33-36): Study the graphs which show the seats won the percentage of valid votes polled for
different political in Gujarat over the year. [MAT 2006]
150 60
125
121

117

50

100 40
82

BJP BJP
67

30
53

52
45

50 Congress Congress
20
33

16

12

Others 10 Others
5

0
0
1990 1995 1998 2002 1990 1995 1998 2002
Seats won Valid votes polled (in %)
33. If the total number of valid votes in 2002 Gujarat 35. In which of the following years did the BJP secure
election was 5 crore then find, the average number of more than 66 2/3% of the total seats?
votes for winning one seat for other political parties: (a) 1990 (b) 1998
(a) 11 Lakh (b) 1.10 lakh (c) 1995 (d) 2002
(c) 1.10 crore (d) Data is inadequate 36. In which of the following years, was the difference
34. In 1998, if 2.24 crore people votes were valid for in the number of valid votes for any two political
BJP whereas in 1990 there were 1.228 crore people parties maximum?
votes valid for Congress by what percent was the
(a) 1990
number of valid votes less in 1990 with respect to
that in 1998? (b) 1998
(a) 20% (b) 24% (c) 1995
(c) 30% (d) 25% (d) Cannot be determined
y
o
u
rs
m
a
h
b
o
o
b
.w
o
rd
Bar Graph 89

p
re
s
s
.c
o
m
Directions (Qs. No. 1-3): Study the following chart 4. For the equipments using Integrated Circuit Board:
carefully and answer. [SNAP 2008] 400, Capacitors: 240 and Printed Circuits boards:
Failure Rates of Electronic Components
45 120 to run with minimum downtime, how many
40
40
spares should be kept in the store respectively?
No. Per Thousand

35 33
30
30
25 (a) 12, 4, 3 (b) 12, 5, 4
20
20
15
15 16 (c) 5, 12, 4 (d) 12, 4, 4
10
5 Directions (Qs. No. 5-6): [SNAP 2009]
0
These questions are based on the graph which shows the
s

es

rs

ts

ds

ts
be

ui

ui
to
ic

ar
tu

demand and production statistics of 5 T.V. companies


rc

rc
ta
ev

Bo
ci

Ci

Ci
es

pa

ts
ur

ed

o
al

ui
Ca

er
ct

gn

at

rc

ic
Pi

gr
Si

Ci

3500 3300
M
te

rid
In

te

3000
yb
in

3000
Pr

2700
2500
1. From the failure data of electronics components 2500 2200
presented here, which statement is true? 2000 1800
(a) Integrated circuits and Printed circuit boards 1500
1500
1200
are more reliable. 1000
1000
(b) Capacitors are more reliable than pictures 600
500
tubes.
(c) Higher number of components failures is 0
A B C D E
likely to be due to pictures tubes. Demand Production

(d) The least reliable component is Hybrid Micro


5. What is the difference between the average demand
circuits.
2. Which of the following components has a failure and the average production of the five companies
rate 25% more than that of signal devices taken together?
(a) Pictures tubes (a) 1400 (b) 400
(b) Capacitors (c) 280 (d) 138
(c) Integrated circuits
6. The production of the company D is how many
(d) Printed Circuits boards.
3. Lowest priority for investing in any changes times of the production of the company A?
or additions to the component manufacturing (a) 1.8 (b) 1.5
units, in the company’s investment plans, may be (c) 2.5 (d) 1.11
given to the following:
Directions (Qs. No. 7-10): Refer to the following bar
(a) Printed circuits boards and hybrid micro circuits.
(b) Capacitors and integrated circuits. diagram showing the sales (in ` Crores) of top market
(c) Pictures tubes and signal devices brands among pain killers in India and answer the
(d) Signal devices and capacitors questions. [SNAP 2010]
y
o
u
rs
m
a
h
b
o
o
b
.w
o
rd
90 Bar Graph

p
re
8. Percentage of increase in sales from

s
30

s
.c
25 2005-2006 is the highest for which brand of a

o
pain killer?

m
20
15 (a) Voveran (b) Volini
2005
10
2006 (c) Dolonex (d) Sumo
5 9. Percentage increase in sales from 2005-2006 is the
0 lowest for which brand of a pain killer?
o
an

m
ise

ex

v
i
l

id (a) Voveran (b) Volini


m

lin
o

oo
fla
er

on
llp

ul
N

Su

Vo

M
r

bi

im
ol
Vo

Ca

(c) Moov (d) Nise


Co

7. What is the difference between the sales of


Voveran in 2006 and those of Calpol in 2005 10. What is the approximate percentage of increase in
(in ` lacs)? the sales of Voveran from 2005-2006?
(a) 1000 (b) 50 (a) 35% (b) 40%
(c) 100 (d) 500 (c) 45% (d) 50%
Direction (Qs. No. 11-15): Answer the questions based on the following graph. [IIFT2008]
Production of P, Q, R & S (1990-95)
180
170 S
160 S
150
140 S
Production in Tonnes

R
130
R
120 S
110 QR
100 Q S
R
90 S
80 R P Q
70 QR
60 P
50 P
Q P Q
40 P
30 P
20
10
0
1990 1991 1992 1993 1994 1995
11. In which year the annual growth rate of total (a) Product (b) Product Q
production (of all products) is highest? (c) Product R (d) Product S
(a) 1991 (b) 1992
13. If four products P, Q, R and S shown in the graph
(c) 1993 (d) 1995
are sold at price of `9, ` 4, `13 and `3 respectively
12. If the stability of the production during 1990 to
during 1990-1995, then the total revenue of all the
1995 is defined as,
products is lowest in which year?
Average Production
(a) 1991 (b) 1992
Maximum Production − Minimum Production
then which product is most stable? (c) 1993 (d) None of the above
y
o
u
rs
m
a
h
b
o
o
b
.w
o
rd
Bar Graph 91

p
re
14. Individual revenue of P, Q, R and S for the entire (a) 12

s
s
.c
period (1990-1995) is calculated based on the price (b) 15

o
of `9, `4, `13 and `3 respectively. Which product (c) 20

m
fetches the lowest revenue? (d) Cannot be determined.
(a) Product P (b) Product Q 17. Which of the five states manufactured liquor at the
(c) Product R (d) Product S lowest cost?
15. Four products P, Q, R and S shown in the graph (a) Tamil Nadu
are sold at price of `9, `4, `13 and `3 respectively (b) Delhi
during 1990-1995. Which of the following (c) The states which has the lowest value for
statements is true? (wholesale price-Excise duty) per litre
(a) Product R fetches second highest revenue (d) Cannot be determined.
across products in 1991. 18. If Excise duty is levied before the goods leave the
(b) Sum of revenue of P, Q and S is more than the factory (on the value of the liquor), then which
revenue of R in 1994. of the following choices shows distilleries in
(c) Cumulative revenue of P and Q is more than ascending order of the excise duty paid by them
the revenue of S in 1993. for the year 1996? (Assume the total liquor in TN
(d) None of the above is supplied by only these 5 distilleries).
Directions (Qs. No. 16-19): The following graph shows (a) ECABD
the value of liquor supplied by the 5 states in 1996 (b) ADEBC
and the excise duty rates in each state. [CAT 1990] (c) DCEBA
350 323.3
(d) Cannot be determined.
300
19. If the Tamil Nadu distillery, with the least average
248 240
simple annual growth in amount of liquor supplied
250
214 in the given period had shown the same rate
187.7
200 of growth as the one which grew fastest, what
150 would that company’s supply have been in 1998,
100 80 in lakh liters?
60
50 25
52 39 (a) 13
0
(b) 15.11
TN AP Maharashtra MP Delhi (c) 130
Total Value (` Crore) Excise Duty (` per Litre) (d) Cannot be determined.
Amount of liquor supplied in Tamil Nadu Distilleries Directions (Qs. No. 20-22): Refer to the following Bar-
A, B, C, D, E (from bottom to top) in lakh litres. chart and answer the questions that follow : [CAT 1990]
50 Project Exports: Contracts Secured
45 4.21 Column
40 5.79 160
35 3.57 140 143.9
30 5.75 11.92 141
25 120
7.26 100
20
12.07 100.5 Column
15 9.35 80
2.45
10 1.64 1.05 60
3.15 67 65
5 10.78 12.89
6.41 40
0
1996 1997 1998 20
A B C D E 0
16. What is the lowest percentage difference in the 1984 1985 1986 1987 1988
excise duty rates for any two states?
y
o
u
rs
m
a
h
b
o
o
b
.w
o
rd
92 Bar Graph

p
re
20. What is the average value of the contract secured Directions (Qs. No. 27-30): are based on the graph given

s
s
during the years shown in the diagram?

.c
below: [CAT 1994]

o
(a) `103.48 crore (b) `105 crore Number of Engineering Students (in hundreds) at

m
(c) `100 crore (d) `125.2 crore institutions of different kind
21. Compared to the performance in 1985 (i.e. 300
taking it as the base), what can you say about the
performances in the years ‘84, ‘85, ‘86, ‘87, ‘88 250

respectively, in percentage terms? 200


(a) 150, 100, 211, 216, 97 Private Engg College
150 Govt. Engg College
(b) 100, 67, 141, 144, 65 Regional Engg College
(c) 150, 100, 200, 215, 100 100 IITS
(d) 120, 100, 220, 230, 68 50
22. Which is the year in which the highest percentage
decline is seen in the value of contract secured 0
1988-89 1989-90 1990-91
compared to the preceding year? 27. What was the total number of engineering students
(a) 1985 (b) 1988 in 1989 – 90?
(c) 1984 (d) 1986 (a) 28500 (b) 4400
Directions (Qs. No. 23-26): Study the graph below (c) 4200 (d) 42000
and answer the questions. [CAT 1993] 28. The growth rate in students of Govt. Engg.
Total Assets are defined as Net Fixed Assets + Net Colleges compared to that of Private Engg.
Current Assets + Investments Colleges between 1988 – 89 and 1989 – 90 is
35
(a) more (b) less
30
4 (c) equal (d) 3/2
25 1 2 29. The total number of Engg. Students in 1991 – 92,
20 2 assuming a 10% reduction in the number over the
15 16 15
17
previous year, is
13
(a) 5700 (b) 57000
10
(c) 44800 (d) None of these
5
7 8 7.5 9 30. In 1990 – 91, what percent of Engg. Students were
0 studying at IIT’s?
1990 1991 1992 1993 (a) 16 (b) 15
Net Fixed Assets Net Current Assets Investments (c) 14 (d) 12
23. What is the approximate simple annual growth rate Directions (Qs. No. 31-35): Answer the questions based
of Total Assets 1990 and 1993? on the following graph. [CAT 1995]
(a) 36% (b) 12% Foreign trade (in billion dollars)
(c) 9% (d) 27%
24. In any two consecutive years, the growth rate is 1987-88
lowest for
1988-89
(a) Net Fixed Assets (b) Net CurrentAssets.
(c) Investments. (d) Total Assets. 1989-90
25. Between 1991 and 1992, the highest growth rate 1990-91
was seen for Exports
1991-92
(a) Net Fixed Assets. (b) Net CurrentAssets. Imports
(c) Investments. (d) Total Assets. 1992-93
26. The only item which has not shown a negative 1993-94
growth in every year between 1990 and 1993 is
1994-95
(a) Net Fixed Assets. (b) Net Current Assets.
(c) Investments. (d) Total Assets. 0 5 10 15 20 25
y
o
u
rs
m
a
h
b
o
o
b
.w
o
rd
Bar Graph 93

p
re
31. In which year was the trade deficit highest? 37. In 1992, what per cent of the total revenue came

s
s
.c
(a) 1987-88 (b) 1988-89 from books?

o
(c) 1989-90 (d) 1990-91 (a) 45% (b) 55%

m
32. In how many years was the trade deficit less than (c) 35% (d) 25%
the trade deficit in the succeeding year? 38. The number of years in which there was an increase
(a) 1 (b) 2 in revenue from at least two categories is
(c) 3 (d) 4 (a) 1 (b) 2
33. Export earning in 1990-91 is how many per cent of (c) 3 (d) 4
imports in 1991-92?
39. If 1993 were to show the same growth as 1992
(a) 82% (b) 85%
over 1991, the revenue in 1993 must be
(c) 90% (d) 15%
(a) `194 lakh (b) `187 lakh
34. In the last three years the total export earnings have
(c) `172 lakh (d) `177 lakh
accounted for how many per cent of the value of
the imports? 40. The growth in total revenue from 1989 to 1992 is
(a) 80% (b) 83% (a) 21% (b) 28%
(c) 95% (d) 88% (c) 15% (d) 11%
35. Which of the following statements can be inferred Direction (Qs. No. 41-45): Answer the questions based
from the graph? on the following information. [CAT 1996]
I. In all the years shown in the graph, the trade 250
220
deficit is less than the export earning. 200
II. Export earnings increased in every year 200
165
185

between 1989-90 and 1991-92. 150 145


135 140
150
130
III. In all the years shown in the graph, the earning 120
102 110 115
125

by exports is less than the expenditure on 100


70
imports in the preceding year. 50
60
50 40
(a) I only (b) II only 20 25 30
(c) III only (d) I and III only 0
1989 1990 1991 1992 1993 1994 1995
Directions (Qs. No. 36-40): Answer the questions Profit Revenue Expenditure
based on the following graph. 41. The average revenue collected in the given 7 years
Revenue obtained by a publishing house while is approximately
selling books, magazines and journals (` in lakh). (a) `164 lakh (b) `168 lakh
80
(c) `171 lakh (d) `175 lakh
42. The expenditure for the 7 years together form what
70
60
per cent of the revenues during the same period?
50
Journals
(a) 75% (b) 67%
40
Magazimes (c) 62% (d) 83%
30 Books 43. Which year showed the greatest percentage
20 increase in profit as compared to the previous year?
10 (a) 1993 (b) 1994
0
1989 1990 1991 1992 (c) 1990 (d) 1992
36. Which year shows the highest change in revenue 44. In which year was the growth in expenditure
obtained from journals? maximum as compared to the previous year?
(a) 1989 (b) 1990 (a) 1993 (b) 1995
(c) 1991 (d) 1992 (c) 1991 (d) 1992
y
o
u
rs
m
a
h
b
o
o
b
.w
o
rd
94 Bar Graph

p
re
45. If the profit in 1996 shows the annual rate of growth Directions (Qs. No. 52-55): Answer the questions

s
s
.c
as it had shown in 1995 over the previous year, based on the following information.

o
then what approximately will be the profit in 1996? The following bar chart gives the growth percentage

m
(a) ` 72 lakh (b) ` 82 lakh in the number of households in middle, upper-middle
(c) ` 93 lakh (d) ` 78 lakh and high income categories in the four regions for the
Directions (Qs. No. 46-51): Answer the questions period between 1987-88 and 1994-95. [CAT 1998]
based on the following graph. 450
The graph given below gives the yearly details of 400
money invested in producing a certain product over the
350
years 1991 to 1995. It also gives the profit (in ‘000
rupees). [CAT 1997] 300

250
250
200
200 Profit
Interest
150
150
Overheads
100
100 Wages

50
Raw-material 50

0 0
North South East West
–50 Middle Income Category
1991 1992 1993 1994 1995
Upper middle income category
46. In which year was the increase in raw material High income category
maximum?
Number of Average Growth in average
(a) 1992 (b) 1993 households in household household income
(c) 1994 (d) 1995 1987-88 (in income in (1994-95 over 1987-
47. In which period was the change in profit maximum? thousands) 1987-88 88)
(a) 1991-92 (b) 1992-93 Middle-
40 ` 30,000 50%
(c) 1993-94 (d) 1994-95 income
Upper -
48. Which component of the cost production has 10 ` 50,000 60%
income
remained more or less constant over the period?
High -
(a) Interest (b) Overheads 5 ` 75,000 90%
income
(c) Wages (d) Raw material
52. Which region showed the highest growth in
49. In which year were the overheads, as a percentage
number of households in all the income categories
of the raw material, maximum?
for the period?
(a) 1995 (b) 1994
(a) North (b) South
(c) 1992 (d) 1993 (c) West (d) None of these
50. What percentage of the costs did the profits form 53. What was the total household income in northern
over the period? region for upper-middle class?
(a) 3% (b) 5% (a) ` 50 lakh (b) ` 500 million
(c) 8% (d) 11% (c) ` 300 million (d) Cannot be determined
51. If the interest component is not included in the 54. What is the percentage increase in total number of
total cost calculation, which year would show the households for the northern region (upper-middle)
maximum profit per unit cost? over the given period?
(a) 1991 (b) 1992 (a) 100 % (b) 200 %
(c) 1993 (d) 1995 (c) 240 % (d) Cannot be determined
y
o
u
rs
m
a
h
b
o
o
b
.w
o
rd
Bar Graph 95

p
re
55. What was the average income of the high-income (a) 20% (b) 36%

s
s
.c
group in 1987-88?
(c) 25% (d) Cannot be determined

o
(a) ` 75,000 (b) ` 25,000

m
(c) ` 2,25,000 (d) Cannot be determined 57. The average income for the northern region in
Directions (Qs. No. 56-57): The numbers of households 1987-88 was
in each category were equally distributed in all the regions.
(a) ` 37,727 (b) ` 37,277
56. The ratio of total income for the high-income
category to the upper-middle class increased by (c) ` 35,000 (d) Cannot be determined
how much percentage in the given period?

Directions (Qs. No. 1-4): Answer these questions based (b) Absolute value of foreign equity inflows was
on the data presented in the figure below. [CAT 2000] higher in 1998 for Thailand and lower for
12 china than the corresponding values in 1997.
10.67
9.92 (c) Absolute value of foreign equity inflows was
10
lower in 1998 for both India and china than
8 the corresponding values in 1997.
5.96 5.82
6
4.8 5.09 1997 (d) None can be inferred.
4
1998 3. It is known that china’s GDP in 1998 was 7%
1.71 2.16 2.5 higher than its value in 1997, while India’s GDP
2
0.72 grew by 2% during save period. The GDP of South
0
Indis China Malaysia S. Korea Thailand
Korea on the other hand, fell by 5%. Which of the
following statement is / are true?
FEI for a country in a year, is the ratio (expressed as I. Foreign equity inflow to China  were  higher
a percentage) of its foreign equity inflows to its GDP. in 1998 than in 1997.
The following figure displays the FEIs for select Asian II. Foreign equity inflow to China were lower in
countries for the years 1997 and 1998. 1998 than in 1997.
1. The country with the largest charge in FEI in 1998 III. Foreign equity inflow to India were higher in
relative to its FEI in 1997, is: 1998 than in 1997
(a) India (b) China IV. Foreign equity inflow to South Korea
(c) Malaysia (d) Thailand  decreased in 1998 relative to 1997.
2. Based on the data provided it can be concluded that: V. Foreign equity inflow to South Korea
(a) Absolute  value of foreign equity inflows in increased in 1998 relative to 1997.
1998 was higher than that in 1997 for both (a) I, III and IV (b) II, III, IV
Thailand and South Korea. (c) I, III, V (d) II & V
y
o
u
rs
m
a
h
b
o
o
b
.w
o
rd
96 Bar Graph

p
re
4. China›s foreign equity inflows in 1998 were 10

s
Chemicals

s
.c
times that into India. It can be concluded that: 15

o
10

m
(a) China’s GDP in 1998 was 40% higher than
that of India. 5
0
(b) China’s GDP in 1998 was 70% higher than 90 91 92 93 94 95 96 97 98
that of India.
5. Which is the sector with the highest growth during
(c) China’s GDP in 1998 was 50% higher than the period 1989 and 1998?
that of India. (a) Manufacturing
(d) No inference can be drawn about relative (b) Mining and quarrying
magnitudes of China’s and India’s GDPS. (c) Electricity
(d) Chemicals
Directions (Qs. No. 5-10): Answer these questions
6. The overall growth rate in 1991 of the four sectors
based on the data given below: [CAT 2000]
together is approximately:
Manufacturing
15 (a) 10% (b) 1%
10
(c) 2.5% (d) 1.5%
7. When was the highest level of production in the
5
manufacturing sector achieved during the nine-
0
90 91 92 93 94 95 96 97 98 year period 1990 - 1998?
–5 (a) 1998
The figures below present annual growth rate, (b) 1995
expressed as the % change relative to the previous (c) 1990
year, in four sectors of the economy of the Republic (d) Cannot be determined
8. When was the lowest level of production of the
of Reposia during the 9 year period from 1990 to 1998.
mining and quarrying sector achieved during the
Assume that the index of production for each of the four
nine year period 1990 - 1998?
sectors is set at 100 in 1989. Further, the four sector:
(a) 1996
manufacturing, mining and quarrying, electricity and
(b) 1993
chemicals respectively, constituted 20%, 15%, 10%
and 15% of total industrial production in 1989. (c) 1990
Mining and Quarrying (d) Cannot be determined
15
9. The percentage increase of production in the
10 four sectors, namely, manufacturing, mining and
5 quarrying, electricity and chemicals, taken together,
0 in 1994, relative to 1989, is approximately:
–5 (a) 25 (b) 20
90 91 92 93 94 95 96 97 98
(c) 50 (d) 40
Electricity
10 10. It is known that the index of total industrial
8 production in 1994 was 50 percent more that in
6 1989. Then, the percentage increase in production
between 1989 and 1994 in sectors other than the
4
four listed above is:
2
(a) 57.5 (b) 87.5
0
90 91 92 93 94 95 96 97 98 (c) 127.5 (d) 47.5
y
o
u
rs
m
a
h
b
o
o
b
.w
o
rd
Bar Graph 97

p
re
Directions (Qs. No. 11-16): Answer these questions 12. Roughly what percentage of the total work is

s
s
.c
based on the two graphs shown below. [CAT 2001] carried out onsite?

o
Figure 1 shows the amount of work distribution, in (a) 40 percent (b) 20 percent

m
man-hours, for a software company between offshore (c) 30 percent (d) 50 percent
and onsite activities. Figure 2 shows the estimated and 13. The total effort in man-hours spent onsite is nearest
actual work involved in the different offshore activities to which of the following?
in the same company during the same period. [Note: (a) The sum of the estimated and actual effort for
onsite refers to work performed at the customer’s offshore design.
premises and offshore refers to work performed at the (b) The estimated man-hours of offshore coding.
developer’s premises]
(c) The actual man-hours of offshore testing.
500
(d) Half of the man-hours of estimated offshore
coding.
400
14. If the total working hours were 100, which of the
following tasks will account for approximately 50
300 hours?
(a) Coding
200 (b) Design
(c) Offshore testing
100 (d) Offshore testing plus design
15. If 50 percent of the offshore work were to be carried
0
Design Coding Testing out onsite, with the distribution of effort between
Offshore Onsite
the tasks remaining the same, the proportion of
Fig. 1 testing carried out offshore would be
500 (a) 40 percent (b) 30 percent
(c) 50 percent (d) 70 percent
400 16. If 50 percent of the offshore work were to be
carried out onsite, with the distribution of effort
300
between the tasks remaining the same, which of
the following is true of all work carried out onsite?
(a) The amount of coding done is greater than
200
that of testing.
(b) The amount of coding done onsite is less than
100
that of design done onsite.
(c) The amount of design carried out onsite is
0
Design Coding Testing greater than that of testing.
Estimated Actual (d) The amount of testing carried out offshore is
Fig. 2 greater than that of total design.
11. Which of the work requires as many man-hours as Directions (Qs. No. 17-20): Answer these questions
that spent in coding? based on the chart given below. [CAT 2002]
(a) Offshore, design and coding The chart given below indicates the annual sales tax
(b) Offshore coding revenue collections (in crores of rupees) of seven states
(c) Testing from 1997 to 2001. The values given at the top of each
(d) Offshore, testing and coding bar represents the total collections in that year.
y
o
u
rs
m
a
h
b
o
o
b
.w
o
rd
98 Bar Graph

p
re
introduced in 1997. 20% of the mixer-grinders bought

s
50000 44.638

s
.c
42.348 4300 in a particular year are disposed off as junk exactly two

o
40000 years later. It is known that 10 Purana mixer-grinders

m
36.068 3500 6375 WB
29.870
33.168
3118 5270 8015 UP were disposed off in 1997. The following figures show
30000
2704
2844
4458 7000 TN the number of Purana and Naya mixer-grinders in
3985
3473
5604
6113 12034 MA operation from 1995 to 2000, as at the end of the year.
20000 5341 10284
KA
8067 250 236 236
7290 7826 222
5413 GU
4839
10000 3510 3829 4265
AP 200
6300 182
5000 4026 4402 4796 5400
162
3526 4728 5251 6055 7202
0 150 134
1996-97 1997-98 1998-99 1999-2000 2000-01 120 124 Purana
100
17. If for each year, the states are ranked in terms of 80 Naya
descending order of sales tax collections, how 50
30
many states don’t change the ranking more than
once over the five years. 0
1995 1996 1997 1998 1999 2000
(a) 1 (b) 5
23. How many Naya mixer-grinders were purchased in
(c) 3 (d) 4
1999?
18. Which of the following states has changed its
(a) 44 (b) 50
relative ranking most number of times when you
(c) 55 (d) 64
rank the states in terms of the descending volume
24. How many Naya mixer-grinders were disposed off
of sales tax collections each year?
by the end of 2000?
(a) Andhra Pradesh (b) Uttar Pradesh
(a) 10
(c) Karnataka (d) Tamil nadu
(b) 16
19. The percentage share of sales tax revenue of which
(c) 22
state has increased from 1997 to 2001?
(d) Cannot be determined from the data
(a) Tamil Nadu (b) Karnataka
25. How many Purana mixer-grinders were purchased
(c) Gujarat (d) Andhra Pradesh
in 1999?
20. Which pair of successive years shows the maximum
(a) 20
growth rate of tax revenue in Maharashtra?
(b) 23
(a) 1997 to 1998 (b) 1998 to 1999
(c) 50
(c) 1999 to 2000 (d) 2000 to 2001
(d) Cannot be determined from the data
21. Identify the state whose tax revenue increased
26. How many Purana mixer-grinders were disposed
exactly by the same amount in two successive pair
off in 2000?
of years?
(a) 0
(a) Karnataka (b) West Bengal
(b) 5
(c) Uttar Pradesh (d) Tamil Nadu
(c) 6
22. Which state below has been maintaining a constant
(d) Cannot be determined from the data
rank over the years in terms of its contribution to
Directions (Qs. No. 27-30): On the basis of the
total tax collections?
information given below: [CAT 2005]
(a) Andhra Pradesh (b) Karnataka
(c) Tamil Nadu (d) Uttar Pradesh A management institute was established on January
Directions (Qs. No. 23-26): Answer the questions on 1, 2000 with 3, 4, 5 and 6 faculty members in the
the basis of the information given below. [CAT 2004] Marketing, Organizational Behavior (OB), Finance,
Purana and Naya are two brands of kitchen mixer- and Operations Management (OM) areas respectively,
grinder available in the local market. Purana is an old to start with. No faculty member retired or joined the
brand that was introduced in 1990, while Naya was institute in the first three months of the year 2000. In
y
o
u
rs
m
a
h
b
o
o
b
.w
o
rd
Bar Graph 99

p
re
the next four years, the institute recruited one faculty The bar chart below shows the revenue received,

s
s
.c
member in each of the four areas. All these new faculty in million US Dollars (USD), from subscribers to a

o
members, who joined the institute subsequently over particular Internet service. The data covers the period

m
the years, were 25 years old at the time of their joining 2003 to 2007 for the United States (US) and Europe.
the institute. All of them joined the institute on April The bar chart also shows the estimated revenues from
1. During these four years, one of the faculty members subscription to this service for the period 2008 to 2010.
retired at the age of 60. The following diagram gives 1000
the area-wise average age (in terms of number of 900
completed years) of faculty members as on April 1 of 800
2000, 2001, 2002, and 2003. 700
60 600
52.5 500
49.33 50.551.5 50.249
50 46 47.8 US
44 45 45 46 45 45 400
43 44
40 Europe
2000
300
30 2001 200
2002 100
20
2003 0
03 04 05 06 07 08 09 10
10
Year
0 31. While the subscription in Europe has been growing
Marketing OB Finance OM
steadily towards that of the US, the growth rate
27. From which area did the faculty member retire? in Europe seems to be declining. Which of the
(a) Finance (b) Marketing following is closest to the percent change in
(c) OB (d) OM growth rate of 2007 (over 2006) relative to the
28. Professors Naresh and Devesh, two faculty growth rate of 2005 (over 2004)?
(a) 17 (b) 20
members in the Marketing area, who have been
(c) 35 (d) 60
with the Institute since its inception, share a
(e) 100
birthday, which falls on 20th November. One was
32. The difference between the estimated subscription
born in 1947 and the other one in 1950. On April 1
in Europe in 2008 and what it would have been if
2005, what was the age of the third faculty member, it were computed using the percentage growth rate
who has been in the same area since inception? of 2007 (over 2006), is closest to :
(a) 47 (b) 50 (a) 50 (b) 80
(c) 51 (d) 52 (c) 20 (d) 10
29. In which year did the new faculty member join the (e) 0
Finance area? 33. In 2003, sixty percent of subscribers in Europe
(a) 2000 (b) 2001 were men. Given that woman subscribers increase
(c) 2002 (d) 2003 at the rate of 10 percent per annum and men
30. What was the age of the new faculty member, who at the rate of 5 percent per annum, what is the
joined the OM area, as on April 1, 2003? approximate percentage growth of subscribers
(a) 25 (b) 26 between 2003 and 2010 in Europe? The subscription
(c) 27 (d) 28 prices are volatile and may change each year.
Directions (Qs. No. 31-34): Answer the following (a) 62 (b) 15
questions based on the information given below: (c) 78 (d) 84
[CAT 2008] (e) 50
y
o
u
rs
m
a
h
b
o
o
b
.w
o
rd
100 Bar Graph

p
re
34. Consider the annual percent change in the gap

s
40

s
.c
between subscription revenues in the US and 35

o
Europe. What is the year in which the absolute

m
30
value of this change is the highest? 25
DPS
(a) 03-04 (b) 05-06 20 DAV
15 CS
(c) 06-07 (d) 08-09 KV
10
(e) 09-10
5
Directions (Qs. No. 35-36): [CAT 2009] 0
ABC ltd is established in 1999. Its revenue, Expenditure 2002 2003 2004 2005

and profit for the four consecutive year is as given in 37. In which of the following year & group there is
the bar graph. Every year it has paid a tax. Tax paid as maximum % increase in number of students?
per the following formula-
(a) DPS 2004-05 (b) DAV 2004-05
Revenue = Exp + Profit + Tax
250 (c) CS 2003-04 (d) KV 2002-03
38. What is the maximum percentage change in the
200
number of students in top 100 from DAV?
150 Revenue (a) 25% (b) 40%
Exp
100 Profit (c) 55% (d) None of these

50
39. DAV should merge with which group so that their
combined number of students is more than 50.
0
1999 2000 2001 2002 (a) DPS (b) CS
35. For which year percentage change in tax is (c) KV (d) CS & KV
maximum than from previous year.
Directions (Qs. No. 40-41): [CAT 2011]
(a) 2000 (b) 2001
ABC ltd is established in 1999. Its revenue, Expenditure
(c) 2002 (d) 2001 & 2002
and profit for the four consecutive year is as given in
36. In which year the ratio of revenue to tax is the the bar graph. Every year it has paid a tax. Tax paid as
maximum? per the following formula-
(a) 2000 (b) 2001 Revenue = Exp + Profit + Tax
(c) 2002 (d) 1999
250
Directions (Qs. No. 37-39): Are based on the graph
given below: [CAT 2010] 200

The graph below shows the number of students from 150


4 different schools came in top 100 in class X exam in Revenue
Exp
a state. There are total 5 groups out of these 4 schools 100
Profit
mentioned and remaining are from other schools as
50
example in 2002-03 total number of students from
these 4 schools are 15+25+15+30= 85 hence remaining 0
15 are from other schools. 1999 2000 2001 2002
y
o
u
rs
m
a
h
b
o
o
b
.w
o
rd
Bar Graph 101

p
re
40. For which year percentage change in tax is 42. Which of the following year exhibited highest

s
s
.c
maximum than from previous year. percentage decrease over the preceding year

o
(a) 2000 (b) 2001 exhibited highest percentage decrease over the

m
(c) 2002 (d) 2001 & 2002 preceding year in the automobile production?
41. In which year the ratio of revenue to tax is the
(a) 2005 (b) 2006
maximum?
(a) 2000 (b) 2001 (c) 2007 (d) 2008
(c) 2002 (d) 1999 43. Assume whatever that is not sold domestically was
Directions (Qs. No. 42-47): Answer the questions exported, then which year has registered highest
based on the following graphs [IIFT 2009] growth in exports of automobiles?
Automobile Production Trends (Number in ‘000) (a) 2005 (b) 2006
1400 (c) 2007 (d) 2008
44. If the ratio of the domestic sale price of a
1200 commercial vehicle, a passenger vehicle, and a
three wheeler is 5:3:2 then what percent of earnings
1000 (approximately) is contributed by commercial
vehicle segment to the overall earnings from
800 domestic sales during the period 2004-2008?
(a) 45% (b) 43%
600 (c) 11% (d) 27%
45. For which year were the domestic sales of
400 automobiles closest to the average (2004-2008)
domestic sales of automobiles?
200 (a) 2005 (b) 2006
(c) 2007 (d) 2008
0
46. Which of the following years exhibited highest
2004 2005 2006 2007 2008
percentage increase over the preceding year in the
Passenger Vehicles Commercial Vehicles Three Wheelers
automobile sales?
Automobile Domestic sales Trends (Number in ‘000) (a) 2005 (b) 2006
1200
(c) 2007 (d) 2008
1000
47. The ratio between absolute increase in domestic
sales over preceding year and absolute increase
800 in production over the preceding year in highest
during which year?
600 (a) 2005 (b) 2006
(c) 2007 (d) 2008
400
Directions (Qs. No. 48-50): Following graph
200 represents the cost per square feet of four retailers
from the financial year 2004 to 2012. The expected
0 cost per square feet for year 2010, 2011 and 2012 are
2004 2005 2006 2007 2008
Passenger Vehicles
forecasted figures. [IIFT 2010]
Commercial Vehicles Three Wheelers
y
o
u
rs
m
a
h
b
o
o
b
.w
o
rd
102 Bar Graph

p
re
s
s
1783 1804

.c
FY12E 2711
2411

o
1729 1889

m
FY11E
1724 1832
1996
FY10E 2135
1603 1802
2044 2464
FY09
1525 1751
1658 2419
FY08
1482 1581
1417 2197
FY07
1425 1659
1396 2280
FY06
1051 1064
1462 2322
FY05
1052 1065
1501 2068
FY04
1040 1052

0 500 1000 1500 2000 2500 3000

Pantaloon S Stop Vishal Westside

48. Which retailer shows the sharpest decline cost per 51. If the world energy council formulates a norm for
square feet and in which year? high emission countries to reduce their emission
(a) West side 2005 (b) Pantaloons 2008 each year by 12.5% for the next two years then
(c) S.stop 2009 (d) Vishal 2010
what would be the ratio of CO2 emission to per
49. Which retailer has shown the maximum increase in
its cost per square feet and in which year? capita income of US, China and Japan after two
(a) S.stop 2006 (b) S.stop 2007 years.  The per capita income of China, Japan and
(c) Pantaloons 2006 (d) Vishal 2006 US is expected to increase every year by 4%, 3%
50. What is the average rate of change in the cost and 2% respectively.
per square feet of the retail sector, if the sector
(a) 3.5, 3.8, 4.1 (b) 3.4, 3.5, 3.9
is represented by the above four retailers in the
period FY07 to FY 10E? (c) 2.9, 3.1, 3.4 (d) None of these
(a) –8.12 (b) –10.86 52. If US and China, decide to buy carbon credits,
(c) –6.73 (d) None of these from Spain and Ukraine to make up for their high
Directions (Qs. No. 51-54): Study the following graph
emissions, then in how many years US, and China
and answer the questions that follow. [IIFT 2010]
would be able to bring down its ratio of CO2
1400
1200 1200 1180 emission (million tonnes) to per capita income
1080 1120
1000 900
to world standard benchmark of 0.75. (per capita
800 800
income of the given countries remain same, 0.5
CO2 emissions (million tonnes) is compensated
600
600 500 500 500
440 440 400 400
380
400 300 270 260
240 200 260 225 by purchase of 1.25 units of carbon credit, and a
200 180 190 170 160 195
140 165 185
country can buy carbon credit units in three lots of
0
15, 20 and 30 units in a single year.
(a) 3.8 years (b) 38 years
Total CO2 Emission (million ton) 2010 (c) 30 years (d) None of these
Per capita income 2010
y
o
u
rs
m
a
h
b
o
o
b
.w
o
rd
Bar Graph 103

p
re
53. France, South Africa, Australia, Ukraine and Poland 54. Select the wrong statement in reference to the

s
s
.c
form an energy consortium which declares CO2 position of India vis-a-vis countries in the graph

o
emission of 350 million ton per annum as standard in terms of the ratio of CO2 emission to per capita

m
benchmark. The energy consortium decides to sell income (increasing order)
their carbon emission savings against the standard (a) India stands at 5th position if 50 is added to the
benchmark to high carbon emission countries. It is given per capita income figures of each country.
expected that the per capita income of each country
(b) India stands at 5th position at the given CO2
of the energy consortium increases by 2%, 2.5%
emission level and per capita income of each
and 3.5% p.a. for the next three years respectively.
country
The ratio of CO2 emission to per capita income
of the each energy consortium country reduces by (c) India stands at 5th position if 200 million
50% and remains constant for the next three years. ton CO2 emission is deducted from the given
By selling 0.5 CO2 emissions (million ton) the CO2 emission figures of each country.
energy consortium earns 1.25 carbon credits, then (d) India stands at 5th position if 200 million
determine the total carbon credits earned by energy ton CO2 emission is deducted from the given
consortium in three years. CO2 emission figures of each country and 50
(a) 3560 (b) 4506 is added to the given per capita income of
(c) 5060 (d) None of these each country.

Direction (Qs. No. 55-57): The following graph shows population data (males and females), educated people data
(males and females) and number of male in the population for a given period of 1995 to 2010. All data is in million.
From the information given in the graph answer the questions that follow. [IIFT 2010]

2010 570 750 1100


2009
540 720
2008 1050
520 700
2007 1015
515 660
2006 975
495 640
2005 950
480 600
2004 900
475 580
2003 845
450 560
2002 800
440 545
2001 790
430 500
2000 770
420 490
1999 760
400 480
1998 750
380 460
1997 700
350 440
1996 670
320 450
1995 650
300 400
0 600
200 400 600 800 1000 1200
Figures in million population Figures in million educated Figures in million no. males
y
o
u
rs
m
a
h
b
o
o
b
.w
o
rd
104 Bar Graph

p
re
55. In which year the percentage increase in 59. In which month Country XX’s Outward Investment

s
s
.c
the number of females over the previous to Singapore dropped most and what is the ‘month

o
year is highest? on month’ growth in that period?

m
(a) 1996 (b) 1999 (a) April, Approximately negative growth of
(c) 2003 (d) 2004 39 percent
56. In 2002 if the ratio of number of educated male (b) March, Approximately negative growth of
to professionally educated female was 5:4. If the 49 percent
number of educated males increased by 25% in (c) April, Approximately negative growth of
2003. What is the percentage change in number of 49 percent
uneducated females in 2003? (d) None of the above
(a) +25% (b) +35% 60. What is the share of Country XX’s Outward Invest-
(c) +34% (d) 56% ment together in USA and UK in February 2011 of
57. In year 2005 total population living in urban area is its total investment in the world?
equal to sixty eight percent of educated population. (a) 7.24 percent (b) 8.30 percent
The ratio of number of people living in urban area (c) 6.79 percent (d) None of the above
to people living in rural area is 43:12 in 2010. 61. In which month the share of Country XX’s total
What is the ratio of the rural population in 2005 to Outward Investment together in Singapore and UAE
that in 2010? achieved the highest level and what is the value?
(a) 0.8 (b) 0.47 (a) April, 40 percent (b) February, 45 percent
(c) 2.05 (d) None of these (c) March, 45 percent (d) None of the above
Direction (Qs. No. 58-62): Answer the following 62. Between February 2011 and April 2011, to which
questions based on the Diagram below, which reports country did Outward Investment from XX witness
Country XX’s monthly Outward Investment flows to the highest decline?
various countries and the World. The FDI figures are (a) Singapore (b) UK
reported US$ Million. [IIFT 2011] (c) UAE (d) Others
Directions (Qs. No. 63-72): Based on the following
3701

4000
information. [XAT 2011]
3221

3500 350
3075

330
No. of positions occupied

3000 300
2628
2568

2358

2500 250
210
2000 200
1551

1390
1378

150 130
1211

1500
100
1000 65
741

60
686

52
615

50 30 25
378

500 15
286
273

6 4 8 4 9 7
227

157

142
133

125
119

117

0
116
86

70

44
37

30
29

0 1 2 3 4 5
Jan -11 Feb -11 Mar -11 Apr-11 May-11 Organizational leve
USA UK Singapore UAE Others World Total employees Ex-defence servicemen Ex-policemen

58. What is the compound average growth rate of Five years ago Maxam Glass Co. had estimated its staff
Country XX’s overall Outward Investment during requirements in the five levels in their organization as:
Level-I: 55; Level-2: 65; Level-3: 225: Level-4: 255
the period January 2011 and May 2011?
& Level-5: 300. Over the years the company had
(a) Approximately 6 percent recruited people based on ad-hoe requirements, in the
(b) Approximately 3.5 percent process also selecting ex-defence service men and ex-
(c) Approximately 5.75 percent policemen. The following graph shows actual staff
(d) None of the above strength at various levels as on date .
y
o
u
rs
m
a
h
b
o
o
b
.w
o
rd
Bar Graph 105

p
re
63. The level in which the Ex-Defence Servicemen are 66. In 2010, which could be a valid statement about

s
s
highest in percentage terms is:

.c
the revenues (adjusted for inflation) of these three

o
(a) 1 (b) 2 companies? (1 mark)

m
(c) 3 (d) 4
64. If the company decides to abolish all vacant posts (a) Revenues of all three companies were equal.
at all levels, which level would incur the highest (b) Revenues of all three companies could be equal
reduction in percentage terms’? (c) Revenue of Yahoo was definitely less than
(a) 1 (b) 2
Facebook which was definitely less than
(c) 3 (d) 4
65. Among all levels, which level has the lowest Google.
representation of Ex-policemen? (d) Total of Yahoo and Facebook was definitely
(a) 1 (b) 2 higher than that of Google.
(c) 3 (d) 4
67. The difference in the average percentage increase
Directions (Qs. No. 66-68): Based on the following
information. [XAT 2012] in revenues, from 4th to 6th year, of Yahoo and
The following graphs shows the revenue (in Facebook is: (1 mark)
$ million) of three companies in their initial six years (a) 35% (b) 40%
of operations, in an economy which is characterized by
(c) 45% (d) 50%
a persistent inflation.
2500
68. What would have been Facebook’s revenue (in
$ million) in its sixth year of operation if the
2000

2000 company had matched Google’s percentage


1500

growth in revenues from the fifth to the sixth year?


1500
(1 mark). Choose the option that is nearest to the
1250

Yahoo
1000
Facebook answer.
750

Google
600

(a) 1600 (b) 1700


400

500
300
250

(c) 1900 (d) 2100


100

100
50

50
50
20
30
20
20

(e) None of the above.


5

0
1 2 3 4 5 6

Direction (Qs. No. 1-3): Answer the question carefully. the respondents were undergraduates and the rest were
A survey on three popular Cigarettes namely- Classic, graduates. One –fifth of the graduates respondents
Navy Cut and Gold Flake. The survey team wants to see were unemployed. One fourth of the undergraduate
the consumptions of different brands. 1200 young male were employed.
generation was the sample. It is known that one third of
y
o
u
rs
m
a
h
b
o
o
b
.w
o
rd
106 Bar Graph

p
re
Below the Graph 1 shows the Employed Responds. Directions (Qs. No. 5-6): The graph given below

s
s
.c
(all values are in percentage) contains data pertaining to number of electronic

o
commerce transactions that have taken place in the

m
70
60 last six months of the financial year 2005. This graph
50 contains data related to private consumption and does
40 used regularly not include corporate electronic commerce activities.
30 used occasionaly Numbers mentioned above the bar graphs are in
20 Never used millions and average price per unit in mentioned in the
10 brackets. [XAT 2007]
0
CLASSIC NAVY CUT GOLD FLAKE 50
45
The graph below is for undergraduate respondents
(all values are in percentage) 40
35
70
60 30

50 Used Regularly 25
40 used occasionaly 20
30 Never used
15
20
10
10
0 5
CLASSIC NAVY CUT GOLD FLAKE
0
Asia Pacific Europe North Latin South
1. What was the number of undergraduates who had America America Africa
used Classic? Book($5) Video/DVD Games ($10)
(a) 254 (b) 289 Airline Tckt Reservation ($20) Clothing etc ($15)
(c) 364 (d) 388
2. how many employed youth never used cigarettes? 5. For which product category is the revenue
(a) 132 (b) 111 contribution of Asia Pacific region the maximum?
(c) 98 (d) 74 (a) Books
3. What is the ratio of the number of undergraduates (b) Video/DVDs/Games
who had occasionally used CLASSIC to the (c) Airline Tickets/ Reservation and Clothing/
number of employed who had occasionally used Accessories/shoes
Gold Flake? (d) Airline tickets/ Reservation
(a) 9 : 10 (b) 24 : 37 6. If the airline ticket purchases made through
(c) 21 : 37 (d) 36 : 25 internet increase by 20% and the average price
4. Assume that 30% of undergraduate used three of the airline ticket increases by 25% then the
brands than what is the ratio of undergraduate net increase in revenues from the e-commerce
regular user of all the brands to the undergraduates activities worldwide will be ___________ percent
never user and unemployed ? of the corresponding pre-price- increase revenues.
(a) 15 : 26 (b) 18 : 23 (a) 18 (b) 19
(c) cannot be determine (d) None of these (c) 20 (d) 21
y
o
u
rs
m
a
h
b
o
o
b
.w
o
rd
Bar Graph 107

p
re
Directions (Qs. No. 7-11): On the basis of the data 8. Suppose the total global production increased (year

s
s
[XAT 2009]

.c
given in two charts. on year) from 2005 to 2008 by the amount Tata

o
Sodium carbonate, also called as soda ash is an Chemicals’ synthetic production (year on year)

m
important ingredient for glass, soaps and detergents, increased in the same period. By what percentage
and many other products. There were two ways of did the total global production increase from 2007
producing soda ash. The first is producing soda ash to 2008?
from trona obtained naturally. The second method was (a) Cannot be calculated at all from the tables
producing soda ash from common salt through Solvay above.
process. Soda ash produced thus was called synthetic (b) Increased by 10.16%
soda ash. Tata Chemicals was one of largest producer (c) Increased by 9.48%
of soda ash. Given below are two charts- first chart
(d) Did not increase at all.
shows production of two varieties of soda ash at Tata
9. Which of the following statements are true?
Chemicals. The second chart shows production of two
varieties of soda ash in the world. 1. Proportion of natural soda ash to synthetic soda
Tata Chemicals Soda Ash Production (MT) has decreased from 2001 to 2006 globally.
3.2 2. Proportion of natural soda ash to synthetic soda
ash has increased from 2001 to 2006 globally.
2.2 2.2
3. Proportion of synthetic soda ash to total soda
ash has decreased for Tata Chemicals from
Natural soda Ash
0.9 Synthetic soda Ash 2005 to 2007 globally.
0.3 4. Proportion of synthetic soda ash to total soda
0
2005 2007 2008
ash has increased for Tata Chemicals.
Natural Soda Ash Synthetic Soda Ash (a) 1 and 3 (b) 1 and 4               
Global Soda Ash production (MT)
(c) 2 and 4 (d) 2 and 3
35 10. What is Tata Chemicals’’ share of global production
30 in 2008?
25
(a) 12.86% (b) 17.42%             
20
15
(c) 59.34% (d) Incomplete data       
10 11. Suppose total global production of soda ash in
5 2008 was 40 MT and Tata Chemicals was second
0 highest producer of soda ash globally after another
2001 2002 2003 2004 2005 2006
company called Solvay. FMC Wyoming was the
Synthetic Soda Ash Natural Soda Ash
third highest producer. Two Indian giants, Tata
7. It was expected that global soda ash production Chemicals and Nirma have a combined production
would be same for 2006, 2007 and 2008 (only for capacity of 8.8 MT. Which of the following
this question). What could be a possible reason for statements are right?
different patterns of production in Tata Chemicals 1. Solvay’s market share was more than 20.66%
and the world?
2. Solvay’s market share was more than 13.5%
(a) Tata Chemicals built new plants of 2.2 MT
3. FMC’s share was less than 10.33%
natural soda ash capacity in 2007.
(b) Tata Chemicals built 3.2 MT of natural soda 4. FMC share was less than 13.5%
ash capacity from 2005 to 2008. 5. Nirma, which was sixth largest producer, had
(c) Tata Chemicals produced 2.7% of total soda a share of less than 8.5%
ash in the world. Choose the right option.
(d) None of the above conclusions could possibly (a) 1 and 3 (b) 1 and 5
be drawn. (c) 2 and 4 (d) 1, 3 and 5
y
o
u
rs
m
a
h
b
o
o
b
.w
o
rd
108 Bar Graph

p
re
s
s
.c
o
m
Concept Applicator 11. (d) No such ratio is given so cannot be determined.
1. (d) Total population of State B in all the years is 12. (b) Total number of students in 2007 for all
50 + 40 + 60 + 70 + 80 + 90 + 100 = 490. State the three subjects = 40,000. Therefore,
40 40, 000
B in 2002 = × 100 = 8.16 ×100 = 8.79% i.e., approximately 9%.
490 455030
2. (c) Total population of State A for the years 13. (d) Total number of students who opted for Hindi
= 145; and of State B for the years = 270. and Mathematics IN 2006, 2007 and 2009 =
Ratio is 145 : 270 = 29 : 54 (5 + 35 + 15 + 15 + 20 + 5) × 1000 = 95,000
3. (a) The per cent rise in population from the 14. (d) Number of students for mathematics in 2005
previous year was the highest in State B – and 2008 are 45,000 and total number of
students in all the three subjects in 2005 and
2003 i.e. 60 − 40 ×100 = 50%
40 2008 are 1,40,000. Therefore,
70 − 60 2 45, 000
4. (b) ×100 = 16 % ×100 = 32.14 i.e., 32 (Approx.)
60 3 140000
40 + 45 + 60 + 50 + 70 + 62 + 80 407 15. (a) Total number of students who opted for
5. (d) = = 58.14
7 7 English in 2006 and 2008 are 55000; and
i.e. approximately 58. Hindi in the years 2005 and 2009 are 25000.
6. (c) Number of candidates appeared from B, C, Ratio is 55000 : 25000 = 11 : 5
D and F = 1400+ 700 + 1200 + 600 = 3900 16. (c) Sales turnover of 1993 – 1991 i.e., 4,20,000 –
and the number of candidates passed from A, 3,20,000 = 1,00,000
E and G = 1200 + 1200 + 500 = 2900. The 17. (b) Sales in 1991 is 0.8 times to those of 1992 i.e.
difference is 3900 – 2900 = 1000. 320
7. (a) The average number of candidates passed = 0.8
400
from all the institutions together is 18. (d) 1994 shows the least per cent increase to that
1200 + 1000 + 400 + 400 + 1200 + 200 + 500 of the preceding year by 4.76%.
7 440
4900 19. (c) The sales in 1994 are × 100 =110% of
= = 700 those in 1992. 400
7
8. (e) The number of candidates passed from 20. (b) the average sales for the years 1992 to 1995 is
institutions C and E together = 1600 and 400 + 420 + 440 + 400
= 415
total number of candidates appeared from 4
institutions A and G together = 2400. 21. (b) It is clearly visible from the graph that the
1600 sales turnover of 1992 and 1995 is same.
Therefore, ×100 = 66.67 approx. 67
2400 22. (a) It can be seen from the graph that Section E
9. (c) It is clearly visible that State D has the highest has the largest number of students.
difference of 1200 – 400 = 800. 23. (d) Section E has twice the number of students as
10. (b) Number of candidates failed from institution compared to Section A i.e. 20 × 2 = 40.
B = 1400 – 1000 = 400 and the candidates 24. (a) only Section A would show the most drastic
appeared from F = 600. Therefore, the ratio is change.
400 : 600 = 2 : 3
y
o
u
rs
m
a
h
b
o
o
b
.w
o
rd
Bar Graph 109

p
re
25. (b) 2 students have to be moved from one section profit declined as compared to the

s
s
.c
to another. D has 38 students we have to add preceding year.

o
2 students to B and subtract 2 from E so that (c) Is true as it is visible in the bar graph.

m
Section. 16. (b) The difference between the profit of 1999 and
2000 is 26815. So in 2000 the reduction in the
Concept Builder expenses should have been 26,817.
1. (c) The total slum population of Calcutta in 1991 9.6
17. (d) 36.2 – 26.6 = 9.6; ×100 = 36.09 i.e.
35 26.6
was 91.9 × = 32.16 i.e. 32 lakhs. approx.. 36%
100 18. (c) It is visible that USA has the lowest rate of
2. (c) Slum population of Bangalore is 10% of 29.2
inflation.
= 2.92 lakhs and Hyderabad is 21% of 25.5 20.2
= 5.35. The difference is 5.35 – 2.92 = 2.43 lakhs 19. (c) 36.2 – 16 = 20.2; ×100 = 126.25% .
16
3. (b) It can be seen from the graph that Calcutta has since the only option nearest to it is 125% so
the highest population and more slum% too. the answer is c.
35% of 91.9 = 32.16 lakhs 20. (c) The control on inflation rate continues to be
4. (d) Mumbai and Calcutta has nearly equal slum same for both developed nations & USA i.e.
population of 31.31 and 32.16 14.2 – 11.8 = 2.4 and 12.7 – 10.3 = 2.4
5. (a) Slum population of Delhi is 17.19 lakhs, 21. (b) Production of item A increases from 150 to 250
Hyderabad is 5.35, Ahmedabad is 6.63, during April to May. Therefore, the percentage
Bangalore is 2.92 and Chennai is 13.72. It 100
increase is 100 × 100 = 66.66% i.e. 66%
can be clearly seen that the slum population 150
of Delhi is more than 3 times of Hyderabad 22. (a) It is visible in the graph that Product A has
5.35 × 3 = 16.05 maintained a rise over the three months.
6. (d) 17.19+5.35+6.63+2.92+13.72+32.16+31.31 23. (b) Overall production in April is 450 and May is
= 109.28 lakhs. Slum population of all cities is 600. The ratio is 450 : 600 = 3 : 4
109.28 which nearly equaled the total population 24. (d) 3 companies A, C, E have more demand
of Mumbai and Ahmedabad i.e. 107.9. than production and 3 companies have more
7. (b) Ratio of Calcutta is 0.35 and Bangalore is 0.1. production than demand. Therefore, the
therefore, Calcutta is 3.5 times i.e. 0.35 / 0.1 answer is 3 : 2.
the ratio in Bangalore. 25. (b) Average demand
8. (d) Hyderabad has the second least slum 3000 + 600 + 2500 + 1200 + 3300 10600
= = = 2120
population of 5.35 lakhs. 5 5
9. (b) We know that the normal rainfall is 100% and and average production
that was experienced in 1994 and 1995. 500 + 1800 + 1000 + 2700 + 2200 8200
= = = 1640 ,
10. (a) It is clearly visible that the year 1991 witnessed 5 5
the least rainfall of 91%. required difference = 480
11. (c) Years experienced above normal rainfall are 26. (c) According to question, 500 x = 2700 or x =
1990, 1994, 1996, 1997 and 1998. 5.4 times
12. (d) Data is inadequate 600
27. (b) The demand of company B is ×100 =24%
13. (a) Profit is highest in 1998 i.e. 13120958 – 2500
12819750 = 301208. of the demand of company C.
14. (c) In 1998 and 2001 the profit was larger than the 28. (d) Demand of A is 3000, maximum production is
preceding year. 2700. So no company can fulfill the demand
15. (c) (a) Is false as there was a loss in 1997. of A entirely.
(b) Is false because in the year 2000 the
y
o
u
rs
m
a
h
b
o
o
b
.w
o
rd
110 Bar Graph

p
re
i.e. Production of company D is 1.8 times the

s
30

s
29. (d) × 100 = 16.67%

.c
production of company A.
180

o
925 Solutions from 7-10: The sales (in Crores) of the

m
30. (a) 150 + 120 + 180 + 220 + 255 = = 185
5 different brands in the two years are as follows.
31. (a) Average of the total salaries paid Brand 2005 2006
500000 + 800000 + 1000000 + 1200000 + 2500000
Voveran 16.5 23.0
5
= 1, 200, 000 Calpol 13.0 18.0

Nise 15.0 18.5
and total salary paid by the company in the year
1997-98 = 1,000,000.The difference is 2,00,000. Combiflam 9.5 14.0
32. (d) Total expenditure in 1995-96 = 3000000. Dolonex 7.0 10.0
Average of the total expenditures over the Sumo 5.0 7.5
years = 3,440,000. Therefore, the percentage Volini 7.0 9.5
3, 000, 000
= × 100 = 87.20% i.e., approx. 87% Moov 3.75 5.0
3, 440, 000
Nimulid 3.5 5.0
33. (d) Data is inadequate
34. (a) In 1998 45% of x = 2.24 crore; 7. (a) Difference between sales of Voveran in 2006
2.24 and Calpol in 2005 = 23.0 - 13.0 = 10.0 Crores
x= ×100 = 4.98 crore = 1000 Lacs. 
45
In 1990 let the valid votes be y, 31% of 8. (c) 66.66%
1.228 9. (d) From the table we can find The percentage
y = 1.228 crore; y = ×100 = 3.96 crore increase is the lowest for Nise as it is less than
31
4.98 − 3.96 25% and all other values are more than 25.
×100 = 20.48% approx. 20%
4.98 8
10. (d) × 100 = 50%
35. (d) It is visible from the graph that either 2002 16
and 1998 secured more than 66 2/3%. In 2002 11. (b) Annual growth rate = summation of all
– BJP has 125 seats, congress has 52 seats, products/ total production of all the years
125 12. (d) Stability = average production / max.
others has 10 seats. × 100 =66.8%
187 Production – mini. Production
36. (b) The difference in the number of valid votes Stability of product P = 0.93
for any two political parties in the year 1998 Stability of product Q = 1.05
between BJP and others 45% - 20% = 25%. Stability of product R = 1.45
Concept Cracker Stability of product S = 1.78
13. (c) Total revenue means summation of production
1. (d)    2. (b)   3.  (c)   4. (b) in tonnes of P, Q, R ans S multiplied with their
5. (c) Total demand = 3000 + 600 + 2500 + 1200 + respective rates.
3300 = 10600 units. Total revenue in 1991 = 2072
Total production = 1500 + 1800 + 1000 + Total revenue in 1992 = 2661
2700 + 2200 = 9200 units. Total revenue in 1993 = 1831
Difference between the average demand and 14. (b) During the given period for the production of
the average production = 2120 – 1840 = 280 Q fetches the lowest revenue
6. (a) Production of company D = 2700.
15. (c)
Production of company A = 1500.
y
o
u
rs
m
a
h
b
o
o
b
.w
o
rd
Bar Graph 111

p
re
16. (d) The answer cannot be determined as the data and with lowest growth rate is E. So had the

s
s
.c
for only five states is given and we don’t know amount of liquor manufactured by E grown

o
the excise duty rates for other states. by 313.41% in the 2 year period ie. Grown by

m
17. (d) We have been given the total value in the 616.82% overall its supply in 1998 would be
graph, but nothing is mentioned about the (2.45 × 616.82 /100) = 15.11 liters.
amount of liquor manufactured by states other
20. (a) (100.5 + 67 + 141 + 143.9 + 65)/5 = 103.48
than TN.
18. (c) Since Excise duty is levied on the total value 21. (a) The key here is figuring out that the only
of liquor produced by the 5 distilleries, this performance which is less than the 1985
will be in the same order as the order of the performance is the 1988 performance. Hence
amount of the liquor produced by them (as the the percentage corresponding to 1988 should
excise duty rate remains constant). Hence the be less than 100. Thus we see that (c) cannot
correct order is DCEBA. be the answer. Also option (b) cannot be the
19. (b) The simple average annual growth for the 5 answer as it shows two of the years having
distilleries in TN is as shown : less than 100%. Between options (a) and (d),
the correct answer is (a), This is so because
Distillary Calculation Rate
the difference between the 1985 and 1988
A (12.89 – 6.41)/(6.41 × 2) 50.54% performance is only 2 units on 67 units. Hence
B (12.07 – 3.15)/(3.15 × 2) 141.58% percentage wise it has to be 97% and not 68%.
C (11.92 – 1.64)/(1.64 × 2) 313.41%22. (b) The highest percentage decline over the
D (5.79 – 1.05)/(1.05 × 2) 225.71% previous year is seen for the year 1988, as in
this year the performance almost halved. In
E (4.21 – 2.45)/(2.45 × 2) 35.91%
other year you won’t find this happening.
So the distillery with highest growth rate is C Solutions from 23-26 :
The above graph can be represented in the following manner :
Net Fixed Growth Net Current Growth Investments Growth Total Growth
Assets(NFA) Rate of Assets Rate of Rate of Assets Rate of
NFA (NCA) NCA Investments (TA) Total Assets
19 7 - 13 - 2 - 22 -
19 8 14.25% 16 23% 1 -50% 25 13.63%
19 7.5 -6.25% 15 -6.25% 2 100% 25 -
19 9 20% 17 13.33% 4 100% 30 20%
23. (b) The growth rate of total assets between 1990-93 = (30-22)/22 = 36%. But this is over a 3 year period.
Hence simple average annual growth rate =
36/3 = 12%. College 1988-89 1989-90 1990-91
24. (c) It cam be seen that the growth rate is lowest
Private Engg. College 120 180 250
for investments in 1990-91 viz. 50% decrease.
25. (c) Between 1991 & 1992, the highest growth rate Govt Engg. College 80 120 130
was seen for investments viz.100% increase.
26. (d) It can be seen that very individual item has Regional Engg. College 40 75 100
shown a decrease in some year or the other.
IIT 30 40 80
Only Total Assets has not shown this trend.
Solutions from 27-30: The data given the graph can 27. (d) Total number of students in 1989–90 = (180 +
be tabulated as given below : 120 + 75 + 40) x 100 = 41500 = 42000 (approx.)
y
o
u
rs
m
a
h
b
o
o
b
.w
o
rd
112 Bar Graph

p
re
28. (c) Growth rate in number of students in Govt.

s
1989 1990 1991 1992

s
.c
Engg. College = (120 – 80)/80 = 50%

o
Growth rate in number of students in Private Journals 46 47 45 44

m
Engg. College = (180 – 120)/120 = 50%. Magazines 31 39 45 50
Hence the growth rate is equal. Books 73 77 79 79
29. (d) Total number of students in 1990–91 =
Total 150 163 169 173
(250 + 130 + 100 + 80) 100 = 56000
Hence the total number of students in 1991- 36. (c) The highest change in the revenue obtained
92 = 0.9 x 56000 = 50400. Hence (d) from journals is (47 – 45) = 2 in 1991.
is the correct answer. 37. (a) In 1992 percentage of total revenue that
30. (c) % of IIT students in 1990 – 91 = 80 / 570 = 1/7 came from books = 79/173 = 45.6% = 45%
= 14% (approx.) (approximately).
Solutions from 31-37: The graph given in the question 38. (b) In 1990 the revenue increased in all three
can be expressed as a table given below. categories. In 1991 it increased for magazines
and books. And in 1992 it increased only for
Year Import Export Trade Deficit
magazines. So the answer is b, viz. 2 years.
1987-88 17 11 6  173 − 169 
39. (d) Growth rate in 1992 over 1991 = 
1988-89 19 12 7  169 
1989-90 21 16 5 = 2.36%. If this rate were to remain same in
1990-91 24 18 6 1993 as well, then the revenue in 1993 will be
1991-92 20 18 2   2.36  
173 × 1 + 100   = ` 177 lakh.
1992-93 22 18 4
1993-94 23 21 2 40. (c) Percentage growth in the total revenue from
1994-95 27 24 3  173 − 150 
1989 to 1992 =  = 15.33% = 15%
 150 
173 138 (Approx.)
31. (b) Trade deficit = Imports – Exports, is highest Solutions from 41-45: To handle this type of questions,
for the year 1988-89, viz. 7 billion dollars. the best way is to express the data in tabular form.
32. (d) Trade deficit is less than that in the succeeding Year 1989 1990 1991 1992 1993 1994 1995
years in 1987-88, 1989-90, 1991-92 and Revenue 120 130 145 165 185 200 220
1993-94.
Expenditure 102 110 115 125 135 140 150
33. (c) Required percentage =(18/20) = 90%
34. (d) In last three years, Imports = (22 + 23 + 27) Profit 20 25 30 40 50 60 70
= 72 and Exports = (18 + 21 + 24) = 63. 41. (b) The average revenue collected in the given 7
Hence, the required percentage = 63/72 years
= 87.5% = 88% (approximately).
 120 + 130 + 145 + 165 + 185 + 200 + 220 
35. (a) The first statement is obviously true as the =   = 166.42
7
trade deficit in each year is less than the export

which is approx. 168 lakh
earning. The export earning has remained
constant for three years between 1990 and 42. (a) Expenses of 7 years add up to 877. Revenue
1993. Hence, statement II is not true. Even of 7 years add up to 1165. Hence, the required
statement III is not true as the exports in 1994- 877 880
answer is = = 75%
95 is more than the imports in 1993-94. 1165 1170
Solutions from 36-40: The graph given in the question 43. (d) We need to find the profit in each year.
can be depicted in the following table:
y
o
u
rs
m
a
h
b
o
o
b
.w
o
rd
Bar Graph 113

p
re
47. (c) The change in the profit is maximum in 1993-

s
Year Profit Percentage

s
.c
94. In this year, there is a 50 points drop in the
1990 (5/20) × 100 = 25%

o
profits.

m
1991 (5/25) × 100 = 20% 48. (a) It can be seen that the interest has remained
1992 (10/30) × 100 = 33.33% more or less constant over the given period.
1993 (10/40) × 100 = 25% 49. (c) Year Raw Mat (RM) O.H. OH/RM x 100
1994 (10/50) × 100 = 20% 1991 60 10 16.66%
1995 (10/60) × 100 = 16.66% 1992 50 20 40%
From the above table, clearly, the answer is 1993 65 15 23.07%
1992, as in 1992 the profit is maximum, i.e. 1994 75 25 33.33%
33.33%. 1995 80 20 25%
44. (d) The growth in expenditure over the previous
Thus, it can be seen from the above table that
year can be expressed as:
the overheads as a percentage of raw material
Year Growth Expenditure is maximum for 1992.
1990 (8/202) × 100 = 7.8% 50. (b) The total profits over the period
1991 (5/110) × 100 = 4.5% = (15 + 25 + 20 – 30 + 15) = 45
1992 (10/115) × 100 = 8.7% Total costs = (330 + 290 + 90 + 260) = 970.
Hence, profit/costs = 45/970 = 4.6% = 5%
1993 (10/125) × 100 = 8%
(Approximately)
1994 (5/135) × 100 = 3.7%
51. (b) If the interest component is not included in the
1995 (10/140) × 100 = 7.14% cost, the data can be represented as follows.
Hence, it is maximum for 1992. Year Cost Profits P/C x 100
45. (b) Profit in 1994 = 60. Profit in 1995 = 70. 1991 115 15 13.04%
Growth percentage in profit in 1995 over
1992 125 25 20%
 10 
1994 =   × 100 × 100 = 16.66%. Profit in 1993 140 20 14.28%
 60 
1996 will be (16.66 % of 70) + 70 = ` 82 lakh. 1994 165 -30 -
Solutions from 46-51: 1995 165 15 9.09%
The values in the graph can be represented in the table
Hence, we can see from the table that
given below. Here O.H. is overheads and Int. is interest,
maximum profit per unit cost is in 1992.
P/C is profit/cost.
52. (b) It can be seen from the graph that the southern
Year Raw Material Wages O.H. Int. Profit region showed the highest growth in number
1991 60 45 10 50 15 of households in all the income categories for
the period.
1992 50 55 20 55 25
53. (d) We only know the total number of households
1993 65 60 15 55 20
for all four regions combined. Nowhere have
1994 75 65 25 50 -30 they given the region-wise break-up of this
1995 80 65 20 50 15 value. In the light of this, the given question
Total 330 290 90 260 45 cannot be answered.
54. (b) It is very clear from the graph that the
46. (b) We can see that the increase in raw material percentage increase in total number of
has been maximum in 1993, viz. 15 points households for the northern region for upper
increase. middle income category is 200%.
y
o
u
rs
m
a
h
b
o
o
b
.w
o
rd
114 Bar Graph

p
re
55. (a) As seen from the table, the average income of Hence, the ratio of total income for these two

s
s
.c
high income group in 1987-88 is ` 75,000. categories in 1994-95 = 2476/2410 = 1.02.

o
56. (b) The total income of high income category in Hence, percentage increase in ratio

m
1987-88 is ` (5000 × 75000). (1.02 − 0.75)
The total income of upper-middle class = = 36%.
0.75
category in 1987-88 is ` (10000 × 50000). 57. (a) For northern region, we can draw the
Hence, the current ratio of their total incomes following table for 1987-88.
= 3 : 4 = 0.75 Total
Since the number of households in each Average
Households Income
category were equally distributed in all Category Household
in 1987-88 (Rs. in
Income
regions, we can have the following table for Millions)
high income category. Middle
10000 ` 30,000 300
Income
Households Percentage Households
Region Upper-
in 1987-88 increase in 1994-95 2500 ` 50,000 125
middle
North 1250 240% 4250 High
1250 ` 75,000 93.75
South 1250 425% 6562.5 Income
East 1250 175% 3437.5 Total 13750 518.75

West 1250 150% 3125 Hence, the average income for northern region
Total 5000 17375 518.75
= × 10 6 =
` 37, 727
13750

The average household income for high-
income category increased by 90%. Hence, Concept Deviator
average household income for this category in 1. (a) This question should be solved by observation
1994-95 = (75000 × 1.9) = ` 1,42,500 rather than by finding out the exact value. For

Hence, the total income for high-income India, FEI (1998) – (1997) / FEI (1997)
category in 1994-95 = (17375 × 142500) = 0.99 / 1.71 = 0.579
= ` 2,476 million For China 1.16/5.96 = 0.1946

The same table can be drawn for upper-middle For the rest, the numerators are higher and
class category as follows: denominators lower than India’s figures, so
Region Households Percentage Households their values are lower than those of India
in 1987-88 Increase in 1994-95
2. (d) Unless GDP values for these countries for
North 2500 200% 7500 both years are provided, we cannot compare
South 2500 340% 11000 the absolute FEI levels.
East 2500 125% 5625 3. (d) Consider (1) let for India FDI( 1997) = a and
West 2500 140% 6000
GDP (1997) = b then GDP (1998) = 1.07b as
it increases by 7%, From the graph for 1997
Total 10000 30125 a/b = 5.96 for 1998 FDI/1.07b =4.8 Or 4.8 x

The average household income for upper- 1.07 = 5.136 < 5.96 and hence it is false.
middle class category increased by 60%. Similarly (2) True because 5.136 < 5.96
Hence, the average household income for this Similarly (3) False because 0.72 x 1.02 =
category in 1994-95 = (50000 ×1.6) = ` 80,000 0.7344 < 1.71

Hence, the total income for high-income And (4) False because 2.5 x 0.95 = 2.375 <
category in 1994-95 = (30125 × 80000) 2.16
= ` 2,410 million But on the same logic (5) is true
y
o
u
rs
m
a
h
b
o
o
b
.w
o
rd
Bar Graph 115

p
re
4. (c) Let GDP of China = x and Let GDP of India Using sector weights of 20%, 15%, 10%, &

s
s
.c
= y 15% respectively.

o
Then 4.8 x x = 10 x (0.72 x y) Total rise in production = (0.20 × 1.278) +

m
Or x / y = 7.2 / 4.8 = 1.5 (0.15 × 1.1107) + (0.1 × 1.444) + (1.5 × 1.92)
Or (x-y) X 100/y= 50% = 0.7586 / 0.6 = 1.2643
5. (c) Again this is the type of question that should Total % rise would then be = 1.2643 – 1 / 1 ×
be solved by observation of the graphs and not 100 =25% (Approx.)
by calculating the exact values. From the four 10. (b) The given 4 sectors = 10+15+15+20=60%. So
bar charts, we can clearly see for electricity, other sectors account for (100 – 60) = 40% of
the percentage increases every year (also, the industrial production 1.50 = (0.6 × 1.25 + 0.4
% increase is more than 5%). Hence, in the × X)
given period, the highest growth will be that X = Total rise in other sector production
of electricity. X = 1.875
6. (b) Manufacturing 20% (decreased by 1 %) Hence 1.875 – 1 / 1 × 100 = 87.5% 
Mining & Quarrying – 15% (Increased by 11. (a) Refer to figure 1 of the question
2%) Electricity – 10% (Increased by 8.5%) Men hrs spent in coding (off-shore & onsite)
Chemicals – 15% (Increased by 0.5%) So = 430 + 100 = 530
total change in the year is given by –1% of 20 Now we will find the values for each of the
+ 2% of 15 + 8.5% of 10 + 0.5% of 15 = 0.2 + options
0.3 + 0.85 + 0.75 =1.7% (a) Offshore- design and offshore coding
7. (a) Again this is the type of question that should =100 +430 =530
be solved by observation of the graphs and
(b) Offshore coding = 430
not by calculating the exact values. From the
(c) Testing 280 +150 =430
bar graph we can find that annual growth is
(d) Offshore testing and coding = 280 + 430
always +ve except 1991, so barring 1991, the
= 710
value has increased year by year, or in other
Obviously (1) is correct. [All values are taken
words the value in a year is always more than
approximately]
more than that of previous year hence highest
value has to be that of last year i.e 1998. 12. (c) Total work onsite = 80 + 100 + 150 = 330
8. (b) We start with level of production at 100 Total work off shore = 100 + 430 + 280 = 810
and arrive at the figures with the percentage Hence percentage of onsite work = 330/330 +
increase/decrease given (approx..). 810) × 100 = 330/1140 × 100 = 28.94. 30% is
Mining and quarrying in various years is the closest value.
1989 – 100   1992 -108    1995 – 121     1998 - 122.5 13. (c) Total man-hours spent on site = 80 + 100 +
1990 - 105    1993 - 104   1996 - 118 150 = 330. Now we will find the values for
1991 - 107    1994 - 111   1997- 125 each of the options
9. (a) We will calculate the percentage increase in (a) The sum of the estimated and actual
each of the sectors. Production increase in effort for offshore design.= 80+100 =180
manufacturing sector = (1.09 × 0.99 × 1.025 (b) The estimated man-hours of offshore
× 1.07 × 1.08) = 1.278. Production increase coding.= 400
in mining sector = (1.05 × 1.02 × 1.01 × (c) The actual man-hours of offshore testing
0.96 × 1.07) = 1.1107. Production increase = 280
in electricity sector = (1.085 × 1.09 × 1.05 × (d) Half of the man-hours of estimated
1.0675 × 1.09) = 1.444. Production increase offshore coding= 400
in chemicals = (1.08 × 1.01 × 1.025 × 1.06 × (e) is the nearest to the 330 hence is the
1.08) = 1.920 correct answer
y
o
u
rs
m
a
h
b
o
o
b
.w
o
rd
116 Bar Graph

p
re
14. (a) From the given figure total man hours = 1140, Solution of 23 to 26.

s
s
.c
in this question it is given equal to 100 days 250 236 236

o
hence 50 day is equivalent to 570 man hours. 222

m
Now again we will check with the given 200 182
options and will find out the one nearest to it. 150
162

We will find that Coding with man hours 120 124 134
430 + 100 = 530 is the nearest to it. 100
80
15. (b) From the figure total off shore work = 100
+ 430 + 280 = 810. Given that 50% of the 50
30
offshore work carried out onsite and that is 0
equal to 810/2 = 405. 1995 1996 1997 1998 1999 2000
Now we will find out the amount of work Purana Naya
distribution. 23. (b) In 1999, total number of Naya mixer-grinder
Man hours for Design = 80 + 100 = 180 = 124
Man hours for Coding = 430 + 280 = 530 Number of Naya mixer-grinder disposed =
Man hours for Testing = 280 + 150 = 430 20% of 30 = 6
Hence they are in the ratio of 18 : 53 : 43
Number of Naya mixer-grinder carried over
Given that 50% i.e 405 off shore work carried
from 1998 = 80-6 =74
out onsite. Then effort by testing = 43/(18 + 53
Number of mixtures bought in 1999 = 124 –
+ 43) × 405 = 152.76 = 153. Off shore testing
74 = 50
work = 50% of total = 50% of 280 = 140.
24. (b) Number of Naya mixer grinder bought in
Hence proportion of testing offshore = 140/
1997 =30
(140+140+153) × 100 = 32.33% closest to 30%
Number of Naya mixer grinder bought in
16. (a) From the solution of previous question the
1998 = 80 - 30 = 50
ratio of design : coding : testing = 18:53:43.
The number of Naya mixer grinder disposed
As in the previous question 405 man hours is
at the end of 2000 = (30 * 0.2) + (80–30)*0.2
carried out onsite.
= 16
Then distribution of onsite work is as follows
25. (a) It is given that 10 Purana mixer-grinders were
Design = 80+ [18/114 × 405] = 144,
disposed off in 1997, the number of mixer-
Coding = 100 + [53/114 × 405] = 288,
grinders in circulation in 1997 should have
Testing = 140 + [43/114 × 405] = 293
been 162 – 10 = 152. But it is given as 182
Then amount of coding = 288 + 430/2 = 503
which imply that 30 were purchased.
Amount of testing = 293 + 270/2 = 433
Number of mixer-grinders disposed off in
Hence amount of coding done is greater than
1999 should be 6. (= 20% x 30)
amount of testing done
17. (b) We get the following ranking table: Number of mixer-grinders in circulation in
1999 should have been 222 – 6 = 216. But
Years WB UP TN MA KA GU AP from the table it is 236,
1996 – 97 7 6 2 1 5 3 4 Hence the number of purana mixer-grinders
1997 – 98 7 5 2 1 6 4 3 purchased in 1999 is 236 – 216 = 20.
1998 – 99 7 5 2 1 6 4 3 26. (d) Since data of the purana mixture grinder are
not available before 1995, so new purchasing
1999 – 2000 7 5 2 1 6 4 3 can not be determined.
27. (a) If neither any one joins or leave the institute,
2000 – 2001 7 4 2 1 6 5 3
the average age should increase by 1 every
18. (b)   19. (d)  20. 
(c)  21. (a)  22. (c) year since each member of the group becomes
y
o
u
rs
m
a
h
b
o
o
b
.w
o
rd
Bar Graph 117

p
re
older by 1 year. Hence ideally average age 30. (c) The break in the trend of the average age

s
s
.c
should increase by 1 year, every year, but if happened in the year 2001. Hence, on April

o
average age decreases that indicates one of the 2001, newly appointed faculty was 25 years

m
two possibilities old and this means he was 27 years old in April
Case (1): One member joins the group with 2003.
age 25 31. (c) Growth rate from 2006 to 2007 = (500
Case (2): One member retire from the group –380)/380 × 100 = 31.6% (we can approximate
at the age of 60 it to 30% as options given in the question are
not very close)
Since one break in each of the departments Growth rate from 2004 to 2005 = (270-
(of the average age) in four areas is due to 180)/180 × 100= 50%
the new faculty member, but if average age Hence required % change = (50–30)/50 × 100
decrease twice then we can say that it is the = 40%
area from which the faculty retired. The actual value is less than because we have
Two breaks in the trend occurred only for reduced the growth from 2006 to 2007, only
Finance. one option 35% is close to the 40% hence that
28. (d) Average age of marketing faculty is 49 1/3 at is the answer.
1st april 2000 and it has 3 members hence total 32. (a) Percentage growth from 2006 to 2007 = (500-
age = 148. Total age of Naresh and Dinesh is 380)/380 × 100 =31.6% approx 30%
49 + 52 = 101, hence age of the 3rd faculty Since in the year 2008 subscription in Europe
= 148-101 =47, hence his age on 1st April = 1.3 x 500 =650
2005 would be 47 +5 =52 years. Hence the required difference = 650–600=50
29. (c) Let new faculty joined the institute in 2001, 33. (a) Here again it is given in the question that we
Consider in 2000 total number of faculty in need to find the approximate value.
Finance = 5, and their average = 50.2 , Let total number of subscriber in 2003 is
1000 ( so that we can calculate the % increase
Hence total age in 2000 = 50.2 × 5 = 251
/decrease easily), then number of males in
And total age of these 5 in 2001 should be 251 2003 = 600 and Women 400
+ 5 = 256
YEAR MEN WOMEN
From the bar graph average age in 2001 = 49
and total number of faculty = 6 2003 600 400
Hence total age in 2001 = 49 × 6 = 294, and 2004 600 +30 = 630 400 + 40 =440
hence age of the newly appointed faculty is 2005 661.5 484
294 – 256 = 38years but that is not allowed. 2006 695 532.4
So faculty did not joined on 2001, and hence 2007 730 585.6
the reduction is caused by the retirement of
2008 766 644
one faculty member.
2009 804 708.6
Let the faculty joined in 2002, in 2001 total
age = 49 × 4 = 196 (since after one faculty left 2010 845 779
total number of faculty becomes 4) and total So total so in 2010 total number of subscriber
age of these 4 faculties in 2002 = 196 + 4= 200 = 845 + 779 = 1624
Total age in 2002 (from bar graph) = 45 × 5 Hence 62.4% = approx 62%
= 225, hence age of the newly appointed 34. (d) The gap between subscription revenues in US
faculty = 225 – 200 = 25 years and that and Europe in 2003 = 410 − 110 = 300 The
matches with the given information hence gap between subscription revenues in the US
new faculty member joins in 2002. and Europe in 2004 = 525 − 185 = 340
y
o
u
rs
m
a
h
b
o
o
b
.w
o
rd
118 Bar Graph

p
re
The percentage change in the gap between 37. (c) Given that there are 5 groups

s
s
.c
subscription revenues in the US and Europe DPS = 15→20→15→25 maximum increase

o
in the period of 2003–04 = (340–300)/300 = from 15 to 25 (66.67%)

m
40/300 DAV = 25→25→15→25 maximum increase
Similarly in the period 1005-06 = (270- from 15 to 25 (66.67%)
320)/320 = –50/320 CS= 15→20→35→10 maximum increase
from 20 to 35 (75 %)
In the period 2006- 07 = (210 – 270)/270 =
KV = 30→20→20→15 There is no increase
–60/270
Others = 15→15→15→25 maximum increase
In the period 2008-09 = (110 -180)/180 = from 15 to 25 (66.67%)
–70/180 Hence maximum % increase is in CS in 2003
In the period 2009-2010 = (100 -110)/110 = to 2004
10/110 38. (d) For DAV the number of students is 25, 25, 15 and
Out of these 5 values the change in gap is 25. Percentage change is 0%, 40% & 66.67%.
highest in 1008-09 39. (a) Only in the year 2002 when DAV is merged
Solution from 35-36 with DPS then total number of students
becomes 55.
Revenue Exp Profit Tax
Solution from 40-41
1999 150 90 30 30
Revenue Exp Profit Tax
2000 170 100 30 40
1999 150 90 30 30
2001 200 110 40 50
2000 170 100 30 40
2002 230 140 50 40 2001 200 110 40 50
35. (a) In 2000 % change = 33.33% 2002 230 140 50 40
In 2001 % Change = 25% 40. (a) In 2000 % change = 33.33%
In 2002 % Change = 20% In 2001 % Change = 25%
Hence maximum % change is in 2000 In 2002 % Change = 20%
36. (c) For 1999 the ratio = 150/30 =5 Hence maximum % change is in 2000
41. (c) For 1999 the ratio = 150/30 =5
For 2000 the ratio = 170/40 =4.25
For 2000 the ratio = 170/40 =4.25
For 2001 the ratio = 200/50 =4 For 2001 the ratio = 200/50 =4
For 2002 the ratio = 230/40 =5.75 For 2002 the ratio = 230/40 =5.75
42. (d) 1st complete the table from the given information.
Automobile Production Trends (in thousands)

Year Passenger Vehicles Commercial Vehicles Three Wheelers Total Production


2004 800 500 475 1775
2005 700 550 450 1700
2006 1025 675 475 2175
2007 1200 650 475 2325
2008 1250 600 350 2200

With the help of this table, percentage decrease in automobile production in 2005 over 2004 and in 2008
over 2007 be 4.33 (approx.) and 5.37 (approx.) respectively.
y
o
u
rs
m
a
h
b
o
o
b
.w
o
rd
Bar Graph 119

p
re
43. (b)

s
s
Export of automobile (in thousands)

.c
o
Year Passenger Vehicles Commercial Vehicles Three Wheelers Total Exports

m
2004 100 50 175 325
2005 25 50 100 175
2006 125 50 75 250
2007 150 50 100 300
2008 275 50 0 325


With the help of above data, growth in export of automobiles in the year 2006, 2007 and 2008 be 42.85%,
20% and 7.69% respectively.

44. (a)
Automobile Domestic Sales Trends (in thousands)

Year Passenger Vehicles Commercial Vehicles Three Wheelers Total Sales


2004 700 450 300 1450
2005 675 500 350 1525
2006 900 625 400 1925
2007 1050 600 375 2025
2008 975 550 350 1875
Percentage of earning (approx..) is contributed = 45%

45. (d) Total domestic sales of automobiles during (D) Vishal in 2010 = (1659 – 1064)/1659 > 30%
2004-2008 = 1450 + 1525 + 1925 + 2025 Hence sharpest decline is for Vishal in 2010
+ 1875 = 8800. Average domestic sales of 49. (b) In this question we will evaluate the options
automobiles during the year 2004-2008 = one by one
8800/5 = 1760.
(A) S. Stop in 2006 is 246.
46. (b) From the solutions of question 2, we have
(B) S. Stop in 2007 is 329.
the sales, this is subtracted from the previous
years of sales, we get increase in sales. 2006 (C) Pantaloon in 2006 is 267.
shows the highest increase. (D) Vishal in 2006 = 30.
47. (b) From the previous solutions we get the ratios Hence the maximum increase in the cost per
and observe that in 2008 production and square feet is shown by S. Stop in 2007.
domestic sales both are decreasing. 50. (d) The cost per square feet of the four retailers in
48. (a) In this question we will evaluate the options FY07 is = 2044+2464+1751+1525 = 7784.
one by one The cost per square feet of the four retailers in
(A) Westside in 2005 = (2411 – 1724)/2411 FY10 is = 1396+2230+1064+1051 = 5741.
< 30% Change in cost per square feet is  = 7784–
(B) Pantaloon in 2008 5741 = 2043.
= (2044 – 1656)/2044 = 20% Percentage change 2043/7784 × 100 = 26.2%
(C) S. Stop in 2009 This change happen in 3 year hence the
= (2419 – 2197)/2419 = 10% average rate of change is 8.7% 
y
o
u
rs
m
a
h
b
o
o
b
.w
o
rd
120 Bar Graph

p
re
51. (c) The CO2 emission in US, China and Japan That in 1999 = 9.38%.

s
s
.c
after 2 years would be 918.75, 903.44 and Similarly in 2004 = 5.7% in 2005 = 13.5%

o
861.33 million ton respectively. Hence the highest percentage increase in

m
Similarly from the given condition the per females is in 2005.
capita income of US, China and Japan after 56. (d) From the given data
2 years would be 312.12, 292.03 and 254.61 The number of educated male in 2002 is =
respectively. 302 million.
Hence the ratio of CO2 emission to per capita The number of uneducated female in 2002 is
income in US, China and Japan is 2.9, 3.1, 3.4 approximately 108 million.
respectively. The number of educated male in 2003 is
52. (b) Since it is given that the ratio of CO2 emission approximately is 378 million.
to per capita income is 0.75 hence the The number of uneducated female in 2003 is
CO2 emission of US and China should be 225 approximately 168 million.
and 202.5 million ton respectively. Hence the percentage change in female
The required reduction in CO2 emission uneducated is (168-108)/108 × 100 = 55.5%  
of US and China should be 975 and 977.5 57. (c) From the given information in 2005 the
million tons respectively. It is given that for number of people living in urban population
every 1.25 units of carbon credit, 0.5 million is 68% of 600 = 408.
ton of CO2 emission is compensated. Then the rural population in 2005 = 900 – 408
So in a year, 26 million tons of CO2 emission = 492
is compensated. The number of rural population in 2010 is =
Then the number of years required are 12×1100/55 = 240
977.5/26 = 38 years                                          Hence required ratio = 492/240= 2.05
53. (c) Total CO2 emissions of these countries at the
beginning = 450+450+350+400+400 = 2050 Solutions from 58-62 IIFT 2012
Since the ratio of CO2 emissions to per capita
Production by Tata = 5.4
income of each country becomes 50% of
present value, hence total emission of these Production by Nirma = 3.4
countries = 2050/2 = 1025 Tata’s share in percentage = (5.4/40) x 100 = 13.5%
Total emissions of 5 countries = 350×5 = 1750 Hence from the given condition Solvay’s share has to
They can sell 1750-1025 = 725 be greater than 13.5% And FMC’s share has to be less
In 3 years total CO2 emissions they can sell = than 13.5%
725×3 = 2175
Hence, we can conclude that statement 2 and 4 are
Since for 0.5 emissions carbon credit is
correct and statement 5 is wrong
1.25 hence for 2175 emissions it would be
2175×2.5= 5437. The actual value is less than So Nirma’s share = (3.4/40) x 100 = 8.5%.
this as we have ignored the percentage increase 63. (d) The ratio of Ex-servicemen to the total
of 2%, 2.5% and 3.5% in per capita income. employee are as follows:
54. (b) From eliminating options we will find that (B) 6 8 30 25 60
is correct.                                , , , ,
52 65 210 30 330
55. (d) Evaluate the given options we will get
The percentage increase in number of females
Out of these 25/30 is the maximum hence the
in 1996 = 10% value is the highest for level 4.
y
o
u
rs
m
a
h
b
o
o
b
.w
o
rd
Bar Graph 121

p
re
64. (d) From the given bar graph we can conclude that Out of these four hundred (400), 100 were employed.

s
s
1

.c
For level 1 vacant posts = 3. Number of unemployed, who were graduates = x

o
For level 2 vacant posts = 0. 5

m
800 = 160. So, total number of graduate employed =
For level 3 vacant posts = 15. 800 – 160 = 640.
For level 4 vacant posts = 225. 1. (c) Required number of graduates = (55+36)% of
For level 5 vacant posts = 0. 400 = 364.
Hence the highest reduction = 225–0 = 225. 
2. (b) Employed never used cigarette = 15% of (100
65. (c) Similar to question number 60 from the graph + 640) = 111.
we will find only in level 3 the percentage of
3. (b) Required ratio = 36% of 400 : 30% of (100 +
ex-policemen less than 4.5% of the employees
640) = 144 : 222 = 24 : 37
at that level.
66. (e) Data Insuuficient… 4. (a) Required ratio = 30% of 400 : [12% of 400 +
160] = 120 : [48+160] = 120 : 208 = 15 :26.
67. (a) Revenue of Yahoo in 2004 = 250 and in 2006
= 1200 5. (d) Maximum Revenue = Avg. price X no. of
So, percentage increase = (950/250) × 100 transactions = 20 × 25 = 500
= 380% this increase is in two years.
Means, Air ticket gives higher revenue.
Average percentage increase = 380/2 = 190 6. (c) Avg. price of airline tickets = ` 20.
Revenue of Facebook in 2004 = 350 and in
Earlier Revenue = (115 × 20) After increment
2006 = 2000 Revenue becomes = 115 × (20 × 125/100) ×
So, percentage increase = (1650/350) × 100 1.2 = 115 × 25 × 1.2% change = (115 × 30-
= 235.71 115)/115 × 100 = 21%
So, percentage difference in both the values 7. (d) Let us eliminate the options one by one.
= [(235.71 – 190)/190] × 100 = 24.05 (A) → The statement is wrong as per the
None of the options are less than 35%,so the given information.
question is incorrect.
(B) → 3.2 MT of natural soda ash was built
However, “None of the above” is not one of from 2005 to 2008 and that is the reason for
the given options and hence we will have to the increase and hence the reason for change
take the option closest to 24.05 as the correct or increase in the trend of Tata. Hence it is true.
answer.
(C) → Since the year is not mentioned hence
68. (a) Percentage increase in Google’s growth
the data given in this statement is incomplete.
between 5th and 6th year = (3250 – 1500)/1500
× 100 = 116.67. Hence required value of (D) → The data given in this statement
Facebook revenue after 6th year = 750 × may be correct as Tata chemical might have
216.67 = 1625.05 acquired 0.3 MT of natural soda ash in 2007.
Since there is no data given about the year
So, the closest answer is 1600.
2006. So, depending on the data of production
Concept Eleminator in 2006 it might have acquired 0.3MT of
natural soda ash in 2007. So, this also leads
Solutions For Question 1-4:
to change in trend of Tata chemicals.
According to question, The total number of
1
So, two statements are true but no unique
undergraduates = × 1200 = 400. reason can be derived.
3
y
o
u
rs
m
a
h
b
o
o
b
.w
o
rd
122 Bar Graph

p
re
8. (d) This is one of the questions that could be Hence Statement 1 is true, 2 is false

s
s
.c
solved by logic. Since it is given that trend Statement 3: SS/TS in 2005 = 0.9/0.9 = 1

o
of total global production is the same as SS/TS in 2007 = 2.2/2.5 = 22/25

m
Tata’s synthetic production, and Tata has 0% And 22/25 <1 hence Proportion has decreased.
increased from 2007 to 2008 And hence Statement 3 is true
Increase in global production = 0%. 10. (d) In order to find Tata Chemicals share of the
9. (a) Let us evaluate given statement one by one global production in 2008, we require the
Statement 1: NS/SS in 2001 = 10/25 = 2/5 global production in 2008. However, the data
Also NS/SS in 2006 = 10/30 = 1/3 is given in question no 68 but it is mentioned
The proportion has decreased from 2/5 (i.e that the data is for that particular question only.
40%) to 1/3 (i.e 33.33%) 11. (c) Given that Total Production = 40,
y
o
u
rs
m
a
h
b
o
o
b
.w
o
rd
4
Percentage of girls in various subjects

p
re
Law 30%

Pie

s
14%

s
Computers 10%

.c
30%

o
Maths 2%

m
14%

Chart
Political Science 30
10% Arts 14%
30%
2% Biology 14%

Topics No. of Questions Level


Concept Applicator 25 BANK PO + MAT (basic)
Concept Builder 41 BANK PO + MAT (advance)
Concept Cracker 27 CAT, IIFT, XAT (basic)
Concept Deviator 18 CAT, IIFT, XAT (advance)
Concept Eleminator 11
Total 122

INTRODUCTION
Pie charts are the circular representation of the data. Generally the data in the question represents the break
ups of a whole into its parts. The share of each part in the pie diagram is proportionate to its share of the whole
θ θ
data. As we know that the total angle is 360 degree. Hence angle of sector = × πr2 or × total value
360 360
Example:
Breakup of 1500 Employees
I Breakup of 800 Male
Across The Scales Employees Across The Scales
II I
8%
22% 10% II
III
14% 9% 24% III
IV
12% IV
12% 15% V 18% V
11%
10% VI VI
19% 16%
VII VII
1. How many females are working in Scale II ? 2. What is the ratio of male to female employees
working in Scale V?
(a) 144 (b) 81 (a) 7 : 8 (b) 2 : 1
(c) 96 (d) 138 (c) 8 : 7 (d) 1 : 2
Sol. (b) Total no. of employees in scale v are 12%
Sol. (b) As total number of employees are 15% of of 1500 = 180 total no. of male employees
1500 = 225 total male employee in scale II in scale v are 12% of 800 = 96 total no. of
are 18% of 800 = 144 therefore, total no of female employees in scale v are (180 – 96)
96
female employee in scale II = 225 – 144 = 81 = 84 required ratio = = 8 : 7
84
y
o
u
rs
m
a
h
b
o
o
b
.w
o
rd
124 Pie Chart

p
re
s
s
.c
o
m
Directions (Qs. No. 1-5): Study the following pie- 5. How many students prefer beverage B and
chart and answer the questions given below: Beverage E together?
[IBPS PO – 2011,Memory Based] (a) 2312 (b) 2313
(c) 2315 (d) 2318
A
F, 21

Directions (Qs. No. 6-10): Study the following pie –


A, 79.2° B chart carefully to answer these questions:
.6

E, [IOB PO 2011]
°

C
Total number of passengers in six different trains = 4800
B, 57.6° D Percentage wise distribution of passemngers
D, 122.4° A B C D E F
C,

E
14
.4°

F 12%
22%
nA
Preferences of students for six beverages A, B, C, D, E Tra
i
9%
Train F
and F in terms of degrees of angle in the pie-chart in B
Tra
Total No. of students = 6800
1. What is the difference between the total numbers of Train E Train C
students who prefer beverage A and C together and 19% 17%
the total number of students who prefer beverages Train D
D and F together? 21%
(a) 959 (b) 955
(c) 952 (d) 954
2. What is the ratio of the number of students who 6. What was the average number of passengers
prefer beverage F to the number of students who travelling in Train A, Train C and Train F together?
prefer beverage A? (a) 816 (b) 826
(a) 3 : 11 (b) 3 : 13 (c) 824 (d) 812
(c) 6 : 11 (d) 5 : 11 7. If the cost of one ticket is `124, what is the total
3. The number of students who prefer beverage E and amount paid by passengers of Train B? (Assuming
F together is what per cent of the total number of all the passengers purchased tickets and cost of
students? each ticket is equal)
(a) 18 (b) 14 (a) `53,658 (b) `53,568
(c) 26 (d) 24 (c) `53,558 (d) `53,468
4. The number of students who prefer beverage C 8. The number of passengers in Train E is
is approximately what percent of the number of approximately what percentage of the total number
students who prefer beverage D? of passengers in Train B and Train D together?
(a) 7 (b) 12 (a) 63 (b) 69
(c) 18 (d) 22 (c) 75 (d) 54
y
o
u
rs
m
a
h
b
o
o
b
.w
o
rd
Pie Chart 125

p
re
9. What is the difference between the number of 14. What is the cost estimated by the family on painting

s
s
and flooring together?

.c
passengers in Train C and the number of passengers

o
in Train A? (a) 36,500 (b) 34,800

m
(a) 280 (b) 250 (c) 36,000 (d) 34,500
(c) 230 (d) None of these 15. The family gets a discount on furniture and pays
10. What is the total number of passengers in Train D, 12% less than the estimated cost on furniture. what
Train E and Train F together? is the amount spent on furniture?
(a) 2796 (b) 3225 (a) 13,200 (b) 14,526
(c) 2976 (d) 3125 (c) 13,526 (d) 13,728
Directions (Qs. No. 11-15): Study the following pie – Directions (Qs. No. 16-20): Study the following pie-
chart carefully and answer the questions given below. chart carefully to answer these questions.
[Corporation Bank 2011]
[Andhra Bank PO 2011]
Total Number of passengers = 8500
Cost estimated by a family in renovation of their house
Architect's fees
Percentage of Passengers
11%
Train A
Grills & Windows Interior 13% Train L
10% decoration 15%
19%
Curtains & Train Q
Cushions Painting 19% Train M
10% 15% 20%

Flooring Train R
Miscellaneous Furniture 14% 9%
8% Train S
13% 24%
Cost estimated by a family in renovation of their house
Total Estimated cost is `1,20,000 16. What was the approximate average number of
11. What is the difference in the amount estimated passenger in Train-S, Train-M and Train-L together?
(a) 1521 (b) 1641
by the family on interior decoration and that on
(c) 1651 (d) 1671
architect’s fees?
17. If in Train-R 34 percent of the passengers are
(a) 10,000 (b) 9,500
females and 26 percent are children, what is the
(c) 7,200 (d) None of these number of males in that train?
12. During the process of renovation, the family (a) 306 (b) 316
actually incurred miscellaneous expenditure of (c) 308 (d) 318
` 10,200. The percentage of the miscellaneous of 18. The number of passengers I Train-Q is
the total estimated cost is? approximately what percentage of the total number
(a) 9.5% (b) 9% of passengers in train-A and Train-R?
(c) 8.5% (d) 10.55% (a) 90 (b) 70
13. Other than getting the discount of 12% on (c) 75 (d) None of these
the estimated cost of furniture and the actual 19. Which train has the second highest number of
miscellaneous expenditure being ` 10,200 instead passengers?
of the estimated one, the family’s estimated cost (a) A (b) Q (c) S (d) M
is correct. What is the total amount spent by the 20. How many more per cent (approximately) number
family in renovating its house? of passengers are there in Train-M as compared to
(a) ` 1,16,728 (b) 1,15,926 the number of passengers in Train-L?
(c) 1,19,500 (d) None of these (a) 29 (b) 49 (c) 43 (d) 33
y
o
u
rs
m
a
h
b
o
o
b
.w
o
rd
126 Pie Chart

p
re
Directions (Qs. No. 21-25): Study the pie – chart 21. What is the approximate percentage of boys in the

s
s
.c
carefully to answer the following questions. school?

o
Percentage of students enrolled in different (a) 34 (b) 56

m
activites in a school Total number of students = 3000 (c) 28 (d) 42
22. How many boys are enrolled in singing and craft
Swimming
Craft
together?
16%
25% (a) 505 (b) 610
Singing (c) 485 (d) 420
21% 23. What is the total number of girls enrolled in
dancing swimming and drawing together?
24%
Drawing (a) 480 (b) 525
14%
(c) 505 (d) 495
Percentage break - up of girls enrolled in these 24. The number of girls enrolled in dancing is what per
activities out of the total students cent of the total numbers of students in the school?
Total number of girls = 1750
(round off to two digits after decimal)
Swimming Craft
(a) 12.35 (b) 14.12
14% 22% (c) 11.67 (d) 10.08
25. What is the ratio of the number of girls enrolled
Singing in swimming to the number of boys enrolled in
28% Dancing swimming?
20%
(a) 47 : 49 (b) 23 : 29
Drawing
16% (c) 29 : 23 (d) 49 : 47

Directions (Qs. No. 1 - 4): In the chart given below, A, 1. What is the angle of the pie chart showing the
B, C, D and E represents the costs of paper, printing, expenditure incurred on royalty?
binding, miscellaneous and the royalty respectively in (a) 15° (b) 24°
publishing a book. (c) 48° (d) 54°
2. The marked price of the book is 20% more than the
A 20% CP. If the marked price of the book is Rs. 30 then
15% 20% what is the cost of the paper used in a single copy
B 25% of the book?
10% (a) `6 (b) `5
C 30% (c) `4.5 (d) `6.50
25% 3. Which two expenditures together will form an
D 10% angle of 108° at the centre of the diagram?
30%
(a) A and E (b) B and E
E 15%
(c) A and D (d) D and E
y
o
u
rs
m
a
h
b
o
o
b
.w
o
rd
Pie Chart 127

p
re
4. If the difference between the two expenditures

s
(a) `50,000 (b) `1,00,000

s
.c
are represented by 18 degrees in the diagram then (c) `45,000 (d) `60,000

o
these expenditures must be: 11. For the same data as given in the previous question,

m
(a) B and E (b) A and C what is the cost of editorial content development ?
(c) B and D (d) none of these (a) `45,000 (b) `30,000
Directions (Qs. 5-11): Following 7 questions are based (c) `15,000 (d) `20,000
on the following pie chart which gives the expenditure Directions (Qs. No. 12-19): These questions are to
incurred in printing a magazine. be answered on the basis of the pie chart given below
Printing Costs showing how a person’s monthly salary is distributed
over different expense heads. [MAT. 1999]
24% Paper Costs
Petrol
30%
Miscellaneous 15% 10%
Entertainment
Transportation 15%
10% Savings
Binding
2% 30% Clothes
18% 4% Promotion Costs
12% 23% House Rent
Editorial Contgent
7%
Development Food
5. What is the angle for the sector representing paper 12. For a person, whose monthly salary is `6,000 p.m.
cost ? how many items are there on which he has to spend
(a) 10° (b) 36° more than `1,000 p.m. ?
(c) 23½° (d) 45° (a) 1 (b) 2
6. What should be the central angle of the sector (c) 3 (d) 4
representing transportation charges ? 13. For the same person, an expenditure of `1,800
(a) 4° (b) 8.4° p.m. takes place on :
(c) 12.4° (d) 14.4° (a) Petrol (b) House Rent
7. If the editorial content development cost is `30,000, (c) Food (d) Clothes
then the cost of transportation can be expected to 14. The annual savings for such a person will be
be approximately :
(a) `4,000 (b) `400 (a) `5,000 (b) `10,000
(c) `12,000 (d) `2,000 (c) `15,000 (d) `16,560
8. For a given issue of the magazine, the miscellaneous 15. The monthly salary for a person who follows the
cost is `2000 and the print-run is 12500 copies. same expense pattern, but has a petrol expense of
What should be the sale price per copy would have ` 500 p.m is
been : (a) `2,500 (b) `3,000
(a) `5 (b) `7.50 (c) `5,000 (d) `6,500
(c) `8 (d) `8.40 16. The percentage of money spent on clothes and
9. If for the same data as given in the previous savings is equal to which other single item of
question, the print-run is 50000 copies. What expense?
should be the sale price per copy would have been : (a) Petrol (b) House Rent
(a) `5 (b) `2 (c) Food (d) Entertainment
(c) `2.10 (d) `2.20 17. The angle made at the center of the pie chart by the
10. If the promotional costs for a given issue of the sector representing the expense on petrolis :
magazine is `9,000, then the total expenditure in (a) 30° (b) 45°
bringing out that issue of the magazine is : (c) 36° (d) 90°
y
o
u
rs
m
a
h
b
o
o
b
.w
o
rd
128 Pie Chart

p
re
18. Given that the pie chart is for a salary of `6,000 22. For Political Science, what is the ratio of boys and

s
s
.c
p.m., what would be the ratio of the radius of this girls?

o
pie chart to a pie chart for a person with a salary of (a) 4 : 3 (b) 3 : 4

m
`1,500 p.m. ?
(c) 2 : 3 (d) 4 : 5
(a) 2 : 1 (b) 1 : 2
(c) 1 : 2 (d) 2 2 : 1 23. The number of girls studying Arts is what percent
19. For a person with a salary of Rs. 1,500 p.m., the more than the number of boys studying Arts?
annual savings would be : (a) 170% (b) 150%
(a) ` 4,140 (b) `2,500 (c) 80% (d) 250%
(c) `2,100 (d) `4,000
Directions (Qs. 24-28): Study the following graphs
Directions (Qs. No. 20-23): Study the following pie-
which show the number of workers of different categories
charts carefully to answer the following questions.
Percentage of Students in a college studying various of a factory for two different years. The total number
subjects and the percentage of Girls out of these of workers in 1998 was 2000 and in 2000 was 24000.
Total students: 1800 1998 2000
(1200 girls + 600 boys) P 13% 8% 13% 10% U 10%
Q 18% 18% P 11%
Percentage of students in various subjects 11%
R 15% 22% 18% Q 25%
Law 20% S 24% 20% R 16%
25%
13% 20% Computers 15% T 22% 24% 15% 16% S 20%
U 8% T 18%
12% Maths 5%
24. In which of the categories is the number of workers
15% Political science 35% same in both the years?
35% 5% Arts 12% (a) P (b) S
(c) R (d) T
Biology 13%
25. Find the percentage increase in the number of
workers in category U in 2000:
Total Girls: 1200 1
(a) 25% (b) 33 %
Percentage of girls in various subjects 3
2
Law 30% (c) 50% (d) 66 %
3
14% Computers 10% 26. What is the total number of increased workers for
30%
Maths 2% the categories in which the number of workers has
14%
Political Science 30 been increased?
(a) 468 (b) 382
10% Arts 14%
30% (c) 408 (d) 168
2% Biology 14% 27. Which categories have shown decrease in the
number of workers from 1998 to 2000?
20. The number of girls studying Arts in college is (a) P (b) Q
(a) 242 (b) 168 (c) R (d) T
(c) 120 (d) 276 28. Find the maximum difference between the number
21. For which subject is the number of boys are of workers of any two categories taken together for
minimum? any one year and that of any two for the other year.
(a) Law (b) Biology (a) 660 (b) 416
(c) Arts (d) Maths (c) 636 (d) 502
y
o
u
rs
m
a
h
b
o
o
b
.w
o
rd
Pie Chart 129

p
re
Directions (Qs. 29-31): The pie charts given below A = Private sector

s
s
shows the funding arrangements for National Highways B = Socially-oriented sector (Plan)

.c
o
development projects-phase1. Study the chart carefully C = Socially-oriented sectors (Non-Plan)

m
to answer these questions. D = Securities guaranteed by the central
Funding arrangements for NHDP - 1 government
(total cost : Rs. 30,000 Crores) E = State government securities
1,690 F = Central government securities
Cess/Market borrowings
1,902 32. The percentage of gross investments in State
16,546
2,009 Government Securities is nearly :
B-External assistance 7,862 (a) 7.1% (b) 7.8%
A 2,009 (c) 8.6% (d) None of these
16,546
7,862 33. The central angle made by the sector indicating the
SPVP 1,902 investments made in the socially oriented sectors is
Toll 1,690 nearly:
(a) 123 (b) 132
29. Near about 25% of the funding arrangement is (c) 126 (d) None of these
through 34. The investment in Private sector is nearly how
(a) External assistance much per cent higher than the investment in the
(b) Cess/market borrowing State Government Securities ?
(c) Annuity (a) 66% (b) 54%
(d) SPVS (c) 46% (d) None of these
30. The angle of the segment formed at the centre of 35. The ratio, of the area of the circle above ROU to
the pie chart, representing Cess/Market borrowings the area of the circle below it is nearly :
is approximately
(a) 1 (b) 0.966
(a) 100° (b) 90°
(c) 0.94 (d) 0.92
(c) 80° (d) 200°
36. The investment in Socially oriented sectors (plan
31. If the toll is to be collected through an out
and non plan) is ……. Than the investment in the
sourced agency by allowing a maximum of
Government Securities (central and the state) by …..
10% commission. How much amount should
be permitted to be collected by the outsourced (a) More, 4 crore (b) More, 1 crore
agency, so that the project is supported with Rs. (c) More, 11 crore (d) Less, 106 crore
1,690 Crores? Directions (Qs. No. 37-41): The pie charts below show
(a) `1878 crores (b) ` 1,690 crores the break-up of the total revenue and total sales volume
(c) `16,900 crores (d) Inadequate data for a company, for the year ended 2003.
Directions (Qs. No. 32-36): The gross investment Total revenue = `11, 39,200 million and total sales
of Life Insurance Corporation of India (in crore of volume = 653610 unit. Based on the data provided, in
rupees) in different sectors are shown in the pie chart both the figures, answer the questions that follow.
given below. The letters denoting the various sectors Total revenue
are: Esteem, 17%
B A
A 17%
107 183 3% Zen, 3%
F 48%
Omni, 21%
E 21%
C 458 Alto, 11%
454 F
D
C M 800, 48%
227 110 11%
D B Break-up of total revenue
E
y
o
u
rs
m
a
h
b
o
o
b
.w
o
rd
130 Pie Chart

p
re
39. If volume of sales of Omni was increased by 25%

s
Total sales volume

s
and this sales increase ‘was’ entirely at the cost of

.c
Esteem, 20%

o
M 800, then what is the percentage decrease in M

m
20% Zen, 3% 800 volume sales?
3% (a) 1.3% (b) 2.4%
58% Omni, 11% (c) 4.3% (d) 4.74%
11% 40. If the net revenue from Alto was to be excluded,
Alto 7% what percent of the revenue would Zen form?
7% (a) 3.27% (b) 3.17%
M 800, 58%
(c) 3.37% (d) 3.47%
Break-up of total sales 41. Which of the following is true?
(i) The sum of the revenue generated by Zen and
37. What is the net revenue per Esteem sold,
Omni is lesser than twice that of Alto.
approximately?
(ii) If the revenue of Esteem and Alto be
(a) `14, 81,000 (b) `13, 48,000 interchanged, Alto’s revenue per unit would
(c) `15, 32,000 (d) `13, 26,000 be more than that for Zen.
38. The highest net revenue per vehicle comes from: (iii) The revenue per unit of Esteem is greater
than that for M 800.
(a) M 800 (b) Omni (a) (i) only (b) (ii) only
(c) Esteem (d) Zen (c) (iii) only (d) Both (ii) and (iii)

Directions (Qs. No. 1-3): The following graphs Gross Receipts for 2001 : ` 75,00,000
present receipts of all the major fast food restaurants Gross Receipts for 2002 : ` 85,50,000
from 2001-2003. Use them to answer these questions: Gross Receipts for 2003 : ` 81,00,000
2001 MB – Mega Bite
12.80 7.30 Others
CC – Crunchy Chop
% % PP – Pizza Pie
CC
1. The gross receipts for 2001 are approximately
33.60 46.30
MB what per cent of the gross receipts for all the three
% %
years?
PP (a) 30%
(b) 46.3%
5.3% 9.30% (c) 46.7%
24.6 Others (d) Cannot be determined
12.30%
% 40.6 37.00% CC 2. Over all the three years, the average percentage
29.50 % MB of gross receipts for Crunchy Chop exceeds the
%
41.40%
PP average percentage of gross receipts for pizza pie
by approximately:
2002 2003 (a) 53% (b) 30% (c) 23% (d) 8%
y
o
u
rs
m
a
h
b
o
o
b
.w
o
rd
Pie Chart 131

p
re
3. The gross receipts earned by the other restaurants Directions (Qs. No. 8-11): Refer to the pie-chart given

s
s
.c
in 2002 amount to precisely: below: [CAT 1994]

o
(a) `18, 10, 650 (b) `7,53,300

m
Distribution of material in Ghosh Babu's
(c) `5,47,500 (d) `4,53,150 body (as % of of total body weight
Directions (Qs. No. 4-7): These questions are based
on the following pie-charts. Other Dry
Total Investment Funds = `11 crore 5 lakh. Material
15%
17.90% Govt bonds &
Protein
securities 48.30 % 15%
High risk stocks Water
70%
24.90% 48.30% 8.9%
Mutual Funds
24.90%
Blue-chip stocks
8.90% Occurance of Proteins in different
17.90%
Total Investment profile organ's of Ghosh Babu's Body
Govt Bonds & securities
Municipal 7% -9% yield Skin
bonds 56% 17%
26%
yield 20%
11%
Treasury 18% Eyes
18% 56% 65% Bones
notes 18% yield 17.9% 34%
21%
State issued
bonds 26%
4. According to these graphs, approximately how Muscles
much money from the investment portfolio was 34%
invested in high-risk stocks?
(a) `98,00,000 (b) `10,10,000 8. What fraction of Ghosh babu’s weight consists of
(c) `9,00,000 (d) None of these muscular skin protein?
5. Approximately how much money belonging to the (a) 1/13
investment portfolio was invested in State-issued (b) 1/30
bonds? (c) 1/20
(a) `4,50,00,000 (b) `3, 39, 50,000 (d) Cannot be determined
(c) `2,87,00,000 (d) None of these 9. Ratio of distribution of protein in muscle to the
6. Which of the following earned the least amount of distribution of protein in skin is
money for the investment portfolio? (a) 34 : 11 (b) 3 : 10
(a) Government bonds and securities 1
(b) State-issued bonds (c) 1 : 3 (d) 3 : 1
(c) Municipal bonds 2
10. What percent of Ghosh Babu’s body weight is
(d) none of the above
made up of skin
7. Which of the following was the greatest?
(a) The amount of money invested in high-risk (a) 0.15
stocks (b) 10
(b) The amount of money invested in State- (c) 1.2
issued bonds (d) Cannot be determined
(c) The amount of money invested in municipal 11. In terms of total body weight, the portion of
bonds which yielded between 7% and 9% material other than water and protein is closest to
(d) The amount of money invested in municipal (a) 3/20 (b) 1/15
bonds which yielded over 9% (c) 85/100 (d) 1/20
y
o
u
rs
m
a
h
b
o
o
b
.w
o
rd
132 Pie Chart

p
re
Direction (Qs. No. 12-16): Answer the questions based

s
(a) `104 lakh (b) `9 lakh

s
.c
on the following pie charts. [CAT 1995] (c) `12.8 lakh (d) `15.6 lakh

o
Operating Profit ` 160 Lakh (1991-92)

m
Directions (Qs. No. 17-21): Refer the given pie chart
Interest 40% Tax 12% Retained Profit 20% to answer these questions.
Dividend 8% Depreciation 20% X’s Domestic Budget
A = Expenditure on food
B = House rent
20% C = Entertainment
40% D = Education
8%
E = Material
20% F = Savings
12% A = 25%
12% B = 18%
Operating Profit ` 130 Lakh (1990-91) 25%
13% C = 9%
Interest 30% Tax 9% Retained Profit 25%
D = 23%
Dividend 8% Depreciation 28% 23% 18% E = 13%
9% F = 12%

28% 30% X earns a salary of ` 9,288 plus 10% HRA per month
17. What is X’s actual house rent ?
8% 9% (a) `1,661 (b) `1,827
25% (c) `1,287 (d) None of these
18. If X wants to save at least `30,000 in two years,
12. The operating profit in 1991-92 increased over that then how much extra should he save in the second
in 1990-91 by year? (Currently, the rate of interest for savings is
(a) 23% (b) 22% 12% p.a. paid annually)
(c) 25% (d) 24% (a) `1,000
13. The interest burden in 1991-92 was higher than (b) `500
that in 1990-91 by (c) He does not need to save extra
(a) 50% (b) `25 lakh (d) None of these
(c) 90% (d) `41 lakh 19. After his savings and payment of house rent, how
14. If on an average, 20% rate of interest was charged much money is left with him for other things ?
on borrowed funds, then the total borrowed funds (a) `7,106 (b) `7,601
used by this company in the given two years (c) `7,016 (d) None of these
amounted to 20. How much does X spend on entertainment and
(a) `221 lakh (b) `195 lakh food?
(c) `368 lakh (d) `515 lakh (a) `415 (b) `835
15. The retained profit in 1991-92, as compared to that (c) `914 (d) None of these
in 1990-91 was 21. If in the next year, a 10% increase in the cost of
(a) higher by 2.5% (b) higher by 1.5% food and entertainment occurs due to inflation,
(c) lower by 2.5% (d) lower by 1.5% then what is the new percentage of savings with
16. The equity base of these companies remained the same salary ?
unchanged. Then the total dividend earning by the (a) 8.2% (b) 8.6%
share holders in 1991-92 is (c) 9.2% (d) None of the above
y
o
u
rs
m
a
h
b
o
o
b
.w
o
rd
Pie Chart 133

p
re
Directions (Qs. No. 22-26): Study the diagram given

s
(a) `86 lakh (b) `113 lakh

s
.c
below and answer these questions. [MAT. 2001] (c) `115 lakh (d) `111 lakh

o
0.50% 26. If the total cost of labour increases from 0.6 lakh to

m
2.4 lakh, then what percentage of increases does it
11% A represent ?
(a) 75% (b) 25%
B (c) 200% (d) 300%
49% Directions (Qs. No. 27): Refer to the following pie
39.50% C chart and answer the question that follows. The chart
shows the no. of units produced in degrees, by Company
D
X in different States of India for the quarter July-Sep
2010. [IIFT]
A = Raw material cost WB
B = Packing material cost Bihar
9
C = Fixed manufacturing expenses 45
UP
D = Labour cost
104.4 MP
22. If the total value in rupees of all the sectors is 36
`128.3 lakh, then calculate the value of D in rupees, Rajasthan
(a) `0.06 lakh (b) `0.6 lakh 32.4
Orissa
(c) `0.006 lakh (d) `6.0 lakh 10.8
18
Delhi
23. If the total cost of production doubles in a period of 21.6 Haryana
one year, then what will be the value of D ? 7.2 28.8 18
Punjab
10.8
(a) `10.3 lakh (b) `1.3 lakh 7.2 10.8 Maharashtra
(c) `570 lakh (d) `50.7 lakh 27. By how many units does the number of units
24. If packing cost increased by 2% how much amount produced in Bihar exceed the number of units
will be involved in packing cost ? produced in Madhya Pradesh, if the total production
(a) `25.6 lakh (b) `52.5 lakh in the quarter is 72,000 units?
(c) `52 lakh (d) `50 lakh (a) 2300 (b) 2520
25. Packing and raw material costs together represent (c) 3516 (d) 2860
`……… of the total cost of production.

Directions (Qs. No. 1-6): These questions are based


G : Other East
on the pie charts given below. [CAT 1999] A : USA D : U.K. J : Other L.D.Cs
Europe
Consider the information provided in the figure below
relating to India’s foreign trade in 1997 98 and the first
B : Germany E E : Japan H : OPEC K : Others
eight months of 1998-99. Total trade with a region is
defined as the sum of exports to and imports from that
region. Trade deficit is defined as the excess of imports C : Other E.U. F : Russia I : Asia L : Asia
over exports. Trade deficit may be negative.
y
o
u
rs
m
a
h
b
o
o
b
.w
o
rd
134 Pie Chart

p
re
Source of Imports 1998-99 Imports into India : (April-November)

s
s
.c
$28126 million

o
1998-99 Exports from India 1%

m
A
(April-Nov): $21436 million 5% 9% B
1% A 5% C
5% B 16%
D
23% C 13% E
18% D F
E 17% 6% G
5% F
10% 6% H
G I
2% 14% 1% 21%
H J
12% 5% I 1. What is the region with which India had the highest
5%
J    total trade in 1997-98?
(a) USA (b) Other E.U
(c) OPEC (d) Others
1997-98 Exports from India : 2. In 1997-98 the amount of Indian exports, in
$33979 million millions US $, to the region with which India had
the lowest total trade is approximately
1% A (a) 750 (b) 340
5% B (c) 220 (d) 440
19% C 3. In 1997-98, the trade deficit with respect to India,
20% D in billions of US $, for the region with the highest
6% E trade deficit with respect to India, is approximately
F equal to
10% 14% G (a) 6.0 (b) 30
3% H (c) 45 (d) 7.5
10% 6% 6% I 4. What is the region with the lowest trade deficit
J with India in 1997-98?
(a) USA (b) Asia
(c) Others (d) Other E.U
Destination of Exports Additional data for questions 5 and 6
Assume that the average monthly exports from India
1997-98 Imports into India : and imports to India during the remaining four months
$40779 million of 1998-99 would be the same as that for the first eight
1% months of the year.
A
4% 9% 5. What is the region to which Indian exports
B
5% registered the highest percentage growth between
14% C
1997-98 and 1998-99?
D
12% (a) Other East Europe
E (b) USA
F (c) Asia
23% 6% G (d) Exports have declined, no growth
5% H 6. What is the percentage growth rate in India›s total
19% I trade deficit between 1997-98 and 1998-99?
2% J (a) 43 (b) 47
(c) 50 (d) 40
y
o
u
rs
m
a
h
b
o
o
b
.w
o
rd
Pie Chart 135

p
re
Directions (Qs. No. 7-9): These questions are based

s
Chart 1

s
.c
on the pie charts given below. [CAT 2001]

o
India

m
9% 22% 12% 20%
Road Switzerland
11% Airfreight
49% Ship 15% USA
9% Pipeline
Rail Turkey
20%
16%
China
Chart 1: Volume transpored 17%
Pakistan
12% 6% 7%
10% Road
Airfreight Chart 2
Ship
65% Pipeline India
Rail 16%
26% Switzerland
Chart 2: Cost of Transportation USA
17%
Chart 1 shows the distribution of twelve million tonnes Turkey
11%
of crude oil transported through different modes over a
China
specific period of time. Chart 2 shows the distribution 15%
of the cost of transporting this crude oil. The total cost 15%
Pakistan
was `30 million.
7. The cost in rupees per tonne of oil moved by rail
and road happens to be roughly 10. The country which has the highest average price, is
(a) 3 (b) 1.5 (a) USA (b) Switzerland
(c) 4.5 (d) 8 (c) Turkey (d) India
8. From the charts given, it appears that the cheapest 11. The average price is Euro / kg for Turkey is roughly
mode of transport is (a) 6.20 (b) 5.60
(a) Road (b) Rail (c) 4.20 (d) 4.80
(c) Pipeline (d) Ship
9. If the costs per tonne of transport by ship, air and Direction: Refer to the four pie charts relating to the
road are represented by P, Q and R respectively, sectoral composition of output and their respective
which of the following is true? share in employment in 1970 and 1994 and answer
Questions 12 – 14: [CAT 2009]
(a) R > Q > P (b) P > R > Q
(c) P > Q > R (d) R > P > Q Sectoral composition of India’s Employment in 1970
Directions (Qs. No. 10-11): Answer these questions
based on the pie charts given below. [CAT 2002]
11%
15%
Chart 1 shows the distribution by value of top 6 Services
suppliers of MFA Textiles in 1995. Chart 2 shows the
distribution by quantity of top 6 suppliers of MFA Agriculture
Textiles in 1995. The total value is 5760 million Industry
74%
Euro (European currency). The total quantity is 1.055
million tonnes.
y
o
u
rs
m
a
h
b
o
o
b
.w
o
rd
136 Pie Chart

p
re
Sectoral composition of India’s Employment in 1994 13. Between 1970 and 1994 the GDP of India in

s
s
.c
agriculture

o
(a) Decreased by 15%

m
14% 23% (b) Increased by about 9 times
Services
(c) Increased by about 9 %
Agriculture (d) Increased by about 90%
63% 14. Which of the following statement is not true?
Industry (a) Services sector has increased by more than
17 times.
(b) Industry sector has increased in almost same
ratio as service sector.
Sectoral composition of India’s output in 1970 (% (c) Agriculture sector has increased by about 9
of GDP) times.
GDP = ` 42,222 crores (d) Agriculture sector has the highest % increase.
Directions (Qs. No. 15-16): Based on the following
22% information. The following pie chart shows the
Services
33% percentage distribution of runs scored by a batsman in
Agriculture a test innings. [XAT 2012]
3.92
45% Industry
1
2
43.14 36.6 3
Sectoral composition of India’s output in 1994
(% of GDP) 4
GDP = ` 570,954 crores 12.42 6
3.92

Services 15. If the batsman has scored a total of 306 runs, how
28% many 4s and 6s did he hit?
42% Agriculture (a) 31 and 3 respectively
30%
Industry (b) 32 and 2 respectively
(c) 32 and 3 respectively
(d) 33 and 2 respectively
12. Between 1970 and 1994 the Indian GDP, in Rupee 16. If 5 of the dot balls had been hit for 4s, and if two
terms of the shots for which the batsman scored 3 runs
each had fetched him one run instead, what
(a) Has increased only marginally in services would have been the central angle of the sector
(b) Has increased about 11 times in services corresponding to the percentage of runs scored in
(c) Has risen by more than 16 times for both 4s? (1 mark)
services and industry (a) 160 (b) 163
(d) None of the above (c) 165 (d) 170
y
o
u
rs
m
a
h
b
o
o
b
.w
o
rd
Pie Chart 137

p
re
Directions (Qs. No. 17-18): Study the following pie 17. If the 2010 sales for all car models is 80,000 and

s
s
.c
charts regarding to sales of 5 models of cars for the these have grown by 25% in 2011, then what is the

o
years 2010 and 2011, and answer the question approximate increase in the number of Figo cars

m
sold in 2011 over 2010?
2010 2011 (a) 4,860 (b) 12,200
Figo Figo (c) 4,500 (d) 2,200
Honda 9% Alto
44° Alto 18. If the 2010 sales for all car models is 80,000 and
City Honda 111°
14% 32% City these have grown by 25% in 2011, then how many
56° models have grown more than the average growth
i10 rate for all the models taken together?
21% i10 Swift
Swift 82° 67 ° (a) 2 (b) 3
24%
(c) 4 (d) None of these

Directions (Qs. No. 1-4): Answer the questions on the Questions asked to A (Total 3000)
basis of the information given below.
10% B
At Pioneer Career there is a unique interactive sessions
in which students are allowed to asked question from 30% C
25%
each other. In the entire session of CAT preparation D
six friends A, B, C, D, E, & F asked question to each E
other. The following pie charts pertain to the details 15%
20% F
regarding the number of questions asked by these
persons within the group. The first pie chart gives the Questions asked by different students (Total N)
break-up of total number of questions received by
A from his five friends according to the friend from 15% B
whom he received the questions and the second pie 24%
C
chart gives the break-up of the total questions asked by
these five persons within the group (i.e., excluding A) 20% D
according to the person asking the question.  E
25%
16% F
y
o
u
rs
m
a
h
b
o
o
b
.w
o
rd
138 Pie Chart

p
re
1. Who has asked minimum number of question to A 8. If salary of one of them increased by 33.33% then

s
s
.c
(a) B (b) C find the minimum % savings of the family.

o
(c) D (d) E (a) 4.76% (b) 5%

m
2. Find the least possible value of N (c) 0% (d) None of these
(a) 3800 (b) 3750
(c) 3600 (d) None of these Directions (Qs. No. 9-11):

Additional information for question numbers 3 and 4 At the end in Dhoom -2 Mr.A & Sunahri got married,
but what after that. Here is the remaining story:- They
If B has asked minimum 25% of questions to A and
started earning well with their restaurant Dhaba-E-
E has asked a maximum of 40% questions to A then:-
Dhokha. They decided to give a separate account and
3. What is the maximum possible number of persons
give an amount as salary to both Mr. A and Su-Nahri.
who have asked more than 80% of their questions
Following Pie Chart shows how they spend their income
to A?
and last pie chart shows their total expense in tours.
(a) 2 (b) 3
Mr. A
(c) 4 (d) 5
4. What percentage of his total questions D has asked 12% Food
25%
to A? ENT
13%
(a) 30 (b) 40 Tax
(c) 50 (d) None of these 5% Tours
Directions (Qs. No. 5-8): Answer the questions on the Others
18% 27%
basis of the information given below. Clothes
In a family of 4 earning members A, B, C and D the
distribution of income is given in pie chart 1 and
expense in pie chart 2. Su-Nahri
Income Expense Food
13%
25% ENT
A 20% A Tax
30% 25% 35%
B B 19%
Tours
C C 12% Others
15% 25%
D 30% D 10%
20% Clothes
21%

5. What is the minimum percentage total savings?


(a) 0% (b) 100% Total Tour Exp.
(c) 25% (d) None of these
Food
6. If ratio of savings of C to that of D is 1:3 then find 13%
25% ENT
the percentage savings of B
(a) 25 (b) 40 Tax
19%
(c) 50 (d) None of these Tours
12%
7. What is the minimum percentage savings of A ? Others
10%
(a) 40% (b) 30% Clothes
21%
(c) 25% (d) None of these
y
o
u
rs
m
a
h
b
o
o
b
.w
o
rd
Pie Chart 139

p
re
9. If Mr.A’s expense on tours is twice the total expense (a) 72% (b) 65%

s
s
.c
of couple on others section in tours. By what % Mr
(c) 55% (d) None of these

o
A is earning more than Sunahri?

m
11. If the expenditure on clothes by Mr. A and that by
(a) 50% (b) 75%
Su-nahri are in the ratio 2 : 5, then By what % Mr
(c) 80% (d) None of these
A is earning more than Sunahri?
10. If Mr A is earning 200% more than Su-Nahri then
(a) 12% (b) 25%
total expense of couple on Japan tour is what % of
total expense made by Mr.A in Tours. (c) 33.33% (d) None of these
y
o
u
rs
m
a
h
b
o
o
b
.w
o
rd
140 Pie Chart

p
re
s
s
.c
o
m
CONCEPT APPLICATOR 7. (b) Cost of the ticket is ` 124. Number of
Alternative Solution : passengers in B is 432. Therefore, the total cost is
Solution for 1-5 : 432 × 124 = 53,568.
8. (a) Number of passengers in B and D is 1440. E is
Degree % Value 912
× 100 = 63.33% approx.. 63% of B and
A 79.2 22 1496 1440
D together.
B 57.6 16 1088
9. (d) The difference in C and A is 816 – 576 = 240.
C 14.4 4 272 10. (c) Total number of passengers in D, E and F are
D 122.4 34 2312 1008 + 912 + 1056 = 2976.
E 64.8 18 1224 11. (d) Difference between the amount spent on
F 21.6 6 408 Architect Fee’s and Interior decoration is
22,800 – 13,200 = 9,600.
1. (c) A prefer 22%, C prefer 4%, D prefer 34% 12. (c) Miscellaneous expenditure is 10,200. 10,200
and F prefer 6% therefore, A + C = 1496 + 10, 200
is × 100 = 8.5% of the total
272 = 1768 and D + F = 2312 + 408 = 2720 1, 20, 000
2720 – 1768 = 952. estimated cost.
Alternative Method : 13. (d) According to question, 13% of 120000 + 8%
(A + C) = 79.2° + 14.4° = 93.6° of 120000 = 25200 (which is estimated cost of
furniture)
(D + F) = 122.4° + 21.6° = 144°
Then actual cost of furniture is 15600 - 12%
Difference = 144° – 93.6° = 50.4°
of 15600 = 13728/-
50.4° So total actual cost (furniture and miscellane-

Then × 6800 =
952
360° ous) = 13728 + 10200 = 23928
2. (a) Ratio of F : A = 408 : 1496 i.e. 3 : 11 Actuale cost = 120000 – ( 25200 – 23928)
3. (d) People who prefer E and F together is 1224 + = 118728
1632 14. (b) the cost incurred on painting and flooring
408 = 1632. Therefore, × 100 = 24% together is 18,000 + 16,800 = 34,800.
6800
15. (d) Let, 12% discount on estimated cost of
Alternative Method : furniture is 12% of 15,600 = 1872. Then, The
Let E + F = 64.8 + 21.6 = 86.4° actual amount spent of furniture is 15600 –
86.4 1872 = 13728.

Required percentage = × 100 = 24% 16. (d) The average number of passengers is
360 2040 + 1700 + 1275
272 = 1671.66
4. (b) C is × 100 =11.76 approx. 12% of D. 3
2312 17. (a) In Train R, number of females are 34 % of
5. (a) B + E = 1088 + 1224 = 2312. 765 = 260 and children are 26% of 765 = 199.
6. (a) The average of A, C and F are 576 + 816 + Therefore, number of males = 756 – (260 +
1056 / 3 = 2448/3 = 816 199) = 306.
y
o
u
rs
m
a
h
b
o
o
b
.w
o
rd
Pie Chart 141

p
re
18. (d) Number of passengers in A and R 1105 + 765 6. (d) Angle for sector representing transportation

s
s
.c
360
= 1870. Q is 1615 × 100 =86.36% of A and R = × 4 = 14.4°

o
1870 100

m
together.
7. (a) 30% = 30,000
19. (d) The second highest number of passengers is
1% = 1000
in M.
∴ 4% = ` 4000
20. (d) Train M has 425 more passengers than Train
8. (d) Miscellaneous cost = 2% = 2000
L. Therefore, 425 × 100 = 33.33% ∴ Total cost = ` 100000
1275
100000
21. (d) From the pie, we can calculate, number ∴ CP per copy = =`8
12500
of boys = 3000 – 1750 = 1250. Boys are
SP per copy = ` 8.40
1250
× 100 =41.66% approx. 42% of the 0.40
3000 Profit % = × 100 = 5%
total number of students. 8
100000
22. (a) Total number of boys in singing and craft 9. (c) Cost price per copy = =`2
50000
together is 140 + 365 = 505. SP at a profit of 5%
23. (b) Total number of girls in Swimming and SP = 2 × 1.05 = ` 2.10
drawing together is 245 + 280 = 525 9000 × 100
10. (a) Total cost = = `50,000
24. (c) Number of girls enrolled in dancing is 18
11. (c) 18% = 9000
350 9000
×100 = 11.666 approx. 11.67% of the 30% = × 30 = `15000
3000 18
total number of students.
12. (b) On two items savings and house rent he has to
25. (d) The ratio of girls enrolled in swimming to boys invest more than `1000
enrolled in swimming is 245 : 235 = 49 : 47. 1800
13. (b) × 100 = 30%. He invests `1800 on
6000
CONCEPT BUILDER house rent.
360° 14. (d) Saving per month = 6000 × 23% = `1380
1. (d) Required Angle for Royalty = × 15 = 54° ∴ Annual saving = 1380 × 12 = `16560
100
2. (b) Let cost price be C.P. 15. (c) 10% = 500
 120  100% = `5000

then C.P. ×  30 (m.p)
=
 100  16. (b) Money spent on clothes + saving = money
spent on house rent.
C.P. = ` 25 360

then cost of paper is 20% of 25 = ` 5 17. (c) Angle made by sector Petrol = × 10 = 36°
100
30 × 360° 18. (a) Let the two radii be r1 and r2
3. (c) A and D = 20 + 10 = = 108° π r 2 6000 4
100 Then required ratio 12 = =
4. (d) None of these π r2 1500 1
∴ r :r =2:1
5. (b) Angle for sector representing paper cost 1 2
23
360 19. (a) Saving p.m. = 1500 × = `345
= × 10 = 36° 100
100
∴ Annual saving = 345 × 12 = `4140
y
o
u
rs
m
a
h
b
o
o
b
.w
o
rd
142 Pie Chart

p
re
(20 to 23)

s
110

s
32. (a) Required % = × 100 = 7.1%

.c
Subject Students Girls Boys 1539

o
m
Arts 216 168 48 107 + 458
33. (b) Required angle = × 360 =132°
1539
Biology 234 168 66 183 − 110
34. (a) % increase = × 100 = 66% (nearly )
Law 360 360 Nil 110
Computers 270 120 150 183 + 454 + 110 747
35. (c) Required ratio = = = 0.94
107 + 458 + 227 792
Maths 90 24 66
36. (b) Investment in socially oriented sector (Plan
Political Science 630 360 270 and Non Plan) = (107 + 458) = `65 crores
Total 1800 1200 600
investment in central and State Government
Securities = (454 + 110) = ` 514crores
20. (b) from the above table we can answer the ∴ Difference = (565 – 564) = ` 1 crore
question easily. 17
37. (a) Total revenue for Esteem = × 1139200
21. (a) 100
million
270 20
22. (b) =3:4 Total number of Esteem sold = × 653610
360 = 130722 100
(168 − 48) 120 × 100 17 1139200
23. (d) × 100 = = 250 % Revenue per unit Esteem = ×
48 48 100 130722
24. (b) Number of workers in category S in 1998 million
Unit = 481 million = `14,81,000
24
= 2,000 × = 480 38. (d) For M 800
100 48
25. (c) Number of workers is category U in 1998 Revenue = × 1139200 = 546816 million
8 100
= 2,000 × = 160 58
100 = × 653610
Number of workers in category U in 2000 100
546816 ×100
10 Revenue per vehicle = = 1.44
= 2,400 × = 240 million 58 × 53610
100 For Omni
240 − 160
Required % = × 100 = 50% 21
160 Revenue = × 1139200 = 21 × 1139200
26. (c) Number of workers in 1998 million 100
U = 160, P = 260, Q = 360, R = 300, S = 480, 11

Number of units = × 653610
T = 440 Number of workers in 2000 100
U = 240, P = 264, Q = 600, R = 384, S = 480, 21 ×11392 ×100
T= 432 Required number = 80 + 4 + 240 + 84 = 408 Revenue per vehicle =
11 ×653610
27. (d) = 3.32 million
28. (a) Required number = (600 + 480) – (160 + 260) For Zen
= 660 3
29. (a) External assistance Total revenue = × 1139200
100
7862 × 100 3
= × 25% (aprox.) Number of units = × 653610
30300 100
16846 × 360
30. (d) Required angle = × 200° (aprox.) 3 100
30300 Revenue per unit = × 1139200 × ×
1690 100 3
31. (a) Amount required to be permitted = = 1
0.9 = 1.74 million
`1878 crore 653610
y
o
u
rs
m
a
h
b
o
o
b
.w
o
rd
Pie Chart 143

p
re
s
11 34176

s
39. (d) Units of Omni = × 653610 Percentage for Zen = × 100

.c
100 1139200 − 125312

o
1 11 34176

m
Increase in the number of units = × × = × 100 =3.37%
4 100 1013888
653610 = 17974
41. (d) (i) It is not true as Zen + Omni is 24% of
58
Units of M 800 = × 653610 = 379094 total revenue which is more than 2 × 11
100 = 22%
17974
Percentage decrease = × 100 = (ii) Revised revenue per unit for Alto =
379094
4.74% = ` 1,25,312 million 17 100
× 1139200 × =3.7 million
11 100 8 × 653610
Revenue from Alto = × 1139200
100 (iii) From the calculation for Qs. 55 and 56
= ` 1,25,312 millions Revenue per unit Esteem = 1.48 million
3 and
40. (c) Revenue from Zen = × 1139200 Revenue per unit M 800 = 1.44 million
100
= ` 34,176 million Hence, both (ii) and (iii) are true.

CONCEPT CRACKER
7500000
1. (a) Required % = × 100
7500000 + 8550000 + 8100000
750
= 2415 × 100 ≈ 30%
2. (b) Average percentage of gross receipts for crunchy chop for the year 2001 to 2003
46.3% of 7500000 + 40.6% of 8550000 + 12.3% of 8100000
= × 100
7500000 + 8550000 + 8100000
7940100
= × 100 = 32.87%
241500
Average percentage of gross receipts for Pizza pie for the year 2001 to 2003
12.8% of 7500000 + 24.6% of 8550000 + 37% of 8100000 6060300
= × 100 = × 100 = 25.09%
2415000 2415000
∴ % increase = 32.87 − 25.09 × 31.01 ≈ 30%
25.09
3. (d) Gross receipt earned by the other restaurants in 2002 = 8550000 × 5.3% = `453150
11, 0500000 ×8.9
4. (a) Required investment in high-risk stock = = `98,34500
100
48.3 26
5. (d) Required investment = 11, 05, 00, 000 × × = `1,38,76,590
6. (d) High-risk stocks 100 100
7. (c)
8. (c) Ghosh babu is 100 kg of weight
Then protein is 15 kg
Then mineral protein is 34% of 15 kg= 5 kg (approx.)
5 1
So required fraction = =
100 20
y
o
u
rs
m
a
h
b
o
o
b
.w
o
rd
144 Pie Chart

p
re
9. (a) Required Ratio = 34 : 11 (approx.). 20. (d) Expenses on entertainment and food = (9 +

s
s
25) = 34% i.e 10150.80 × 34% = `3451.27

.c
10. (d) We can determine only the percentage of skin

o
protein in Ghosh Babu’s total body weight. But 21. (b) New saving % = 12 – 3.4 = 8.6%

m
there is no data given about the percentage of 22. (b) D represents = 0.5%
skin in Ghosh Babu’s body. Hence the answer Share of D = 128.3 × 0.5% = 0.6 lakh
is (d). (approximately)
11. (a) Proportion of material other than water & 23. (b) The value of D would be approximately =
protein in Ghosh Babu’s body is 15/100 = `1.3 lakh
3/20. 24. (c) Packing cost after increase = (39.5 + 39.5 ×
12. (a) Percentage increase = (160 – 130)(100/130) 2%) = 40.29%
= 300/13 = 23%. ∴ Amount involved in packing cost = 128.3 ×
13. (b) Interest in 1990-91 = 30% of 130 = `39 lakh 40.29% = `51.69 = `52 lakh (approximately)
Interest in 1991-92 = 40% of 160 = `64 lakh 25. (b) Packing and raw material cost together = 88.5%
Hence, difference = (64 – 39) = `25 lakh 128.3 × 88.5% = `113 lakh approximately
14. (d) Total interest = (30% of 130) + (40% of 160) 2.4 − 0.6
26. (d) % increase = × 100 = 300%
= (39 + 64) = `103 lakh. 0.6
If this total interest is charged on borrowed 27. (b) The difference in the degrees between Bihar
funds, then (20% of borrowed funds) and Madhya Pradesh is 12.6°
= 103. Hence, borrowed funds We know that in a pie chart total value is
= (5 × 103) = `515 lakh. 360° hence 360°= 72000
15. (d) Retained profit in 1990-91 = (25% of 130) Or 12.6°  =2520.
= `32.5 lakh
CONCEPT DEVIATOR
Retained profit in 1991-92 = (20% of 160) =
`32 lakh 1. (c) We know that total trade = Import + Export
Hence, percentage change in retained profit Total trade with USA = 9 + 19 = 28%
 32.5 − 32  Total trade with other EU = 12 + 14 = 26%

=  = 1.5% lower. Total trade with OPEC = 23 + 10 = 33%
 32.5 
16. (c) Total dividend earned by shareholders in Total trade with others = 1 + 1 = 2%
1991-92 Hence OPEC is the region with which India
= (8% of 160) = `12.8 lakh. had the highest total trade.
17. (b) Total income of the person = 9228 × 1.1 = 2. (b) From the observation of the pie chart we
`10150.80 can conclude that India had the lowest total
∴ Share of house rent = 10150.80 × 18% = trade with k and that is 1% of 33979 = 340
`1827.14 (approximately)
18. (c) Saving of Ist year = 10150.80 × 12% ×12 = 3. (a) With the observation we can find out that
`14617.15 trade deficit for the region H is the highest.
Interest on Ist year income = 14617.15 × 12% We will calculate the deficit = 940779 × 0.09
= `1754.06 –33979 × 0.100 = 5.9. The closest option is 6.0
Saving in IInd year = `14617.15 4. (a) The lowest trade deficit will be for that country
Total saving in two year = (14617.15 + which will have more exports wrt imports and
14617.15 + 1754.06) = `30988.36 the difference should be maximum wrt others.
19. (a) Total income `10150.80 Here it is for A.
Investment in saving and house rent = (18 + 5. (b) Here we have to compare both the pie-charts
12) = 30% for exports. We can observe that only A and G
∴ Money left = 70% i.e, have registered growth. Further we can notice
10150.80 × 70% = `3451.27 that A has registered more growth.
y
o
u
rs
m
a
h
b
o
o
b
.w
o
rd
Pie Chart 145

p
re
6. (c) We know that deficit = total import – total 16. (d) As per the information given in question total

s
s
export. Now for the year 1998-99, Total

.c
runs = 306 + 20 – 4 = 322.

o
import from India = 28126 × 12/8 = 42189 From the previous question, runs scored in

m
for the year 1998-99, Total export from India fours = 132.
= 21436 × 12/8 = 32154 hence deficit for this So, runs scored in fours = 132 + 20 = 152.
year = 42189 – 32154 = 10035. So angle made = (152/322) × 360 = 169.93
Similarly deficit in 1997-98 = 40779 – 33979 = 6800 = 170 (approximately)
Hence percentage growth = (10035 – 6800)/ 17. (a) Sales of Figo cars in 2010
6800 × 100 = 47.57% or closest to 50%.  9 
7. (b) Volume transported by rail = 9% and by road =   × 80000 = 7200
100 
= 22% so by both together it will be 9 + 22
= 31% of total volume transported = 31% of Sales of Figo cars in 2011
12m = 31 × 12/100 = 3.72 million tonnes  44   125 
=  × × 80000 =12222
Cost of transportation by rail = 12% and by  360   100 
road = 6% so by both together it will be 12 Hence the increase in sales is 12222 – 7200
+ 6 = 18% of total cost of transportation = 5022. The closest option is 4860.
= 18% of 30m = 18 × 12/100 = ` 5.4million. 18. (b) We need to find the number of models for
Hence cost per tonne = 5.4/3.72 = 1.45 the which the sales in 2011 are 25% more than
closest option is 1.5 the sales in 2010. For ease of calculations let
8. (a) Cost of transportation by Road = 6/22 3/11, us assume that the total sales in 2011 are 360.
by Rail = 12/9 = 4/3, by pipeline = 65/49 and 360
by ship is 10/9, hence the cheapest way of Then the sales in 2010 are = 288
1.25
transportation = Road
9. (c) From the answer of the previous question cost Sales in Percentage
Model Sales in 2010
of transportation can be arrange as ship > Air 2011 Increase
> Road, hence P > Q > R Figo 2.88 × 9 = 25.92 44 > 25%
10. (b) Considering percentages above, we see that Alto 2.88 × 32 = 92.16 111 < 25%
Switzerland has average price of 20/11 which Swift 2.88 × 24 = 69.12 67 < 25%
is > 1. All others are less than 1.
I10 2.88 × 21 = 60.48 82 >25%
11. (b) (16% of 5760)/(15% of 1.055) = 5.60.
Solution from 12 to 14 Honda
2.88 × 14 = 40.32 56 >25%
We can tabulate the given information as follows City

42222 670954 % change Hence the sales grew by more than 25% only
for three models – Figo, i10 and Honda City
Services 14074 239800 1603
Industry 9288 159867 1621 CONCEPT ELIMINATOR
Agriculture 19000 171286 801 1. (d) Question asked by B to A is 10% of 3000 = 300.
12. (c) From the above table Option C is correct. Similarly, for C,D,E and F are 750,600,450
13. (b) From the above table Option B is correct and 900 respectively.
(Increased by 800% is equal to increased by 9 Now percentage of questions asked by B to A
times). 300 2000
= =
14. (d) From the above table Option D is not correct. 15% of N N
15. (d) Total runs scored by the batsman = 306. Similarly, for C,D ,E and F be 3750 , 3750 ,
Now 43.14% of 306 = (43.14/100) × 306 1800 3750 N N
= 132 runs. This is equivalent to 33 fours and .
N N
33 × 4 = 132. 3.94% of 306 is approximately
equal to 12 runs or 2 sixes. Hence E is the answer.
y
o
u
rs
m
a
h
b
o
o
b
.w
o
rd
146 Pie Chart

p
re
2. (a) Questions asked by B to A is 10% of 3000 =
Say

s
s
.c
300.
Value of X 1 2

o
Similarly, for C,D,E and F are 750,600,450

m
and 900 respectively . Value of Y 1 1
According to the question, total questions When X = Y = 1, % of savings of A be
asked by each person must not be less than ( 25X − 20Y) × 100 = 20%
the questions faced by A. so for B: 15% of
25x
N ≥ 2000, similarly for others. so the value
When X = 2 and Y = 1, then % of saving of A
of N that satisfies all the above cases is N ≥
3750. But if total questions are 3750, then the be ( 25X − 20Y) × 100 = 60%
questions asked by F and D are not integers.. 25x
hence the least possible value is 3800. So minimum savings be 20%.
3. (b) The values of B and E are 2000/N and 1800/N 9. (c) As Mr. A expends 5% on tour which is twice
respectively. than the other expenses of the couples (15%)
As it is given that B has asked minimum 25% i.e 30% of their combined tour and for Su-
questions to A then 2000/N ≥ 25%. Similarly nahri it was 70%. So the ratio of their tour be
for E 1800/N ≥ 40%. then N lies between – 3:7
4500 ≥ N ≥ 8000. So no of questions will be 5% of the income of A 3
Or, = ,
maximum for N = 4500. Therefor, questions 21% of the income of Su-Nahri 7
asked by B, C , D. E and F be 675, 900, 720, the income of A
1125 and 1080 respectively. = 9/5,
the income of Su-Nahri
Hence, the percentages are 44.44%, 83.33%,
83.33% , 40% and 83.33% respectively. Then % of A ’income more than Su-nahri
As we can observe that three persons have = 4/5 x 100 = 80%.
asked more than 80% questions to A. 10. (a) As it is given that Mr. A’s income is 200%
4. (c) With the help of above solution, we came to more than that of Su-nahri, that means their
know that the value of N lies between 4500 ratio of income must be 3:1. As Mr. A invested
to 8000. The required values of D be 3750/N, 5% of his income on tour and Su-nahri on
3750 21% on tour. Hence their rato of investment
so we can write to 3750 i.e 83.33%
4500 8000 be 5% of total investment(3) : 21% of total
to 46.87%. so from the option 505 is the only investment(1) = 5 : 7
possible value of D. Now total expense must be 5+7 = 12
5. (a) Let the total income of the house is 100x and Investment on the japan tour = 30% of total
expense is 100y. for saving condition should investment on tour together = 30% of 12 = 3.6
satisfy100 (x-y) 3.6
Required percentage = × 20 = 72%
Therefore, for A : 25x – 20y. similarly for 5
others. NOW the percentage of saving varies 11. (b) 12% of expenditure of Mr A : 25% of income
with the difference of income and expense. of Su-nahri = 2:5
So, minimum % can be 0%. Therefore, income of Mr. A: income of Su-
6. (d) As ratio of savings of C and savings of D be nahri = 5:6
15 x − 20 y Let Mr. A’s income is 5K and Sunehri’s
= 1/3  ⇒  3x = 5y.
30 x − 35 y income be 6K. frome it we can say that
Now B’s savings be 30x-25y = 30x – 25(3x/5) sunehri’s income is 20% more than Mr. A.
= 15x. so 15 % of his income B saves. 2000
hence Mr. A’s income is = 25% less
7. (d) When saving is there, it means that always 100 − 20
than sunahri.
X ≥ Y.
y
o
u
rs
m
a
h
b
o
o
b
.w
5
60

o
rd
50

Mixed

p
40

re
s
30

s
.c
20

o
Graph

m
10

0
SUN MON TUES WED THRUS FRI SAT
10am to 2pm 2pm to 6pm 6pm to 10pm

Topics No. of Questions Level


Concept Applicator 30 Bank Po basic level
Concept Builder 38 Bank Po and MAT advance level
Concept Cracker 09 XAT basic level
Concept Deviator 10 CAT, XAT advance level
Concept Eleminator 07 Advance level
Total 94

INTRODUCTION Production Level ('000s)

Mixed graph is the another type of questions which is 7


6
usually asked in different types of competitive exams
5
to see whether the candidate is capable to interpret 4
information given in different types of multi-graphs . 3
Generally two or more graphs (either bar chart with pie 2
or line graph with data table) are combined together. 1
Let’s start the exercise to understand the different 0
levels of difficulties. 2010 2011 2012 2013 2014
Production Level ('000s)
Example: Study the line and the bar graph and answer
Q. No. of cars produced = ( cost of production + profit
the question: margin)/(price per car). If during 2014 , the cost of
Price per car (` lakhs)
production was ` 1200 cr. Find profit margin?
3 (a) 10 crs (b) 20 crs
2.5 (c) 30 crs (d) 40 crs
2
Sol. (c) Using the information given in the questions,
we know that –
1.5 Cost of production in year 2014 = 120 Crs
1 Production level = 6000
0.5
Price per car = 2.5 lakhs
Now using the formula, No. of cars produced
0 = (cost of production + profit margin)/(price
2010 2011 2012 2013 2014
per car)
price per car (` lakhs) Or, profit margin = no. of cars produced x
The two graphs above pertain to ABC LTD , a famous price per car – cost of production = 6000
car maker of india. Follow the graph and questions × 2.5 lakhs – 120 crs = 15000 lakhs – 120,
based on graphs. 00,00,000 = 15000,00,000 –120,00,00,000 =
30,00,00,000 = 30crs
y
o
u
rs
m
a
h
b
o
o
b
.w
o
rd
148 Mixed Graph

p
re
s
s
.c
o
m
Directions (Qs. No. 1-5): Study the following 2. What is the difference between the total number of
information carefully to answer these questions. employees in IT and that in Operations Department?
[Andhra Bank PO 2009] (a) 75 (b) 150
Percentage of employees in various departments of an (c) 100 (d) 50
organization and the male-female ratio 3. What is the ratio of the total number of males in
HR and Marketing to the total number of females
Total Number of Employees = 2500 in these two departments?
Printing Logistics
6% (a) 13 : 15 (b) 15 : 13
Administration 11%
12% (c) 13 : 17 (d) 17 : 14
Operations 4. How many female employees are there in the HR
Accounts 18% Department?
8%
(a) 250 (b) 120
HR IT
16% 14% (c) 125 (d) 150
5. What is the difference between the numbers of male
Marketing and female employees in Logistics Department?
15%
(a) 50 (b) 25
Male: Female Ratio (c) 75 (d) 100
Department Male : Female
Directions (Qs. No. 6-10): Study the following graph
Administration 7:5 and table carefully and answer the questions given
Accounts 2:3 below them.
Chart Title
Obese women and Obese Children

HR 5:3 40
Percentage of Obese men,

Marketing 7:8 35
IT 3:4 30
Operations 5:4 25
Logistics 6:5 20
15
Printing 2:1
10
1. What is the ratio of male employees in Administra- 5
tion to those in Printing Department? 0
2004 2005 2006 2007 2008 2009
(a) 7 : 4 (b) 4 : 7
(c) 3 : 4 (d) 7 : 3 Obese men Obese Women Obese Children
y
o
u
rs
m
a
h
b
o
o
b
.w
o
rd
Mixed Graph 149

p
re
Total Number of Men, women and Children in the state

s
Percentage of Students

s
.c
over the years F A

o
m
Years Men Women Children 6% 12%
B
2004 54,000 38,000 15,000 E 9%
2005 75,000 64,000 21,000 29%
2006 63,000 60,000 12,000 C
D 26%
2007 66,000 54,000 16,000 18%
2008 70,000 68,000 20,000
2009 78,000 75,000 45,000 Number of boys out of 6000 students in
eachs chool separately
6. What was the approximate average of obese men,
School F
obese women and obese children in 2007?
School E
(a) 12,683 (b) 12,795
School D
(c) 12,867 (d) 12,843
School C
7. The number of obese men in the year 2009 was
what percent of the men not suffering from obesity School B
in the same year? School A
(a) 55 (b) 60 0 500 1000 1500
(c) 50.5 (d) 65.5 Number of boys out of 6000 students
in each school separately
8. What was the ratio of the obese women in the year
2006 to the obese men in the year 2008? 11. What is the sum of the number of girls in School C,
the number of girls in School E and the number of
(a) 6 : 7 (b) 21 : 65 boys in School D together?
(c) 15 : 73 (d) 48 : 77 (a) 1700 (b) 1900
9. What is the difference between the number of (c) 1600 (d) 1800
obese women and obese children together in the 12. What is the ratio of the number of boys in School
year 2006 and the number of obese men in the C, the number of girls in School B and the total
same year? number of students in School E?
(a) 5475 (b) 5745 (a) 45 : 7 : 97 (b) 43 : 9 : 97
(c) 4530 (d) 31,650 (c) 45 : 7 : 87 (d) 43 : 9 : 87
10. What was the total number of children not suffering 13. What is the difference between the total number
from obesity in the year 2004 and 2005 together? of students in School F and the number of boys in
(a) 4350 (b) 31560 School E?
(a) 820 (b) 860
(c) 4530 (d) 31650
(c) 880 (d) None of these
Directions (Qs. No. 11-15): Study the following pie- 14. In which of the following schools is the total
chart and bar diagram and answer the following number of students equal to the number of girls in
questions. Percentage- wise distribution of Students in School E?
six different schools. [Allahabad bank PO 2011] (a) A (b) B
Total number of students = 6000 (c) C (d) D
y
o
u
rs
m
a
h
b
o
o
b
.w
o
rd
150 Mixed Graph

p
re
15. The number of girls in School A is approximately 18. In which district is the number of male teachers

s
s
.c
what percentage of the total number of students in more than the number of female teachers?

o
School B? (a) B only (b) D only

m
(a) 55 (b) 50 (c) Both D and E (d) Both E and F
(c) 35 (d) 41 19. What is the difference between the number of
female teachers in District F and the total number
Directions (Qs. No. 16-20): Study the following
of teachers (both male and female) in district E?
pie-chart and bar-graph and answer the following
(a) 625 (b) 775
questions. Percentage wise distribution of teachers in
(c) 675 (d) 725
six different districts
20. What is the ratio of the number of male teachers
Total number of teachers = 4500 in District C to the number of female teachers in
Percentage of Teachers
District B?
District F (a) 11 : 15 (b) 15 : 11
6% District A (c) 15 : 8 (d) 30 : 13
14% Directions (Qs. No. 21-25): Study the following pie
District E – chart and table carefully and answer the questions
21% District B given below: [IBPS PO 2012]
16%
Percentage wise distribution of the number of mobile
District D
15% phones sold by a shopkeeper during six months
District C
28% Total number of mobile phones sold = 45000
December
16%
Number of males out of the 4500 teachers in July
each district separately 17%
November
District A 12%
August
District B 22%
October
District C September
8%
District D 25%
District E The ratio between the numbers of mobile phones sold
District F of Company A and Company B during six months
0 100 200 300 400 500 600 700 Month Ratio
Number of males out of the 4500 teachers in July 8:7
each district separately August 4:5
16. What is the total number of male teachers in September 3:2
District F, female teachers in District C and female October 7:5
teachers in District B together?
November 7:8
(a) 1080 (b) 1120
(c) 1180 (d) 1020 December 7:9
17. The number of female teachers in District D is 21. What is the ratio of the number of mobile phones
approximately what per cent of the total number of sold of Company B during July to those sold during
teachers (both male and female) in District A? December of the same company?
(a) 70 (b) 75 (a) 119 : 145 (b) 116 : 135
(c) 80 (d) 91 (c) 119 : 135 (d) 119 : 130
y
o
u
rs
m
a
h
b
o
o
b
.w
o
rd
Mixed Graph 151

p
re
22. If 35% of the mobile phones sold by Company The table given below represents the ratio of the

s
s
.c
A during November were sold at a discount, how production (in tonnes) of Company A to the production

o
many mobile phones of company A during that (in tonnes) of Company B, and the ratio of the sales

m
month were sold without a discount? (in tonnes) of Company A to the sales (in tonnes) of
(a) 882 (b) 1635 Company B.
(c) 1638 (d) 885
Year Production Sales
23. If the shopkeeper earned a profit of ` 433 on each
mobile phone sold of Company B during October, 2006 5:4 2:3
what was his total profit earned on the mobile 2007 8:7 11 : 12
phones of that company during the same month? 2008 3:4 9 : 14
(a) ` 6,49,900 (b) ` 6,45,900 2009 11 : 12 4:5
(c) ` 6,49,400 (d) ` 6,49,500 2010 14 : 13 10 : 9
24. The number of mobile phones sold of company
2011 13 : 14 1:1
A during July is approximately what per cent of
the number of mobile phones sold of Company A
26. What is the approximate percentage increase in
during December?
the production of Company A (in tonnes) from
(a) 110 (b) 140
the year 2009 to the production of Company A (in
(c) 150 (d) 130
tonnes) in the year 2010?
25. What is the total number of mobile phones sold
(a) 18% (b) 38%
of Company B during August and September
together? (c) 23% (d) 27%
(a) 10000 (b) 15000 27. The sales of company A in the year 2009 was
(c) 10500 (d) 9500 approximately what per cent of the production of
Company A in the same year?
Directions (Qs. No. 26-30): Study the following (a) 65% (b) 73%
information and answer the questions that follow:
[IBPS CWE PO 2012] (c) 79% (d) 83%
The graph given below represents the Production 28. What is the average production of Company B (in
(in tonnes) and sales (in tonnes) of company A from tonnes) form the year 2006 to the year 2011?
2006 -2011. (a) 574 (b) 649
(c) 675 (d) 593
Production and sales (in Tonnes)

Chart Title
900
29. What is the ratio of the total production (in tonnes)
800
of Company A to the total sales (in tonnes) of
700
Company A?
600
500 (a) 81 : 64 (b) 64 : 55
400 (c) 71 : 81 (d) 81 : 55
300 30. What is the ratio of production of Company B (in
200 tonnes) in the year 2006 to production of Company
100 B (in tonnes) in the year 2008?
0
2006 2007 2008 2009 2010 2011 (a) 2 : 5 (b) 4 : 5
Sales Production
(c) 3 : 4 (d) 3 : 5
y
o
u
rs
m
a
h
b
o
o
b
.w
o
rd
152 Mixed Graph

p
re
s
s
.c
o
m
Directions (Qs. No. 1-10): Study the following pie- 3. The graduate female population of State C is what
charts and table to answer these questions. percent of the Std XII population of all the states
[RBI GRADE B OFFICER 2009] together?
Statewise Details of the adult population of a country (a) 40 (b) 62.5
Graduate and Above Upto XII STD pass (c) 50 (d) 52.5
Total no. = 24 Lakhs Total no. = 32 lakhs 4. The STD XII male population of State C is what
F A percent of the total STD XII population of all the
F A states together?
14% 16% 15%
20% (a) 8% (b) 12%
E B B
20% 18% 16% (c) 11% (d) 9%
E
19% 5. What is the ratio of the Graduate male population
D C C of State E to Std XII female population of that
15% D 18%
17% 12% State?
Male: Female ratio (a) 28 : 35 (b) 35 : 28
(c) 32 : 45 (d) None of these
Graduate & Above Upto XII Std Pass 6. Total graduate population of State F is what per
State
M:F M:F cent of the total Std XII population of State A?
A 7:5 7:9 (a) 56 (b) 72
B 5:3 3:5 (c) 68 (d) None of these
C 5:4 4:5 7. Std XII male population of State E is what per cent
D 9:8 5:7 of the Std XII male population of State F?
E 9:7 9 : 10 (a) 70 (b) 75
F 4:3 3:2 (c) 68 (d) 72
8. What is the ratio of the total Graduate and Std XII
1. What is the difference between the Graduate male male population of State A to the total Graduate
population and XII Std male population from State and Std XII female population of that state?
‘A’? (a) 215 : 216 (b) 214 : 215
(a) 24,000 (b) 14,000 (c) 217 : 215 (d) 215 : 217
(c) 28,000 (d) 36,000 9. What is the ratio of the total; Graduate population
2. What is the ratio of the Graduate female population of State D to the total Std XII population of that
of State E to Std XII female population of State D? State?
(a) 7 : 5 (b) 5 : 7 (a) 17 : 16 (b) 16 : 17
(c) 16 : 15 (d) 15 : 16 (c) 64 : 51 (d) 51 : 48
y
o
u
rs
m
a
h
b
o
o
b
.w
o
rd
Mixed Graph 153

p
re
10. The Graduate female population of State B is what 13. If the area under tea production was less by 10%

s
s
.c
per cent of the Graduate female population of state in 1994 than in 1993, then the approximate rate of

o
E? (rounded off to the nearest integer) increase in productivity of tea in 1994 was:

m
(a) 129 (b) 82 (a) 97.22
(c) 77 (d) 107 (b) 3
Directions (Qs. No. 11-20): Study the following graph (c) 35
and answer these questions given below it.[MAT 2001] (d) Cannot be determined
Tea in India (In Million kg) 14. The average proportion of tea exported to the tea
800 720 produced over the period is:
700 700 660
600 600 (a) 0.87 (b) 0.47
540
500 480 450 (c) 0.48 (d) 0.66
400 400
288
340 15. What is the first half decade’s average per capita
300
180 availability of tea?
200
100 96 (a) 475g (b) 535g
0
1991 1992 1993 1994 1995 1996 (c) 446g (d) 430g
Exports Production 16. In which year was the per capita availability of tea
(Per capita availability in gm) minimum?
600
(a) 1996 (b) 1994
525 (c) 1991 (d) None of these
500 500
450 17. In which year was there minimum percentage of
400 410 400
390 export with respect to production?
300 (a) 1991 (b) 1992
(c) 1993 (d) 1994
200
18. In which year we had maximum quantity of tea for
100 domestic consumption?
(a) 1994 (b) 1991
0 (c) 1993 (d) 1996
1991 1992 1993 1994 1995 1996
19. What approximately was the average quantity of
(Per capita availability in gm)
tea available for domestic consumption during the
11. Which year shows the maximum percentage of period?
export with respect to production? (a) 324.3 million Kg (b) 400 million kg
(a) 1992 (b) 1993 (c) 410.3 million kg (d) 320.3 million kg
(c) 1996 (d) 1995 20. What was approximately the average population
12. The population of India in 1993 was: during the period?
(a) 800 million (b) 1080 million (a) 625 million (b) 624 million
(c) 985 million (d) 900 million (c) 600 million (d) 757 million
y
o
u
rs
m
a
h
b
o
o
b
.w
o
rd
154 Mixed Graph

p
re
Directions (Qs. No. 21-26): Refer to the pie – charts 25. It was proposed to increase the input in the feed

s
s
.c
given below to answer these questions. [MAT 2002] to 25% of the total input for the year 2000 – 01

o
Major Inputs Used in Agriculture approximately, how much reduction in fertilizers

m
input will be required to keep the total input and
1990-91
the percentage-wise other inputs the same?
Seed (a) `3000 crore (b) `2000 crore
Fertilizers
16.2% 19.2%
Electricity & (c) `1000 crore (d) None of these
Diesel Oil 26. In terms of actual financial input in electricity
3.2% and diesel, the increase in the year 2000 – 01 as
Feed compared to 1990 – 91 was roughly:
31% Others
29.7% (a) 2 times (b) 3 times
Total = `7659 Crore
(c) 4 times (d) the same
Directions (Qs. 27-29): Answer these questions based
2000-01 on the pie – chart given below:
Electricity & Expenditure of Kanpur Industries
Diesel Oil Seed
12.2% 1980 (` 3087 million)
7.2%
Fertilizers Legal
Affairs
31.6% 12%
Compensation
26% Research
15%
Others Taxes Misc.
10%
30.4% Feed Materials
5%
18.6% Construction 18%
2%
Advertising
12%
Total = ` 14,610 crore 1990 (` 4851 million)
21. The total expenditure on electricity and diesel
Research
oil in the year 2000 – 01 exceeded the similar 14%
expenditure in 1990 – 91 by approximately `.
Compensation
(a) 815 Crore (b) 950 Crore 38%
Materials
22%
(c) 1000 Crore (d) 2000 Crore
22. The actual input in fertilizers in the year 2000 Taxes Construction
2%
-01 exceeded the input in the year 1990-91 by 5%
Legal Affairs Advertising
approximately: 4% 15%
(a) 1.5 times (b) 2 times 27. The amount spent on materials for 1980 was 120%
(c) 2.75 times (d) 4 times of the amount spent on:
23. The total input in fertilizers and feed in the year (a) research in 1980
1990 – 91 amounted to approximately: (b) advertising in 1990
(a) `3800 crore (b) `3900 crore (c) compensation in 1980
(c) `4000 crore (d) `3650 crore (d) legal affairs in 1980
24. The input in the feed in the year 2000-01 from that 28. The fraction of the total expenditures for 1980 and
in the year 1990 – 91 has approximately: 1990 spent on compensation was about:
(a) decreased by 55% (b) increased by 15% (a) 1/2 (b) 1/3
(c) increased by 40% (d) decreased by 30% (c) 1/4 (d) 1/5
y
o
u
rs
m
a
h
b
o
o
b
.w
o
rd
Mixed Graph 155

p
re
29. The amount spent in 1980 for materials, advertising 31. How many years witnessed a decline in FNI and an

s
s
.c
and taxes was about the same as : increase in FSI?

o
(a) the amount spent on research and construction (a) 2 (b) 3

m
in 1990 (c) 4 (d) 5
(b) the amount spent for compensation in 1990 32. During 1994-2002 the greatest proportion of FNI
(c) the amount spent on materials in 1990 was put to commercial use in:
(a) 1994 (b) 1996
(d) 5/3 of the amount spent on advertising in 1990
(c) 1999 (d) 2002
Directions (Qs. No. 30-32): Refer to the given line
Directions (Qs. 33 to 38): Study the following figures
graph and the pie-charts and answer these questions:
to answer these questions:
(1000 hectars) [MAT 2005] Manufacturers installed
160 Capacity shares 2001-02
140
120 HL 5% RP
100 FNI 18%
80 HP
FSI 4%
60
40
20 TC
0
RP 46%
1994 1995 1996 1997 1998 1999 2000 2001 2002
33%
FNI – Distribution of Forest Land in North India
FSI – Distribution OF forest Land in South India
4%
Forest land in north india Chart Title
1994 (hectares) 1996 (hectares) 2,000
Commercial
2000 1,800
Commercial
15,000 1,600
Housing Agriculture
Agriculture 1,400
2000 6000 30,000 1,200
Housing
20,000 1,000
Wasteland 800
1000 Wasteland
5,000 600
1999 (hectares) 2002 (hectares) 400
200
Agriculture 0
5,000
Commercial Agriculture Commercial 2000-01 2001-02 2002-03 2003-04 2004-05
30,000 9,000 Wasteland Installed Capacity
35,000 Production Consumption
5,000
Housing
33. When consumption is more than the production,
10,000 the government has to import the shortfall. What
percentage as consumption was imported in 2003 -04?
Housing Wasteland
15,000 5,000 (a) 5.77% (b) 7.36%
(c) 6.12% (d) 5.65%
30. How many hectares of FSI has been distributed 34. What was the percentage increase in installed
between 1994–2002? capacity in the year 2003-04?
(a) 3,00,500 (b) 2,77,500 (a) 10.09% (b) 11.22%
(c) 6,57,000 (d) 9,00,000 (c) 12.14% (d) 13.35%
y
o
u
rs
m
a
h
b
o
o
b
.w
o
rd
156 Mixed Graph

p
re
35. Capacity utilization was the maximum in which of 38. If TC will double its installed capacity of 2003-

s
s
.c
the following years? 04 in the year 2005-06 then what would be the

o
(a) 2000-01 (b) 2001-02 installed capacity of the industry in 2005-06?

m
(c) 2002-03 (d) 2004-05 (Assume that no other manufacturer adds to their
36. In which year was the production as a percentage capacity in 2005-06 and that TC had a 46% share
of installed capacity, the least? in 2003-04).
(a) 2000-01 (b) 2001-02
(c) 2002-03 (d) 2003-04 (a) 2723
37. The installed capacity of HP (In ‘000 tonnes) in (b) 2763
2003-04 was (Assume the same share of capacity
for HP as it had in 2001-02) (c) 2827
(a) 68 (b) 76 (d) Can’t be determined
(c) 78 (d) 83

Instructions: Answer the question no. 1 and 2 on the basis of the data given in the chart. [XAT 2009]

40
35 14
Amount in Rupees

30 12 12
25 10 10 10
8 12
20 10.2 10.2
8.5 7 8.5
15 6 6 8.5
5 6.8
10 6 10
5 8 4.2 8 5 9 7.5 9
5 5 6
4 3.5 4
0
Veg. Patties

Spring Roll

Cream Roll
Veg. Pizza
Mistisukh
Kachouri

Kachouri

Chicken

Chicken

Chicken

Chicken
Patties

Spring
Titbit

Pizza
Panir

Fish

Fish

Prodn. Cost 6 10 5 7 10 6 8 12 14 10 12
Producer's Selling Price 5 8.5 4.2 6 8.5 5 6.8 10.2 12 8.5 10.2
Retailer Selling Price 4 8 3.5 5 8 4 6 9 10 7.5 9
 
y
o
u
rs
m
a
h
b
o
o
b
.w
o
rd
Mixed Graph 157

p
re
The chart above gives per unit selling prices and costs

s
Variable Cost Estimates of Mulch and Textiles

s
in rupees of 11 items prepared by a sweet shop. Margins

.c
Output (Square feet) Labour Cost (`) Material Cost (`)

o
are calculated on percentage basis. Based on the chart

m
above, answer the questions that follow: 125000 95000 54750
1. Which of the following conclusions can be made? 150000 111000 65700
(a) Producer’s margin for panir kachouri is less
175000 133000 76650
than retailer’s margin.
(b) Producer’s margin for chicken pizza is more 200000 160000 88000
than retailer’s margin. Apart from labour and material costs Mulchand
(c) Producer’s margin for fish spring roll is more Textiles incurs administrative costs of ` 40,000 per
than retailer’s margin. month, and electricity costs. Mr. Sharma recalled that
(d) Producer’s and retailer’s margin are highest estimate data of variable electricity cost had certain
for panir kachouri alone. peculiar characteristics. Values at every 25000 sq
(e) Retailer made losses in a few products. ft of output increased in geometric progression till
2. Which of the following conclusion can be drawn
150000 sq ft of output, after which values increased in
from diagram above?
arithmetic progression for every 25000 sq ft of output.
(a) Retailers’ selling price for mistisukh was more
Mr. Sharma remembered that the electricity cost was
than producer’s selling price for chicken titbit.
estimated to be ` 3800 for 25000 square feet of output,
(b) Difference between retailer’s selling price
and producer’s selling price for fish kachouri ` 5700 for 50000 square feet of output and ` 38856.50
was more than that of cream roll. for 175000 square feet of output
(c) There are three types of margins for all items. 3. The estimated cost per square feet of output is least
(d) Of all the margins, both for retailer and for: [3]
producer, producer’s margin for chicken (a) 75000 square feet output
pizza was the maximum. (b) 100000 square feet output
(e) The three lines that connect different points, (c) 125000 square feet output
in the diagram above are superfluous. (d) 150000 square feet output
Directions (Qs. No. 3-5): Based on the following (e) 175000 square feet output
information: [XAT 2011] 4. The estimated material cost given in the table titled
Mulchand Textiles produces a single product of only “Variable Cost Estimates of Mulchand Textiles”
one quality- waterproof synthetic fabric. Mr. Sharma, included the cost of material that gets spoiled in the
the cost accountant of Mulchand Textiles, estimated production process. Mr. Sharma decomposed the
the costs of Mulchand Textiles for different possible estimated material cost into material spoilage cost
monthly output levels. Before he could tabulate his and material usage cost, but he lost the data when
estimates his computer crashed, and Mr. Sharma lost his computer crashed. When he saw the following
all data. Fortunately he had some printouts of some line diagram, he recalled that he measured the
incomplete tables, charts and diagrams. The table estimate of material spoilage cost per square feet
titled “Variable Cost. Estimates of Mulchand Textiles” of output on the y-axis and monthly output on the
provided the estimates of labour and material costs. x-axis.
0.043
Variable Cost Estimates of Mulch and Textiles 0.042
Output (Square feet) Labour Cost (`) Material Cost (`) 0.041
25000 21500 11050 0.04
0.039
50000 41500 22000
0.038
75000 60000 33000
0.037
100000 78000 44000 0.036
y
o
u
rs
m
a
h
b
o
o
b
.w
o
rd
158 Mixed Graph

p
re
Estimated material usage cost per square feet of (c)

s
0.35

s
.c
output- 0.3

o
0.25
(a) decreases up to 125000 square feet of monthly

m
0.2
output and then increases monotonically. 0.15
(b) decreases up to 50000 square feet of monthly 0.1
output, remains constant between 50000 0.05
0
and 100000, decreases between 100000 and
(d) 0.35
125000, remains constant between 125000
0.3
and 175000 and finally increases between 0.25
175000 and 200000 of monthly output. 0.2
(c) remains constant for all levels of monthly 0.15
0.1
output.
0.05
(d) increases up to 50000 square feet of monthly 0
output, remains constant between 50000 (e) 0.35
and 100000, increases between 100000 and 0.3
125000, remains constant between 125000 0.25
0.2
and 175000 and finally decreases between
0.15
175000 and 200000 of monthly output. 0.1
(e) increases up to 100000 square feet of monthly 0.05
output and then decreases monotonically. 0

5. Mr. Sharma found some printouts of line diagrams. Directions (Qs. No. 6-9): These questions are based
The axes of the graphs were not marked, but Mr. on the information given below:
Sharma remembered that he measured monthly Amit an MBA student did his Summer internship
output on the x-axis. Which of the following project in Procter and Gamble for two months. The
diagrams would represent the estimates of marketing department’s regional head has given a
electricity cost per square feet of output versus project to study about the sales and market shares of
monthly output? different branded detergents in india and to know the
most popular brands. Amit started his work and and
(a) 0.35
Study on two FMCG giants in India and came to know
0.3
about the following facts-
0.25
0.2 Detergents: Market share for 2013 = 3200Cr
0.15 Surf Wash Boosters
0.1 Surf Ultra
0.05 Sun light
Rin Power White
0
wheel Green
(b) 0.35 Wheel Blue
0.3 Ariel Super Soaker
Ariel Super Soaker
0.25
Nirma
0.2 Henko Stain Champion
0.15 white Giant
0.1
0 5 10 15 20 25 30 35 40
0.05
Percentage
0
y
o
u
rs
m
a
h
b
o
o
b
.w
o
rd
Mixed Graph 159

p
re
& Gamble and spic Fine Chemicals over the

s
Brand Owners & Percentage Change

s
.c
Over the Previous Year’s Sale previous year in 2014 is the same as it was in 2013.

o
(a) 6.5% (b) 4%

m
1. Hindustan Lever 2. Procter & Gamble
• Surf Wash Boosters –15.1 • Ariel 2.1 (c) 8% (d) Indeterminable

• Surf Ultra 1.5 • Ariel Super Soaker 364


7. By what amount is the market share of Hindustan
Lever more that of Procter & Gamble in the year
• Sun Light –3.2 3. Nirma Chemical Works 2013 (` Crore)?
• Rin Power White 420 • Nirma –9 (a) 760 (b) 860
• Wheel Green –5.3 4. Spic Fine Chemicals (c) 960 (d) None of these
• Wheel Blue 86 • Henko Stain Champion 15 8. By how many `crores has the market share of Spic
Fine Chemicals changes over the previous year?
• White Giant 100
(a) 11.2 (b) 9.6
Detergent Market This Year : ` 3200 crore
(c) 5.4 (d) 7.3
Only the given companies are the players in the detergent
market. 9. If both varieties of Wheel show the same trend
next year, then what will be their combined market
6. If the detergent market decreases by 18.75% in the share (`crore), assuming that the total detergent
year 2014 and Procter and Gamble takes over Spic market remains constant?
Fine Chemicals, what will be the percentage change
(a) 500 (b) 600
in the sales of Procter & Gamble in 2014? Assume
that percentage change in the year’s sales for Procter (c) 700 (d) Indeterminable

Directions (Qs. No. 1-3): The following line graph 60


gives the number of customers visiting a CD parlour on
50
various days of the week during different time periods.
The table gives the data about the total amount 40
collected on various days of the week during different
30
time slots( Slot 1: 10am to 2pm, slot 2: 2pm to 6pm, slot
3: 6pm to 10pm) only one CD per customer is allowed 20
while each Hindi CD costs ` 40 and each English CD
10
cost ` 80. Further more English CDs are available only
in slot 2 and slot 3. A person entering a particular slot 0
leaves the parlour in the same slot. SUN MON TUES WED THRUS FRI SAT
10am to 2pm 2pm to 6pm 6pm to 10pm
y
o
u
rs
m
a
h
b
o
o
b
.w
o
rd
160 Mixed Graph

p
re
s
Amount Collected ( in ` )
Store House Reserve (in million tonnes)

s
.c
DAY 2500
10am to 2pm 2pm to 6pm 6pm to 10pm

o
m
Sunday 800 1360 1520 2000
Monday 320 200 320
1500
Tuesday 400 360 440
Wednesday 240 440 320 1000
Thursday 240 520 440 500
Friday 400 640 1480
Saturday 620 1040 1600 0

ra
na

ab

r
P

ha
al
ga
U

ht
ya

nj

er
1. On which day of the week the percent of customers

Bi
en

as
Pu
ar

ar
tB
H
renting CDs is the maximum?

ah
es

M
W
(a) Sunday (b) Saturday
Store House Reserve (in million tonnes)
(c) Friday (d) Indeterminable
2. on Thursday which slot can have the ratio of 8 wheat reserve in different state store houses
customers renting CDs to customers Visiting the 4. In the beginning of the year, Kerala has Wheat
shop during the slot as the minimum? stock of 3 million tonnes which rose to 5 million
(a) Slot 1 (b) slot 2 tonnes at the end of the year. Approximately, what
(c) Slot 3 (d) indeterminable percentage of its production is exported?
3. A day is said to be normal day if the number of (a) 50% (b) 44.44%
customers visiting the parlour go on increasing as
(c) 33.33% (d) 30%
we progress from Slot 1 to Slot 3. How many days
in the week given are normal days? 5. If Haryana imports its entire wheat short fall from
(a) 0 (b) 3 Bihar what percentage of Bihar exports is not to
(c) 4 (d) None of these Haryana? (Assume no change in wheat stock).
Directions (Qs. No. 4-6): The following graphs give (a) 50 (b) 40
the production and consumption of wheat in certain (c) 30 (d) 20
part of India, as well as the reserves in store houses. 6. If Maharashtra produces 5% of the wheat
Shortfall /excess between production and consumption production of INDIA, what is the percentage share
which is met by imports/exports. of Punjab?
45
(a) 1.25 (b) 2
40
(c) 2.5 (d) 3
35
30 Directions (Qs. No. 7-10): Answer the questions based
25 on the following information. [XAT 2013]
20 Data on an outgoing football league of a country is
15 given here. 20 teams are playing in the league. The
10 rules of the league are as follows :
5 1. Each team plays all the other teams twice, once in
0 its home ground, and once in the opponent’s home
a

ra
b

r
P

ground. These matches are known as the “Home”


l
an

al

ha
ja

ga
U

ht
er
n
y

Bi
en

as
Pu
ar

match and the “Away” match respectively.


ar
tB
H

ah
es

M
W

production consumption 2. A win results in 3 points, a draw in 1 point, and a


* production and consumption is in million tones loss in 0 point for team.
y
o
u
rs
m
a
h
b
o
o
b
.w
o
rd
Mixed Graph 161

p
re
3. The number of goals a team scores is termed as

s
Overall

s
“Goals for” and the number of goals it concedes

.c
Team M W D GF GA

o
is termed as “Goals Against”. We get the “Goal

m
Difference” by subtracting “Goals Against” from EV 1 1 0 1 0
“Goals For” . MU 1 1 0 3 2
The ranking of the teams is decided on the total points. TOT 2 0 2 2 2
If two teams are tied on their total points, the team
NW 1 0 1 1 1
which has higher Goal Difference gets the higher rank.
If the tie cannot be resolved on Goal Difference, Goals AS 1 0 1 0 0
For is checked followed by Goal Against. If the tie ST 1 0 1 0 0
persists, the teams are ranked in the ascending order
of their names. Goal Difference - Overall
Table 1: Provides data on the current top 13 teams 9
based on the overall situation, i.e., by taking into 8
account both home matches and away matches of each 7
team. 6
Table 2: Provides data on the current top 13 teams
based on home matches only. 5

Chart 1: Provides a plot of the goal difference of each 4


of the 13 teams based on the overall situation. 3
Overall 2
Team M W D GF 1
CH 3 3 0 8 0
SW 3 2 1 10 -1 AS CH EV FU MC MU NC ST SU SW WB WG WH
WB 3 2 1 6
-2
MC 3 2 1 8 Goal Difference - Overall
MU 3 2 0 6
7. Considering away matches only, which of the
WH 3 2 0 4
following teams is the second ranking team?
EV 3 2 0 4 (a) AS (b) CH
AS 3 1 2 2 (c) WG (d) SW
WG 3 1 1 4 (e) WB
NC 3 1 1 3 8. Considering away matches only, the least number
FU 3 1 0 7 of teams with either 0 or 1 point is:
ST 3 0 3 3 (a) 2 (b) 3
SU 0 0 2 2 (c) 4 (d) 5
(e) 6
Overall 9. Let us define a term pos as the difference between
Team M W D GF GA “home rank” and “away rank”. Which of the
WB 1 2 0 5 0 following has the maximum value of pos?
WH 2 2 0 4 0 (a) AS (b) WB
(c) WH (d) MC
CH 2 2 0 6 2
(e) SW
MC 2 2 0 6 3 10. How many unique values of goal difference are
SW 2 1 1 5 2 there for away matches?
NC 2 1 1 3 2 (a) 5 (b) 6
(c) > 6 (d) > 6 and < 13
FU 1 1 0 5 0
(e) > 6 and < 14
y
o
u
rs
m
a
h
b
o
o
b
.w
o
rd
162 Mixed Graph

p
re
s
s
.c
o
m
Directions (Qs. No. 1-3): A marketing survey team 2. What percentage of the total voters would have
visited five different colleges of Kolkata where the been from Lady Bovine, if the number of voters
respective students of Asutosh college, South Point from south point increased by 15%?
college, St. Xavier’s college, Lady Bovine and South (a) 38.61% (b) 38.55%
Kolkata Government college have given their votes (c) 34.45% (d) 34.55%
for their favorite Actors. The number of male voters of 3. Find the difference between the number of female
South point increased that of St. Xavier by 40500. voters from Lady Bovine and the number of male
Voters according to their Colleges voters from Asutosh?
5% Asutosh (a) 24000 (b) 17750
(c) 12750 (d) 11250
Asutosh college
20% SCGC
South point Directions (Qs. No. 4-7): The following table gives
25% South
point the amount of loans disbursed by the various banks
St. Xavior
10%
in the year 2010- 2014. The bank which disburses the
40% Lady St. Xavier
Lady Bovine
maximum amount in a particular year is said to be a
Bovine
SCGC market leader.
% of Female Voters of Corresponding colleges Constituents of others in 2014
70%
cosmos -11%
60% 11%
18% Rupee-14%
50% 14% union-16%
40% western -13%
28% 16%
30% co-oper -28%
13%
20% other smaller banks -18%
10%
0% BANK 2010 2011 2012 2013 2014
South point St. Xavier Lady Bovine SCGC ICICI 88 98 113 129 146
% of Female Voters of Corresponding colleges HDFC 98 118 128 138 130
1. Find the ratio of number of voters from SCGC to CBI 47 43 53 49 58
the number of male voters from St. Xavier? SBI 78 83 88 94 103
(a) 5 : 3 (b) 5 : 1 BOM 46 53 48 59 71

(c) 13 : 9 (d) None of these OTHERS 89 111 123 133 145


y
o
u
rs
m
a
h
b
o
o
b
.w
o
rd
Mixed Graph 163

p
re
6. If in 2014, the Rupee and western banks go

s
4. In 1999 all the major banks had same

s
.c
percentageshare in auto-finance as their respective bankrupt on account of increasing non-performing

o
assets then the market Shares of these banks gets

m
shares in total loan disbursement. How much was
the share of HDFC greater than the share of SBI if equally distributed to other banks. What is the
percentage increase in the market share of ICICI
the total auto-finance market was of 180 Crore?
bank?
(a) 8.8 cr (b) 11 cr
(c) 13 cr (d) 15.6 cr (a) 5.2% (b) 8.3%
5. In the year 2015, all the banks retained their (c) 11.2% (d) Indeterminable
respective market shares as they had in 2014.
The merger of which of the following three banks 7. By what percentage did the total market of loan
disbursement increase in the year 2013 with
would make them the market leader?
respect to 2012?
(a) CBI, cosmos, jan kalian
(b) CBI, Rupee, Union (a) 8.08% (b) 9.28%
(c) Cosmos, Rupee, SBI (c) 13.5% (d) Indeterminable
(d) Rupee,union,SBI
y
o
u
rs
m
a
h
b
o
o
b
.w
o
rd
164 Mixed Graph

p
re
s
s
.c
o
m
CONCEPT APPLICATOR 9. (a) Obese women in 2006 are 60000 ×
20
12000
=
1. (a) The number of male employees in Administra- 100
tion department is 175 and Printing department and obese children in 2006 = 12000 ×
25
3000
=
is 100. Therefore, the ratio is 175:100 = 7:4 100

Alternative method of calculation : The total is 15000. Obese men in 2006 are
32.5
7 63000 × = 20475 . The difference between
× 12% of 2500 7 100
12 Obese men and obese women children
Required ratio = 2 =
4 together is 20475 – 15000 = 5475
× 6% of 2500
3 10. (d) We can calculate like, 85 % of 15000 + 90 %
2. (c) The total number of employees in IT are
of 21000 = 31650.
350 and Operations are 450. The difference
between them is 450 – 350 = 100. 11. (d) Total number of girls in C = 26 % of 6000 –
3. (d) Number of male employees in HR and 900 = 1560 – 900 = 660, similarly for E and
Marketing are 250 + 175 = 425. D. then C, E and Band girls in D are 660 + 540
Number of female employees in HR and + 600 = 1800.
Marketing are 150 + 200 = 350. The ratio 12. (c) Ratio of boys in C, girls in B and total number
between the male and female employees is of students in E = 900 : 140 : 1740 = 45 : 7 : 87.
425 : 350 = 17 : 14.
13. (d) The difference between total number of
4. (d) The number of female employees in HR are 150. students in F and boys in E is 1200 – 360 = 840
5. (b) The difference between the number of male 14. (b) Total girls students in School E is 29% of
and female employees in logistic department
6000 – 1200 = 540. So,without calculating the
is 150 – 125 = 25.
total students of different schools. 9% of 6000
6. (c) The average of Obese men, Obese = 540
women and obese children in 2007 is
23100 + 13500 + 2000 15. (d) The number of girls in school A are
= 12867 (approx.) 220
3 ×100 = 40.74% of the total number of
7. (b) The obese men in 2009 is 37.5% of 78000 i.e. 540
29250. Persons not suffering from obesity are students in B.
78000 – 29250 = 48750. The number of obese 16. (a) The total of male teachers in F and female
29250 teachers in C, B are 100 + 660 + 320 = 1080.
men is × 100 = 60%
48750
20 17. (d) Female teachers in district D = 15 % of 4500
8. (d) Obese women in 2006 = 60000 × =12000 – 100 = 675 – 100 = 575
100
27.5
Total no. of teachers in District A = 145 of
and obese men in 2008 = 70000 × =19250
100 4500 = 630.
The ratio between them is 12000 : 19250 575

Required Percentage = × 100 = 91.2%
630

= 48 : 77.
y
o
u
rs
m
a
h
b
o
o
b
.w
o
rd
Mixed Graph 165

p
re
18. (a) 26. (d) Production of Company A in the year 2009 =

s
s
.c
Option Male Female 550 and production of company A in the year

o
a B = 400 320 2010 = 700.

m
b D = 100 575 Required percentage = (700 – 550)/550 × 100
= 27.27% or approximately 27%
c D & E = 600 1021
27. (b) Sales of Company A in 2009 = 400 and
d E & F = 600 615 production = 550

Only B has more male. Required percentage = 400/550 × 100 = 72.72
of approximately 73%
19. (b) The difference between male and female 28. (c) Company A : 5 × x = 750
teachers in E and female teachers in F is 21%
x = 150
of 4500 – {(6% 4500) – 100} = 945 – 170 = 775.
Company B : 4 × x = 4 × 150 = 600
20. (c) The ratio of male teachers in C and female So, production of company B be 600, 700,
teachers in B = 600 : 320 = 15 : 8 800, 600, 650, 700 respectively.
21. (c) Total no. of mobile phone sold in the month Sum of production = (600 + 700 + 800 + 600
july = 17% of 45000 = 7650. + 650 + 700) = 4050
Total number of mobiles sold in the month Average = 4050/6 =675
December = 16% of 45000= 7200 29. (d) Required ratio = total production of company
Mobile phone sold by company B in the A : total sales of company A = 4050 : 2750 =
month of July = 7/15 × 7650 = 3570 81 : 55.
For Company B in the month of December = 30. (c) Production of company B in the year 2006
7200 × 9/16 = 4050 and 2008 are 600 and 800 respectively
Required ratio = 600 : 800 = 3 : 4
The ratio of the mobile phones of B in July
and B in December = 3570 : 4050 = 119 : 135 CONCEPT BUILDER
22. (c) The number of mobile phone sold in the 1. (b) The difference between the male graduates
month of November = 12% of 45000 = 5400 and XII pass male candidates are 224000 –
Number of mobile sold by company A in 210000 = 14000.
November = 5400 × 7/12 = 2520. the number 2. (d) The ratio of female graduates in state E and
of mobile phones of company A during XII pass female candidates of state D is
November is 2520 and the discount was 35%. 210000 : 224000 = 15 : 16
The number of mobile phones sold without 3. (e) Female graduates of state C is
35 160000
giving discount is 2520 × = 882 ; 2520 – ×100 = 5% of XII pass population
100 3200000
882 = 1638.
of all the states together.
23. (d) The shopkeeper earned a profit of 433 on each
4. (a) Std XII male candidates of state C is
mobile of company B, the total profit earned
256000
during October is 1500 × 433 = 6,49,500. ×100 = 8% of Std XII population of
3200000
24. (d) The number of mobiles of company A during
all the states together
4080
July is × 100 = 129.52 i.e. 130% approx. 5. (d) The ratio of Male graduate of state E and XII
3150 pass female candidate of the same state is
Of the same company during December.
270000 : 32000 = 27 : 32
25. (a) Total number of mobile phones of company B 336000
6. (d) Total Graduates in state F is ×100 =70%
during august and September is 5500 + 4500 480000
= 10000 of total Std XII population of state A.
y
o
u
rs
m
a
h
b
o
o
b
.w
o
rd
166 Mixed Graph

p
re
7. (b) XII male population of state E is 1991 ⇒ 480 – 96 = 384

s
s
.c
288000 1993 ⇒ 720 – 288 = 432
×100 = 75% of the XII male popula-

o
384000 1994 ⇒ 700 – 340 = 360

m
tion of state F. 1996 ⇒ 660 – 450 = 210
8. (c) Required Ratio 19. (a) The average quantity of tea available for
(graduate + XII) male domestic consumption is 384 + 432 + 360 +
=
(graduate + XII) female 1946
210 + 360 + 200 = = 324.3 million kg
6
 7  7 20. (d) Population of all the years :-
 24 × 16% ×  +  32 × 15% × 
= 2 16 480 − 96
 5  9 1991 ⇒ = 985 million
 24 × 16% ×  +  32 × 15% ×  0.39
12 16 540 − 180
1992 ⇒ = 878 million
= 217 : 215 0.41
9. (e) The ratio of total graduate population in 720 − 288
state D and total XII population in state 1993 ⇒ = 1080 million
0.4
D = 4,08,000 : 3,84,000 = 51 : 48
700 − 340
10. (c) Female graduates in state B is 1994 ⇒ = 800 million
162000 0.45
× 100 = 77% (approx..) of the female 600 − 400
210000 1995 ⇒ = 400 million
graduates in state E. 0.5
660 − 450
11. (c) 1996 shows maximum percentage of 68.18% 1996 ⇒ = 400 million
export with respect to production. 0.525
12. (b) Tea available in India 1993 = 720 – 288 = Therefore, the average population
432 and Per capita availability in 1993 = = 985 + 878 + 1080 + 800 + 400 + 400 = 757 million

0.4kg. therefore, the population in India is 6
21. (a) Expenditure of electricity and diesel oil in
432 1990 – 91 = 245 (approx..) and in 2000 – 01
= 1080
0.4 = 1052 (approx.) the expenditure exceeded by
13. (d) Cannot be determined since there is no data 1052 – 245 = 807 (approx..) which is nearly to
given about area. 815 crore
14. (b) Tea exported over the period = 96 + 180 + 288 22. (c) The actual input in fertilizers in 2000 – 01 =
+ 340 + 400 + 450 = 1754 million kg and tea 4617 and in 1990 – 91 = 1241. 4617 – 1241
produced over the period = 480 + 540 + 720 = 3376 more than that in 1990 – 91. It is 2.72
+ 700 + 600 + 660 = 3700 million kg. The times approximately 2.75 times.
1754 23. (d) Total input in fertilizers and feed in 1990 – 91
average proportion = = 0.47
3700 is 1241 + 2374 = 3615 which is close to the
15. (b) The average per capita availability of tea is option 3650 crore.
390 + 410 + 400 + 450 + 500 + 525 24. (b) The input in the feed in the year 2000 – 01 has
= 535 g 2717 − 2374
5 increased by × 100 = 12.62%
16. (c) It is clearly evident from the graph that 1991 2717
which is nearly 15%.
has the minimum per capita availability of tea.
25. (d) Input in feed year 2000-01 = 2717 there is a
17. (a) There was minimum percentage of export 25
with respect to production is in 1991. increase of 25% 2717 × = 679 . we have
100
18. (c) Quantity of tea for domestic consumption in to reduce the same amount from fertilizers to
the following years:- keep the input and the percentage same.
y
o
u
rs
m
a
h
b
o
o
b
.w
o
rd
Mixed Graph 167

p
re
26. (b) The input of electricity and diesel oil in the 36. (d) Referring to the previous question, production

s
s
.c
year 2000 – 01 is 1052 and in 1990 – 91 is
as a percentage of installed capacity was least

o
807
254 and 1052 – 254 = 807, = 3.29 this

m
245 in the year 2003-2004.
is approximately 3 times.
27. (a) The amount spent on materials for 1980 = 37. (b) The installed capacity of HP in 2003 – 04 was
555.66 and research in 1980 = 463.05 of whose 4
4% of 1893 i.e. × 1893 =
75.72
120% is 555.66 [463.05 × 120% = 555.66] 100
28. (a) Rxpenditure of compensation in 1980 = approximately 76.
802.62 and in 1990 = 1843.38. and the fraction 38. (b) We know that the installed capacity of
1 industry in 2003-04 = 100% installed capacity
of this is (approx..)
2 of industry in 2005-06 = 146 % therefore,
29. (d) Amount spent in materials, advertising and 146% of 1893 = 2763
taxes = 1234.80 million and amount spent
on advertising in 1990 = 727.65. therefore, CONCEPT CRACKER
5 1. (c) Consider option A-Producer’s margin for
× 727.65 = 1212.75 which is close to1234.80
3 Paneer Kachori is 25% and retailer’s margin
30. (d) 40 + 60 + 80 + 120 + 100 + 110 + 120 + 150 for the same is 20%. Hence this statement (A)
+ 120 = 9 ,00, 000 hectares of FSI has been is wrong.
distributed between 1994-2002. Option B Producer’s margin for Chicken
31. (a) It is clearly visible from the graph that the Pizza is 13.33% and retailer’s margin for the
common years which witnessed a decline in same is 17.1%
FNI and increase in FSI is 1997 and 2000 Hence statement (B) is wrong.
32. (c) The greatest proportion of FNI in commercial Option C Producer’s margin for fish spring
30000 roll is 20% which is more than retailer’s
use is in 1999 = = 1 and the rest are
lower than this. 30000 margin 16.66%
33. (a) Consumption is more than the production Hence, C is correct. We will check other
only in the year 2003 – 04. Required % options as well and prove that rest all options
1576 −1485 are wrong.
consumption = × 100 = 5.77% 2. (e) Again we will eliminate the options one by
1576
34. (b) The % increase in installed capacity in the one. Option A - Retailer’s S.P. for Mistisukh
1893 − 1702 is 5 which is the same as producer’s S.P. for
year 2003 – 04 = × 100 =11.22% Chicken titbits
1702
35. (b) Capacity utilization Hence, option ‘A’ is wrong
1363 Option B- Retailer’s S.P. (Fish kachori) –
In 2000-01 = ×100 = 80.08% Producer’s S.P. (Fish kachori) = 1.5
1702
And Retailer’s S.P. (Cream roll) – Producer’s
1523
In 2001-02 = ×100 = 89.48% S.P. (Cream roll) = 1.5
1702 As they are equal, hence option ’B’ is wrong.
1488 Option C As there are only 2 types of margins,
In 2002-03 = × 100 =87.42%
1702 ‘C’ is wrong.
1485 Option D Producer’s margin for Chicken
In 2003-04 = × 100 = 78.44%
1893 pizza is 13.33% which is not the maximum;
1596 hence, D is also wrong
In 2004-05 = × 100 = 84.31%
1893 Option E- it Is correct.
y
o
u
rs
m
a
h
b
o
o
b
.w
o
rd
168 Mixed Graph

p
re
3. (d) The value of different costs at various outputs are given below

s
s
.c
Output (Sq ft) Labour Material Electricity cost Admin cost Total Cost Total cost/

o
m
Cost cost Output
25000 21500 11050 3800 40000 76350 3.054
50000 41500 22000 5700 40000 109200 2.18
75000 60000 33000 8550 40000 141550 1.88
100000 78000 44000 12825 40000 174825 1.74
125000 95000 54750 19237 40000 208987 1.67
150000 111000 65700 28856 40000 245556 1.63
175000 133000 76650 38856 40000 288506 1.64
200000 160000 88000 48856 40000 336856 1.68

The estimated cost per square foot of output is least when output is 150000.
4. (c) Similar to previous question option C is next year will be 94.7% of 18.2% of 3200 =
correct. ` 552 crores. And market share of Wheel Blue
5. (b) From the table of previous question is 2.4% this year and it shows an increase of
Comparing the values of electricity cost for 86% over the next year. So, its market share
various outputs, we can see that only option in the next year will be 1.86 x 2.4% of 3200 =
B give the correct value of electricity cost per ` 143 crores. So, their combined share comes
square foot of output. to approximately ` 700 crores.

Solution from 6 - 9 : CONCEPT DEVIATORS


6. (d) As we do not know the market share of
Solution from 1-3 :
detergents for the year 2014. Hence, the
answer is (d). 1. (d) On any day in the first slot, the people will
borrow only ` 40 CDs. But during the second
7. (a) The market share of Hindustan Lever is
30.7% and that of Procter & Gamble is 6.9% and third slot the number of people renting
this year. So, the difference in their market the CD will depend upon the selection of
shares comes to 23.8% of 3200 crores = CD (English or Hindi) this number cannot be
` 761 crores respectively. defined, as we we don’t know the exact number
8. (b) For Henko Stain Champion over the previous of people who will buy the English CD or
year there is a increase. This year its market Hindi .so, answer should be indeterminable.
share is 0.9% of 3200 = 28.8. So, for the 2. (c) In the first slot revenue generation is ` 240
previous year, it must have been 28.8/1.15 = i.e 6 people renting the CDs whereas the
25. So, the change is 3.2. For White Giant,
people visiting in this Slot were 22. In the slot
over the previous year, there is a 100%
increase. This year its market share is 0.4% of 2, revenue generated is ` 520. The minimum
3200 = 12.8. So, in the previous year it must number od customers renting the CDs is 7
have been 6.4. Here the change is 6.4. So, and visiting customer were 10. In the Slot 3,
total change comes to 9.6. revenue generated ` 440, minimum customer
9. (c) Market share of Wheel Green is 18.2% this rented CDs is 6 and number of Visiters is 25.
year and it shows a change of – 5.3% over So, the minimum possible ratio are 6/22, 7/10
the next year. So, its market share in the and 6/25. Hence, the answer is Slot 3
y
o
u
rs
m
a
h
b
o
o
b
.w
o
rd
Mixed Graph 169

p
re
3. (b) From the graph we can say the number of
The above table is formed from both the table

s
s
.c
people is increasing and just by looking at the given in the questions. From the information

o
graph for various slots, Sunday Wednesday in the question we say that WG is a little bit

m
and Saturday the condition is satisfied. different. WG must have won its match in
4. (c) From the graph, Kerala’s production is 30 any away tie, because WG is not in the top
million tonnes and consumption is 18 million 13 in home matches. Thus for WG of its
tonnes. So Kerala has surplus of 12 million three matches the win must have definitely
tonnes. As it is given in the question there is Occurred in an away match, the draw and loss
an increase in the reserve from 3 million to 5 could happen in either an away or a home tie.
million tonnes which is 2 million ( difference). In case WG has 1 win and 1 draw in away ties,
So kerala may export 10 million tonnes of it would have 4 points is its away matches.
wheat. Which is 33.33% of 30 million. However that would place it at a possible best
position of 3rd inn the table above – since
5. (d) Haryana’s wheat import is 38 million tonnes
WG cannot be placed above SW since the
consumption and 22 million tonnes of wheat is
maximum possible goal difference for SW
production, it has shortage of 16 million tonnes
would be 4. Thus for the teams ranked 3rd
of wheat which Haryana has to importes. So
onwards in the table above, there might be a
the same way Bihar export 20 million tonnes.
possibility of a shifting down of their ranks

Required percentage (% of wheat of Bihar not – that’s wy we have mentioned CH’s rank in
20 − 16 away matches as 3+ and Not 3.
exported to Haryana) = × 100 = 20 %
20
7. (d) We can say that team SW ranks second.
6. (a) Let assume that X is the India’s production of
wheat. So 5% of X production = 40 million 8. (d) Team WB,MC,WH,NC and FU scored 0 or 1.
May be there are more that % teams but these

Then x = 800 million tonnes
fives are certain.
10

Therefore punjab’s share = × 100 = 1.25% 9. (a) As has maximum value of pos.
800
Solution from 7 - 10: 10. (b) We observe SU closely than we say SU
Team M W D GF GA GD Points Rank
played 2 drawn matches. Its goal difference is
0 and it has scored 2 goals. Hence it must have
CH 1 1 0 2 0 2 3 3+
conceded two goals. This leads two cases-
SW 1 1 0 5 0 5 3 2
case 1: SU drew one match 2-2 and match
WB 1 0 1 1 1 0 1 8+
0-0. And case 2: SU drew both its matches
MC 1 0 1 2 2 0 1 7+ 1-1. So in both the cases SU did not played
MU 2 1 0 3 3 0 3 5+ both the matches at homke- as Su is not in top
WH 1 0 0 0 3 -3 0 10+ 13 teams on a home basis. Atleast one match
EV 2 1 0 3 3 0 3 4+ it has played away. So goal difference be 0.
AS 2 1 1 2 0 2 4 1
WG Data not present
CONCEPT ELEMINATOR
NC 1 0 0 0 2 -2 0 9+ Solution from 1-3 :
FU 2 0 0 2 6 -4 0 11+ Let the total number of voters be x as from the question,
ST 2 0 2 3 3 0 2 6+ we came to know that 70% of 25% of x – 40% of 10 %
SU Data not available of x = 40500 or, x = 300,000
y
o
u
rs
m
a
h
b
o
o
b
.w
o
rd
170 Mixed Graph

p
re
1. (b ) Total number of voters from SCGC = 20% of 4. (d) In 1999 HDFC and SBI disbursed 128 crore

s
s
.c
300000 and 88 crore respectively. So their market

o
Then the number of male voters of St.xavier = share for loans were 128 / 553 = 23.14% and

m
40% of 10% of 300000 88/553 = 15.9% respectively. So the difference
Required ratio = 0.2/ 0.04 = 5:1 between their shares is 21.33 – 15 = 7.2%.
Auto finance market in 1999mwas of 180
2. (b) Total number of voters from Lady bovine =
crore, so 7.2% of 180 crore is approximately
40% of 300000
13 crore. Hence, the answer is (d).
New addition to the total number of voters =
5. (d) Market Share of Rupee, Union and SBI =
15% of 25% of 300000
103 + (14 % + 16 %) of 146.5 crore, which
New total number of voters be = 300000 +
is more than ICICI, the market leader. Hence,
15% of 25% of 300000 the answer is (d).
Hence, required %
6. (d) Since we do not know the total number of
40% of 300000 banks, the answer be determined. Hence, the
= × 100
300000 + 15% of 25% of 300000 answer is (d).
= 38.55% 7. (b) Total amount disbursed in the year 1998 was
3. (c) The number of female voters from lady 506 crore and for the year 1999 it was 553
Bovine = 20% of 40% of 300000 = 24000 crore, hence percentage increase = (553 –
The number of male voters from asutosh 506) × 100/506 = 9.28%. Out of the given
college = 75% of 5% of 30000 = 11250 answer option, the most appropriate answer is
Required difference = 24000 – 11250 = 12750. (b).
y
o
u
rs
m
a
h
b
o
o
b
.w
o
rd
60

p
re
50

s
40

s
Data

.c
30

o
m
20

10
Sufficiency
0
SUN MON TUES WED THRUS FRI SAT
10am to 2pm 2pm to 6pm 6pm to 10pm

Topics No. of Questions Level


Concept Applicator 11 SNAP
Concept Builder 8 XAT
Concept Cracker 51 CAT
Concept Deviator 129 CAT, XAT advance level
Total 199

INTRODUCTION Tip 2 : Identify the parent question, means the main


question that is asked.
Data sufficiency is one of the most important as well
Tip 3 : Look at the statement (i) and information given
as most difficult type of question. Questions based
in that, see whether it is sufficient enough to
on data sufficiency requires deeper knowledge in the
answer the parent question.
subject area. Data sufficiency questions can be from
any topic viz Numbers, Arithmetic, Algebra, Geometry, Tip 4 : Look at the statement (ii) and information
and puzzles. These questions are not only difficult but given in that, most important is that when
also confusing. Students generally get confuse with the you see statement (ii) then just forget the
given information. Some general tips to solve the data information given in statement (i) , then see
sufficiency questions are discussed below. whether it is sufficient enough to answer the
parent question.
Tip 1 : 1st and foremost is understanding of options. In
Tip 5 : When the given statements are not sufficient
GMAT options are fixed but in other aptitude
then use information given in both the
test exams these options vary.
statements.
y
o
u
rs
m
a
h
b
o
o
b
.w
o
rd
172 Data Sufficiency

p
re
s
s
.c
o
m
Directions (Qs. No. 1-7): Each of the following (I) P secured one-third marks of the total of Q, R
questions is followed by two statements. Mark and S.
[SNAP 2010] (II) Average marks obtained by Q and R are
(a) if statement I alone is sufficient to answer the 20 more than that secured by S.
question. 6. How many ice cubes can be accommodated in a
(b) if statement II alone is sufficient to answer the container?
question. (I) The length and breadth of the container is 20
(c) if both statement I and II together are necessary to cm and 15 cm respectively.
answer the question. (II) The edge of the ice cube is 2 cm.
(d) if both statements I and II together are not sufficient 7. Ram got ` 1500 as dividend from a company. What
to answer the question. is the rate of interest given by the company?
(I) The dividend paid last year was 10%.
1. Is ‘b’ positive?
(II) Ram has 350 shares of Rs 10 denomination.
(I) a + b is positive.
Directions (Qs. No. 8-11): Each of questions consist
(II) a – b is positive.
of question followed by two statements numbered
2. In a general body election, 3 candidates, p, q and r I and II [SNAP 2010]
were contesting for a membership of the board.
(a) if data in Statement I alone is sufficient to answer
How many votes did each receive?
the question but the data in Statement II alone is
(I) p received 17 votes more than q and 103 not sufficient to answer the question.
votes more than r.
(b) if data in Statement II alone is sufficient to answer
(II) Total votes cast were 1703. the question but the data in Statement I alone is not
3. If C1 and C2 are the circumferences of the outer sufficient to answer the question.
and inner circles respectively. What is C1 : C2? (c) if data in Statement I and II together are necessary
(I) The two circles are concentric. to answer the question.
(II) The area of the ring is 2/3 the area of greater (d) if data in Statement I and II together are not
circle. sufficient to answer the question.
4. What is the middle number of 7 consecutive 8. ∆ABC and ∆PQR are congruent
whole numbers? (I) Area of ∆ABC and ∆PQR are same
(I) Product of number is 702800. (II) ∆ABC and ∆PQR are right angle Triangles
(II) Sum of the number is 105. 9. Salary of A and B is in ratio 3:4 and expenditure is
5. Total marks obtained by P, Q, R and S in in ratio 4:5. What is the ratio of their saving?
Mathematics is 360. How many marks did P (I) B’s saving is 25% of his salary.
secure in Mathematics? (II) B’s salary is ` 2500.
y
o
u
rs
m
a
h
b
o
o
b
.w
o
rd
Data Sufficiency 173

p
re
10. What is the average height of the class? 11. Ram is taller than Shyam and Jay is shorter than

s
s
.c
(I) Average height of the class decreases by 1 Vikram. Who is the shortest among them?

o
cm if we exclude the tallest person of the (I) Ram is the tallest.

m
class whose height is 56 cm.
(II) Shyam is taller than Vikram.
(II) Average height of the class increases by 1
cm if we exclude the shortest person of the
class whose height is 42 cm.

Directions (Qs. No. 1-2): are followed by two statements Directions (Qs. No. 3-4): are followed by two
labelled as I and II. Decide if these statements are statements labeled as I and II. You have to decide if
sufficient to conclusively answer the question. Choose these statements are sufficient to conclusively answer
the appropriate answer from the options given below: the question. Choose the appropriate answer from
[XAT 2011] options given below: [XAT 2008]
(a) Statement I alone is sufficient to answer the (a) If statement I alone is sufficient to answer the
question. question.
(b) Statement II alone is sufficient to answer the
(b) If statement II alone is sufficient to answer the
question.
question.
(c) Statement I and Statement II together are sufficient,
but neither of the two alone is sufficient to answer (c) If statement I and statement II together are
the question. sufficient but neither of the two alone is sufficient
(d) Either Statement I or Statement I alone is sufficient to answer the question.
to answer the question. (d) If either statement I or statement II alone is
(e) Neither Statement I nor Statement 11 is necessary sufficient to answer the question.
to answer the question.  (e) Both statement I and statement II are insufficient to
1. Let PQRS be a quadrilateral. Two circles 01 and answer the question.
02 are inscribed in triangles PQR and PSR 3. The base of a triangle is 60 cms, and one of the
respectively. Circle 01 touches PR at M and circle base angles is 60°. What is length of the shortest
02 touches PR at N. Find the length of MN. sideof the triangle?
I. A circle is inscribed in the quadrilateral (I) The sum of lengths of other two sides is 80
PQRS. cms.
II. The radii of the circles 01 and 02 are 5and 6 (II) The other base angle is 45°.
units respectively. [5]
4. A, B, C, D, E and F are six integers such that E <
2. Given below is an equation where the letters
F, B > A, A < D < B. C is the greatest integer. Is A
represent digits. (PQ). (RQ) = XXX. Determine
the smallest integer?
the sum of P + Q + R + X.
I. X = 9. (I) E + B < A + D
II. The digits are unique. [3] (II) D < F
y
o
u
rs
m
a
h
b
o
o
b
.w
o
rd
174 Data Sufficiency

p
re
Directions (Qs. No. 5-8): are followed by two 6. ABC is a triangle with ∠B = 90°. What is the

s
s
.c
statements labelled as (1) and (2). You have to decide length of the side AC?

o
if thesestatements are sufficient to conclusively answer 1. D is the midpoint of BC and E is the midpoint

m
the question. Choose [XAT 2007] of AB.
(a) If statement (1) alone is sufficient to answer the 2. AD = 7 and CE = 5
question. 7. Five integers A, B, C, D and E are arranged in such
(b) If statement (2) alone is sufficient to answer the a way that there are two integers between B and C
question. and B is not the greatest. There exists one integer
(c) If statement (1) and Statement (2) together are between D and E and D is smaller than E. A is not
sufficient but neither of the two alone is sufficient the smallest integer. Which one is the smallest?
to answer the question. 1. E is the greatest
(d) If either statement (1) or Statement (2) alone is 2. There exists no integer between B and E.
sufficient to answer the question. 8. Le f : N → N (N is the set of all natural numbers).
(e) Both statement (1) and statement (2) are insufficient How many solutions are there to the equation f (x)
to answer the question. = 1485 ?
5. What is he maximum value of a/b? 1. For a, b ∈N f(10a + b) = f(a) + 12b
1. a, a + b and a + 2b are three sides of a triangle. 2. The maximum value of b is 9
2. a and b both are positive.

Directions (Qs. No. 1-10): Each question is followed B. Exclusion of the heaviest and the lightest
by two statements, A and B. Answer each question members from the class does not change the
using the following instructions: [CAT 1999] average weight of the students. 
2. A small storage tank is spherical tank in shape.
(a) if the question can be answered by using one of the
What is the storage volume of the tank?
statements alone, but cannot be answered using
 A. The wall thickness of the tank is 1cm.
the other statement alone.  B. When the empty spherical tank is immersed
(b) if the question can be answered by using either in a large tank filled with   water, 20litres of
statement alone. water overflow from the large tank.
(c) if the question can be answered by using both 3. Mr. X starts walking northwards along the
statements together, but cannot be answered using boundary of a field, from point A on the boundary,
either statement alone. and after walking for 150 metres reaches B, and
(d) if the question cannot be answered even by using then walks westwards, again along the boundary,
both statements together. for another 100 metres when he reaches C. What is
the maximum distance between any pair of points
1. The average weight of students in class is 50 kg..
on the boundary of the field?
What is the number of students in the class? A.   The field is rectangular in shape.  
A. The heaviest and the highest members of the B.   The field is a polygon, with C as one of its
class weigh 60 kg and 40 kg respectively. vertices and A the mid point of a side.
y
o
u
rs
m
a
h
b
o
o
b
.w
o
rd
Data Sufficiency 175

p
re
4. A line graph on a graph sheet shows the revenue 9. What is the distance x between two cities A and B

s
s
.c
for each year from 1990 through 1998 by points in integral number of Kms?

o
and joins the successive points by straight line A. x satisfies the equation log2x =  x    

m
segments. The point for revenue of 1990 is labelled
B. x < 10 Kms
A, that for 1991 as B, and that for 1992 as C. What
is the ratio of growth in revenue between 91-92 10 Mr. Mendel grew one hundred flowering plants
and 90-91? from black seeds and white seeds, each seed giving
rise to one plant. A plant gives flowers of only one
A. The angle between AB and X-axis when
colour. From a black seed comes a plant giving red
measured with a protractor is 40 degrees,
or blue flowers. From a white seed comes a plant
and the angle between CB and X-axis is 80
giving red or white flowers. How many black seeds
degrees.
were used by Mr. Mendel?  
B. The scale of Y-axis is 1cm = 1000 Rs.
A. The number of plants with white flowers was
5. There is a circle with centre C at the origin and
10.
radius r cm. Two tangents are drawn from an
external point D at a distance d cm from the centre. B. The number of plants with red flowers was
What are the angles between each tangent and the 70.
X-axis? Directions (Qs. No. 11-20): Each question is followed
A.   The coordinates of D are given by two statements, A and B. Answer each question
B.   The X-axis bisects one of the tangents. using the following instructions: [CAT 2000]
6. Find a pair of real numbers x and y that satisfy (a) if the question can be answered by using one of the
the following two equations simultaneously. It is statements alone, but cannot be answered using the
known that the values of a, b, c, d, e and f are non- other statement alone.
zero. (b) if the question can be answered by using either
ax + by = c statement alone.
dx + ey = f (c) if the question can be answered by using both
A. a = kd and b = ke, c = kf, k ≠ 0 statements together, but cannot be answered using
B. a = b = 1, d = e = 2, f ≠ 2c either statement alone.
7. Three professors A, B and C are separately (d) if the question cannot be answered even by using
given three sets of numbers to add. They were both statements together.
expected to find the answers to 1+1, 1+1+2, and 11. Consider three real numbers, X, Y and Z. Is Z the
1+1 respectively. Their respective answers were smallest of these numbers?
3, 3, and (2) How many of the professors are
A. X is greater than at least one of Y and Z.
mathematicians?
B. Y is greater than at least one of X and Z.
A. A mathematician can never add  two numbers
correctly, but can always add three numbers 12. Let X be  a real number. Is the modulus of X
correctly. necessarily less than 3?
B. When a mathematician makes a mistake in a A. X (X + 3) < 0
sum, the error is +1 or – (1) B. X (X – 3) > 0
8. How many among the four students A, B, C and D 13. How many people are watching TV programme P?
have passed the exam? A. Number of people watching TV programme
 A. The  following is a true statement : A and B Q is 1000 and number of people watching
passed the exam. both the programmes, P and Q, is 100.
B The following is a false statement. At least B. Number of people watching either P or Q or
one among C and D has passed the exam.   both is 1500.
y
o
u
rs
m
a
h
b
o
o
b
.w
o
rd
176 Data Sufficiency

p
re
14. Triangle PQR has angle PRQ equal to 90°, what is A. The average speed of the plane is 700

s
s
.c
the value of PR + RQ? kilometres per hour.

o
A. Diameter of the inscribed circle of the triangle B. The flight distance is 10,500 kilometres.

m
PQR is equal to 10 cm. 20.   What are the ages of two individuals, X and Y?
B. Diameter of the circumscribed circle of the A.      The age difference between them is 6 years.
triangle PQR is equal to 18 cm. B.      The product of their ages is divisible by 6.
15. Harshad bought shares of a company on a certain Directions (Qs. No. 21-27): Each item is followed by
day, and sold them the next day. While buying and two statements, A and B. Answer each question using
selling he had to pay to the broker one percent of the following instructions: [CAT 2001]
the transaction value of the shares as brokerage.
What was the profit earned by him per rupee spent (a) if the question can be answered by one of the
on buying the shares? statements alone and not by the other.
A. The sales price per share was 1.05 times that (b) Choose b   if the question can be answered by using
of its purchase price. either statement alone.
(c) Choose c if the question can be answered by
B. The number of shares purchased was 100.
using both the statements together, but cannot be
16. For any two real numbers:
answered by using either statement alone.
a + b = 1 if both a and b are positive or both a and (d) Choose d if the question cannot be answered even
b are negative. by using both the statements together.
=  –1 if one of the two numbers  a and  b is 21. What are the value of m and n?
positive and the other negative. A. n is an even integer, m is an odd and m is
What is (2 + 0) Å (–5 + –6)? greater than n.
A. a + b is zero if a is zero B. Product of m and n is 30.
B. a + b = b + a 22. Is Country X’s GDP higher than country Y’s GDP?
17. There are two straight lines in the x-y plane with A. GDP’s of the countries X and Y have grown
equations: over the past 5 years at compounded annual
ax + by = c rate of 5% and 6% respectively.
dx + ey = f B. 5 years ago, GDP of country X was higher
Do the two straight lines intersect? than that of country Y.
A. a, b, c, d, e and f are distinct real numbers. 23. What is the value of X?
B. c and f are non-zero  A. X and Y are unequal even integers less than
18. O is the centre of 2 concentric circle. ae  is a chord 10 and X/Y is an odd integer.
of the outer circle and it intersects the inner circle  B. X and Y are even integers each less than 10
at points b and d. C is a point on the chord in and product of X and Y is 12.
between b and d. What is the value of ac/ce? 24. On a given day a boat ferried 1500 passengers
A. bc/cd = 1 across the river in 12 hrs. How many round trips
did it make?
B. A third circle intersects the inner circle at b
and d and the point c is on the line joining the A. The boat can carry 200 passengers at any
centres of the third circle and the inner circle. time.
B. It takes 40 minutes each way and 20 minutes
19. Ghosh Babu has decided to take a non-stop flight
for waiting time at each terminal.
from Mumbai to No-man’s-land in South America.
He is scheduled to leave Mumbai at 5 am, Indian 25. What will be the time for downloading software?
Standard Time on December 10, 2000. What is the A. Transfer rate is 6 kilobytes per second.
local time at No-man’s-land when he reaches there? B. The size of the software is 4.5 megabytes.
y
o
u
rs
m
a
h
b
o
o
b
.w
o
rd
Data Sufficiency 177

p
re
26. A square is inscribed in a circle. What is the A. After reducing the listed price by 10 percent,

s
s
.c
difference between the area of the circle and that the dress sold for a net profit of 10 dollars.

o
of the square? B. The dress sold for 50 dollars.

m
A. The diameter of the circle is 25 2 cm. 32. Is 500 the average(arithmetic mean) score on the
B. The side of the square is 25 cm. GMAT?
27. Two friends, Ram and Gopal, bought apples from a A. Half of the people who take the GMAT score
wholesale dealer. How many apples did they buy? above 500 and half of the people score below
A. Ram bought one-half the number of apples 500.
that Gopal bought. B. The highest GMAT score is 800 and the
B. The wholesale dealer had a stock of 500 lowest score is 200.
apples. 33. Is |x – 2| < 1?
A. |x| > 1
Directions (Qs. No. 28-35): Each item is followed by B. |x – 1| < 2
two statements, A and B. Answer each questions using 34. People in a club either speak French or Russian
the following instructions. [CAT 2002] or both. Find the number of people in a club who
(a) if the question can be answered by one of the speak only French.
statements alone but not bythe other. A. There are 300 people in the club and the
(b) if the question can be answered by using either number of people who speak both French and
statement alone. Russian is 196.
(c) if the question can be answered by using both B. The number of people who speak only
thestatements together, but cannot be answered by Russian is 58.
us either statement alone. 35. A sum of ` 38,500 was divided among Jagdish,
(d) if the question cannot be answered by either of the Punit and Girish. Who received the minimum
statements. amount?
28. In a hockey match, the Indian team was behind by A. Jagdish received 2/9 of what Punit and Girish
2 goals with 5 minutes remaining. Did they win the together received.
match? B. Punit received 3/11 of what Jagdish and
A. Deepak Thakur, the Indian striker, scored 3 Girish together received.
goals in the last five minutes of the match.
B. Korea scored a total of 3 goals in the match. Directions (Qs. No. 36-39): In each question there are
29. Four students were added to a dance class. Would two statements A and B
the teacher be able to divide her students evenly (a) if the question can be answered by one of the
into a dance team (or teams) of 8? statements alone but not by the other.
A. If 12 students were added, the teacher could (b) if the question can be answered by using either
put everyone in teams of 8 without any statement alone.
leftovers. (c) if the question can be answered by using both the
B. The number of students in the class is statements together but cannot be answered using
currently not divisible by 8. either statement alone.
30. Is x = y ? (d) if the question cannot be answered even by using
A. (x + y)( 1 / x + 1 / y) = 4 both the statements A and B. [CAT 2003]
B. (x – 50) 2 = ( y – 50) 2 36. F and M are father and mother of S, respectively. S
31. A dress was initially listed at a price that would has four uncles and three aunts. F has two siblings.
have given the store a profit of 20 percent of the The siblings of F and M are unmarried. How many
wholesale cost. What was the wholesale cost of the brothers does M have?
dress? A. F has two brothers.
B. M has five siblings.
y
o
u
rs
m
a
h
b
o
o
b
.w
o
rd
178 Data Sufficiency

p
re
37. A game consists of tossing a coin successively. There 40. Four candidates for an award obtain distinct scores

s
s
.c
is an entry fee of `10 and an additional fee of ` 1 in a test. Each of the four casts a vote to choose the

o
for each toss of the coin. The game is considered winner of the award. The candidate who gets the

m
to have ended normally when the coin turns heads largest number of votes wins the award. In case
on two consecutive throws. In this case the player of a tie in the voting process, the candidate with
is paid `100. Alternatively, the player can choose the highest score wins the award. Who wins the
to terminate the game prematurely after any of the award?
tosses. Ram has incurred a loss of ` 50 by playing A: The candidates with top three scores each vote
this game. How many times did he toss the coin? for the top scorer amongst the other three.
A. The game ended normally. B: The candidate with the lowest score votes for
B. The total number of tails obtained in the the player with the second highest score.
game was 138. 41. Zakib spends 30% of his income on his children’s
38. Each packet of SOAP costs ` 10. Inside each packet education, 20% on recreation and 10% on
is a gift coupon labelled with one of the letters S, O, healthcare. The corresponding percentages for
A and P. If a customer  submits four such coupons Supriyo are 40%, 25%, and 13%. Who spends
that make up the word SOAP, the customer gets more on children’s education?
a free SOAP packet. Ms. X kept buying packet A: Zakib spends more on recreation than
after packet of SOAP till she could get one set of Supriyo.
coupons that formed the word SOAP. How many B: Supriyo spends more on healthcare than Zakib.
coupons with label P did she get in the above 42. Tarak is standing 2 steps to the left of a red mark
process? and 3 steps to the right of a blue mark. He tosses
A. The last label obtained by her was S and the a coin. If it comes up heads, he moves one step to
total amount spent was ` 210 the right; otherwise he moves one step to the left.
B. The total number of vowels obtained was 18. He keeps doing this until he reaches one of the two
39. If A and B run a race, then A wins by 60 seconds. marks, and then he stops. At which mark does he
If B and C run the same race, then B wins by 30 stop?
seconds. Assuming that C maintains a uniform A: He stops after 21 coins tosses.
speed what is the time taken by C to finish the B: He obtains three more tails than heads.
race? 43. In a class of 30 students, Rashmi secured the third
A. A and C run the same race and A wins by 375 rank among the girls, while her brother Kumar
metres studying in the same class secured the sixth rank in
B. The length of the race is 1 km. the whole class. Between the two, who had a better
overall rank?
Directions (Qs. No. 40-45): Each question is followed
A: Kumar was among the top 25% of the boys
by two statements, A and B. Answer each question
merit list in the class in which 60% were boys.
using the following instructions: [CAT 2004]
B: There were three boys among the top five
(1) if the question can be answered by using one of rank holders, and three girls among the top
the statements alone but not by using the other ten rank holders.
statement alone. 44. Nandini paid for an article using currency notes of
(2) if the question can be answered by using either of denominations ` 1, ` 2, ` 5 and ` 10 using at least
the statements alone. one note of each denomination. The total number
(3) if the question can be answered by using both of five and ten rupee notes used was one more than
statements together but not by either statement the total number of one and two rupee notes used.
alone. What was the price of the article?
(4) if the question cannot be answered on the basis of A: Nandini used a total of 13 currency notes.
the two statements. B: The price of the article was a multiple of ` 10.
y
o
u
rs
m
a
h
b
o
o
b
.w
o
rd
Data Sufficiency 179

p
re
45. Ravi spent less than ` 75 to buy one kilogram each 48. In a football match, at the half-time, Mahindra and

s
s
.c
of potato, onion, and gourd. Which one of the three Mahindra Club was trailing by three goals. Did it

o
vegetables bought was the costliest? win the match?

m
A: 2 kg potato and 1 kg gourd cost less than 1 kg A. In the second-half Mahindra and Mahindra
potato and 2 kg gourd. Club scored four goals.
B: 1 kg potato and 2 kg onion together cost the B. The opponent scored four goals in the match.
same as 1 kg onion and 2 kg gourd. 49. In a particular school, sixty students were athletes.
Ten among them were also among the top academic
Directions (Qs. No. 46-49): Each question is followed performers. How many top academic performers
by two statements, A and B. Answer each question were in the school?
using the following instructions: [CAT 2007] A. Sixty per cent of the top academic performers
(a) If the question can be answered by using the were not athletes.
statement A alone but not by using the statement B B. All the top academic performers were not
alone. necessarily athletes.
(b) If the question can be answered by using the Directions (Qs. No. 50-51): [CAT 2008]
statement B alone but not by using the statement A (a) if Question can be answered from A alone but not
alone. from B alone.
(b) if Question can be answered from B alone but not
(c) If the question can be answered by using either of
from A alone.
the statements alone.
(c) if Question can be answered from A alone as well
(d) If the question can be answered by using both
as from B alone.
the statements together but not by either of the
(d) if Question can be answered from A and B together
statements alone. but not from any of them alone.
(e) If the question cannot be answered on the basis of (e) if Question cannot be answered even from A and B
the two statements. together.
46. Five students Atul, Bala, Chetan, Dev and Ernesto In a single elimination tournament, any player is
were the only ones who participated in a quiz eliminated with a single loss. The tournament is played
contest. They were ranked based on their scores in in multiple rounds subject to the following rules:
the contest. Dev got a highet rank as compared to (i) If the number of players, say n, in any round is
Ernesto, while Bala got a higher rank as compared even, then the players are grouped in to n/2 pairs.
to Chetan. Chetan’s rank was lower than the The players in each pair play a match against each
other and the winner moves on to the next round.
median. Who among the five got the highest rank?
(ii) If the number of players, say n, in any round
A. Atul was the last rank holder.
is odd, then one of them is given a bye, that is,
B. Bala was not among the top two rank holders. he automatically moves on to the next round.
47. Thrity per cent of the employees of a call centre are The remaining (n − 1) players are grouped into
males. Ten per cent of the female employees have (n – 1)/2 pairs. The players in each pair play a
an engineering background. What is the percentage match against each other and the winner moves on
of male employees with engineering background? to the next round. No player gets more than one
A. Twenty five per cent of the employees have bye in the entire tournament.
engineering background. Thus, if n is even, then n/2 players move on to the next
B. Number of male employees having an round while if n is odd, then (n + 1)/2 players move on
engineering background is 20% more than to the next round. The process is continued till the final
the number of female employees having an round, which obviously is played between two players.
engineering background. The winner in the final round is the champion of the
tournament
y
o
u
rs
m
a
h
b
o
o
b
.w
o
rd
180 Data Sufficiency

p
re
49. What is the number of matches played by the 50. If the number of players, say n, in the first round

s
s
.c
champion? was between 65 and 128, then what is the exact

o
A: The entry list for the tournament consists of value of n?

m
83 players. A: Exactly one player received a bye in the
B: The champion received one bye. entire tournament.
B: One player received a bye while moving on
to the fourth round from third round

Directions for Data Sufficiency 6. Is P + Q a rational number?


(a) If statement 1 alone is sufficient to answer the (1) P and Q both irrational
question, but statement 2 alone is not sufficient (2) PQ is rational
(b) If statement 2 alone is sufficient to answer the 7. When 0.abc is written in the form of P/Q then is
question, but statement 1 alone is not sufficient P + Q divisible by 9?
(c) If both statements together are needed to answer the (1) Three digit number “abc” is divisible by 9
question, but neither statement alone is sufficient
(2) Any two digit number formed by digits a, b
(d) If either statement by itself is sufficient to answer
or c is divisible by 9.
the question
8. Is (a^b + b^a) positive number?
(e) If not enough facts are given to answer the question
(1) a + b is positive.
1. Is K an integer?
(2) ab is positive
(1) (PK) is a positive integer where P is a prime
9. Is four digit number abcd divisible by 4?
number.
(1) abcd is divisible by 1st 4 prime numbers.
(2) (K/P) is a positive integer
(2) The four digit number “abcd” is divisible by
2. If A and B (B ≠ 0) are integers, is A/B an integer?
n^3-n^2 where n is odd and more than 77.
(1) A is an even integer and B is a prime number
10. Is a + b + c a two digit number.
BA
(2) is an integer. (1) all a, b and c is more than 3 but less than 30
AB (2) if 7 < a < b < c < 11
3. If N is a perfect square less than 1000 what is the
value of N? 11. Is three digit number N divisible by 9?
(1) N is odd. (1) All the digits in N is distinct and sum and
(2) N is a perfect cube. product of digits is 6
4. Is abc + def is an even number (2) N is divisible by only 1st 4 prime numbers.
(1) abc def + def abc is an even number 12. If a number N = a.bcd when written in the form
(2) a^b^c + d^e^f is an even number P/Q then is P-Q divisible by 9
5. Is a four digit number “abcd” divisible by 9 (1) Two digit number ‘dc’ is divisible by 9.
(1) (a^b)(b^c) divisible by 9 (2) When two digit number “dc” divided by 6
resultant is not divisible by 3.
(2) (a + b)(c + d) is divisible by 9
y
o
u
rs
m
a
h
b
o
o
b
.w
o
rd
Data Sufficiency 181

p
re
13. Is number P a prime number if its square root is K. 22. Is number N2 has more than 35 factors?

s
s
.c
abcd…… (1) N has 12 factors

o
(1) P is not divisible by any prime number less (2) In 4 ways we can write N as product of two of

m
than K its co-prime factors.
(2) P is not divisible by any prime number more 23. Out of three non zero digits A, B and C which one
than K has highest number of factors?
14. Is A + B a rational number? (1) If A is prime then B and C is perfect square.
(1) A is sum of square roots of two prime (2) If B is odd then A and C are even.
numbers 24. How many numbers are less than N and co prime
(2) B is difference of square roots of two prime to N?
numbers (1) N is a two digit prime number with sum of
15. Is product of three numbers a, b and c a positive digits 16
integer? (2) N is a two digit prime number with product
(1) At least 2 of three is a positive integer. of digits 42
(2) At least 1 of the three are negative integer. 25. Is N/6 has odd number of factors?
16. Is product of three numbers a, b and c a positive (1) N/24 has odd number of factors
even number? (2) 24 N has odd number of factors
(1) At least 2 of the three is an even positive 26. If N has 81 factors then how many factors does N3
integer. has.
(2) At most 1 of the three is an even negative (1) In 4 ways we can write N14 as product of 2 of
integer. its co-prime factors.
17. What is minimum value of A/B + B/A? (2) N2 has maximum possible number of factors
(1) The product AB > 0 27. Is number of factors of N divisible by 2 more than
(2) The sum A +B > 0 that divisible by 3
18. Is ab + bc + ca positive? (1) N is not divisible by 24 but is divisible by 28.
(1) At least two of the three given numbers is (2) Nis not divisible by 54 but is divisible by 63
positive 28. If N has 81 factors then find the number of factors
(2) At least one of them is negative of 2N
19. Is product of 5 consecutive two digit natural (1) N is not divisible by 24 but is divisible by 28.
numbers divisible by 25? (2) N is not divisible by 54 but is divisible by 63.
(1) Sum of those 5 natural numbers is more than 29. If N has 16 factors then find the value of N
239 (1) N is divisible by 64.
(2) Sum of those 5 natural numbers is less than (2) N is divisible by 72.
260 30. What is the ratio of number of factors of N2 to that
20. A “centum set a” is define as set of numbers whose of N
100th digit is a. Find the number of such sets. (1) N3 has 112 factors
(1) If all the members of the set is 3 digit numbers (2) Number of factors of N2 is more than 64 but
and any member of the set has only prime less than 81.
digits. 31. Find the number of factors of N
(2) If all the members of the set is a 5 digit (1) N/2 has 12 factors
number. (2) N is a perfect cube
21. Is number N has odd number of factors? 32. N has 18 factors is it divisible by 18?
(1) N is divisible by only one prime number. (1) N is divisible by 32 but not by 512
(2) 61 < N < 67 (2) N is divisible by 5
y
o
u
rs
m
a
h
b
o
o
b
.w
o
rd
182 Data Sufficiency

p
re
33. Find the number of factors of N 45. What is remainder when N is divided by 9?

s
s
(1) P = NQ

.c
(1) N give remainder 1 when it is divided by 3 or 4.

o
(2) Number of factors of P = 6Q2+5Q +1 (2) N when successively divided by 3, 4, 5 and 6

m
34. Find the number of odd factors of N it leads remainder 1 in every step.
(1) Number of even factors of N is 17 46. Is ratio of LCM to HCF of two numbers more than 8?
(2) 2N has 17 odd factors (1) One of the numbers is 5 and their LCM is 40.
35. How many factors of number N has unit digit 5 if (2) HCF of two numbers is 5 and one of the
N is divisible by 5. number is 15
(1) N has 21 odd factors 47. Is (ABC) and (DEF) co-prime to each other?
(1) HCF of (AB), (CD) and (EF) is 1.
(2) None of the factors divisible by 10.
(2) HCF of (A and D), (B and E) and (C and F) in
36. How many ways N can be written as product of 2
pairs is 1.
of its co-prime factors?
48. Find the value of f (20) if f (n+2) is LCM of f (n)
(1) Only single digit prime numbers divide N.
and f (n+1) here n is a natural number.
(2) N is divisible by 9 but not by 10. (1) f (7) = 36
37. Is number of even factors of N more than number (2) f (1) = 1
of odd factors? 49. Is HCF of a and b more than 1, here a and b are
(1) Given that number is divisible by 10 distinct positive integers?
(2) Number of factors of N is odd (1) a is divisible by 9 but not by 18
38. Is integer k > 5 ? (2) b is divisible by a2 but not by a2
(1) K is a factor of 30. 50. Is A & B co-prime to each other ?
(2) K is not a factor of 120 (1) LCM of A & B is 36 and one of the number is
39. How many factors of number N is a two digit prime.
number? (2) Number of factors of their HCF is 1.
(1) N has total number of factors as 16. 51. What is the number of students in class 8? If number
(2) N has 4 single digit prime factors. of students in a class cannot be more than 100.
40. Is number of factors of N more than 27? (1) When students are divided in a group of 4
(1) N is a perfect square with at least 3 prime only 2 students are left but when divided in a
factors. group of 11, only 9 students are left.
(2) If N is a perfect cube then it is a perfect (2) When students are divided in a group of 5
square also. only 2 students are left but when divided in a
41. Is A > B if their LCM is 30. group of 7, no student is left.
52. How many days in the month March both Maths
(1) HCF of A and B is 1 and when (a + b) divided
and English faculty will come together?
by 9 remainder is 4.
(1) Maths faculty comes at an interval of 3 days
(2) AB is odd.
and English faculty at an interval of 4 days.
42. What is HCF of (A + B) and (A – B) given that A is
Mathsfaculty came on 3rd, 6th, 9th January
not equal to B? and so on.
(1) HCF of A and B is 4 English faculty came on 4th, 8th, 12th, January
(2) LCM of A and B is 36 and so on.
43. Is HCF of A and B more than 4? (2) Maths faculty comes at an interval of 3 days
(1) LCM of A and B is 48. and English faculty at an interval of 7 days.
(2) The ratio of A:B is 1;2 Maths faculty came on 3rd, 6th, 9th January
44. Are A and B co-prime to each other? and so on.
(1) AB and B are co-prime to each other. English faculty came on 7th, 14th, 21st,
(2) HCF of AB and BA is 1 January and so on.
y
o
u
rs
m
a
h
b
o
o
b
.w
o
rd
Data Sufficiency 183

p
re
53. If H1 is the HCF of (11111….P times) and 61. Find the unit digit of x y

s
s
.c
(333333…… Q times) and H2 is the HCF of (1) x and y are prime digits

o
(2222…. R times) and (5555…. S times) is H1 > H2? (2) unit digit of yx is 0

m
(1) HCF of P & Q is 5, P & R is 7 and P & S is 14. 62. What is the unit digit of 7abcd
(2) HCF of P & Q is 5 and R & S is 7. (1) Four digit number cbad is divisible by 4.
Consider Q > P then H1 = 11111….P times, but if
(2) When 4 digit number ‘abcd’ divided by 72 it
P > Q then H1 = can be 1111… Q times or
leaves remainder 1.
54. Is R > 12
3x
(1) When 252, 612 and 1212 divided by N 63. What is the unit digit of 3 .
remainder is R in each case. (1) x is a prime number.
(2) When 255, 612 and 1215 divided by N, (2) x is a three digit prime number
remainder is R in each case.
x x −1
55. Is HCF of A, B, C and D more than 5. 64. What is the unit digit of 4
(1) HCF of A & B is 20 and HCF of C and D is 35 (1) (x – 1) has only 2 factors.
(2) HCF of A & B is 10, A & C is 20 and A & D (2) (x + 1) has only 2 factors.
is 30 d
bc
56. Is HCF of A, B, C and D more than 5 if they are 65. What is the unit digit of 2a
distinct non-prime number? (1) All the numbers a, b, c and d are even
(1) LCM of A & B is 50 and that of B & C is 25 numbers.
(2) LCM of A& C is 50 and that of B & D is 75. (2) All the numbers a, b, c and d are odd numbers.
57. What is remainder when (x + y + z) divided by 3 if
cd
x, y and z are distinct positive integers? ab
66. What is the unit digit of 3
(1) A number gives remainder x, y and z when
divided by 4, 8 and 16 respectively. (1) All the numbers a, b, c and d are prime
(2) HCF of xy, yz and zx is 3. numbers.
58. Is HCF of A. B and C one? (2) All the numbers a, b, c and d are three digit
(1) HCF of AB, BC, and CA (i.e product of two numbers.
numbers taken at a time) is 1 67. If unit digit of ab is 1 then what is the value of a?
(2) HCF of (AB)C, (BC)A, and (AC)B is 1. (1) b is even number
59. If F(3k) = Product of F(3k – 2) and F(3k – 1); (2) a is divisible by 9
F(3k + 1) = LCM of F(3k – 1) and F(3k); 68. Find the unit digit of a 11111
F(3k + 2) = HCF of F(3k) and F(3k + 1) here k is a (1) Unit digit of a66 is 1
natural number. (2) Unit digit of a999 is 9
Find the HCF of F(3n), F(3n + 3), F(3n + 6) …. 69. Find the unit digit of ab
F(3n + 3p) (1) Unit digit of a333 is 9
(1) F(3n+9) = 1248 (2) Unit digit of a555 is 6
(2) HCF of F(3n – 6) and F(3p – 6) is 1224 if p > n 70. Find the unit digit of 9x + 9y.
60. If A = a4444 – 1 and B = a4444 + 1 find unit digit of (1) x + y is odd
LCM of A and B. (2) (x.y) is odd
(1) When a2 divided by 8 remainder is 1. 71. What is the unit digit of (xy)xy ? Here “xy” and
(2) a can be expressed as difference between “yx” is a two-digit number.
squares of 2 consecutive natural numbers, (1) Two digit number xy is divisible by 4.
and a99 is not divisible by 625 (2) (xy)xy is not divisible by 100.
y
o
u
rs
m
a
h
b
o
o
b
.w
o
rd
184 Data Sufficiency

p
re
72. If unit digit of 3x + 3y is 0 then find the unit digit of

s
 5x 

s
3xy 82. Find the remainder of  

.c
9

o
(1) 7x has unit digit either 1 or 7 (1) x is a 2 digit prime no.

m
(2) 2y has unit digit either 4 or 8 (2) x when divided by 12 gives remainder 9.
73. Find the unit digit of 3x – 3y  a 7777 
(1) Both x is in the form of 4p +1 while y is in the 83. What is the remainder of  
 7 
form of4p + 3 here p & q are positive integers. (1) given that a is in the form of 5x + 1
(2) x when divided by 4 gives remainder 1 and (2) a is in the form of 4x + 1
when y divided by 4 gives remainder 3  ab 
74. Find the unit digit of (x7)(7x) 7
84. What is the remainder of   here a, b are
(1) Unit digit of x7 + 7x is 6 distinct integers.  9 
(2) Unit digit of x7 + 7x is 8 (1) a, b both are prime numbers.
75. Find the unit digit of x y + yx (2) a, b both are even integers.
(1) Unit digit of x y is 3  xy 
85. What is the remainder of  
(2) Unit digit of y x is 1.  z 
(1) |x – z| = 1
76. Find the Last two digit of 2x
 x3 
(1) x gives remainder 4 and 5 when divided by 5 (2) Remainder of   is 1
or 6.  z 
(2) x gives remainder 3 and 4 when divided by 4  x
86. Is remainder of   is 1
or 5. z
77. Find the last two digit of (8181)xyz here xyz is a  x ^ 2
(1) Remainder of  z  is 1
three digit number.  
(1) Three digit number ‘xyz’ is divisible by 5  x3 
(2) Remainder of  z  is 1
and 7  
(2) Three digit number ‘xyz’ is divisible by 5  3x 
and 23 87. If R1 is remainder of   and R2 is the remainder
7
78. Find the last two digits of a^b^c^d^e.  3y 
of   is R1>R2 here x and y both are positive
(1) Last two digit of a^b is 76 7
(2) Last two digits of a ^b is 12 integers.
79. What is the last two digit of (ab)ba here ‘ab’ and (1) x = 6y
‘ba’ are two digit numbers and difference between (2) x = 4y
 2 x3 y 
ab and ba is not divisible by 27. 88. What is the remainder of  
 7 
(1) ab has unit digit 1
(1) x and y both divisible by 3.
(2) ba has unit digit 4
(2) both x and y are odd.
80. Find the last two digits of (aa)bb  yz  
(1) Last two digits of ab is 29   x  
89. What is the remainder of    ?
(2) Last two digits of ab is 56
 xy 
 3x  x
81. What is the remainder of  (1) Remainder of   is 1.
  y
7
(1) x is in the form of( 4k + 2)  y
(2) Remainder of   is 5
(2) x is in the form of (3k + 1) x
y
o
u
rs
m
a
h
b
o
o
b
.w
o
rd
Data Sufficiency 185

p
re
s
xyz  bc 
90. What is the remainder of   here ‘xyz’ is a

s
a

.c
 x + y + z 
 97. What is the remainder of  c 
three digit number.

o
 b 

m
(1) x + y + z = 25 (1) a, b, c are all even numbers.
(2) Unit digit of x y is same as that of y z (2) b is a prime number while a and c are
 xy  composite numbers.
91. If remainder of 
 7 
 is 1 then what is the
98. What is the remainder of 
{
 abc abc 
}
 here ‘abc’
 x  xa + yb + c 
remainder of   ?
7 is a three digit number and x, y are +ve integers.
(1) Given that y is a two digit prime number. (1) x is 10 times of y
(2) Given that y is four digit composite number (2) y is smallest two digit number that has 4
not divisible by any digit except 1. factors.

92. What is the remainder of 


 x97531 
 24 
 
99. What is the remainder of 
{ }
 ( n!) 2 

 2 n + 1 
 x (1) 2n + 1 is a prime number and n is odd
(1) Remainder of   is 4
6 (2) 2n + 1 is a prime number and n is Even
 x 100. Find the unit digit of a! + b! + c!
(2) Remainder of   is 3
4 (1) Each of a, b and c is a two digit number.
 xy  (2) Each of a, b and c is a prime number in the
93. What is the remainder of   form of 6k ± 1.
 36 
x  x  a !
(1) Remainder of   is 1 and that of   is 1 101. Find the remainder of  
18  8  b! 
(1) Given that a ≥ b
x  x
(2) Remainder of   is 17 and that of   is (2) Given that a ≤ b
18  8
7 102. Find the highest power of 7 in N!.
 x ! N
94. Find the remainder of   (i) Quotient of   is 7
y 9
(1) Given that |x – y| =2 N
(2) y is a three digit prime number and more than (ii) Quotient of   is 5
11
x. 103. Find the last two digits of a! + (a + 1)! + (a + 2)!


95. Find the remainder of 
{ }
 ( x!) 2 

(1) a is a prime number.
(2) a is a two digit number.
 2x + 1 
104. Is a! divisible by 72?
(1) Given that x is Even.
(1) a! is divisible by 18
(2) Given that x is Odd
(2) a is divisible by 4 but not by 8, a is divisible
 ( aa )bb 
96. What is the remainder of   here a, and b by 3 but not by 9.
 100  105. Is a! divisible by 68?
are digits
 ab  (1) a! is divisible by 17.
(1) Remainder of   is 29 (2) a! is divisible 26.
100 
106. What is highest power of 3 in a! ?
 ab  (1) Highest power of 9 in a! is 5
(2) Remainder of   is 56
100  (2) Highest power of 9 in a! is 6
y
o
u
rs
m
a
h
b
o
o
b
.w
o
rd
186 Data Sufficiency

p
re
107. What is highest power of 9 in a! ? 118. What is number of zeros when N! is converted in

s
s
.c
(1) Highest power of 27 in a! is 12 hexadecimal system (Base 16)

o
(2) Highest power of 27 in a! is 9 (i) Number of zeros at the end is 8 when N! is

m
108. Find the unit digit of (N!) converted to base 9.
(ii) Number of zeros at the end is 8 when N! is
(1) Unit digit of (N-5)! Is not 0.
converted to base 12.
(2) N is divisible by 3 but not by 6. 119. What is last two non zero digits of nCr
109. Find the number of zeros at the end of N! (1) n = r + 2
(1) Highest power of 2 in N! is 97 (2) Highest power of 2 and 3 in n! is 11 and 6
(2) Highest power of 4 in N! is23 respectively.
110. Find the number of zeros at the end of N! 120. Is A > 1 ?
(1) Highest power of 10 in (N+1)! Is 12 (1) Log A X > Log A X2 ( X is an integer)
(2) Highest power of 10 in (N-1)! Is 10 (2) A2 < A
111. What is HCF of a! and b! ? 121. What is the value of (a + b)3 ?
(1) a is root of quadratic equation x2 – 5x – 6 = 0 (1) a2 + b 2 = 13
(2) b is a two digit even number while ‘a’ is a (2) a*b = 6
single digit even number. 122. What is the value of X?
(1) if log 2, log (2x -1 ) and log ( 2x + 3 ) are in
112. Find the number of zeros at the end of (a!)(b!)
AP
(1) Highest power of 2 in a! is 41 and highest
(2) if X is real number
power of 5 in b! is 12
123. Is X > Y ?
(2) Highest power of 2 in a! is 46 and highest (1) X2 + X = 2 and Y2 + 2Y = 15
power of 5 in b! is 24
(2) X2 – 2 X = 8 and Y2 – 3Y = 18
113. Find the number of zeros at the end of (a! + b!) 124. If X and Y are positive integers then is ( X2 + Y2 )
(1) Highest power of 2 in a! is 41 and highest 1 1
power of 5 in b! is 6 greater than  +  ?
 x y
(2) Highest power of 2 in a! is 46 and highest (1) X > Y
power of 5 in b! is 24 (2) XYZ = 1
114. Find the unit digit of (a!)(b!) 125. X2 – 32X + 216 = 0 what is the value of X?
(1) a = b + 7 (1) X is perfect cube
(2) b = a + 4 (2) X2 is perfect cube
115. What is the last two digits of [a!/b!] if a > b ? 126. What is the value of x?
(1) log ( X2 + 10X + 25 ) = 2
(1) Given that a = b + 7 and b is not a multiple of 5
(2) log ( X3 – X2) = 2
(2) Given that a = b + 4 and b is a multiple of 5
127. Is X > Y?
116. What is the largest prime number that has exponent (1) X /Y = 8/7
2 in N!. (2) X2 > Y2
(i) 17 is the largest prime number that has 128. What is the value of C in this equation X2 + X +
exponent 2 in (N/2)!. C=0
(ii) 19 is the largest prime number that has (1) the equation has two roots
exponent 2 in (N/3)!. (2) the sum of the roots is -1
117. What is the highest power of 3293 in N! 129. Is X3 + Y3 + Z3 = 3XYZ ?
(ii) Highest power of 17 in (N/2)! is 3. (1) X +Y+Z = 0
(i) Highest power of 37 in (N/2)! is 2. (2) X2 + Y2 + Z2 = XY +YZ + ZX
y
o
u
rs
m
a
h
b
o
o
b
.w
o
rd
Data Sufficiency 187

p
re
s
s
.c
o
m
Concept Applicator Now if we consider Statement I alone
Product of these 7 integers = 702800
1. (d) If we look at Statement I then we will get
Since 702800 = 24  52  (251)(7), it cannot be
If a = 3 and b = 2, a + b > 0. Here b > 0
the product of 7 consecutive whole numbers.
If a = 3 and b = –2, a + b > 0. Here b < 0 Hence I alone is insufficient.
Hence I alone is not sufficient. Now if we consider Statement II alone
Now if we look at Statement II only then we Given that their sum = 105 = 7n or n = 15 and
will get then 7 consecutive integers are 12, 13, 14, 15,
If a = 3 and b = 2, a – b > 0. Here b > 0 16, 17, 18
If a = 3 and b = –2, a – b > 0. Here b < 0 So, II alone is sufficient.
Hence II alone is not sufficient. 5. (a) Since sum is 360 hence P + Q + R + S = 360
Now by using statements Ι and ΙΙ together From statement I alone we will get P =
If a = 3 and b = 2, a – b > 0 and a + b > 0. Here (Q + R + S)/3 from this we can find the value of
b>0 P hence statement I alone is sufficient enough.
If a = 3 and b = –2, a – b > 0. and a + b > 0. From statement II alone we can not find the
Here b < 0. value of P.
Hence Ι and ΙΙ together are also insufficient.  6. (d) Statement  Ι  is not sufficient as the size of
2. (c) If we look at Statement I the ice cube and height of the container is
i = p – 17 and r = p – 103 not known hence statement I is not sufficient
alone.
Hence, we cannot find how many each
Statement  ΙΙ  is also not sufficient as the
received so this statement is not sufficient
dimension of the container is not known.
enough.
We cannot answer the question even by
Now by considering Statement II alone.
combining both the statements as the height
p + i + r = 170 of the container is not known.
Hence, we cannot find how many each 7. (b) It is given that Ram got a dividend of ` 1500.
received. so this statement is not sufficient Statement I
enough Knowing the dividend paid last year, we
Using I and II together, we get p + (p – 17) + cannot find the dividend paid this year.
(p – 103) = 170. Statement ΙΙ
Solving the above equation we get the value Given that Ram bought 350 shares of face
of p and the values of q and r. value  ` 10, and so, their total face value is
3. (b) If we look at Statement I ` 3500. So here we know the investment and
It is given that the circles are concentric. the return hence we can find out the rate of
But nothing is given about their dimensions. interest.
Hence Ι alone is not sufficient. 8. (d) Consider Statement I alone.
In statement II ratio of area is given hence we Given that Area (∆ ABC) = Area(∆PQR)
can find the required ratio. since nothing about the sides or angles
4. (b) Let the 7 consecutive whole numbers be is mentioned, we cannot say if they are
(n ± 3), (n ± 2) (n ± 1), n. congruentHence, Ι alone is not sufficient.
y
o
u
rs
m
a
h
b
o
o
b
.w
o
rd
188 Data Sufficiency

p
re
Consider Statement ΙΙ alone Consider statement I alone:-

s
s
.c
∆ABC and ∆PQR are right triangles.Nothing we know that Ram is not the shortest, either

o
about the sides is given, hence, ΙΙ alone is not Shyam or Jay is the shortest.

m
sufficient. Hence (Ι) alone is not sufficient.
Now using both Ι and ΙI Consider statement I aloneShyam > Vikram.
Now we have two right angled triangle with From the given information and the information
same area we may have different combination in (ΙΙ), we get Ram > Shyam > Vikram > Jay.
as only product of base and height is same. Hence, (ΙΙ) alone is sufficient.  
Hence even by using both the statement we
can not find the answer. Concept Builder
9. (a) Given that their salaries are in the ratio of
1. (a) Statement Ι alone is sufficient.
3 : 4 and expenditure is in the ratio of 4 : 5
Statement ΙΙ alone is not sufficient, for we can
hence we can assume that salary of A and B
have more than one value of MN possible.
are 3x and 4x and their expenditures are 4y
and 5y. 2. (e) Given relationship is (PQ)(RQ) = XXX
Now we need to find the ratio of (3x – 4y)/ Since X can take 9 values from 1 to 9 hence
(4x – 5y) we have 9 possibilities
Consider statement I alone:- 111 = 3 × 37  444 = 12 × 37  777 = 21 × 37
Saving of B is 25% of his salary hence his 222 = 6 × 37  555 = 15 × 37  888 = 24 × 37
expenditure must be 75% so ¾(4x) = 5y or 333 = 9 × 37  666 = 18 × 37  999 = 27 × 37
3x = 5y from this we can find the required But out of these 9 cases only in 999, we get
ratio hence this statement is sufficient. the unit’s digit of the two numbers the same.
Consider statement II alone:- Since it is a unique value, hence we need
Given that 4x = 2000 or x =500 but from this neither statement Ι nor statement ΙΙ to answer
we can not find the value of y and hence we the question.
can not find the ratio of their savings. 3. (d) C
10. (c) Let x be the average height of the class and n
be the number of students in the class.
Consider statement I alone:- a cms
b cms
xn –  56 = (x – 1)(n – 1)
⇒ x + n = 57 …(1)
Hence, the value of x cannot be found. 60°
So, Ι alone is not sufficient. A B
Consider statement II alone:- 60 cms
xn –  42 = (x + 1)(n – 1) Let, a cm and b cm are the the two unknown
⇒ x – n = 41 …(2) sides as shown in the fig.
Hence, the value of x cannot be found. From statement 1, a + b = 80 cm, hence
So, ΙΙ alone is not sufficient. b = (80 – a) cm
Both the statements together are sufficient as Now using cosine rule.
the value of x can be found by solving (1) and (2) Cos 60° = ( AB2 + AC2 – CB2 )/2 AB
11. (b) Given that Ram > Shyam, Vikram > Jay. 1
∴ = [602 + b2 – (80 – b)2]/120
Hence from this we can conclude that neither 2
Ram nor Vikram is the shortest. And we have By solving this we get, b = 28 cm. Hence,
to find the shortest among them - statement 1 is sufficient to answer.
y
o
u
rs
m
a
h
b
o
o
b
.w
o
rd
Data Sufficiency 189

p
re
From statement 2, If we combine the two statements; only case

s
s
.c
Since ∠B = 45° hence ∠C = 75° (iii) is possible.

o
According to sine rule: we know that a/sin a Hence (c) is correct option.

m
= b/sin b = c/sin c = a/sin45° = b/sin60° 8. (e) From statement 1: F(x = 10a + b) = f (a) + 12b
= 60/ sin75° = 1485, from this equation we will have
From this we can find the value of the sides. different values of a, and b and hence many
Hence statement 2 is sufficient to answer the values of x. From statement 2: Maximum
question. value of b = 9, since we don’t know the exact
4. (a) From statement I, value of b so we cannot determine. From both
E + B < A + D, we easily say that E is less than equation we cannot solve due to unknown
A, because B > D and as the statement suggest value of f (a).
E + B < A + D.  ∴  E < A.
∴  A is not the smallest integer. Concept Cracker
Statement I is sufficient to answer. 1. (d) Statement A: Alone – it gives highest and
From statement II, D < F lowest weight of members of the class. This
This statement is not sufficient to find the does not give any other information so we
relation between A and E. cannot determine the number of students.
5. (e) Since ‘b’ is the common difference of three Hence this statement is not sufficient enough.
sides of a triangle, a/b can take any values in Statement B: Alone – It gives that by
the given range of real; numbers.
eliminating the highest and lowest members,
Suppose a = 1000
b=1 the average weight does not change. This is
Then there sides will be 1000, 1001, 1002 also not sufficient to answer anything.
Here a/b = 1000/1 = 1000 Both the statements together cannot determine
Hence the values of ‘a’ and ‘b’ can be the number of students as the average weight
varied to any values making the ratio a/b of students (after eliminating highest and
undeterminable. lowest) cannot determine the total number.
6. (c) From statement 1: Let AB=2x and BC=2y, 2. (c) To find the volume, we need radius of sphere.
then AC2 = 4(x2 + y2), since we don’t know the Statement A alone - does not give any useful
values of x and y hence we cannot calculate information.
the AC, hence this statement is not sufficient Statement B alone – it says 20 litres of water
enough. are displaced when the tank is fully immersed.
Statement 2: AD=7 and BD=5; We don’t get Hence we will get the outer volume of the
answer from this spherical tank. Now with this the outer radius
Now combining both the information we can
of the tank can be found. But this is not
write (2x + y)2 = 49 and (2y + x)2 =25, hence
we can calculate the value of x and y and sufficient to get an answer.
hence we can calculate the value of AC On combining we can find the inner radius
7. (b) We have three possible cases because the wall is 1cm thick. And hence the
Case (i) DBEAC storage volume can be determined.
Case (ii) BADCE 3. (c) Statement A alone – It is given here that the
Case (iii) CADBE shape of the field is rectangular but exact
From statement (1), either case (ii) or case location of point A, B and C is not given hence
(iii) could be valid hence we cannot get a we cannot find the from this information.
unique value from this statement alone. Statement B alone – It is given that the shape
From statement (2), either case (i) or case (ii) is polygon but we don’t know whether it is
is possible hence we cannot get a unique value polygon of n = 4, 5, 6 or anything else, hence
from this statement alone. this is not sufficient to get an answer.
y
o
u
rs
m
a
h
b
o
o
b
.w
o
rd
190 Data Sufficiency

p
re
By combining statement A and B we know 8. (c) Statement A states that A and B passed.

s
s
.c
that shape is rectangle and C and B are its Statement B actually means neither C nor D

o
vertices and A is the mid point of other side, has passed.

m
hence together both the statement is sufficient Hence from both the statements we can
to get answer. determine that the total number of students
4. (a) Here we need to find the ratio of growth and who passed was 2 and these are A and B.
that will be equal to the ratio of the slope of 9. (c) Statement A alone – it gives the following
the line. Statement A- From this statement equation Log2 X = √X X = 2√X. This
we know the slope and hence we can find out is not sufficient to answer the question.
the ratio of slopes. So this is sufficient to get Because there are many values of X
answer. Statement B- From the scale only we may satisfy the equation, e.g., 4, 16 etc.
can not calculate the ratio.
Statement B alone – it says X ≤ 10. This is
5. (a) Statement A- alone- With the coordinates of
not sufficient by itself to answer the question.
point D given it helps to find out the equation
But on combining both the statement A and B
of the straight line passing from the two points
we get only the integer 4 that shall satisfy the
where the tangents meet the circle, i.e., (r, 0)
equation.
and (0, r). And if we know the equation of a
straight line then we can calculate its slope and 10. (d) Since Black Seed gives Blue flower or Red
angle between tangent and x-axis and hence flower. White seed gives White flower or red
this statement is sufficient enough. Statement flower. Statement A alone – it says plants with
B does not give any answer. So answer is (1) white flowers are 10. Hence this statement is
6. (d) Statement A gives (kd)x + (ke)y = k f, k ≠ 0 not sufficient enough to answer the question.
⇒ dx + ey = f ⇒ These represent one and  the Statement B alone – it says number of plants
same equations, hence infinite pairs can satisfy. with red flowers is 70. Again nothing can be
hence this is not sufficient to get an answer. concluded. On combining we get 10 plants
Statement B gives x + y = c, 2x + 2y = f with white flowers and 70 with Red flowers.
⇒ f = 2c. but it is given that f ≠ 2c ⇒ So plants with Blue flowersshould be 20 (As
Inconsistent. Combining the two statement A total number of plants is 100). Now blue
and B also we cannot determine the answer. flowers shall come from Black seeds only. So
7. (b) Statement A alone- it says a mathematician there are at least 20 Black seeds used. 10 white
can never add two numbers correctly, but can flowers can come only from white seeds. So at
always add three numbers correctly. Let there least 10 white seeds have been used. Now the
are three professor A, B and C, remaining 70 red could have come from either
A gives an answer 1 + 1 = 3, hence he is a black or white seeds. So data of this statement
mathematician. is insufficient.
B gives an answer 1 + 1 + 2 = 3, hence he is Even by using both the statement we cannot
not a mathematician. determine the answer.
C gives an answer 1 + 1 = 3, hence he is a 11. (c) A alone : X may be greater than Z. But doesn’t
mathematician. say anything about Z being smallest.
So this statement is sufficient enough B alone : Y may be greater than Z. But says
Statement B- says when a mathematician nothing about Z being smallest
makes a mistake in a sum, the error is +1 or
Now we will use both statement A and B
–1. Here two professors madea mistake of +1
If X > Y, then  Y > Z and   X > Y > Z
and –1, the third got it correctly. Hence this
statement is sufficient enough Hence each But if X > Z, then Y > X and  Y > X > Z
statement individually is able to answer the In either of these two cases, Z is the least and
question. hence the question can be answered.    
y
o
u
rs
m
a
h
b
o
o
b
.w
o
rd
Data Sufficiency 191

p
re
12. (a) From A alone: x(x + 3) < 0 hence  0 > x >  – 3 Statement A: Gives us the radius of

s
s
.c
Hence, |x| is necessarily less than 3 in this inscribed circle = 5,but with this information

o
range. we cannot calculate PR & QR.

m
From B alone : x (x – 3) hence x > 3 or x < 0 Statement B : Gives hypotenuse PQ = 18 cm
In this condition, any given |x| value need (dia of circumcircle).
not necessarily be less than 3. So B is not
Now, ∠ P + ∠ Q = 180° – 90° = 90°
sufficient enough.
13. (c) Consider A alone : from the formulan ( P U Q) So ∠P/2 + ∠Q/2 = 45° but this is not sufficient
= n(P) + n(Q) – n(P ∩ Q) enough.
We get n (P  U  Q) = n (P) + 1000 – 100. Now, If IS = 5cm (in radius = 5), then
From A alone, we get 2 unknowns and 1 Using both the information we can calculate
equation.  Hence  A alone is not insufficient 15. (a) Consider A alone : Given S.P / C.P = 1.05 and
enough. brokerage of 1%
Consider B alone: n(P U Q) = 1500 given, but
from Balone, n(P) cannot be determined. Let CP =100x brokerage = x so total CP = 101x,
Consider both together :1500 = n(P) + 1000 – and SP = 105x
100 AND n(P) = 1500 – 900 = 600 A is sufficient.
Hence both the statement required to solve Consider B alone : No. of shares = 100, since
the question. no price is given, profit cannot be calculated
Alternately- From statement A alone hence this statement is not sufficient enough.
P Q = 1000 16. (c) From the main information a@b is not define
when a or b is zero.
a c Consider A alone : from this information we
b
see that
a @ b   = 0 only if a is zero.
However in the expression (2 + 0) : a ≠ 0. A is
It is given that b + c = 1000 and b = 100 hence not sufficient.
c = 900 but we cannot calculate a, so this B alone is not sufficient enough as it does not
statement is not sufficient enough. provide important information.
From statement B alone a + b + c = 1500, Consider A and B together a @ b = b @ a
and this statement is also not sufficient enough hence 2 @ 0 = 0 @ 2
to find the value of a + b
If we use both the statement then we can So (2 @ 0 ) = 0 and (– 5 @  – 6) =1
calculate the value of a + b Hence A and B together is sufficient enough
14. (c) P 17. (d) Two straight line intersect if they are not
parallel hence if a/d ≠ b/e
R Now consider statement A alone, it does not
S say specifically that a/d ≠ b/e
From statement B alone, if c and f both are
zero then also we cant say that a/d ≠ b/e

I Even with both the statement together we
 cant determine whether the two straight line
R Q
intersect or not.
y
o
u
rs
m
a
h
b
o
o
b
.w
o
rd
192 Data Sufficiency

p
re
18. (b) From statement A 21. (c) Statement A- it is not sufficient to give the

s
s
.c
answer as we will get the pairs (15,2), (10,3),

o
(6,5). There are infinitely many values.

m
But using the statement B we can say the that
value of (m, n ) may be, (1,30), (2, 15), (3, 10),
O
B D (5, 6) hence this is not sufficient enough.
A E
C By combining A and B we will get unique
answer m = 15, n = 2.
22. (d) Statement A- as we don’t know that what
We see that BC = CD or C is at the center of were their GDPs 5 years ago. Hence we cant
B and D    compare its present value.
So C will also be the centre of A and E, Hence Statement B- we know that GDP of country X
(A) is sufficient to answer. is greater than that of Y but we do not know
From Statement (B) by how much. Hence even using both the
statement we cant determine.
23. (a) Considering statement A alone the possible
pairs are (5,1), (7,1), (9,1), (2, 6), (9, 3). So there
B
OC
D is only one pair with both even numbers (2, 6).
A E Now considering statement B the possible
T pairs are (2, 6) and (6, 2). So there is no unique
solution by using (B) alone. We can get direct
answer from statement A alone.
Let T be the center of the third circle. Then 24. (a) From statement A- We cannot make out the
since C is the line joining O and T implies C is number of rounds, as maximum capacity is
midpoint of B and D or bisects B and D. given 200, but actual value is not given.
Similar to last case now we can say that this
Statement B- However, we can find out the
statement is also sufficient enough.
trip time using the given values and can thus
19. (d) Consider statement A alone : Avg. speed calculate the total number of trips since total
of plane says nothing about local time in no- time is fixed 12 hrs.
mans land. Statement B alone : Distance also
25. (c) Statement A: alone it is not sufficient enough
says nothing about the local time.
because it gives only the data transfer rate but
Now if we use both the statement together does not give the size of the software to be
: Time required for flight can be obtained but downloaded.
still we cannot find the local time.
Statement B: alone it is also not sufficient
20. (d) From statement A alone it is given that x – y
because it gives only the size of software but
= 6 or y – x = 6, from this we cannot find out a
does not give the rate of data transfer.
unique value of x or y, hence this statement is
Combining both the statements, we can get
not sufficient enough.
the answer.
From statement B xy = 6k, again we cannot
find out a unique value of x or y, hence this 26. (b) We know that if diameter is given, then side
statement is also not sufficient enough. of square can be found. Similarly if side of the
square is given then the radius of circle can be
Now if we use both the statement together
found.
then, let x – y = 6 and xy = 6k, or x = 6k/y or
6k/y – y 6 or y2 + 6y = 6k, and again we will Hence, both the statements individually can
not get a unique value of x and/or y. provide the difference in areas.
y
o
u
rs
m
a
h
b
o
o
b
.w
o
rd
Data Sufficiency 193

p
re
27. (d) Ratio of number of apples bought by Ram A tells us F has 2 brother then they must be

s
s
.c
and Gopal is given and total quantity at dealer unmarried. Rest two uncle must be brother of

o
is given, but what part of the total quantity M and married. Rest aunt of s should be M `s

m
they bought is not given hence even both sibling.
statements are not sufficient to get the number We get answer from A.
of apples bought by Ram and Gopal. From B is not correct to get answer.
28. (d) We do not know whether Korea scored a goal 37. (b) Let the total no. of time coin tosses be n.
in the last 5 minutes or it may be possible that Total amount spent by ram = (10 + n × 1 )
india at begining has 0 goal and korea has 2, = 10 + n
so at the end they may draw, even from both First he paid ` 100 but he suffered ` 50 losses.
statement we can not conclude. Then amount left with him,
29. (a) From Statement A it gives us that by adding (10 + n) – 100 = 50 ⇒ n =140
4, the number (4, 12, 20…) would be divisible We get result using A.
by 8. Hence it is sufficient enough For statement B we have 2 cases
30. (a) From statement A - we get (x + y)(x + y/xy) = Case (1): Game ended normally – this case is
2
4; or (x + y) = 4xy ; similar to statement A

2
Or ( x – y ) = 0, hence x = y. Case (2): Game ended prematurely – this case
From statement B- (x – 50)2 = (y – 50)2, is not possible with 138 tails.
or x – 50 = +/– (y – 50) Hence both the statements are sufficient
Hence we will get x = y or x + y =100 hence enough
We cannot get the answer from the second 38. (c) Price of each soap= ` 10
statement. From statement 1 : She spent total ` 210.
31. (a) Statement A – After reducing the listed price So, she bought 210/10=21 soaps. And get last
by 10 percent, the dress sold for a net profit letter S. We cannot say anything from it.
of 10 dollars and given that net profit is 20% From statement 2: Total vowels = 18 means
hence we can get the CP. he got no. of A and O=18
But statement B just gives the SP but we do From it we cannot say anything
not know the discount. But using both, no. of P she got = 21– (18 + 1)
32. (d) We cannot arrive at the even with the use of =2
both the statement. Average since we do not 39. (c) Race of A and C, A wins by 375 m and A wins
know individual scores or number of students. by (60 + 30) = 90s
33. (d) After putting different values in the given Speed of c = (375/90) m/s. Time cant be
statements. We find that the question cannot calculated because we don’t know the length
be answered even by the use of both the of track.
statement as we get different answers. From statement B, length of track = 1 Km.
34. (c) If we draw a Venn diagram we will find that, from it nothing is concluded.
we need both statements. But using A and B we get speed of C = 1000/
Hence by using both the statement we can (375/90) .
solve it. 40. (a) Assume A, B, C, D get score 10, 8, 6, 4
35. (c) From statement A we get only J’s share. ABCD
Statement B also alone can not solve it 10 8 6 4
But by combining the statements we get the Statement A:
values of each student. With the conditions A will give vote to B
36. (a) From above question, Father and Mother of With the conditions B will give vote to A
S is F and respectively. Total no. of uncle and With the conditions C will give vote to A
aunts are 4 and 3 respectively. Even if D gives to A/B/C–2 situation arises.
y
o
u
rs
m
a
h
b
o
o
b
.w
o
rd
194 Data Sufficiency

p
re
Either A will win or there will a tie when D 46. (d) Let the abbreviation of their name as A,B,C,D

s
s
.c
gives vote to B. and E,

o
Even then A will win. From main information D>E, B >C, and the

m
So we are getting the answer. rank of C is either 4th or 5th in descending
Statement B: Cannot conclude anything. order. And we need to find the highest ranker.
41. (a) We have to find out, which one greater: 30% Now From Statement 1- it is given that A
of Z or 40% of S. ranked 5 and any one of the B or D can be
Statement A: 20% of Z > 25% of S; So, we the top ranker, this statement is not sufficient
can not conclude anything about 30% of z : enough as any one of B or D can be top ranker.
40% of s; Statement 2- B’s rank is either 3, 4 or 5, again
Statement B: 13 % of S > 10% of Z => 39 % we don’t have clear information that who is
of S > 30 % of Z the topper.
Which further implies, 40% of S > 30 % of Z Combining the statement 1 & 2 we can
42. (b) Statement A: We can find, there are 12 Tails conclude that D is the top ranker as from
and 9 Heads. statement 2 B cannot be the topper.
After tosses he will reach at blue point. So 47. (c) Let the number of employee be 100x
statement A is sufficient. Then the number of female employee is 70x,
Statement B: 3 more Tails greater than and those with engineering background 7x
Heads. So he will reach at blue point after Statement I- the no of employee with
tosses. So statement B is also sufficient. engineering background is 25x
43. (a) Statement A: Cannot say anything. The % of male employees with engineering
Statement B: Because amongst the Top 5 → background out of the males = (25x-7x)/30x
3 are boys, 2 are girls. And Rashmi is third ×100
among the girls and Kumar is 6th. Similarly the % of employee out of total
So, B is sufficient. employees can be calculated.
44. (d) Statement A: 13 currency notes will give Hence this statement is sufficient enough.
different possible Values and hence it is not Statement II- the number of male employees
sufficient. Statement B: Multiple of 10 and having engineering background is 84x
will give many values. Similar to above calculation this statement is
Even if you combine the statement, we can also sufficient enough.
have various values. 48. (e) Statement I- this statement doesn’t give
45. (c) Statement A: 2 kg potato cost + 1 kg gourd information about the number of goals scored
cost < 1 kg potato cost + 1 kg gourd cost or by the opponent team, hence this statement is
1 kg potato cost < 1 kg gourd cost. Hence not sufficient enough.
statement A is not sufficient. Statement II- This statement doesn’t give
Statement B: 1 kg potato cost + 2 kg onion information about the number of goals scored
cost = 1 kg onion cost + 2 kg gourd cost by Mahindra & Mahindra club, hence this
Hence 1 kg potato cost + 1 kg onion cost = 2 kg statement is not sufficient enough.
gourd cost. Combining both the statement even it is not
So statement B is also not sufficient as we can given the number of goals scored by Mahindra
not conclude. & Mahindra in 1sthalf, hence even by using
Combining both statements we get 1 kg potato both the information we cannot find the result.
cost < 1 kg gourd cost (From statement A)…(i) 49. (a) From statement I- 40% of total is given 10
1 kg potato cost + 1 kg onion cost = 2 kg hence total is 10/0.4 = 25.
gourd cost (From statement B) …(ii) This statement is sufficient enough.Statement
So we can conclude that the onion is costliest. II- It does not give any significant information.
y
o
u
rs
m
a
h
b
o
o
b
.w
o
rd
Data Sufficiency 195

p
re
50. (d) Statement A alone: Since we do not know the 3. (e) From statement (1) we cannot conclude

s
s
.c
number of byes got by the champion. Hence, anything.

o
statement A alone is not sufficient to answer From statement (2) since N is a perfect square

m
the question. as well as perfect cube hence N = P6 so
Statement B alone : Since we do not know possible values of N = 16 = 1, or 26 = 64, or
the exact number of players in the tournament. 36 = 729.
Hence, statement B alone is not sufficient to From both the statement we can have N = 1 or
answer the question. 729, hence we can find exact value of N
Combining both the statements together:
4. (a) From statement (1) alone since sum of two
If there are 83 players, then there will be 6
terms is even hence either both of them are
rounds in the tournament and we know that
even or both of them are.
the champion received only one bye, therefore
the total number of matches played by the Case (i) when both the terms are even then
tournament will be 6 – 1 = 5. Given in option 4 abc = even and def = even and hence abc + def
51. (d) Statement A alone : When n = 127, exactly is even
one bye is given in round 1. When = 96, Case (ii) when both the terms are odd then
exactly one bye is given in round 6. As no abc = odd and def = odd and hence abc + def
unique value of n can be determined hence, is even
statement A alone is not sufficient. Hence from this statement we can say that
Statement B Alone: Since we do not know abc + def is even.
exactly how many bye’s are given, so, we From statement (2) alone again we have two
cannot determine the value of n, uniquely. cases
Combining statement A and B: using both
Case (i) when both a and d are even then both
the statement. There is a unique value of n =
abc and def are even hence abc + def = even
120, for which exactly 1 bye is given from the
third round to the fourth round. Case (ii) when both a and d are odd then we
cannot say anything about ‘abc’ or ‘def’ as we
Concept Deviator don’t have information about b, c, e and f.
1. (c) From statement (1) alone we cannot say Hence this statement alone is not sufficient
anything as, K = 1/3 (When P = 3) or K can be enough.
integer. 5. (e) From statement (1) alone we can’t answer
From Statement (2) alone we can’t deduce the question.
anything as nothing is given about P From statement (2) alone again we can’t
From both the statement if P is prime and K/P conclude anything.
is integer then K must be an integer. 6. (e) From statement (1) consider P = 2 + √3 and Q
2. (c) From statement (1) alone we know that A is = 2 – √3 then P + Q is rational but if P = √2
an even number and B is prime from this we and Q = √3 then P + Q is irrational hence this
cannot conclude about A/B. statement is not sufficient enough.
From statement (2) alone consider A = 2 and From statement (2) consider P = 2+√3 and Q
B = 4 then A/B is not an integer but if A = 4, = 2 – √3 then PQ and P + Q both rational but
B = 4 then A/B is an integer hence this if P = √2 and Q =2 √2 then PQ is rational but
statement is not sufficient enough. not P + Q hence this statement is not sufficient
If we combine both the statement then only enough.
possibility is A =4 and B =2 or A = B = Prime
number in both cases A/B is an integer. Even we use both statements then also we
will have same situation.
y
o
u
rs
m
a
h
b
o
o
b
.w
o
rd
196 Data Sufficiency

p
re
7. (c) From the given information 0.

s
abcd − ab

s
12. (d) Since a.bcd =

.c
−abc − ab 100a + 10b + c − 100a − b P

= abc = = 900

o
900 900 Q
{(1000a + 100b + 10c + d ) − (10a + b )}

m
Hence P + Q = 90a + 9b + c + 900 is divisible =
900
by 9 only when c = 0 or 9
From statement (1) alone a + b + c is divisible 990a + 99b + 10c + d
= = P/Q
by 9 from this we can not conclude about c 900
whether it is 0 or 9. Hence P – Q = 990a + 99b +10c + d – 900
From statement (2) alone a + b, b + c, and is divisible by 9 that means 10c + d should
c + a are divisible by 9 but from this we can be divisible by 9 or two digit number ‘cd’ is
not conclude about c whether it is 0 or 9. divisible by 9 or c + d divisible by 9.
From statement (1) two digit number ‘dc’ is
If we combine both the statements then since
divisible by 9 hence c + d divisible by 9 hence
a + b and a + b + c both divisile by 9 hence c
this statement is sufficient enough to find the
must be divisible by 9.
answer.
8. (c) From statement (1) alone we can conclude From statement (2) alone when two digit
that at least one of a or b must be positive number “dc” is divisible by 6 then resultant is
hence a^b + b^a may be positive or negative. not divisible by 3 hence “dc” is not divisible
From statement (2) alone either both of them by 9 so this statement is sufficient enough.
are positive or both of them are negative, 13. (d) Here in this case either statement is sufficient
hence a^b + b ^a may be positive or negative. enough to answer the question.
If we use both statements then we will get From statement (1) and (2) alone we are sure
both a and b are positive. that the number is prime
9. (b) From statement (1) alone we can say that the 14. (c) From statement (1) we do not have any
number is divisible by 2, 3, 5 and 7 i.e. 210, information about B and similarly statement
from this we can’t conclude that “abcd” is (2) alone is also not sufficient enough.
divisible by 4 or not. From statement (1) and (2) both we are sure
that even if A + B cannot be rational.
From statement (2) alone the number is
15. (e) None of the statement says that a, b and c are
divisible by (n –1)n(n +1) where n is odd hence
integers hence even with both the statements
the number is divisible by two consecutive
we cannot answer.
even numbers hence it must be divisible by 4.
16. (e) From statement (1) and (2) alone no
10. (e) From both the statements (1) and (2) it is not information is given about third number it
clear whether a, b and c are integers hence may be integer or it may be fraction.
we can’t conclude even by using both the If we combine both the statements then we
statements together. have 2 even integers and we may have 1 or
11. (a) From statement (1) alone digits are 1, 2 and none even negative integers, hence even by
3. Since sum of digit is 6 hence it can’t be using both the statements we cannot answer
divisible by 9 hence this statement alone is the question.
sufficient enough. 17. (a) Here we have to find the minimum value of
From statement (2) alone the number is A/B + B/A let us assume that A/B = t then
divisible by 210 hence possible values of N is A/B + B/A = t +1/t, we know that its minimum
210, 420, 630 and 840, out of these only 630 value is 2 if t is positive i.e. A/B is positive
is divisible by 9 hence we cannot conclude i.e. either both of them are positive or both of
from this statement. them are negative.
y
o
u
rs
m
a
h
b
o
o
b
.w
o
rd
Data Sufficiency 197

p
re
From statement (1) alone since AB > 0 hence 22. (c) From statement (1) alone:- If a number has 12

s
s
factors then possible ways of writing N is a11,

.c
either both of them are positive or both of

o
them are negative hence this statement alone a1b5, a2b3, a1b1c2

m
is sufficient enough to find the answer. Then N2 must be one of a22, a2b10, a4b6,
From statement (2) alone at least one of them a2b2c4 and corresponding number of factors
is positive from this we cannot be sure about of N2 is 23, 33, 35, 45, hence statement (1)
the parity of A/B. alone is not sufficient enough.
18. (e) From statement (1) alone we can’t determine From statement (2) alone number N has three
as third one may be positive or negative. prime factors but from this we can not say
Similarly from the 2nd statement. anything about N.
If we use both the statement then we will If we combine both the statements then
get one negative and two positive but we number must be in the form of a1b1c2 or
don’t know magnitude of these numbers i.e. N2 = a2b2c4 has 3 × 3 × 5 has 45 factors.
(negative is how much is it –1 or –1000) 23. (e) From statement (1) alone A is prime hence it
hence we can’t answer even with both the has only 2 factors and B and C must be either
statements. 4 or 9 (3 factors)
19. (c) From statement (1) alone: - since 46 + 47 + From statement (2) alone we can not find any
48 + 49 + 50 = 240 hence 1st of these natural conclusion.
numbers must be more than 45. If we combine both the statements then we
Similarly from statement (2) alone: - since 50 will find that B = 9 and C = 4 but both has
+ 51 + 52 + 53 + 54 = 260 hence smallest of same number of factors, hence even by both
these numbers must be less than 50. the statements we can not find out the result.
From both the statements together we are 24. (b) From statement (1) alone we have two prime
sure that one of the number must be 50 hence numbers 79 and 97 hence this statement
product must be divisible by 25. alone is not sufficient enough to answer the
20. (d) From statement (1) alone question.
235, 253, 237, 273, 257, 275, From statement (2) alone we have 67 and 76
325, 352, 327, 372, 375, 357 but out of these two only 67 is prime hence
523, 532, 527, 572, 537, 573 this statement alone is sufficient enough.
723, 732, 725, 752, 735, 753 25. (d) Odd number of factors means perfect square.
Hence we have 24 such numbers and 4 such If N/6 has odd number of factors then N/6 =
sets K2 or N = 6K2
Similarly from statement (2) alone we can From statement (1) alone N/24 = P2 or N =
find out numbers of such sets. 24P2 = 6(2P)2 hence N/6 = (2P)2 a perfect
21. (c) Odd number of factors means number should square hence has odd number of factors, so
be perfect square this statement alone is sufficient enough.
From statement (1) alone we can’t sure about From statement (2) alone 24N = Q2 or N =
the number is perfect square or not. Q2/24 hence N/6 = (Q/12)2 is a perfect square.
From statement (2) alone number N must be Hence this statement alone is sufficient
one of 62, 63, 64, 65, or 66, out of these only enough.
64 is a perfect square. 26. (d) If N has 81 factors then N must be in one
If we combine both the statements then only of the following formats : a80, a2b26, a8b8,
64 is a number that is divisible by only one a2b2c8, a2b2c2d2
prime number and it is a perfect square hence From statement (1) alone N14 has 3 prime
by using both the statements we can answer factors hence only possibility is N = a2b2c8
the question. hence this statement is sufficient enough.
y
o
u
rs
m
a
h
b
o
o
b
.w
o
rd
198 Data Sufficiency

p
re
From statement (2) alone N2 has maximum (3r +1) … etc hence if N3has 112 factors then

s
s
possible number of factors, N2 must be one only possibility is that 112 = 4 x 4 x 7 or N3

.c
o
of the following formats a160, a4b52, a16b16, = a3b3c6 and from this statement we can find

m
a4b4c16, a4b4c4d4 for maximum possible required ratio.
factors N2 = a4b4c4d4 from this we can find From statement (2) alone:- N2must be in the
out the number of factors of N3 hence this form of a2p x b2q x c2r… etc. and its number
statement alone is sufficient enough. of factors is given by (2p + 1)(2q + 1)(2r +1)
27. (c) From statement (1) alone we can conclude hence in this case N may be a2b4c4 or a22b2
that highest power of 2 is 2 but from this we hence this statement alone is not sufficient
do not know about power of 3. Hence this enough.
statement alone is not sufficient enough. 31. (e) From statement (1) alone since N/2 has 12
From statement (2) alone we can only factors hence it must be in one of the following
conclude that highest power of 3 in N is 2 formats a11, ab5, a2b3, a1b1c2
hence this statement is not sufficient enough. If N = a11 the 2N may be 212 or 21a11
If we combine both the statements then highest If N = a1b5 then 2N may be 21a1b5 or 22b5 or
power of 2 and 3 is 2 hence number must be in a1b6
the form of 22327p… since highest power of 2 If N = a2b3 then 2N may be 21a2b3 or 23b3 or
and that of 3 is same hence number of factors a224
divisible by 2 and by 3 must be same, so we If N = a1b1c1 then 2N may be 2a1b1c1 or 22b1c1
can answer that Number of factors divisible So from this statement alone we can not say
by 2 is not more than that divisible by 3. anything about N.
28. (a) From statement (1) alone we can say that From statement (2) alone definitely we cant
number is in the form of N = 22 x 7p x….. find factors of N
Number of factors of N will be 3(P+1) x ….. If we combine both the statement (1) and (2)
And 2N = 23 x 7p x….. and its number of then we have N = 212 or 23b3 hence we cant
factors are 4 x (P + 1) x…. = (81/3)4 = 27 x 4 answer.
= 108 hence this statement alone is sufficient 32. (c) From the given information N must be in the
enough. form of a17 or a1b8 or a2b5 or a1b2c2
From statement (2) alone we don’t know From statement 1 alone N is divisible by
whether N is divisible by 2 or not hence we 32 = 25 but not by 512 or 29 hence N may
cant determine the number of factors of 2N be in the form of a128 or a225 but in both the
29. (b) Since N has 16 factors hence N must be in cases we don’t know a hence this statement
one of the following formats- a15, a1b7, a3b3, alone is not sufficient enough.
a1b1c3, a1b1c1d1 Statement (2) is definitely not sufficient
From statement (1) alone N is divisible by 64 enough, if we combine both the statements
hence minimum power of 2 will be 6, then in then we will get a =5 then N = 2851 or 2552
this case we have two numbers that satisfy but in both the cases it is not divisible by 18.
this condition 215 and a1 × 27 hence from this 33. (c) None of the statement alone is sufficient
statement alone we can not find the value of N. enough. From 2nd statement number of factors
From statement (2) alone number is divisible of P is 6Q2 + 5Q + 1 = (2Q + 1)(3Q + 1) hence
by 72 or 2332 hence only possibility is that the p must be in the form of a2Qb3Q = (a2b3)Q,
number is in the format of 23x33 hence this Now if we combine statement (1) then we will
statement alone is sufficient enough. get N = a2b3 and its number of factors is 12.
30. (a) From statement (1) alone:- a number N3 must 34. (b) Let N is in the form of 2paqbr…. then number
be in the form of a3p x b3q x c3r x… and its of even factors are p(q + 1)(r + 1)… = 17 only
number of factors is given by (3p + 1) (3q + 1) possibility is that p = 1 and q + 1 = 17 or p = 17
y
o
u
rs
m
a
h
b
o
o
b
.w
o
rd
Data Sufficiency 199

p
re
Hence N must be in the form of 21a16 or in the 38. (b) From statement (2) alone if k is not a factor

s
s
form of 217 hence we can not find out number

.c
of 120 then k > 6 as 120 is divisible by all

o
of odd factors so this statement alone is not the numbers < 7. Hence this statement alone

m
sufficient enough to find the answer. is sufficient enough to answer the question.
From statement (2) alone :- Let 2N is in the 39. (d) From statement (1) alone if a number has
form of 2xaybz then number of odd factors is 16 factors then minimum such number is
(y + 1)(z + 1) = 17, and that is equal to number 2 × 3 × 5 × 7 =210 hence no such two digit
of odd factors of N. factor exist.
35. (e) Let N is in the form of 2p5qarbs…. Number From statement (2) alone minimum number is
of factors that has unit digit 5 are all the odd again 2 × 3 × 5 × 7 = 210 hence this statement
factors divisible by 5. Number of factors of N alone is sufficient enough.
that has unit digit 5 = q(r + 1)(s + 1) … 40. (e) From statement (1) alone if it has only three
From statement (1) alone (q +1)(r +1)(s +1) prime factors then number of factors may
… = 21 here we have three possibilities. N = be 27 if number is in the form of a2b2c2 or
2p520 or 2p52a6 or 2p56a2 from this number otherwise more than 27 hence we can not
of factors that has unit digit 5 is given by 20, answer the question from this statement alone.
14 or 18 hence this statement alone is not From statement (2) alone we can not be sure
sufficient enough. hence this statement is not sufficient enough.
From statement (2) alone:- we can only From both the statement again number may
conclude that N is not divisible by 2 or p = 0, be in the form of a2b2c2 (as it is given that If
from this also we cant find out the required N is a perfect cube then it is a perfect square
number of factors. also but it is not given that vice-versa is also
If we combine both the statement then also true)
we have three values 20, 14 and 18. 41. (e) From statement (I) alone the pairs are
36. (e) In this case we need to find the number of (1, 30), (2, 15) (3, 10) and (5, 6) now from
co-prime factors of N, From both the statement 2nd condition we may short the pairs as (1, 30)
we don’t know whether N is divisible by 7 or and (3, 10) but from this we can not conclude
not hence even by using both the statement we a definite relation between A and B, hence this
can not answer the question. statement alone is not sufficient enough.
37. (c) Let number is 2pbqcr… then number of factors If LCM of A and B is 30 then possible pair
is (p + 1)(q + 1)(r + 1), is (1, 30), (2, 15) (3, 10) (5, 6) (2, 30) (3, 30)
Number of even factors = p(q +1)(r +1) (5,30) (6, 30) (10, 30) and (15, 30)
Number of odd factors = (q +1)(r +1) Now if AB is odd then A must be odd, this
Number of even factors is more if p > 1. statement alone not sufficient enough as in
From statement (1) we are sure that minimum (1, 30) here A = 1 and B =30 while in (2, 15)
value of p is 1 but we cant say p >1 or not A = 15 and B = 2.
hence this statement alone is not sufficient Even by using both the statement we can not
enough. find relationship between A and B.
From statement (2) alone we can only say 42. (c) From statement (I) alone, two numbers A = 4x
that number is perfect square hence it is not and B = 4y, A + B = 4(x + y) and A – B = 4(x – y)
sufficient enough. in this case since we don’t know the HCF of
If we use both the statement then we will get (x + y) and (x – y) so we can not find HCF of
minimum value of p as 2 and hence number of A and B.
even factors will be more than number of odd From statement (II) alone, we can not
factors. conclude any thing.
y
o
u
rs
m
a
h
b
o
o
b
.w
o
rd
200 Data Sufficiency

p
re
If we combine both the statements then LCM If we try to combine both the statement

s
s
.c
= 4xy = 36 or xy = 9 so possible pair of (x, y) = together then we will get data inconsistent as

o
(1, 9) another pair (3, 3) is ruled out as A cant if one of the number is 15 then LCM can not

m
be equal to B so possible value of A + B is 40 be 40.
and corresponding value of A – B is 32 and 47. (e) Statement (I) alone is not sufficient enough,
their HCF is 8. we may take two examples for the values of
43. (c) From statement (I) alone, The pairs (A, B) is (A, B, C, D, E,F ) as ( 3, 4, 6, 5, 2 and 8)
(3, 16), (6, 16), (12, 16) (16, 24) and (F, 48) in this case required HCF is 8 but if we take
here F is a factor of 48, From this we can not a different case as each of them is prime
conclude about HCF of A and B. numbers then required HCF is 1.
From statement (II) alone numbers are in the From statement (II) alone, it is given that A
ratio of 1:2 so numbers can be (1, 2) or can is co-prime to D but may or may not be co-
be (8, 16) so we cant say anything about their prime to E and F hence from this statement
HCF. also we can not conclude the answer.
If we use both the statements then only From both the statement together also we can
possible pair is (24, 48) and their HCF is more not conclude the answer.
than 4. 48. (a) From the given condition if we assume f (1) = hx
44. (d) From statement (I) alone since AB and B are and f (2) = hy, here h is the HCF of f (1) and
co-prime to each other that means A and B has f (2) then from the given condition f (3) = hxy,
no common prime factors hence their HCF is f (4) = hxy and so on each term is hxy.
1, so statement (I) alone is sufficient enough From statement (I) alone we can be sure that
to answer the question. f (10) = f (7) = 36.
From statement (II) alone similar explanation From statement (II) alone since we don’t
as above this statement also sufficient enough know the value of f (2) hence we can not find
to answer the question. the value of f (10)
45. (b) From statement (I) alone we can conclude that 49. (b) From statement (I) above we can only
number is in the form of 12K+1 but we cant conclude that a is an odd number.
find unique remainder when it is divided by From statement (II) alone we can conclude
9, hence this statement alone is not sufficient that there must be a prime common factor
enough. exist hence HCF must be more than 1. Hence
From statement (II) alone minimum such this statement alone is sufficient enough.
number is [{(1 × 5 +1) × 4 +1} × 3 + 1] = 76 50. (b) From statement (I) alone possible pairs are
and any number that satisfy this condition (1, 18) (2, 18) (3, 18) (6,18) (9, 18) (2, 9) (6, 9)
must be in the form of (3 × 4 × 5 × 6)K + 76 = Since given that one of the number is prime
360K +76 and any number in this form when hence possible pairs are (2, 18)(3, 18)(2, 9)
divided by 9 will always gives remainder but in each case they are not co-prime to
4, hence this statement alone is sufficient each other hence statement (I) alone is not
enough. sufficient enough
46. (e) From statement (I) alone possible pairs are From statement (II) alone since number of
(5, 8) and (5, 40) in 1st case ratio between factor is 1 only in the case if number is 1,
LCM and HCF is 40 (which is more than 8) hence their HCF is 1 or they are co-prime to
but in 2nd case it is 8 (Not more than 8) hence each other. So statement (II) alone is sufficient
this statement alone is not sufficient enough. enough.
From statement (I) alone we can not get a 51. (d) From statement (I) alone N is the total number
unique value of 2nd number/LCM, hence this of students then N/4 gives remainder 2 and
statement alone is not sufficient enough. N/11 gives remainder 9. Hence N must be in
y
o
u
rs
m
a
h
b
o
o
b
.w
o
rd
Data Sufficiency 201

p
re
the form of: k(L.C.M of 4, 11) – 2 = 44k-2 if Maximum value of N is HCF of 360 and 600

s
s
.c
k = 1 then N = 42, if k = 2 then N = 86 hence is 120, so maximum value of R is 15, hence

o
this statement alone is not sufficient enough. we can not answer that R is more than 12 or

m
From statement (II) alone N is the total not.
number of students then N/5 gives remainder 55. (d) From statement (I) alone HCF of A, B, C and
2 and N/7 gives remainder 0 7 is the smallest D is same as HCF of 20 and 35 i.e 5, hence
number that satisfy above conditions. Hence this statement alone is sufficient enough.
N must be in the form of: k(L.C.M of 5, 7) From statement (II) alone, HCF of A, B, C and
+7 = 35k + 7 if k =1 then N = 42 and if k=2 D must be 10 hence this statement is sufficient
then N = 77, hence this statement alone is not enough.
sufficient enough. 56. (d) From statement (I) alone only possibility
If we use both the statements together then we of B and C is 1 & 25 and if any one of the
will get N = 42 will satisfy all the condition. number is 1 then required HCF is 1, hence this
52. (b) We don’t know here that the given year is a statement alone is sufficient enough to answer
leap year or not. So here we have two cases:- the question.
Case (1) :- if it is leap yaer then March will be From statement (II) alone, we will find
from 61st day to 91 day. out maximum possible value of HCF then
Case (2) If it is a non leapyaer then March possible pair of A, B, C & D is ( 25, 15, 50 &
will be from 60th day to 90th day. 75) or (10, 25, 50, 75) in both the cases HCF
From statement (I) alone both of them will is 5, hence their HCF is not more than 5, this
come together after an interval of 12 days statement alone is also sufficient enough.
i.e 12th, 24th and so on, so they will come 57. (d) From statement (I) alone, as per the given
together on 60th, 72nd and 84th day. information number must be in the form of
If it is a leap year then in a March they will 16p + z and when it is divided by 8 remainder
come on 2 days (72nd and 84th) is y hence z = 8 + y,
If it is a non leap year then in a March they Since number when divided by 8 gives
will come on 3 days (60th, 72nd and 84th) remainder y hence it must be in the form of
Hence this statement is not sufficient enough 8q + y and remainder when it is divided by 4
to give the exact answer. is x hence y = 4 + x
From statement (II) alone both of them will Hence z = 12 + x and y = 4 + x
come together after an interval of 21 days In this case possible pairs of (x, y z) = (1, 5, 13)
i.e 21st, 42nd and so on, so they will come (2, 6, 14) (3, 7, 15) in each case (x + y + z)/3
together on 63rd, 84th. remainder is 1.
Here in both the cases they will meet twice So this statement (II) alone we can only
in the month of March. Hence this statement conclude that at least 2 of the three is divisible
alone is sufficient enough. by 3 but we can not get a definite information
53. (e) about third number. Such example is (x, y, z)
54. (a) From statement (I) alone, It is HCF type = ( 4, 3, 9) in this case x + y + z is not divisible
question, here N must be a factor of by 3 and gives remainder 1 but if we take (x, y,
612 – 252 = 360 and 1212 – 612 = 600, z) as (5, 3, 9) then (x + y + z) gives remainder
Maximum value of N is HCF of 360 and 600 2 when divided by 3, hence this statement
is 120, so maximum value of R is 12, hence alone is not sufficient enough to answer the
we can answer that R is not more than 12. question.
From statement (II) alone, It is HCF 58. (d) From statement (I) alone Minimum HCF of
type question, here N must be a factor of AB and BC is B hence if HCF of (AB, BC and
615 – 255 = 360 and 1215 – 615 = 600, AC) is 1 that means B is co-prime to AC or to
y
o
u
rs
m
a
h
b
o
o
b
.w
o
rd
202 Data Sufficiency

p
re
A and C, similarly A & C are co-prime to each Hence this statement is not sufficient enough.

s
s
.c
other hence they all are co-prime to each other From statement (I) alone, a must be an odd

o
and their HCF is 1. number but not divisible by 5 hence not ends

m
From statement (II) alone similar explanation with 5, then condition is similar to previous
as above A, B and C are co- prime to each case except case 3 (i.e 4, 6) hence unit digit
other. of LCM is 0, so this statement is sufficient
59. (d) Let F(1) = hx, and F(2) = hy then enough.
F(3) = h2xy, F(4) = h2xy, F(5) = h2xy, 61. (e) From statement (i) alone we have following
F(6) = h4x2y2, F(7) = h4x2y2, F(8) = h4x2y2 cases:-
F(9) = h8x4y4, F(10) = h8x4y4, F(11) = h8x4y4 (x, y) = (2,3)
and so on. xy -> 23 -> 8 or 32 -> 9
Hence required HCF is same as F(3n) We will not get unique value of unit digit
From statement (I) alone since F(3n + 9) = hence statement (i) alone is not sufficient
1248, hence F(3n + 6) = 1224, F(3n + 3) = 1212 enough.
and F(3n) = 126, hence this statement alone is Statement (ii) alone y is a multiple of 10.
sufficient enough.
If we use both the statement then also we will
From statement (II) alone, since p>n so
not get a unique value of unit digit.
3p-6 > 3n-6 and hence F(3n-6) and F(3p-6) is
62. (b) We know that cyclicity of unit digit of 7 is
F(3n-6) = 1224 then F(3n-3) = 1248 and F(3n)
four hence we will divide the given power by
= 1296. This statement alone is sufficient
4 if remainder is 1, 2, 3 and 0 then unit digit is
enough.
same as that of 71, 72, 73, or 74.
60. (b) From statement (I) alone, if a2 divided by 8
remainder is 1 hence a must be odd number, So in this question we have to find the
so A and B are two consecutive even numbers remainder when ‘abcd’ is divided by 4 i.e
then their HCF must be 2 and hence their when two digit number ‘cd’ divided by 4.
LCM will be (AB)/2 so unit digit of LCM From statement (i) alone two digit number
will be unit digit of (AB)/2 since A and B are ‘ad’ is divisible by 4. Or we can say that d
two consecutive even numbers hence unit must be an even number. This statement alone
digits of A & B must be (0, 2), (2, 4), (4, 6), is not sufficient enough.
(6, 8), (8, 0) From statement (ii) alone since ‘abcd’ divided
If a ends with 1 then a4444 ends with 1 and by 72 it leaves remainder 1 hence when it is
unit digit of (A, B) = (0, 2) and unit digit of divided by 4 it leaves remainder 1 and so
(AB)/2 is 0 required unit digit is 7. Statement (ii) alone is
If a ends with 3 then a4444 ends with 1 and sufficient enough.
unit digit of (A, B) = (0, 2) and unit digit of 63. (b) Since cyclicity of unit digit of 3 is 4 hence we
(AB)/2 is 0 will divide 3x by 4, we know that 3odd divided
If a ends with 5 then a4444 ends with 5 and by 4 remainder is 3 and 3even divided by 4
unit digit of (A, B) = (4, 6) and unit digit of remainder is 1, hence what all we have to find
(AB)/2 is 2 is that x is even or odd.
If a ends with 7 then a4444 ends with 1 and From statement (i) x may be even (if = 2)
unit digit of (A, B) = (0, 2) and unit digit of otherwise it is odd hence this statement is
(AB)/2 is 0 alone not sufficient enough.
If a ends with 9 then a4444 ends with 1 and From statement (ii) alone x is a three digit
unit digit of (A, B) = ( 0, 2) and unit digit of prime number hence it must be odd hence this
(AB)/2 is 0 statement alone is sufficient enough.
y
o
u
rs
m
a
h
b
o
o
b
.w
o
rd
Data Sufficiency 203

p
re
64. (c) If a number has only 2 factors then it must be 67. (e) From statement (i) alone we can say that unit

s
s
a prime number. And we know that 4odd has

.c
digit of a ends with either 1 or with 9. Hence

o
unit digit 1 and 4even has unit digit 6. this statement alone is not sufficient enough.

m
From statement (i) alone, x –1 is prime number We can not conclude anything from statement
P then x = P + 1 if P is 2 then x = 3 (odd) or (ii) alone.
if P is any other number than 2 (say 3) then If we use both the statements then we can say
x = odd hence this statement is not sufficient that a is a multiple of 9 that ends with 9, so
enough. even by using both the statements we can not
From statement (ii) alone, x + 1 is prime find the unique value of a.
number P then x = P – 1, again as in the 68. (b) From statement (i) alone since 66 is in the
previous case x may be even or may be odd, form 4k +2 hence unit digit of a66 is same as
hence this statement alone is not sufficient that of a2 hence unit digit of a is either 1 or 9.
enough. Hence this statement alone is not sufficient
If we use both the statements then x – 1 and enough.
x + 1 both are prime numbers hence that prime From statement (ii) alone since 999 is in the
number can not be 2, (such pairs of prime form of 4k + 3 hence unit digit of is same as
numbers are 3 & 5 when x = 4, 5 & 7 when that of which is given 9 hence unit digit of a
x = 6) hence x must be even, so by using both must be 9 and hence required unit digit of is 9
the statements we cans say that x is even and so statement (ii) alone is sufficient enough.
required unit digit is 6. 69. (b) From statement (i) alone since 333 is in the
65. (a) We know that cyclicity of unit digit of 2 is 4 form of 4k + 1 hence unit digit of a333 is
hence we will divide the given exponent by 4. same as that of a hence a = 9 since we don’t
From statement (i) alone since a, b, c and d know the value of b hence we can not find the
are even numbers hence must be divisible by required unit digit.
4, so this statement is sufficient enough to From statement (ii) alone since 555 is in the
answer the question. form of 4k + 3 hence unit digit is same as that
From statement (ii) alone is in the form of of a3 since it is given 6 hence unit digit of a
oddodd when this is divided by 4 remainder must be 6 and hence required unit digit is 6.
may be 1 or 3 hence this statement alone is not Statement (ii) alone is sufficient enough.
sufficient enough to answer the question. 70. (d) We know that unit digit of 9odd is 9 and 9even
66. (e) Since cyclicity of unit digit of 3 is 4 hence is 1.
we will divide the given power by 4 and its From Statement (i) alone since x + y is odd
remainder will decide the unit digit. hence one of them must be even and one of
From statement (i) alone we don’t know them must be odd and one of them is even,
whether a is even (2) or odd (other prime hence required unit digit is given by 9odd +
numbers) hence this statement is not sufficient 9even has unit digit 1+ 9 = 10 or required unit
enough. digit is 0. Statement (i) alone is sufficient
From statement (ii) alone we can not conclude enough.
anything as three digit number may be even or From statement (ii) alone since product x.y
odd. is odd hence both x and y must be odd hence
If we use both the statements then we can 9odd + 9odd or required unit digit is 8, statement
conclude that number is a three digit prime (ii) is alone sufficient enough.
number that means odd number, but we don’t 71. (d) Since two digit number “xy” divisible by 4
know remainder when given exponent divided hence y must be even i.e. (0,2,4,6,8)
by 4. If Y=0 then unit digit will be 0
y
o
u
rs
m
a
h
b
o
o
b
.w
o
rd
204 Data Sufficiency

p
re
Y=2 then unit digit given by 24k which is 6 If x =1 then 17 + 71 has unit digit 8

s
s
Y=4 then unit digit given by 44k which is 6

.c
If x =3 then 37 + 73 has unit digit 0

o
Y=6 then unit digit given by 64k which is 6 If x =5 then 57 + 75 has unit digit 2

m
Y=8 then unit digit given by 84k which is 6 If x =7 then 77 + 77 has unit digit 6
Hence unit digit can be either 0 or 6. So
If x =9 then 97 + 79 has unit digit 6
statement (i) is not sufficient enough.
From statement (ii) alone (xy)xy is not From statement (i) alone summation is 6
divisible by 100, so from this statement we which can be given by two values of x = 7 & 9
cant conclude anything. hence this statement is not sufficient enough.
If we use both the statements then we will get From statement (ii) alone we will get x = 1.
unit digit 6. 75. (e) From statement (i) unit digit is 3 hence we
72. (e) We know that 34k+1 has unit digit 3, 34k+2 has have following cases if unit digit of x is 3
unit digit 9, 34k+3 has unit digit 7, and 34k has then y should be in the form of 4k + 1, if unit
unit digit 1 digit of x is 7 then y should be in the form of
Unit digit 0 is given by either 3 + 7 or 9 + 1 4k + 3, from this statement we cant say
Hence we have two cases – anything about unit digit of y hence this
Case (i) if 3 & 7 then x and y must be in the statement alone is not sufficient enough.
form of 4p + 1 and 4q + 3, then From statement (ii) alone unit digit 1 then we
xy = (4p + 1)(4q + 3) = 16pq + 12p + 4q + 3 have following cases if unit digit of y is 1 then
hence when xy divided by 4 remainder is 3. x can be any number, if unit digit of y is 3 then
Case (ii) if 1 & 9 then x and y must be in the x is in the form of 4k, if unit digit of y is 7
form of 4p and 4q + 2, then xy = (4p)(4q + 2) then x is in the form of 4k, if unit digit of y is
hence when xy divided by 4 remainder is 0. 9 then x is in the form of 2k, as in the previous
From statement (i) alone x may be in the case we cant conclude anything hence this
form of either 4k or 4k + 1 hence we can not statement alone is not sufficient enough.
conclude whether it is in case (i) or in case (ii) If we use both the statement then we also we
hence this statement is not sufficient enough. cant conclude anything
From statement (ii) alone x may be in the form 76. (b) From statement (i) alone x is in the form of
of either 4k + 2 or 4k + 3 hence we can not 30k-1 and cyclicity of last two digit of 2 is
conclude whether it is in case (i) or in case (ii) 20 and a number in the form of 30k-1 divided
hence this statement is not sufficient enough. by 20 will give us remainder 10k-1 which
Even if we use both the statements we can is either 9 or 19 hence this statement is not
not find the unique value of remainder when sufficient enough.
xy divided by 4 hence even by using both From statement (ii) alone x is in the form of
the statements we can not find the value of 20k-1 and any number in this format when
required unit digit. divided by 20 will always give remainder 16
73. (a) Since unit digit of 3 has cyclicity of 4. hence this statement is sufficient enough.
From statement (i) alone unit digit of x is 3
77. (c) We know that unit digit of the given expression
and that of y is 7 but y > x hence unit digit is 4,
is 1 and 2nd last digit is given by unit digit
so statement (i) alone is sufficient enough.
of product of 8 and z, so we have to find the
From statement (ii) alone if x > y then unit
value of z.
digit is 6 and if x < y then unit digit is 4 hence
From statement (i) alone xyz is a multiple of
this statement alone is not sufficient enough.
35 hence we can not find a unique value of z
74. (b) In both the statements summation is even that
(it may be 0 or 5) hence this statement alone
is possible when x is odd, we have following
is not sufficient enough.
cases-
y
o
u
rs
m
a
h
b
o
o
b
.w
o
rd
Data Sufficiency 205

p
re
From statement (ii) alone xyz is a multiple of 81. (d) Since cyclicity of remainder of [3/7x] is 6,

s
s
.c
115 hence we can not find a unique value of z hence we have to divide x by 6 and find the

o
(it may be 0 or 5) hence this statement alone remainder. From statement (i) alone when x is

m
is not sufficient enough. divided by 6, we won’t get a unique remainder.
If we use both the statement then we will Similarly from statement (ii) alone we
get xyz a multiple of 5, 7 and 23 only three cannotget a unique remainder. If we use both
digit number that satisfy this condition is 805. the statement together, then from L.C.M. type
Hence both the statement require to answer 2 x must be in the form of P (L.C.M. of 324)
the question. - 2 i.e. x = 12 P-2. When any number in this
78. (a) Since 76^n has last two digit 76 hence format divided by 6 will give us remainder
statement (i) is sufficient enough to answer -2 (or 4). Final remainder is same as that of
the question. 4th point of cycle or that of [34/7] which is
79. (c) From statement (i) alone ab has unit digit 1, 4. Hence both the statements are required to
hence possible cases are when a = 1 then b can answer the question.
be any digit, when a =3, then b can be 4 or 8, 82. (b) Cyclicity of remainder of [5x/9] is 6. From
when a = 7, then b = 4 or 8, when a =9 then statement (i) alone 2 digit prime no is in the
b = 2, 4, 6 or 8 hence from this statement form of 6k ± 1 hence we cannot find remainder
alone we can not conclude anything. when x divided by 6, so this statement alone is
From statement (ii) alone since unit digit of not sufficient enough.
ba is 4 hence possible cases are, when b =2, From statement (ii) alone number is in the
then a = 2 or 6, when b = 4 then a = 1, 3, 5, 7 form of 12k + 9 ; it is when divided by 6 will
or 9, when b = 8 then a = 2 or 6. Hence this give remainder 3. Hence this statement alone
statement alone is not sufficient enough. is sufficient enough.
If we use both the statement together then we 83. (e) From statement (i) alone a is in the form of
will get possible pairs that satisfy both the 5x + 1 and we can’t find remainder when it
cases are (a, b) = (1, 4), ( 3, 4), (7, 4) or (9, 4) is divided by 7. Hence, this statement is not
Since difference between ab and ba is not sufficient enough similar to statement (i).
divisible by 27 and |a – b| ≠ 3 hence (1, 4) and 84. (e) Since cyclicity of remainder of division
(7, 4) ruled out. So possible pairs of (a, b) is [7^k /9] is 3 hence to get desired remainder
(3,4 ) or (9,4) we have to find the remainder when k divided
Last two digit of 3443 is 04 by 3 but here k = ab, so we have to find the
Last two digit of 9449 is 04, [At this point remainder when ab is divided by 3.
when we are at the end of this chapter please From statement (i) alone since a and b both are
don’t wonder how last two digits of 3443 and prime numbers so we cannotfind remainder
9449 is 04] when ab is divided by 3.
Hence by using both the statement we can be From statement (ii) alone since given that a
sure that required last two digit is 04 and b both are even numbers then also we
80. (e) From statement (i) alone if a = 3 then b = 6 cannotfind the remainder when ab divided by
and if a =9 then b =3 hence this statement 3.
alone is not sufficient enough. Even by using both the statements we cannot
From statement (ii) alone if a =4 then b = 4 find the required remainder. x
85. (c) From statement (i) alone remainder of   is
and if a =2 then b = 8 hence this statement z
alone is not sufficient enough. ±1 hence if y is even then remainder is 1 and
Even by using both the statements we cant if y is odd then remainder is –1. Hence this
answer the question. statement alone is not sufficient enough.
y
o
u
rs
m
a
h
b
o
o
b
.w
o
rd
206 Data Sufficiency

p
re
89. (a) The required remainder is same as that of ‘x’

s
 x3 

s
( ( ))

.c
From statement (ii) alone remainder of    y z −1 
 z 

o
 x 

m
is 1 but since we don’t know the remainder time of remainders of  
 y 
when z divided by 3 hence this statement
alone is not sufficient enough. From statement (i) alone the remainder of
If we use both the statements together then we
x
( ( ))
 y z −1 
 x 
can be sure that remainder of   is 1 hence   is 1 hence actual remainder is
z  y 
required remainder is 1,  x2  x. since we don’t know the exact value of x
86. (e) From statement (i) alone remainder of  
 z  hence this statement alone is not sufficient
is 1 from this we can conclude that remainder enough.
of [x/z] may be +1 or –1, hence this statement From statement (ii) alone again we cannotfind
alone is not sufficient enough.  x3  the required remainder.
From statement (ii) alone remainder of   If we use both the statements then we will get
 z  x = 6 and y =5
is 1 from this we can conclude that remainder
90. (e) From statement (i) alone since x + y + z = 25,
of [x/z] is 1hence this statement alone is
possible values of x, y and z is 9, 9, 8 in any
sufficient enough to answer the question.
order hence we cannotget a unique remainder.
 3k 
87. (a) Cyclicity of remainder of   is 6- From statement (ii) alone we cannotconclude
7 anything.
From statement (i) alone since x = 6y hence If we use both the statements then we will not
both the remainders are same i.e R1 = R2 get any value of x, y z that satisfy both the
hence this statement alone is sufficient enough conditions.
to say that R1 is not more than R2. x
From statement (i) alone since x = 4y, so in 91. (d) Let remainder of   is R then
7  xk 
some cases R1 is more than R2, while in some Let R=1, then cyclicity of remainder of  
cases less and in some cases even it may be 7 
equal, hence this statement is not sufficient is 1. Hence y can be any integer.
 xk 
enough.  2x  Let R=2, then cyclicity of remainder of  
88. (c) Since cyclicity of remainder of   is 3 and is 3. Hence y is in the form of 3p 7 
7 
 3y   xk 
remainder of   is 6, hence we have to
Let R=3, then cyclicity of remainder of  
7  is 6. Hence y is in the form of 6p 7 
find the remainder when x and y divided by 3
 xk 
and 6 respectively.
Let R=4, then cyclicity of remainder of  
From statement (i) alone we cannot find the is 3. Hence y is in the form of 3p 7 
remainder when y divided by 6, hence this  xk 
statement alone is not sufficient enough.
Let R=5, then cyclicity of remainder of  
From statement (ii) alone we cannotfind the is 6. Hence y is in the form of 6 p 7 
remainder when y divided by 6, hence this  xk 
statement alone is not sufficient enough.
Let x = 6, then cyclicity of remainder of  
is 2. Hence y is in the form of 2p 7 
If we use both the statements then we can say
that remainder when x divided by 3 remainder
Hence if y is a multiple of 3 then R can be any
is 0 and when y divided by 6 remainder is 3, integer from 1 to 6, if y is a multiple of 6 then
hence by using both the statements we can R can be either 3 or 5. If y is a multiple of 2
answer the question. then R is 6.
y
o
u
rs
m
a
h
b
o
o
b
.w
o
rd
Data Sufficiency 207

p
re
From statement (i) alone since y is a two digit If we use both the statements then we will get

s
s
remainder 1 from Wilson’s theorem.

.c
prime number hence it is not divisible by 2

o
and 3 so we must have case 1 and R =1, hence 95. (b) From extension of Wilson’s Theorem-

m
this statement alone is sufficient enough. From statement (i) alone remainder is 2x,
From statement (ii) alone since y is a number since we don’t know the value of x, hence this
which is not divisible by any digit so we must statement is not sufficient enough.
have case 1 and R =1, hence this statement From statement (ii) alone remainder is 1, hence
alone is sufficient enough. this statement alone is sufficient enough.
92. (e) From statement (i) alone since remainder 96. (e) This question is exactly same as that
of question number 20 of section Data
of [x/6] is 4 hence x must be in the form of
Sufficiency of Chapter 5.
6k + 4 from this statement alone we cannotfind
From statement (i) alone if a = 3 then b = 6
the remainder when x divided by 24, hence and if a =9 then b =3 hence this statement
this statement alone is not sufficient enough. alone is not sufficient enough.
From statement (ii) alone since remainder of From statement (ii) alone if a = 4 then b = 4
[x/4] is 3 hence x must be in the form of 4k +3 and if a =2 then b = 8 hence this statement
from this statement alone we cannot find the alone is not sufficient enough.
remainder when x divided by 24, hence this Even by using both the statements we cannot
statement alone is not sufficient enough. answer the question.
If we use both the statement, actually we 97. (c) From statement (i) alone we cannotfind
cannotuse both the statements simultaneously remainder when a divided by bc, hence this
as they are not compatible (means from statement alone is not sufficient enough.
statement (i) x is even but from statement (ii) From statement (ii) alone we cannotfind
x is odd) hence we cannotfind the required remainder when a divided by bc, hence this
answer even by using both the statements. statement alone is not sufficient enough.
93. (a) From statement (i) alone x must be in the form If we use both the statements then we will get
of k(L.C.M of 18 & 8) + 1 or in the form of b =2 (the only even prime number), and a is
72k + 1 and any number in this format when a composite even hence minimum value is
divided by 36 given remainder 1, hence final 4, hence is always divisible by, and required
remainder is zero.
remainder is 1 and it is independent from
98. (c) From both the statements we can find that
y. hence this statement alone is sufficient
y =10, then x =100 and then denominator is
enough. xa + yb + c =100a + 10b + c = three digit
From statement (ii) alone x must be in the number ‘abc’. In that case remainder is 0.
form of k(LCM of 18 & 8) - 1 or in the form of Hence by using both the statements we can fid
72k-1 and any number in this format when the required remainder.
divided by 36 given remainder -1 (or 35), 99. (a) From the extension of Wilson’s theorem we
hence final remainder is 1 when y is even know that-
and it is -1 or 35 when y is odd, hence this If 2n + 1 is a prime number then
statement alone is not sufficient enough. Case (1) when n is odd then
94. (c) We know from Wilson’s theorem that
 ( P − 2 ) ! Remainder of 
{ }
 ( n !)2 
 is 1
remainder of   is 1 here P is a prime  2n + 1 
number.  P 
Case (2) when n is Even then
From statement (i) alone we cannotfind the
required remainder, Remainder of 
{ }
 ( n !)2 
 is 2n
From statement (ii) alone we cannotfind the  2n + 1 
required remainder, Hence statement (i) alone is sufficient enough.
y
o
u
rs
m
a
h
b
o
o
b
.w
o
rd
208 Data Sufficiency

p
re
100. (d) From statement (i) alone since a, b and c is From statement (ii) alone a is divisible by

s
s
.c
a two digit number hence unit digit of their 6 and hence this statement alone is also

o
factorial must be 0, and hence required unit sufficient enough.

m
digit is 0, so this statement alone is sufficient 105. (a) We know that a! is divisible by 64 if a ≥ 17
enough. From statement (i) alone if a! is divisible by
From statement (ii) alone a, b and c is a prime 17 then a ≥ 17, hence this statement alone is
number except 2 and 3 hence unit digit of sufficient enough.
their factorial must be 0 and hence required From statement (ii) alone if a! is divisible by
unit digit is zero. Hence statement (ii) alone is 26 then a ≥ 13, hence this statement alone is
sufficient enough. not sufficient enough.
101. (a) From statement (i) alone since a ≥ b hence a! 106. (b) From statement (i) alone since highest power
must be divisible by b! and hence remainder is of 9 in a! is 5 hence that of 3 must be either 9
zero, this statement is sufficient enough. or 10
Highest power of 3 in 24!, 25! And 26! Is
From statement (i) alone since a ≤ b if a = b
10 and that in 21!, 22!, and 23! Is 9 so this
then remainder of [a!/b!] is zero and when
statement alone is not sufficient enough.
a < b then remainder depends on the value
From statement (ii) alone since highest power
of a and b, hence this statement alone is not
of 9 in a! is 6 hence that of 3 must be either 12
sufficient enough. or 13
102. (c) From statement (i) alone 63 ≤ N < 72, if Highest power of 3 in 24!, 25! And 26! Is 10
63 ≤ N <70 then highest power of 7 is 10 and that in 27!, 28!, and 29! Is 13, so only
and if 70 ≤ N < 72 then highest power of 7 possibility is that highest power of 3 is 13, so
in N! is 11,, hence this statement alone is not this statement alone is sufficient enough.
sufficient enough. 107. (a) From statement (i) alone since highest power
From statement (ii) alone 55 ≤ N < 65, if N of 27 in a! is 12 hence that of 3 must be either
=55 then highest power of 7 is 8, if 56 ≤ N <63 36, 37 or 38
then highest power of 7 is 9 and if 63 ≤ N < 65 Highest power of 3 in 78!, 79! And 80! Is 36
then highest power of 7 in N! is 10,, hence this and that in 81!, 82! And 83! Is 40 so highest
statement alone is not sufficient enough. power or 9 must be 18 so this statement alone
If we combine both the statements then we is sufficient enough.
will get 63 ≤ N < 65 and in any case highest From statement (ii) alone since highest power
power of 7 in N! is 10, hence we can answer of 27 in a! is 9 hence that of 3 must be either
the question by using both the statements. 27 or 28 or 29
103. (b) From statement (i) alone it is not known that Highest power of 3 in 57!, 58! And 59! Is 27
a is a two digit number or not (as factorial of and that in 60!, 61!, and 62! Is 28, so highest
a two digit number will have last two digits as power of 9 may be 13 or 14 so this statement
00) hence this statement alone is not sufficient alone is not sufficient enough.
enough. 108. (a) From statement (i) alone minimum value of N
Statement (ii) alone is sufficient enough. is 5, and hence required unit digit is 0 so this
104. (d) If a! is divisible by 18 then minimum value of statement alone is sufficient enough.
From statement (ii) alone N can be any odd
a is 6, as 6! Is the smallest factorial divisible
multiple of 3 hence this statement alone is not
by 72.
sufficient enough.
From statement (i) alone 6! Is the smallest
109. (a) From statement (i) alone N must be either 100
factorial divisible by 18. And in this case it is
or 101 in both the cases highest power of 5
divisible by 72 as well, hence this statement and hence 10 is 24 so this statement alone is
alone is sufficient enough. sufficient enough.
y
o
u
rs
m
a
h
b
o
o
b
.w
o
rd
Data Sufficiency 209

p
re
From statement (ii) alone Highest power of a! but in both the cases it is more than 6 hence

s
s
.c
2 is either 46 or 47 from trail and error we highest power of 10 is 6, so this statement

o
know that possible values of N is 49 and 50, alone is sufficient enough.

m
since highest power of 2 in 49! And 50! Is 46 From statement (ii) alone a must be either
and 47 respectively, and highest power of 5 or 48 or 49 and in both the cases highest power
10 in 49! And 50! Are 10 and 12 respectively of 5 in a! is 10 hence this statement alone is
hence so this statement alone is not sufficient sufficient enough.
enough. 114. (a) From statement (i) alone a ≥ 7 hence unit
110. (e) We know that highest power of 10 in 50! Is 12. digit of a! must be zero and hence required
From statement (i) alone N can be 49, 50, 51, unit digit is zero hence this statement alone is
52, or 53, If it is 49 then answer is 10 or else sufficient enough.
12 hence this statement alone is not sufficient From statement (ii) alone if a = 0 then b = 4
enough and then unit digit of (a!)(b!) is 4 or else unit
From statement (ii) alone N can be 46, 47, 48, digit is zero, hence this statement alone is not
49, or 50, If it is 49 then answer is 10 or else sufficient enough.
12 hence this statement alone is not sufficient 115. (b) From statement (i) alone since a = b +7 hence
enough. [a!/b!] = (b +1)(b + 2)(b + 3)… (b + 7) if
If we use both the statements together then b > 3 then last two digit will be given by
also we can notbe sure as N= 49 and 50 hence 4 x 5 x .. x 9 x 10 which has last two digit as
even by using both the statements we can not zero but if b is not more than 3 then last two
answer the question. digits is not zero, hence this statement alone is
111. (c) From statement (i) alone we can find that not sufficient enough.
roots of the given quadratic equation is 6 or -1 From statement (ii) alone if b =5k then [a!/b!]
hence a = 6 but from this statement we don’t = (5k + 1)(5 k + 2)(5k + 3)(5k + 4) which has
have any information about ‘b’ hence this last two digits 24 irrespective of the value
statement alone is not sufficient enough. of k, hence this statement alone is sufficient
From statement (ii) alone we can not find enough.
a definite value of ‘a’ and ‘b’ hence this 116. (a) From statement (i) alone 19 is the next prime
statement alone is not sufficient enough. number after 17, from the given condition
If we use both the statement then b > a and 34 ≤ N/2 < 37 or 68 ≤ N < 74, since factorial
a = 6 hence required HCF is 6!. of N/2 is defined hence N must be an even and
112. (b) From statement (i) alone since highest power possible values of N is 68, 70, 72. Whatever
of 2 in a! is 41 hence a can be 44 or 45, and the value of N may be highest value of prime
highest power of 5 in 44! Is 9 and that in 45! number that has exponent 2 in N! must be
Is 10, hence we can notfind highest power of 31, hence this statement alone is sufficient
5 in a! so this statement alone is not sufficient enough.
enough. From statement (ii) alone 13 is the next prime
From statement (ii) alone a must be either number after 11, from the given condition
48 or 49 and in both the cases highest power 22 ≤ N/3 < 26 or 66 ≤ N < 78, since factorial
of 5 in a! is 10 hence this statement alone is of N/3 is defined hence N must be divisible by
sufficient enough. 3 and possible values of N is 66, 69, 72 and
113. (d) From statement (i) alone since highest power 75. If N is 66 then largest prime number is 31
of 2 in a! is 41 hence a can be 44 or 45, and and if N is 75 then largest prime number is
highest power of 5 in 44! Is 9 and that in 45! Is 37 hence this statement alone is not sufficient
10, hence we can notfind highest power of 5 in enough.
y
o
u
rs
m
a
h
b
o
o
b
.w
o
rd
210 Data Sufficiency

p
re
117. (a) We know that 3293 = 37 × 89 If we use both the statement together then we

s
s
.c
From statement (i) alone from the given can find the value of n and that of r, and hence

o
condition 51 ≤ N/2 < 68 or 102 ≤ N < 138, we can answer the question by using both the

m
since factorial of N/2 is defined hence N must statements together.
be an even. Whatever the value of N highest 120. (d) Log A X > Log A X2 as x is an integer so X2
power of 89 is 1 and that of 37 is more than 1 > X log of a greater number is less than that
hence highest power of 3293 must be 1, hence of a smaller number only when the base lies
this statement alone is sufficient enough. between 0 and 1, So both the statement I and
From statement (ii) alone from the given II are sufficient to answer. Option( D)
condition 68 ≤ N/2 < 111 or 136 ≤ N < 222,
121. (e) From statement I and II we cannot find the
since factorial of N/2 is defined hence N must
answer, when we combine, we have
be an even. In this case highest power of 89
is 1 or 2 hence this statement alone is not a2 + b 2 + 2ab = 25, (a + b)2 = 25, (a + b) = 5
sufficient enough. or –5. Still we are not having any idea.
118. (d) From statement (i) alone highest power of 9 122. (a) By using Statement I we can calculate the
in N! is 8 hence that of 3 in N! is 16 or 17, value of X, Statement Ii is redundant.
but from trail we will get highest power of 123. (c) From I, X = 1 or –2 and Y = 3 or –5
3 in 33! Is 15 and that in 36! Is 17, hence
From II, X = 4 or –2 and Y = 3 and –6
36 ≤ N < 38, and highest power of 2 in 36!
and 37! Is 34 while that in 38! Is 35. But in Combining both we get X = –2 and Y = 3
all the cases highest power of 16 is 8 and 124. (a) Let X = 2 and Y = 1, than ( X2 + Y2 ) = 5 and
number of zeros when N! is converted to base ( + ) = 1.5, hence I alone is sufficient.
16 is 8, hence this statement alone is sufficient 125. (e) For Statement I : we get X = 27 or 8, from the
enough. value of x we cannot find the unique value of
From statement (ii) alone highest power of X as both the values of x are perfect cube. So
12 in N! is 8 hence that of 3 in N! is 8 and if X is perfect cube X2 must be perfect cube.
highest power of 4 in N! is 8 or that of 2 is 16 Both the statement are not sufficient to
or 17. Possible value of N is 36, 37 or 38 in answer.
any case highest power of 16 in N! is 8 8 and
number of zeros when N! is converted to base 126. (c)
16 is 8, hence this statement alone is sufficient 127. (e) Both the statement are not sufficient to answer
enough. the question as From one X = 8 and Y = 7 than
n! X > y and from II, X = –8 and Y = –7 where
119. (c) Since nCr = Y > X.
{r ! n − r )!}
(
( r − 2 )!
From statement (i) alone nCr = 128. (a) C can take any value except ¼. For X to
{r !2!} = be real, C < ¼. Science we cannot get a
( r + 1)( r + 2 ) hence this statement alone is
unique value of C. Statement I is sufficient
2 but Statement II is not sufficient as no more
not sufficient enough.
information is given.
From statement (ii) alone since highest power
of 2 in n! is 11 hence n = 14 or 15 but highest 129. (d) We know X3 + Y3 + Z3 – 3XYZ
power of 3 in n! is 6 hence n must be 15. From = ( X + Y + Z) (X2 + Y2 + Z2 – XY – YZ – ZX).
this statement we don’t have any information Each statement alone is sufficient to give the
about r hence this statement alone is not answer.
sufficient enough.
y
o
u
rs
m
a
h
b
o
o
b
.w
o
rd
7
60

p
re
50

s
40

s
.c
30

o
Miscellaneous

m
20

10

0
SUN MON TUES WED THRUS FRI SAT
10am to 2pm 2pm to 6pm 6pm to 10pm

Topics No. of Questions Level


Concept Applicator 5 Bank PO basic level [RBI-2011]
Concept Builder 15 IIFT [2014]
Concept Cracker 4 XAT 2015
Total 24

Directions (Qs. No. 1-5): Study the radar-graph 1. What is the average of the monthly salary of Sumit
carefully and answer the questions that follow. Monthly in the year 2008, Anil in the year 2009 and Jyoti in
salary (in thousands) of five different persons in three the year 2010?
different years [RBI Grade ‘B’ Officer’s Exam – 2011] (a) ` 20,000 (b) ` 25,000
(c) ` 45,000 (d) ` 15,000
Sumit
30 (e) None of these
2008
25 2009 2. The total monthly salary of Arvind in all years
20 2010 together is what per cent of the total monthly salary
15
of all the five persons together in the year 2008?
Anil
Poonam 10
(a) 55% (b) 60%
5
(c) 75% (d) 70%
0 (e) None of these
3. Among the five persons, whose earning per month
over all the years together is the second lowest?
(a) Sumit (b) Anil
Jyoti (c) Jyoti (d) Arvind
Arvind
(e) Poonam
y
o
u
rs
m
a
h
b
o
o
b
.w
o
rd
212 Miscellaneous

p
re
4. What is the per cent decrease in the monthly salary 5. If Jyoti’s monthly salary in the year 2010 was

s
s
.c
of Poonam in the year 2009 as compared to her increased by 30 percent what would her monthly

o
monthly salary in the previous year? salary be in that year?

m
(a) 40% (b) 10% (a) ` 36,000 (b) ` 39,000
(c) 20% (d) 80% (c) ` 45,000 (d) ` 42,000
(e) None of these (e) None of these

Directions (Qs. No. 1-5): Read the following information and Tables and answer the questions [IIFT 2014]
BHUBANESHWAR, CHENNAI, KANYAKUMARI, KOCHI, MUMBAI and VIZAG are 6 major Indian cities.
For some reason people use only a certain mode of transport between a pair of cities. The modes of transport are
provided in Table 1, while in Table 2 the distances between different pairs of cities are given. Table 3 provides the
speed of the mode of transport and the cost associated with each of them.
Table 1. Mode of Transport between Cities
Destination
Origin BHUBANESHWAR CHENNAI KANYAKUMARI KOCHI MUMBAI VIZAG
BHUBANESHWAR – Ship Train Airplane Bus Train
CHENNAI Ship – Train Ship Ship Train
KANYAKUMARI Train Train – Train Bus Ship
KOCHI Airplane Ship Train – Train Airplane
MUMBAI Bus Ship Bus Train – Airplane
VIZAG Train Train Ship Airplane Airplane –

Table 2. Distance between cities (KM)


Destination
Origin BHUBANESHWAR CHENNAI KANYAKUMARI KOCHI MUMBAI VIZAG
BHUBANESHWAR – 950 700 798 701 1002
CHENNAI 950 – 999 901 1000 300
KANYAKUMARI 700 999 – 1100 950 250
KOCHI 798 901 1100 – 300 600
MUMBAI 701 1000 950 300 – 500
VIZAG 1002 300 250 600 500 –
y
o
u
rs
m
a
h
b
o
o
b
.w
o
rd
Miscellaneous 213

p
re
Table 3. Mode of Transport and Cost

s
s
.c
Mode of Transport KMPH Cost per KM (in rupees)

o
m
Airplane 60 5
Bus 40 2
Ship 30 1.5
Train 25 2.5

1. For which of the following options, travel time is order. What is the minimum travel cost (in `) the
the least? school should charge from each of the student for
(a) MUMBAI – KANYAKUMARI the entire tour?
(b) BHUBANEHSWAR – CHENNAI (a) ` 4300 (b) ` 5000
(c) CHENNAI – KOCHI (c) ` 7500 (d) ` 6800
(d) MUMBAI – CHENNAI 4. Which of the following cities can be reached
2. Mr. Ranjith lives in MUMBAI and wants to travel from BHUBANESHWAR in least time?
to KOCHI. However, the train services are on halt
(a) CHENNAI (b) KANYAKUMARI
due to laying of track for bullet trains across the
country. In this scenario, which of the following is (c) MUMBAI (d) VIZAG
the least cost route to reach KOCHI? 5. What is the least cost way to reach to VIZAG from
(a) MUMBAI – BHUBANESHWAR – KOCHI KOCHI?
(b) MUMBAI – CHENNAI – KOCHI (a) Take a flight from KOCHI to VIZAG
(c) MUMBAI – KANYAKUMARI – KOCHI (b) Take a ship from KOCHI to CHENNAI and
(d) MUMBAI – VIZAG – KOCHI then take a train to VIZAG
3. A school in Chennai is planning for an (c) Take a train from KOCHI to KANYAKUMARI
excursion tour for its students. They want to and then take a ship to VIZAG
show them KANYAKUMARI, VIZAG and (d) Take a train from KOCHI to MUMBAI and
BHUBANESHWAR, not necessarily in the same then take a flight to VIZAG
Directions (Qs. No. 6-10): Read the following information on ‘ Sectoral Trends in Mergers & Acquisitions in
India (2001 – 02 to 2006 – 07) given in tables below and answer these questions [IIFT 2014]
Table 1. Sector wise Number of Mergers & Acquisitions
Sectors 2001-02 2002-03 2003-04 2004-05 2005-06 2006-07
Food & Beverage 113 77 77 74 63 61
Textile 57 59 59 64 77 55
Chemicals 134 98 112 99 79 62
Drugs & Pharma 64 60 44 50 60 72
Cement 11 7 8 22 0 0
IT & Telecom 153 114 84 80 109 103
Diversified 15 8 13 4 7 5
Financial Services 194 201 160 116 193 177
Other Services 297 280 287 281 271 293
Misc Manufacturing 31 36 31 35 35 24
Non-Metallic Mineral Products 32 24 27 27 47 34
y
o
u
rs
m
a
h
b
o
o
b
.w
o
rd
214 Miscellaneous

p
re
Table 2. Sector wise number of Mergers

s
s
.c
Sectors 2001-02 2002-03 2003-04 2004-05 2005-06 2006-07

o
m
Food & Beverage 17 23 10 19 20 8
Textile 7 7 8 11 21 23
Chemicals 27 15 12 23 24 15
Drugs & Pharma 6 17 14 10 15 12
Cement 0 2 1 3 0 0
IT & Telecom 19 19 13 16 17 12
Diversified 1 0 1 0 0 0
Financial Services 91 107 87 41 75 51
Other Services 90 92 105 81 61 83
Misc Manufacturing 3 13 0 4 11 3
Non-Metallic Mineral Products 3 5 1 5 8 11

6. What is the approximate proportion of ‘mergers’ 10. In which year maximum sectors have exhibited
to ‘acquisitions’ for the entire period (2001-02 to higher number of acquisitions compared to
2006-06)? previous year?
(a) 26% (b) 36% (a) 2003 – 04 (b) 2004 – 05
(c) 30% (d) 20%
(c) 2005 – 06 (d) 2006 – 07
7. For how many sectors is the proportion of ‘mergers’
to ‘mergers & acquisitions’ greater than 20% for Directions (Qs. No. 11-15): Charts given below
the entire period (2001 – 02 to 2006 – 07)? describe the energy scenario of a country. Assume
(a) 2 (b) 3 that the country does not export any form of energy
and whatever is produced and imported is consumed in
(c) 4 (d) 5 the same year. Go through the charts and answer the
8. For how many sectors merger activity (measured questions [IIFT 2014]
by number of mergers) is more in the first three
years as compared to the last three years? Chart 1: Proportion of energy consumption in 2009
(a) 7 (b) 3 2%
2%
(c) 6 (d) 5 5%
Coal – 54%
9. If the turbulence over the period is defined by the 8%
Crude oil – 29%
sum of each of the differences (in absolute terms) in Natural gas – 8%
number of mergers & acquisitions on a year-on-year
Nuclear energy – 2%
basis, then which sector is considered most turbulent 54%
29%
for the entire period (2001 – 02 to 2006 – 07) ? Hydro electricity – 5%
(a) Financial Services Others – 2%
(b) IT & Telecom
(c) Food and Beverage Total energy consumed in the year 2009 is 1000 MTOE
(million tonne oil equivalent)
(d) Other services
y
o
u
rs
m
a
h
b
o
o
b
.w
o
rd
Miscellaneous 215

p
re
Chart 2: Annual (year-on-year) growth rate in con- 11. What was the approximate total import of energy

s
s
.c
sumption of different sources of energy for 2010 to 2012 in 2010?

o
(a) 400 MTOE (b) 300 MTOE

m
25

20 (c) 360 MTOE (d) 430 MTOE

15
12. The import of natural gas in 2012, when compared
to 2010, is approximately:
10
(a) Reduced by 10 MTOE
5 (b) Reduced by 13 MTOE

0
(c) Increased by 10 MTOE
Crude oil Natural Gas Coal Nuclear
energy
Hydro
electricity
others
(d) Increased by 5 MTOE
2010 2011 2012 13. What is the approximate domestic production of
crude oil in 2011?
Chart 3: Percentage of imports to consumption of
different sources of energy during 2010 to 2012. (a) 220 MTOE (b) 190 MTOE
(c) 160 MTOE (d) 280 MTOE
Others
14. What is the approximate proportion of coal in the
Hydro electricity domestic consumption of energy in 2012?
Nuclear energy (a) 52 (b) 54
Coal (c) 58 (d) 56
Natural gas 15. What is the sum of the approximate domestic
production of nuclear energy and hydro electricity
Crude oil in 2011?
0 10 20 30 40 50 60 (a) 75 MTOE (b) 80 MTOE
2012 2011 2010 (c) 90 MTOE (d) 100 MTOE

Directions (Qs. No. 1-4): On the basis of information types – Positive Tweets (in support), Negative Tweets
given below: [XAT 2015] (against) and Neutral Tweets. The following table
Twitter allows its users to post/share and read short presents the Number of Votes and Tweets received by
messages known as tweets. Tweets can be of three certain political parties.
y
o
u
rs
m
a
h
b
o
o
b
.w
o
rd
216 Miscellaneous

p
re
s
s
Number of Votes Tweets (Year 2010)

.c
Parties

o
Negative Tweets

m
Year 2000 Year 2005 Year 2010 Total No. of tweets Positive Tweets (%)
(%)
A 329,700 343,200 364,450 131,021 33.3% 35.4%
B 133,450 154,000 241,325 108,128 30.4% 29.7%
C 196,250 123,200 162,525 96,620 32.5% 26.6%
D 27,475 48,400 54,175 41,524 30.6% 36.1%
E – 30,800 49,250 32,724 21.6% 41.0%
Other Parties* 98,125 180,400 113,275 15,000

Any party which has secured less than 2% of the 3. Between 2000 and 2010, in terms of gain in vote
total votes falls under ‘Other Parties’ category. share which of the following cannot be a possible
For example, Party E secured less than 2% of total value (approximated to one decimal places) for
votes, in the year 2000. any party?
Note: If the vote share (%age of total votes) of a party
changes from 15% to 40%, gain in vote share would be (a) 2.0% (b) 2.5%
25% (=40% - 15%). (c) 3.5% (d) 4.5%
1. Which of the following options correctly arranges
the political parties in descending order of gain in (e) 7.5%
vote share from the year 2005 to the year 2010?
4. In 2010, which of the following options has
(a) EBDCA (b) EBCDA
(c) EBCAD (d) BCEDA maximum difference between the vote share
(e) BCEAD and tweet share?
2. Which of the following parties received maximum (a) Party B (b) Party C
number of “neutral tweets” in the year 2010?
(a) Party B (b) Party C (c) Party D (d) Party E
(c) Party D (d) Party E (e) Other Parties
(e) One of the parties categorized under ‘Other
Parties’
y
o
u
rs
m
a
h
b
o
o
b
.w
o
rd
Miscellaneous 217

p
re
s
s
.c
o
m
CONCEPT APPLICATOR 3. (d) Earning of each one over all the years :
Sumit = ` 60000
1. (a) Monthly salary of Sumit in year 2008 = 15000
Monthly salary of Anil in year 2009 = 15000 Anil = ` 40000
Monthly salary of Jyoti in year 2010 = 30000 Jyoti = ` 75000
60000 Arvind = ` 45000
Average monthly salary = = 20000
3 Poonam = ` 60000
2. (b) Total monthly salary of Arvind in all the years 25 − 20
together = 45000 4. (c) Required percentage = × 100 =
20%
25
Total monthly salary of all the five persons
5. (b) After an increase of 30%, Jyoti’s salary in
together in the year 2008 = 75000
30 × 130
45000 2010 = = ` 39000
Required percentage = × 100 = 60% 100
75000

CONCEPT BUILDER
1. (a) Follow the table
Place Distance Mode Speed time
MUMBAI – KANYAKUMARI 950 km Bus 40kmph 23.75 hrs
BHUBANEHSWAR – CHENNAI 950 Ship 30 kmph 31.66hrs
CHENNAI – KOCHI 901 Ship 30 kmph 30.03 hrs
MUMBAI – CHENNAI 1000 ship 30 kmph 33.33 hrs

2. (b) As from the table 3, we observe that the cost of travelling by ship is least that by other modes i.e.,
` 1.5 per KM.
From the options we find that the route Mumbai to Chennai and Chennai to Kochi is throughout by ship
and total distance is 1000 + 901 = 1901
Cost of journey = 1901 × 1.5 = ` 2851.5. which is least.
3. (a)
Chennai -Vizac Vizac - Kannya Kannya - Bhubaneswar Bhubaneswar - Chennai
Distance 300 250 700 950
Mode Train Ship Train Ship
Cost 300 × 2.5 250 × 1.5 700 × 2.5 950 × 1.5


Total cost = ` 4300 ( this is the amount school can charge from the students )
y
o
u
rs
m
a
h
b
o
o
b
.w
o
rd
218 Miscellaneous

p
re
4. (c) Follow the table for minimum time

s
s
.c
Chennai Kanaykumari Mumbai Vizac

o
m
Bhubaneswar 31.6 hrs 28 hrs 17.5 hrs 20.08 hrs

As speed of bus is much more than train and ship, hence time taken by bus must be less.
5. (b) As cost of the flight is higher so at very first we eliminate option (a) and option (d). By calculating the
cost of the routes of option (b) and option (c) we get option (b) ‘s route is cheaper.
6. (b ) total number of merger and acquisition over the period ( 2001-02) to (2006 -07) of all the sectors be
5646. Total number of mergers over the period ( 2001-02 to 2006-07) of all the sectors is 1499, which is
approximately 36% of 4147 = (5646 – 1499).
7. (d) Follow the given table below:
Merger and Acquisition Merger
Sectors Total M and A (2001-02 to Sectors Total M and A ( 2001-02 to
2006-07) 2006-07)
Food & Beverage 485 Food & Beverage 97
Textile 371 Textile 77
Chemicals 584 Chemicals 116
Drugs & Pharma 350 Drugs & Pharma 74
Cement 48 Cement 6
IT & Telecom 643 IT & Telecom 96
Diversified 52 Diversified 2
Financial Services 1041 Financial Services 452
Other Services 1709 Other Services 512
Misc Manufacturing 192 Misc Manufacturing 34
Non-Metallic Mineral 191 Non-Metallic Mineral 33
Products Products
Bold sectors are the greater than 20% for the entire period (2001 – 02 to 2006 – 07).
8. (d) Follow the table given below.
Sectors Total merger in first 3 years Total merger in last 3 years
Food & Beverage 50 47
Textile 22 55
Chemicals 54 62
Drugs & Pharma 37 37
Cement 3 3
IT & Telecom 51 45
Diversified 2 0
Financial Services 285 167
Other Services 287 225
Misc Manufacturing 16 18
Non-Metallic Mineral Products 9 24
y
o
u
rs
m
a
h
b
o
o
b
.w
o
rd
Miscellaneous 219

p
re
Hence, five such sectors are there, where the merger activity in the first three years as compared to the

s
s
.c
last three years is more.

o
9. (d) Follow the table given below for finding turbulence over the period.

m
Sectors Total M and A ( Sector wise) Total Merger (Sector wise) Turbulence
(a) (b) (a – b)
Food & Beverage 485 97 388
Textile 371 77 294
Chemicals 584 116 468
Drugs & Pharma 350 74 276
Cement 48 6 42
IT & Telecom 643 96 547
Diversified 52 2 50
Financial Services 1041 452 589
Other Services 1709 512 1197
Misc Manufacturing 192 34 158
Non-Metallic Mineral Products 191 33 158
Only other services have more turbulence.
10. (c) Below is the table for Acquisition i.e., (Table 1: merger and acquisition – table 2: merger )
Sectors 2001-02 2002-03 2003-04 2004-05 2005-06 2006-07
Food & Beverage 96 54 67 55 43 53
Textile 50 52 51 53 56 32
Chemicals 107 83 100 76 55 47
Drugs & Pharma 58 43 30 40 45 60
Cement 11 5 7 19 0 0
IT & Telecom 134 95 71 64 92 91
Diversified 14 8 12 4 7 5
Financial Services 103 94 73 75 118 126
Other Services 207 188 182 200 210 210
Misc Manufacturing 28 23 31 31 24 21
Non-Metallic Mineral Products 29 19 26 22 39 23

Dark one is the incensement in acquisition over the previous year. Hence, 2005-06 has the maximum.
11. (c) HYDRO ELECTRIC : 5% OF (110% OF 50)
COAL : 25 % OF (115% of 540)
Natural gas : 50% of (105% of 80)
Crude oil : 50% of (10% of 290)
Now adding these all, we get 360 MTOE, was the imported energy in 2010.
12. (b) In 2012, total energy growth of natural gas was 97 MTOE, 30% of 97 MTOE was imported i.e., 29.1
Similarly, in 2010 total imported gas was 50% of 84 = 42
Hence there was a reduction of (42 – 29) = 13 MTOE.
y
o
u
rs
m
a
h
b
o
o
b
.w
o
rd
220 Miscellaneous

p
re
13. (b) Domestic production of crude oil in the year 2. (a) Neutral tweet for any party = 100 Neutral

s
s
.c
2011 was = 55% of 290 × ( 110 %) × (110 %) tweets = 100% – (Positive tweets + Negative

o
= 193 MTOE tweets)

m
14. (d) 540 of (115%)(110%)(115%) + 290 of (110%) A 41009.6
(110%)(115%) + 80 of (105%)(110%)(105%)
B 43143.1
+ 20 of (120%)(115%)(110%) + 50 of (110%)
(105%)(110%) + 20 of (115%)(115%)(110%) C 39571.6
= 786 + 404 + 97 +30 + 64 +29 D 13827.5
= 1410 MTOE (approx.) E 12238.8
Hence required proportion = (786/1410) × 100 Hence, Party B received maximum number
= 56%
of neutral tweets.
15. (b) Domestic production of nuclear energy and 3. (b)
hydro-electricity in the year 2011 is as follows:
Vote % Vote % Gain in %
20 of (120%)(115%) + 95% of 50 of (110%) in 2000 in 2010 2000 – 10
(105%)
A 42.0% 37.0% – 5.0%
= 82 MTOE = 80 MTOE (approx.)
B 17.0% 24.5% 7.5%
CONCEPT CRACKER C 25.0% 16.5% – 8.5%


Lets find few essential data. D 3.5% 5.5% 2.0%
E 0% - 2% 5.0% 3% - 5%
Year 2000 Year 2005 Year 2010
Other
Total No. of votes 785, 000 880,000 985,000 12.5% 11.5% – 1.0%
parties
As we know, Total no. of tweets – 425, 017

From th question, In 2000 vote percentage
Neutral tweets = 100% - (Positive tweets + Negative for Party E is not given. Thus, maximum vote
tweets) percentage for Party E can be 2% in 2000
Neutral Tweets (if it was under the category ‘Other Parties’).
A 31.30% From Answer Table, we can directly conclude
that possible values are Option A (for Party D)
B 39.90%
and Option E (for Party B) and the value can
C 40.90% be in the range of 3% to 5% for Party E. So,
D 33.30% Options C and D are also possible. Hence,
E 37.40% Option B (2.5% is not possible) is the correct
answer.
1. (d) Follow the table
4. (e) Vote share Tweet share Deviation
Vote % in Vote % in Change in
37.0% 30.8% 6.2%
2010 2005 Vote share
A 37.0% 39.00% - 2.0% 24.5% 25.4% - 0.9%
B 24.5% 17.50% 7.0% 16.5% 22.7% - 6.2%
C 16.5% 14.00% 2.5%
5.5% 9.8% - 4.3%
D 5.5% 5.50% 0.0%
5.0% 7.7% - 2.7%
E 5.0% 3.50% 1.5%
11.5% 3.5% 8.0%

The sequence is BCEDA in descending order.

Das könnte Ihnen auch gefallen